You are on page 1of 228

K nng t hc l k nng

quan trng nht m mt


ngi c th s hu.

"Logic l c s ca hu nh ton
b kin thc m chng ta thu
nhn c.

TONY BUZAN

LEONARD EULER

11 12

06 07
05

09

08

VT L, HNH HC V TRI T TRN


Nguyn i Vit

ST MAYONNAISE V
BU C TNG THNG M
Nils Berglund

BNH LUN THI


OLYMPIC TON QUC T 2016
Nguyn Tin Dng

BT NG THC
TAM GIC, A GIC V A DIN
L T Quc Thng

V CC CHUYN MC KHC

CH BIN:
Trn Nam Dng

BIN TP VIN:
V Quc B Cn
Ng Quang Dng
Trn Quang Hng
Nguyn Vn Huyn
Dng c Lm
L Phc L
Nguyn Tt Thu
ng Nguyn c Tin

11 12

06 07
05

09

08

LI NG
Nng thng 8 vng rc trn nhng con ng, ma h n chn mi. Nhng cuc vui,
nhng chuyn i xa ang ng dp tng bng nht. Nhng u , vo thng tm lng chng,
vi cn ma cht i cht n, vi ngn gi sm mt lnh a n mi v mt ma thu tu trng
chng cn bao xa.
Tp ch Epsilon, ra mt vo nhng tun l cui cng ca ma h, cng l s ra mt ln th 10,
mt con s p v trn vn mi ngi cng nhn li mt chng ng i qua. i vi
nhng ngi trong ban bin tp, s 10, coi nh trm trm mt chu k, v bt u mt chng
ng mi phn u.
Chng ti cng hy vng rng vi bn c, c bit vi nhng ngi gn b vi bng en bc
ging, t nhiu s 10 ny cng s c ngha vo thi khc giao ma. khi thng tm tri
qua, thng 9 g ca, chng ta li cng bt u mt chng mi trn con ng truy tm tri thc
mnh mng.
i nhiu ngi, bn s i rt xa ...

MC LC
Ng Quang Hng
Bt ng thc khng-Shannon . . . . . . . . . . . . . . . . . . . . . . . . . . . . . . .

L T Quc Thng
Bt ng thc tam gic, a gic, v a din . . . . . . . . . . . . . . . . . . . . . . . . .

15

Nguyn Hng Sn
Ton hc v ngh thut tung hng . . . . . . . . . . . . . . . . . . . . . . . . . . . . .

26

ng Minh Tun
H mt m kha cng khai da trn ng cong Elliptic - Mt s ng dng . . . . . . .

33

Nguyn i Vit
Vt l, Hnh hc v Tri t trn . . . . . . . . . . . . . . . . . . . . . . . . . . . . . .

46

Nils Berglund, dch gi: Dng c Lm


St mayonnaise v bu c tng thng M . . . . . . . . . . . . . . . . . . . . . . . . .

50

Dng Trng Tn
Hc cch hc: Mt bi hc quan trng bc nht ang b b qun . . . . . . . . . . . . .

61

Nguyn c Hng
Leonhard Euler - Ngi thy v i . . . . . . . . . . . . . . . . . . . . . . . . . . . . .

64

L Ngc Tu
Gi tr no cho 1 + 1 + 1 + ... ? V cng hay -1/2? . . . . . . . . . . . . . . . . . . . . .

68

Trnh o Chin
Tip ni cu chuyn v mt tng ly tha . . . . . . . . . . . . . . . . . . . . . . . . .

76

Trn Quang Hng, Nguyn c Bo


V mt ton hay trn tp ch THTT . . . . . . . . . . . . . . . . . . . . . . . . . . .

91

Nguyn Trn Hu Thnh


Mt b v phn gic . . . . . . . . . . . . . . . . . . . . . . . . . . . . . . . . . . . 107
Trn Minh Ngc
Cc ng trn c hai im chung trong t gic ni tip . . . . . . . . . . . . . . . . . . 120
Trn Minh Hin
nh l Cauchy-Davenport v ng dng . . . . . . . . . . . . . . . . . . . . . . . . . . 135
L Anh Dng
S dng Modulo trong phng trnh nghim nguyn v bi ton chia ht . . . . . . . . . 156
4

Tp ch Epsilon, S 10, 08/2016


Nguyn Quc Anh
Chng minh BT bng phng php phn tch bnh phng vi s tr gip ca my tnh 174
Gary Antonick, dch gi: Nguyn V Duy Linh
Mt vi im c bit ca phong tro Olympic ton ca M . . . . . . . . . . . . . . . . 193
Nguyn Tin Dng
Bnh lun thi Olympic Ton Quc t (IMO) 2016 . . . . . . . . . . . . . . . . . . . . 198
Trn Nam Dng
Bi ton hay - li gii p . . . . . . . . . . . . . . . . . . . . . . . . . . . . . . . . . . 205
Ban Bin tp
Li gii thi Ton quc t Formula of Unity - The Third Millennium (tip theo) . . . . 209
Ban Bin tp
Cc vn c in - hin i . . . . . . . . . . . . . . . . . . . . . . . . . . . . . . . . 219

Tp ch Epsilon, S 10, 08/2016

BT NG THC KHNG-SHANNON
Ng Quang Hng
LogicBlox

TM TT
Tip theo bi gii thiu v entropy, cc bt ng thc Shannon v vi ng dng trong
Epsilon s 7, bi ny gii thiu mt bt ng thc khng-Shannon cng vi mt s tnh
cht m cc bt ng thc m hm entropy phi tho mn. Trong hnh trnh nho nh ny,
ta s tnh c gp mt mi quan h gia l thuyt s v l thuyt thng tin, v gia bt
ng thc thng tin v quy hoch tuyn tnh.

Trc ht, xin tm tt li mt s k hiu c gii thiu v dng trong bi trc [5]. Ta ch
xt cc phn b xc sut trn n bin ri rc X0 , . . . , Xn , trn cc min 1 , . . . , n tng ng.
Ta dng XS = (X
Qi )iS k hiu mt b bin ngu nhin c ch s trong tp S [n], v
xS = (xi )iS iS i k hiu mt b gi tr c th ca cc bin ny. Entropy ca mt
phn b cho ta mt con s H[XS ] vi mi tp con =
6 S [n]. Do , ta vit H(S) thay v
H[XS ]. Entropy ca mt phn b cho trc l mt hm tp hp H : 2[n] {} R+ . Hm
2n 1
entropy H cng l mt vector trn khng gian R+
, ti v c tt c 2n 1 tp con khc rng
ca [n], v mi tp con l mt to . Vi mt phn b khc th ta li c entropy khc, ngha
n
l mt hm tp hp khc v mt vector khc trong khng gian R2+ 1 . Bi trc chng minh
nh l sau y:
nh l 0.1. Xt mt phn b xc sut lin kt ca n bin tu h. Entropy H ca phn b ny
tho ba tnh cht sau y:
Tnh khng m: H(S) 0, S [n], S 6= .
Tnh n iu: H(S) H(T ), S T [n].
Tnh sub-modular: H(S T ) + H(S T ) H(S) + H(T ), S, T [n].
Ni cch khc, entropy H l mt polymatroid.
Tt c cc bt ng thc c tho mn bi mi polymatroid th tt nhin cng c tho mn
bi mi hm entropy. Ta gi chng l cc bt ng thc kiu-Shannon.

Tp ch Epsilon, S 10, 08/2016

1. Bt ng thc Zhang-Yeung
Trong hn na th k, tt c cc bt ng thc entropy m chng ta bit th u l bt ng thc
kiu Shannon. Nm 1998, Zhang v Yeung [7] khm ph ra mt bt ng thc khng chng
minh c bng cc tnh cht ca polymatroid:
2I(C; D) I(A; B) + I(A; C, D) + 3I(C; D|A) + I(C; D|B).

(1)

Nh rng thng tin tng h l mt hm tuyn tnh ca entropy:


I(X; Y ) = H(X) + H(Y ) H(XY ),
I(X; Y |Z) = H(XZ) + H(Y Z) H(XY Z) H(Z),
cho nn bt ng thc (1) l mt bt ng thc entropy. Bt ng thc ny cho thy s tn ti
ca mt bt ng thc ng vi mi entropy nhng khng ng vi mi polymatroids. Lm th
no m Zhang-Yeung tm ra v chng minh c rng
1. Bt ng thc (1) khng suy ra c t cc tnh cht ca polymatroid?
2. Tt c cc hm entropy u phi tho bt ng thc (1)?

1.1. Quy hoch tuyn tnh


Ta i ng vng tr li cu hi u tin. Nu i ng thng th ch cn ch ra mt polymatroid khng tho mn bt ng thc (1) l xong. Mt bt ng thc suy ra c t cc tnh cht
ca polymatroids nu v ch nu n c tho mn bi tt c cc polymatroids. Nhng ni vy
th qu m m, ta cn mt phng php c h thng no kim tra xem mt bt ng thc kiu
nh H(AB) + H(AC) + H(BC) 2H(ABC) c c tho mn bi tt c cc polymatroids
(trn 3 bin) hay khng.
Nh rng, nh vit trn, mt hm tp hp h : 2[n] R+ cng c xem nh mt vector
n
h R2+ 1 (v h() = 0 trong ng cnh ca ta). Ta dng cc tp con khng rng ca [n]
nh ch s cc to ca vector h. Mt hm tp hp l mt polymatroid nu v ch nu n
nm trong a din P = {Mh 0, h 0}, trong M l ma trn ca cc tnh cht n iu
v sub-modular trn. V d, vi tnh cht sub-modular trn tp S, T th s c mt hng ca ma
trn M tng ng vi bt ng thc
h(S) + h(T ) h(S T ) h(S T ) 0.
Hng ny ca ma trn M c hai s 1 cc to S, T , v hai s 1 cc to S T v
S T.
n 1

Mt bt ng thc tuyn tnh s c dng cT h 0, trong c R2


d, trong bt ng thc

h(AB) + h(AC) + h(BC) 2h(ABC) 0


7

l mt vector h s. V

Tp ch Epsilon, S 10, 08/2016

th vector c c ba s 1 cc to AB, AC, BC, v mt s 2 to ABC. Cu hi th


nht trn tng ng vi cu hi: lm th no bit l cT h 0 ng vi mi h P ?
Lu rng vector 0 P , ta c
cT h 0, h P nu v ch nu min{cT h | Mh 0, h 0} = 0.
Bi ton min{cT h | Mh 0, h 0} l mt bi ton quy hoch tuyn tnh c bn. V ta c th
gii n (bng my tnh) kim tra xem bt ng thc cT h 0 c ng vi mi polymatroids
hay khng. Mt cch khc l ta dng tnh cht i ngu ca quy hoch tuyn tnh; tnh cht ny
ni rng
min{cT x | Ax b, x 0} = max{bT y | AT y c, y 0},
nu nh mt trong hai bi ton c hm mc tiu hu hn. Bi ton min{cT h | Mh 0, h 0}
c quy hoch i ngu ca n vit l
min{cT h | Mh 0, h 0} = max{0T y | MT y c, y 0}.
Bi ton i ngu c hm mc tiu hu hn (bng 0) nu v ch nu n c nghim! Nh vy, ta
va chng minh c b sau1 :
B 1.1. Bt ng thc cT h 0 ng vi mi polymatroid h nu v ch nu h bt phng
trnh sau y c nghim:
MT y c, y 0.
Trong , M l ma trn ca cc bt ng thc sub-modularity v n iu.

Ni cch khc, bt ng thc cT h 0 ng nu m ch nu ta tm c cc h s y khng m


v t hp tuyn tnh dng cc h s y ca cc bt ng thc sub-modularity v n iu suy
ra cT h 0 c. i ngu trong quy hoch tuyn tnh nghe c v hi vang vang, nhng n
l mt tnh cht n gin; nu h bt phng trnh c nghim th ta suy ra c rng
cT h (MT y)T h = yT Mh yT 0 = 0.
Tt nhin, chng minh trc tip chiu ngc li ca b trn m khng dng quy hoch tuyn
tnh th kh hn mt cht; v lm vic ny khng cn thit lm trong ng cnh ca bi vit. Tm
li, B 1.1 cho chng ta mt thut ton kim tra xem mt bt ng thc kiu (1) c phi
l bt ng thc Shannon hay khng? Ta ch cn kim tra xem h bt phng trnh tuyn tnh
tng ng c nghim hay khng; bt k mt LP-solver no (nh cplex, Gurobi) u lm c
iu ny d dng.

1.2. Ln khng gian nhiu chiu hn


By gi ta quay li cu hi th hai: lm th no chng minh rng (1) l mt bt ng thc m
tt c cc hm entropy trn 4 bin u phi tho? y tht s l cu hi mu cht cn mt pht
kin tuyt vi ca Zhang v Yeung. i khi, h xy dng mt bin ngu nhin th 5, gi l R,
v dng mt bt ng thc kiu Shannon cho phn b (A, B, C, D, R). Bin R c mt tnh cht
1

y chng qua l mt dng ca b Farkas

Tp ch Epsilon, S 10, 08/2016

c bit, m nh khi ta chiu bt ng thc kiu Shannon t khng gian (A, B, C, D, R)


xung khng gian (A, B, C, D) th ta c bt ng thc (1).
Mt cch nm na hn, gi n l tp tt c cc hm entropies H ca n bin, v n l tp tt c
cc hm polymatroids ca n bin. nh l 0.1 cho ta bit n n . Ngoi ra, n l mt tp li,
v n khng phi l tp ng, nhng bao ng ca n cng l mt tp li. Mt bt ng thc
nh cT h 0, nu ng vi mi polymatroid, th chng qua l v cT h l mt siu phng nm
ngoi n ; vector c l mt php tuyn ca siu phng ny. Mt bt ng thc nh (1) ng vi
n nhng khng ng vi n th phi l mt siu phng nm ngoi tp n v ct vo trong n .
trn ta chng minh rng ci siu phng tng ng vi (1) ct 4 . chng minh rng n
nm ngoi 4 , ta tm mt siu phng nm ngoi 5 sao cho hnh chiu ca n xung khng
gian (A, B, C, D) chnh l siu phng tng ng vi (1).
C th hn, ta gh li ton b phng php ca Zhang-Yeung dng mt chng minh mi hn [2].
B 1.2. Gi A, B, C, D l bn bin t mt phn b lin kt no . Th, tn ti mt bin
ngu nhin R phn b lin kt vi A, B, C, D, vi cc tnh cht nh sau:
(i) Phn b ngoi vi ca (A, B, C) v (A, B, R) ging ht nhau (vi C thay bng R)
(ii) I(CD; R|AB) = 0.
Tm tt. Gi p(a, b, c, d) l hm cn nng xc sut ca phn b lin kt ca (A, B, C, D). Gi
R l mt bin ngu nhin mi c cng min vi C, v nh ngha hm cn nng xc sut
P
p(a, b, c, d) d p(a, b, r, d)
0
P
p (a, b, c, d, r) =
.
c,d p(a, b, c, d)

P
D thy rng r p0 (a, b, c, d, r) = p(a, b, c, d): ngha l phn b ngoi vi trn (A, B, C, D) ca
p0 chnh l phn b c ca (A, B, C, D). V t ta cng c p0 l mt hm cn nng xc sut
(tng bng 1). T y, kt thc chng minh b ch cn l c bp.
Ta vit li bt ng thc (1) mt cht. Trc ht, chuyn I(C; D) sang v phi v sp xp li,
d thy bt ng thc (1) tng ng vi bt ng thc sau y:
I(C; D)
I(A; B) + 2I(C; D|A) + I(C; D|B) + I(A; C, D) + I(C; D|A) I(C; D)

= I(A; B) + 2I(C; D|A) + I(C; D|B) + H(A) + H(CD) H(ACD)

+H(AC) + H(AD) H(ACD) H(A) H(C) H(D) + H(CD)

= I(A; B) + 2I(C; D|A) + I(C; D|B)

+H(CD) + H(AC) H(ACD) H(D) + H(AD) + H(CD) H(ACD) H(C)

= I(A; B) + 2I(C; D|A) + I(C; D|B) + I(A; D|C) + I(A; C|D).

Sau , i bin A C v B D th ta c (1) tng ng vi (2) di y.


9

Tp ch Epsilon, S 10, 08/2016

nh l 1.3 (Zhang-Yeung). Gi A, B, C, D l cc bin ngu nhin t mt phn b lin kt bt


k, th
I(A; B) 2I(A; B|C) + I(A; C|B) + I(B; C|A) + I(A; B|D) + I(C; D).

(2)

Chng minh. Gi R l bin ngu nhin t B 1.2. Lu rng thng tin tng h (gia hai
bin) v thng tin tng h c iu kin l cc i lng khng m. Ta c
I(A; B)
I(A; B)
+I(C; R|A) + I(C; D|R) + I(AB; R|CD) + I(D; R|B)
+I(A; B|RD) + I(D; R|A) + I(R; C|B) + I(A; B|CR) + I(C; R|ABD)
= 2I(A; B|C) + I(A; C|B) + I(B; C|A) + I(A; B|D) + I(C; D)
(= 0)
+2I(CD; R|AB)
(= 0)
+I(A; B|R) I(A; B|C)
(= 0)
+I(A; R|B) I(A; C|B)
(= 0)
+I(B; R|A) I(B; C|A).

2. Bt ng thc thng tin v bt ng thc nhm


nh ngha 2.1 (Bt ng thc thng tin). Nu bt ng thc
cT h 0,
ng vi mi h n th n gi l mt bt ng thc thng tin.
Do tt c cc hm entropy u l polymatroid, tt c cc bt ng thc Shannon u l cc bt
ng thc thng tin. Ngc li, c mt s v hn [4] cc bt ng thc thng tin khng phi l
bt ng thc Shannon. V d c th l bt ng thc (1). B 1.1 cho ta bit cch (bng
mt thut ton) kim tra xem mt bt ng thc c phi l bt ng thc Shannon hay khng.
T ny ra cu hi rt t nhin l: c kt qu no cho chng ta mt thut ton xc minh mt
bt ng thc thng tin khng? Tic rng cho n nay cha c kt qu no nh vy. Tuy nhin,
c mt kt qu th v ca Chan v Yeung [1] lin kt bt ng thc thng tin v ci gi l bt
ng thc nhm.
nh ngha 2.2 (Hm c tnh nhm). Gi h : 2[n] {} R+ l mt hm tp hp. Hm ny
c gi l hm c tnh nhm2 nu tn ti mt nhm hu hn G, v n nhm con G1 , . . . , Gn ,
sao cho
|G|
h(S) = log2
, S [n], S 6= .
|GS |
T
Trong , GS = iS Gi l mt nhm con ca G. Gi n l tp tt c cc hm c tnh nhm
va nh ngha.
2

Group-characterizable function

10

Tp ch Epsilon, S 10, 08/2016

nh ngha 2.3 (Bt ng thc nhm). Nu bt ng thc cT h 0 ng vi mi h n , th


n c gi l mt bt ng thc nhm. C th hn, mt bt ng thc nhm l bt ng thc
c dng
X
|G|
0,
(3)
c(S) log2
|GS |
6=S[n]

sao
T cho n ng vi mi nhm hu hn G v n nhm con G1 , . . . , Gn . (Nh rng GS =
iS Gi .)

Ta gp nhiu bt ng thc thng tin [5], nhng cha gp bt ng thc nhm no. i
hi (3) ng vi mi nhm hu hn v n nhm con c v rt mnh. C tn ti bt ng thc
nhm no hay khng? Trc ht ta xt mt v d n gin:
V d 2.4. Gi G l mt nhm hu hn bt k, v G1 , G2 l hai nhm con. Ta c
log2

|G|
|G|
|G|
+ log2
log2
0.
|G1 |
|G2 |
|G1 G2 |

chng minh bt ng thc ny, ta vit n dng khc: |G| |G1 G2 | |G1 | |G2 |. nh
ngha G1 G2 = {a b | a G1 , b G2 }3 , th ta c th chng minh
|G1 | |G2 | = |G1 G2 | = |G1 G2 | |G1 G2 | |G| |G1 G2 |.
Bi tp 2.5. Chng minh rng |G1 G2 | = |G1 G2 | |G1 G2 |, vi mi nhm con G1 , G2
ca mt nhm G hu hn.
Kt qu rt p ca Chan v Yeung [1] l nh l sau y:
nh l 2.6. Bt ng thc cT h 0 l bt ng thc thng tin nu v ch nu n l bt ng
thc nhm.
Chng minh. Trc khi chng minh, ta tho lun mt cht v kt qu ny. Thot nhn, n hi
c v la o v n ch chuyn t mt cu hi kh v l thuyt thng tin sang mt cu hi kh
trong l thuyt nhm. N khng cho chng ta thng tin g v phng php xc minh xem mt
bt ng thc c phi l bt ng thc thng tin hay bt ng thc nhm hay khng. Mt khc,
lin h ny li rt th v. chng minh mt bt ng thc nhm mi, ta ch cn chng minh
bt ng thc thng tin mi m khng cn bit g v l thuyt nhm. Quyn sch ca Yeung [6]
c mt vi v d bt ng thc nhm mi chng minh c bng l thuyt thng tin m trc
cc nh i s cha bit. Ngc li, nh nh l ny m cc nh l thuyt thng tin c th
cu cu cc nh i s, nh h chng minh h bt ng thc cho mnh.
l bao ng ca tp , v conv(n ) l bao ng li ca
By gi ta chng minh nh l. Gi
n
n
tp n . Ta quan st rng



n
Bt ng thc cT h 0 l bt ng thc thng tin nu v ch nu n h R2 1 | cT h 0 .
n
Do tp {h R2 1 | cT h 0} l tp ng v li, iu ny tng ng vi


h R2n 1 | cT h 0 .
(4)

n
G1 G2 khng nht thit l nhm con ca G.

11

Tp ch Epsilon, S 10, 08/2016

Tng t nh vy, bt ng thc cT h 0 l bt ng thc nhm nu v ch nu




n
conv(n ) h R2 1 | cT h 0 .

(5)

= conv(n ) bng hai bc.


chng minh rng (4) tng ng vi (5), ta chng minh
n
n v
l mt hnh nn li [6]. B
B 2.7 chng minh rng n n ; do conv(n )
n

2.8 chng minh chiu ngc li n conv(n ).


B 2.7. Ta c n n , ngha l mi hm c tnh nhm u l hm entropy
Chng minh. Gi h n l mt hm c tnh nhm, v G l mt nhm hu hn vi cc nhm
|G|
vi mi 6= S [n]. Xt khng gian xc sut = G
con G1 , . . . , Gn sao cho h(S) = log2 G
S
1
vi phn b u p(g) = |G| vi mi g G. Vi mi i [n], nh ngha bin ngu nhin
Xi : 2G nh sau Xi (g) = gGi l lp tri4 ca nhm Gi . Vi mt tp S [n] v b
(gi )iS bt k, d thy




Prob Xi = gi Gi , i S = Prob gGi = gi Gi , i S
g
g


= Prob g gi Gi , i S
g
T
| iS gi Gi |
.
=
|G|
T
T
Nu iS gi Gi 6= , ly mt phn t a iS gi Gi tu , th ta c
\
\
\
Gi = aGS .
aGi = a
gi Gi =
iS

iS

iS

T
Vy th iS gi Gi hoc l tp rng hoc c kch thc bng ng |GS |. Hn na, c tng cng
|G|/|GS | tp khng rng nh th. Do ,
(
T
T
|GS |
| iS gi Gi |
nu iS gi Gi 6=
|G|
=
T
|G|
0
nu iS gi Gi = .
T , d thy H[XS ] = log2 (|G|/|GS |) v h n .
n conv(n ), vi mi n 1.
B 2.8. Ta c
Chng minh. Ta theo trnh by ca Lun [3]. Gi h n l mt hm entropy tu , ngha l c
n bin ngu nhin X1 , . . . , Xn sao cho H(S) = h(S). n gin, ta gi s l min i ca bin
Xi l min hu hn, v c th hn l cc xc sut u l s hu t. (Trong trng hp tng qut,
ta dng mt chui s hu t tin n s v t.) Ta chng minh rng tn ti mt chui f (r) n
sao cho limr f (r) /r = h.


Gi q l mu s chung ca cc xc sut Prob X[n] = x[n] . Chn r = q, 2q, 3q, . . . l mt bi s
ca q, v A l mt ma trn nr sao cho mi ct x[n] ca A xut hin ng r Prob X[n] = x[n]
ln.
4

Left coset

12

Tp ch Epsilon, S 10, 08/2016

Vi =
6 S [n], gi AS l ma trn con ca A xy dng bng cch ly cc hng
 s i ca A vi
i S. D thy rng, mt ct xS xut hin trong AS ng r Prob XS = xS ln.
Gi G = Sr l nhm hon v ca cc ct ca A. Gi Gi l nhm cc hon v cc ct ca A
sao cho hng th i ca A khng thay i. Vy th GS l nhm cc hon v lm cho ma trn AS
khng i. D thy rng
Y


|GS | =
(r Prob XS = xS )!
xS

Dng xp x Stirling, ta c

iS

|G|
1
log2
r r
|GS |
r!
1


= lim log2 Q
r r
xS (r Prob XS = xS )!
lim

1
= lim
r r
=
=

lim

lim

X
xS

r log2 r

log2 r

X
xS

X
xS

X
xS





r Prob XS = xS log2 r Prob XS = xS + O(log2 r)





Prob XS = xS log2 r Prob XS = xS

!




Prob XS = xS log2 Prob XS = xS





Prob XS = xS log2 Prob XS = xS

= H[XS ]
= h(S).

nh ngha f (r) = log2

|G|
|GS |

th ta c limr f (r) /r = h.

Ti liu
[1] CHAN, T. H., AND YEUNG, R. W. On a relation between information inequalities and group
theory. IEEE Trans. Information Theory 48, 7 (2002), 19921995.
[2] DOUGHERTY, R., FREILING, C. F., AND ZEGER, K. Non-shannon information inequalities
in four random variables. CoRR abs/1104.3602 (2011).
[3] LUN, D. S. A relationship between information inequalities and group theory, 2002.
[4] MATUS, F. Infinitely many information inequalities. In 2007 IEEE International Symposium on Information Theory (June 2007), pp. 4144.
13

Tp ch Epsilon, S 10, 08/2016

[5] NG QUANG HNG. Bt ng thc kiu Shannon v vi ng dng. Epsilon, 7 (Feb 2016).
[6] YEUNG, R. W. A first course in information theory. Information Technology: Transmission,
Processing and Storage. Kluwer Academic/Plenum Publishers, New York, 2002. With a
foreword by Toby Berger, With 1 CD-ROM.
[7] ZHANG, Z., AND YEUNG, R. W. On characterization of entropy function via information
inequalities. IEEE Trans. Inform. Theory 44, 4 (1998), 14401452.

14

Tp ch Epsilon, S 10, 08/2016

BT NG THC TAM GIC, A GIC V A DIN


L T Quc Thng
(School of Mathematics, Georgia Institute of Technology, Atlanta)

1. Bt ng thc tam gic, a gic, v a din


1.1. Khng gian chun d chiu
K hiu R l tp hp s thc, v R+ l tp hp cc s thc dng. Xt khng gian chun d chiu
Rd vi tch v hng
hx, yi =

d
X

x i yi ,

cho x = (x1 , . . . , xd ), y = (y1 , . . . , yd ).

i=1

Khi d = 2 y l mt phng, v d = 3 l khng gian 3 chiu. Mt im x Rd i khi c


gi l mt vector.
nh ngha chun ca mt vector
kxk =

p
hx, xi,

v khong cch gia 2 vector x, y, hoc cn gi l chiu di on [x, y], l kx yk. Mt vector
c gi l vector n v nu n c chun bng 1.
Vi khi nim di ny, ta c th nh ngha din tch ca mt a gic trong R3 cng nh th
tch ca mt a din trong R3 .
Hai vector x, y l song song nu tn ti mt s thc k sao cho x = ky hoc y = kx.
Nu U Rd , a Rd , v k R, ta dnh ngha
a + U = {a + x | x U }
kU = {kx | x U }.

1.2. Bt ng thc tam gic


Nh thng thng, ta ng nht chiu di mt cnh ca a gic vi chnh cnh y.
Mnh 1.1. Trong mt tam gic, mi cnh nh hn tng ca hai cnh cn li.
15

Tp ch Epsilon, S 10, 08/2016

Dng k hiu vector, bt ng thc tam gic thng c pht biu di dng: Nu x, y Rd
khng song song vi nhau, ta c
kx + yk < |xk + yk.
Bt ng thc tam gic c th chng minh kh d dng, trc tip t nh ngha khong cch,
bng cch s dng bt ng thc Cauchy v gi tr trung bnh. Bt ng thc tam gic l mt
bt ng thc rt cn bn trong ton hc. N l nn tng ca nhiu ngnh ton hc. N th hin
nguyn l ng thng l ng cc tiu.
Ta cng c phn o ca mnh v bt ng thc tam gic nh sau, m cc hc sinh cp 2 u
bit qua phng php dng hnh.
Mnh 1.2. Nu 3 s dng tha mn tnh cht mi s nh hn tng ca 2 s cn li th
chng l cnh ca mt tam gic duy nht, ty theo cc ng c (isometry) ca khng gian.

1.3. Bt ng thc a gic


Bt ng thc tam gic c th d dng tng qut ho cho a gic.
Mnh 1.3. (a) Trong mt a gic, mi cnh nh hn tng ca cc cnh cn li.
(b) Ngc li, nu n 3 s dng tho mn iu kin mi s nh hn tng cc s cn li th
chng l cnh ca mt a gic li no .
Bi tp 1.1. (a) Chng minh mnh trn.
(b) Chng minh rng nu n > 3 th ta khng c tnh duy nht trong phn (b) ca mnh trn.

1.4. Bt ng thc a din


Thay v xt tam gic, a gic l cc vt th 2 chiu, ta hy xt a din l cc vt th 3 chiu.
nh l 1.4 (Bt ng thc a din). (a) Trong mt a din, din tch ca mt mt nh hn tng
din tch cc mt cn li.
(b) Ngc li, nu n 4 s dng tho mn iu kin mi s nh hn tng cc s cn li th
chng l din tch ca cc mt ca mt a din li no .
Bi tp 1.2. Chng minh phn (a) ca nh l trn. (Gi : Gi s F l mt mt ca a gic.
Hay chiu trc giao cc mt khc xung mt phng cha F .)
Phn (b) ca nh l trn kh hn phn (a) nhiu. mc sau chng ta s a ra mt chng minh
n gin da vo nh l Minkowski v a din, mt nh l rt hay trong hnh hc khng gian
nhng t c bit n.
n y c gi c th on rng nh l trn c th tng qut ho cho khng gian nhiu chiu.
16

Tp ch Epsilon, S 10, 08/2016

2. nh l Minkowksi
2.1. Trng hp 2 chiu
Gi s P l mt a gic. Nu F l mt cnh ca P , k hiu A(F ) l di ca F , v nh ngha
vector php tuyn ca F , k hiu u(F ), l vector n v vung gc vi F va hng ra ngoi a
gic P .
nh l 2.1 (nh l Minkowski 2 chiu). (a) Gi s P l mt a gic li vi cc cnh
F1 , . . . , Fn . Khi cc vector u(F1 ), . . . , u(Fn ) khng cng nm trn mt ng thng, v
n
X

A(Fi )u(Fi ) = 0 .

i=1

(b) Ngc li: Gi s cc vector n v khc nhau u1 , . . . , un trong R2 v cc s dng a1 , . . . an


tha mn iu kin
u1 , . . . , un khng cng nm trn mt ng thng,
Pn

i=1 ai ui = 0 .

Khi tn ti duy nht mt a gic li P vi cc cnh F1 , . . . , Fn sao cho ai = A(Fi ), ui =


u(Fi ) vi mi i = 1, . . . , n.
nh l ny khng kh chng minh lm, v ta s chng minh n trong mc 3.

2.2. Trng hp 3 chiu


nh ly Minkowski trong trng hp nhiu chiu hon ton tng t. Gi s P l mt a din
li. Nu F l mt mt ca P , k hiu A(F ) l din tch ca mt F , v nh ngha vector php
tuyn ca F , k hiu u(F ), l vector n v vung gc vi F v hng ra ngoi.
nh l 2.2 (nh l Minkowski 3 chiu). (a) Gi s P l mt a din li vi cc mt F1 , . . . , Fn .
Khi cc vector u(F1 ), . . . , u(Fn ) khng cng nm trn mt mt phng, v
n
X

A(Fi )u(Fi ) = 0 .

(1)

i=1

(b) Ngc li: Gi s cc vector n v khc nhau u1 , . . . , un trong R3 v cc s dng a1 , . . . an


tho mn iu kin
cc vector u1 , . . . , un khng cng nm trn mt mt phng, v
17

Tp ch Epsilon, S 10, 08/2016

Pn

i=1

a i ui = 0 .

Khi tn ti duy nht mt a gic li P vi cc mt F1 , . . . , Fn sao cho ai = A(Fi ), ui =


u(Fi ) vi mi i = 1, . . . , n.
Ch 2.3. nh l Minkowksi ng trong khng gian nhiu chiu.
Phn (a) ca nh l trn khng kh lm, v s c chng minh trong mc 3. Phn (b) kh hn
nhiu v l phn th v nht ca nh l. Tnh duy nht ca phn (b) lm nh l l mt kt qu
rt hay. Nu cc vector u1 , . . . , un v cc s dng a1 , . . . an tho mn iu kin ca phn (b)
trong nh l, ta khng d xc nh s cnh ca cc mt ca a gic P ! nh l Minkowksi c
nhiu ng dng trong ton hin i.
Chng ta s tho lun cc chng minh ca nh l Minkowski trong cc mc sau. Trc ht
chng ta s chng minh phn (b) ca nh L 1.4 (nh ly bt ng thc a din) bng cch s
dng nh l Minkowski.

2.3. Chng minh nh l 1.4 (bt ng thc a din)


Chng minh. (a) Chng minh phn (a) c a ra trong bi tp 1.2. Phn (a) cng d dng suy
ra t nh l Minkowski nh sau. T (1), ta c
!
n
X
A(F1 )u(F1 ) =
A(Fi )u(Fi ) .
i=2

Cc vector u(F2 ), . . . , u(Fn ) khng cng nm trn mt ng thng (v nu khng th tt c


u(F1 ), . . . , u(Fn ) s nm trn mt mt phng). T bt ng thc tam gic ta c
A(F1 ) <

n
X

A(Fi ).

i=2

L lun tng t, ta thy mi mt A(Fi ) nh hn tng ca cc s cn li.


(b) Gi s a1 , . . . , an l cc s dng sao cho mi s nh hn tng cc s cn li. Theo nh
l 1.3, tn ti mt a gic li Q vi cc cnh c di a1 , . . . , an . Gi s cc nh ca Q l

3
3
q1 , . . . , qn theo chiu kim ng h. t xi =
q
i qi+1 (vi n + 1 = 1). Gi s : R R l
o
0
php quay quanh ng thng i qua q1 q3 vi gc quay 90 . t x1 = (x1 ) v x2 = (x1 ). Ta
c x01 + x02 = x1 + x2 , v vy nu t x0i = xi voi i > 2, ta c
n
X

x0i = 0 .

i=1

Cc vector x01 , . . . , x0n khng cng nm trn mt mt phng. Theo nh l Minkowski, tn ti


mt a din li P sao cho |x0i | = ai l din tch ca cc mt ca a din li ny.

18

Tp ch Epsilon, S 10, 08/2016

Bi tp 2.1. (a) Hy tm hiu iu kin n 4 c s dng nh th no trong chng minh trn.


(b) Hy chng minh rng nu n 4 s thc dng tho mn mi mt s nh hn tng cc s
cn li th tn ti v hn a gic li m din tch cc mt l cc s cho.

3. Chng minh nh l Minkowski, phn I


Mc d l mt nh l vi pht biu s cp, cc chng minh c bit n ca phn (b) nh l
Minkowski 3 chiu u dng n cng c ton cao cp. nh l Minkowski 2 chiu v phn (a)
ca nh l Minkowski 3 chiu c th d dng chng minh bng phng s cp, v chng ta s
tho lun cc chng minh trong mc ny.

3.1. nh l Minkowski 2 chiu, phn (a)


Chng minh 1. Gi s cc nh ca P theo chiu kim ng h l p1 , . . . , pn . Ta c th gi s Fi

l cnh pi pi+1 . t vi =
p
i pi+1 . Ta c
n
X

vi = 0 .

i=1

t l php quay 90o ngc chiu kim ng h. Ta c (vi ) = A(Fi )ui . V vy nu cc vector
u(Fi ) nm trn mt ng thng th a gic P s nm trn mt ng thng l iu khng th
xy ra. Ta c
!
n
n
X
X

A(Fi )ui =
vi = 0 .
i=1

i=1

Chng minh 2. Mc d chng minh ny di hn, nhng n s d dng tng qut ho cho trng
hp nhiu chiu. tng chnh l nu chiu P ln mt ng thng, th nh ca n c ph 2
ln bi cc im trn chu vi ca P , mt ln t hng trn xung v mt ln t hng di ln.
Gi s v l mt vector n v bt k. K hiu v ng thng vung gc vi v i qua gc
to , v prv l php chiu trc giao ln v . Khi X = prv (P ) l mt on thng. t
= X \ pr(V ), vi V l tp hp cc nh ca P . D dng thy chiu di ca prv (Fi ) c tnh
X
bi
kprv (Fi )k = A(Fi ) |hu(F ), ui|.
(2)
t
F+ =

F =

Fi ,

hu(Fi ),vi>0

Fi .

hu(Fi ),vi<0

tn ti duy nht x+ F+ v duy nht x F sao cho prv (x+ ) =


Mnh . Vi mi x X,
x = prv (x ).
19

Tp ch Epsilon, S 10, 08/2016

Theo mnh trn, ta c


X

hu(Fi ),vi>0

kprv (Fi )k = kprv (P )k =

T (2) v (3), ta c
X
h
A(Fi )u(Fi ), vi =
i

hu(Fi ),vi<0

A(Fi )hu(Fi ), vi +

hu(Fi ),vi>0

(3)

A(Fi )hu(Fi ), vi

hu(Fi ),vi<0

= kprv (P )k kprv (P )k = 0.

V v l vector n v bt k, ta c th kt lun

kprv (Fi )k.

A(Fi )u(Fi ) = 0 .

Bi tp 3.1. Chng minh mnh c s dng trong chng minh trn. (Gi : Gi s ng

thng (prv )1 (x) ct P theo on thng x x+ , vi vector


x
x+ cng phng vi v. Gi s x

Fi . Vector t x hng n x+ l mt vector hng vo trong a gic P . V vy h


x
x+ , ui i < 0.)
Bi tp 3.2. Gi s P la a gic li, vi cc nh p1 , . . . , vn theo chiu kim ng h v cnh
Fi = pi pi+1 . Chng minh rng trn ng trn n v, theo chiu kim ng h bt u t u1 , ta
s ln lt gp u2 . . . , un .

3.2. nh l Minkowski 2 chiu phn (b)


Chng minh. nh s li cc vector u1 , . . . , un sao cho nu i trn ng trn n v theo chiu
kim ng h bt u t u1 , ta s ln lt gp u2 . . . , un . Gi s vi l nh ca ui di php quay
90o cng chiu kim ng h.

Chn im p1 bt k. Ln lt dng cc im p2 , p3 , . . . , pn sao cho


p
i pi+1 = ai vi voi i =
Pn

p
1, . . . , n 1. V i=1 ai vi = 0 , ta cng c an vn =
n p1 . a gic P vi cc nh p1 , . . . , pn l
gic tho mn cc iu kt lun ca phn (b).
Bi tp 3.3. Chng minh tnh duy nht ca phn (b) nh l Minkowski 2 chiu.

Ch 3.1. Ta c th thy l chng minh ny khng th m rng ln cho trng hp 3 chiu.


Khc vi trng hp 2 chiu, trong trng hp 3 chiu, bn cht ca mt mt v bn cht ca
vector php tuyn ca n hon ton khc nhau.

3.3. nh l Minkowski 3 chiu, phn (a)


Phn (a) kh n gin.

20

Tp ch Epsilon, S 10, 08/2016

Chng minh. Gi s v l mt vector n v bt k. K hiu v mt phng vung gc vi v i


qua gc to , v prv l php chiu trc giao ln v . Khi X = prv (P ) l mt a gic li.
= X \ pr(E), vi E l hp ca cc cnh ca P . D dng thy rng din tch ca prv (Fi )
t X
c tnh bi
A(prv (Fi )) = A(Fi ) |hu(F ), ui|.
L lun tng t nh trng hp 2 chiu, ta c
X
A(prv (Fi )) = A(prv (P )) =
hu(Fi ),vi>0

V t ,
X
h
A(Fi )u(Fi ), vi =
i

A(Fi )hu(Fi ), vi +

hu(Fi ),vi>0

A(Fi )hu(Fi ), vi

hu(Fi ),vi<0

= A(prv (P )) A(prv (P )) = 0.

V v l vector n v bt k, ta c th kt lun

A(prv (Fi )).

hu(Fi ),vi<0

A(Fi )u(Fi ) = 0 .

Phn (a) cng c mt chng minh vt l" nh sau (t blog ca GS m Thanh Sn). Mc d
khng c cht ch v mt ton hc, nhng chng minh ny cng ch ra mt s tng th v.
Ta hy nhng a din P vo mt cht lng ng nht. p lc ca cht lng ln mi mt Fi bng
kA(Fi )u(Fi ), vi k l mt hng s khc 0 khng ph thuc i. V P s ng bt ng trong cht
lng (iu ny khng gii thch ton hc c), tng tt cc lc p ln n phi bng ~0. V vy
P

i A(Fi )u(Fi ) = 0 .

4. Chng minh nh l Minkowski, phn II


Phn (b) l phn hay nht trong nh l Minkowski. Cc chng minh phn (b) ca nh l
Minkowski vi s chiu 3 u s dng ton cao cp. C l chnh v vy m mc d c pht
biu di dng s cp, nh l Minkowski t c bit n trong ton s cp. y ta ch chng
minh nh l Minkowski cho trng hp n = 4, trng hp ny ch cn s dng kin thc ca
ton ph thng. Vi mt t kin thc v gii tch nhiu bin, bn c th hiu c chng minh
nh l Minskowski tng qut, xem mc 4.4.

4.1. Tp hp cc a gic c vector php tuyn cho trc


Gi s u1 , u2 , u3 , u4 R3 v cc s dng a1 , a2 , a3 , a4 tho mn cc iu kin ca phn (b)
nh l Minkowski 3 chiu. T gi thit, ta c th thy rng
u2 , u3 , u4 khng cng trn mt mt phng.

(4)

Bi tp 4.1. Hy chng minh rng u2 , u3 , u4 l c lp tuyn tnh, tc l nu cc s thc


k2 , k3 , k4 tha k2 u2 + k3 u3 + k3 u4 = ~0 th k1 = k2 = k3 = 0.
21

Tp ch Epsilon, S 10, 08/2016

Gi s P l mt din thoa iu kin


cc vector php tuyn mt ca P l u1 , . . . , u4 .

(5)

Khi tn ti cc s thc z1 , z2 , z3 , z4 sao cho


P = {x R3 | hx, ui i zi i = 1, 2, 3, 4}.

(6)

Cc s z1 , . . . , z4 xc nh hon ton a din P . Tuy nhin khng phi bt c 4 s thc z1 , . . . , z4


cng xc nh mt a din theo (6), v tp hp nh ngha bi v phi ca (6) c th khng phi
l a din, thm ch c th rng.
Bi tp 4.2. (a)Gi s P c xc nh bi (z1 , . . . , z4 ) theo (6), v v R3 . Chng minh rng
v + P c xc nh bi z10 , . . . , z40 , nh ngha bi zi0 = zi + hv, ui i.
(b) Chng minh rng (z1 , . . . , z4 ) xc nh mt a gic P nu v ch nu tn ti x R3 sao cho
hx, ui i < zi

i = 1, 2, 3, 4.

(c) Chng minh rng z1 = z2 = z3 = z4 = 1 xc nh mt a din no tho mn (5).


V ta s ng nht P vi P + v, ta s chn v sao cho (z1 , . . . , z4 ) l n gin, nh sau. T tnh
cht (4) d dng suy ra rng cc mt phng qua F2 , F3 , F4 ct nhau ti mt im duy nht.
y Fi l mt ca P c vector php tuyn ui . Sau mt php tnh tin, ta c th gi s
cc mt phng qua F2 , F3 , F4 ct nhau ti gc ta ~0.

(7)

Gi s P 0 l tp hp tt c cc a gic tho mn (5) v (7). Vi P P 0 , v cc s z1 , z2 , z3 , z4


ca (6), ta c z2 = z3 = z4 = 0. V vy P c xc nh duy nht bi z1 = z1 (P ). Ta c nh x
z1 : P 0 R. Gi P = z1 (P 0 ) l nh ca P.
Bi tp 4.3. Chng minh P = R+ = {x R | x > 0}, v z1 : P 0 P l song nh.

Nh vy tp hp tt c cc a gic tho mn (5) v (7) c th ng nht vi tp hp P = R+ .


Vi z P = R+ , ta k hiu P (z) P 0 l a gic tho mn z1 = z, z2 = z3 = z4 = 0. Khi
P : P P 0 l song nh.

4.2. Tp hp cc din tch c th c


Gi s Q0 l tp hp tt c (y1 , y2 , y3 , y4 ) (R+ )4 sao cho
y1 u1 + y2 u2 + y3 u3 + y4 u4 = ~0.
Mt vector trong R3 c 3 thnh phn, v vy ng thc trn cho ra 3 phng trinh tuyn tnh vi
4 n s y1 , y2 , y3 , y4 . Ni chung tp hp li gii s l khng gian 1 chiu. Ngoi ra ta cn phi
gii hn yi > 0.
22

Tp ch Epsilon, S 10, 08/2016

t : R4 R l php chiu ln to th nht, tc l


(y1 , . . . , y4 ) = y1 .
t Q = (Q0 ), v k hiu : Q0 Q l gii hn ca trn tp Q0 .
Bi tp 4.4. (a) Chng minh rng Q0 tho mn: nu x, y Q0 v k R+ th x + y Q0 va
kx Q0 .
(b) S dng iu kin (4), chng minh rng l song nh.
(b) Chng minh rng Q = R+ .

4.3. nh x din tch mt


Gi s z P. t Ai (z) l din tch mt Fi ca a gic P (z), v A : Q R4 l nh x
A(z) = (A1 (z), . . . , A4 (z)).
Phn (a) ca nh l Minkowski chng minh rng A(P) Q0 . Phn (b) ca nh l Minkowski
3 chiu tng ng vi mnh sau y m ta s chng minh.
Lemma 4.1. nh x A : P Q0 l song nh.
Chng minh. D dng thy rng P (kz) = kP (z) voi moi k R+ . T suy ra rng
A(kz) = k 2 A(z).
A

(8)

t B : P Q l composition R+ = P Q0 Q = R+ . ng thc (8) chng t rng


B(kz) = k 2 B(z) z R+ & k R+ .
D dng thy rng bt k nh x B : R+ R+ no tho mn iu kin trn l mt song nh.
V l song nh, ta suy ra A cng l song nh.
Ta kt thc chng minh nh l Minkowski 3 chiu cho trng hp n = 4. Trng hp n > 4
c chng minh tng t, mc d phc tp hn, v s dng bt ng thc Brunn-Minkowski.
c gi c th tham kho [1, 2].

23

Tp ch Epsilon, S 10, 08/2016

4.4. S lc v trng hp n > 4


Bng cch nh s li, ta c th gi s
un2 , un1 , un khng cng trn mt mt phng.

(9)

Mt a gic P tha iu kin


cc vector php tuyn mt ca P l u1 , . . . , un

(10)

s c hon ton xc nh bi cc s thc z1 , . . . , zn sao cho


P = {x R3 | hx, ui i zi i = 1, . . . , n}.

(11)

Tuy nhin khng phi bt c n s thc z1 , . . . , zn cng xc nh mt a din theo (11).


Bi tp 4.5. (b) Chng minh rng (z1 , . . . , zn ) xc nh mt a gic P nu v ch nu tn ti
x R3 sao cho
hx, ui i < zi i = 1, . . . , n.
(c) Chng minh rng z1 = = zn = 1 xc nh mt a din no tha mn (10).

Tnh cht (9) suy ra rng cc mt phng qua Fn2 , Fn1 , Fn ct ti mt im duy nht. Sau mt
php tnh tin, ta c th gi s
cc mt phng qua Fn2 , Fn1 , Fn ct nhau ti gc ta ~0.

(12)

Gi s P 0 l tp hp tt c cc a gic tha mn (10) v (12). Voi P P 0 , ta c zn2 = zn1 =


zn = 0, v vy P c xc nh duy nht bi z1 , . . . , zm . O day m = n 3. t P Rm l tp
hp tt c z = (z1 , . . . , zm ) sao cho (z, 0, 0, 0) xc sinh mt a gic P P 0 . nh x P P 0 ,
z P (z) l song nh.
Bi tp 4.6. (a) Chng minh P l mt cone li, tc l nu z, z0 P v k R+ th kz P v
z + z0 P.
(b) Chng minh P l mt tp hp m, tc l nu z P th tn ti > 0 sao cho nu kz0 zk <
th z0 P.
P
Gi s Q0 l tp hp tt c (y1 , . . . , yn ) (R+ )n sao cho ni=1 yi ui = ~0. t : Rn Rm l
php chiu ln m ta u tin. t Q = (Q0 ), v k hiu : Q0 Q l gii hn ca
trn tp Q0 .
Bi tp 4.7. (a) Chng minh rng Q0 l mot cone loi.
(b) S dng iu kin (4), chng minh rng la song nh.
(b) Chng minh rang Q l mt cone li m trong Rm .
24

Tp ch Epsilon, S 10, 08/2016

Vi z P dat Ai (z) la din tch mt Fi ca a gic P (z), v A : Q Rn l nh x


A(z) = (A1 (z), . . . , An (z)).
t B = A. Phn (a) ca nh l Minkowski chng minh rng A(P) Q. Phn (b) ca
nh l Minkowski 3 chiu tng ng vi mnh sau
Lemma 4.2. nh x B : P Q l song nh.
Nh vy ta phi chng minh vi mi a Q tn ti duy nht z P sao cho B(z) = a. Bi
ton tm z s uc a v bi ton tm cc tr ca mt hm s li v trn, v li gii duy nht
ca n c khng nh bi nh li v trn ca hm s. em chi tit ti [2]. Alexandrov c mt
chng minh th v khc, bng cch trc ht chng minh tnh duy nht, ri dng tnh cht tp
ca min xc nh v min gi tr ca B chng minh tnh tn ti. Xem chi tit ti [1].

Ti liu
[1] A. D. Alexandrov, Convex polyhedra (translation of the 1950 Russian original),
Springer, Berlin, 2005.
[2] I. Pak, Lectures on Discrete and Polyhedral Geometry, online at
http://www.math.ucla.edu/~pak/geompol8.pdf

25

Tp ch Epsilon, S 10, 08/2016

TON HC V NGH THUT TUNG HNG


Nguyn Hng Sn

GII THIU
Tung hng l mt b mn ngh thut c xa, dng nh lun ng hnh vi lch s ca
nhn loi. Nhiu bn v m t nhng ngi ph n ang tung hng c tm thy trong
mt ngi m ca Ai Cp c nin i vo khong th k th hai mi trc cng nguyn.
Ch cn mt vi vin v mt cht luyn tp l bn c th tung hng. V vy khng
c g ng ngc nhin khi loi ngi quan tm n mn ngh thut ny t rt lu ri.

Tuy nhin, ch mi gn y kha cnh ton hc ca tung hng mi bt u c quan tm mt


cch nghim tc. Cc nghin cu ton hc v cc m hnh tung hng c tin hnh ln u
tin cng mt lc vo nhng nm 80 ca th k XX ti vi trng i hc, trong c i hc
California ti Santa Cruz, Caltech v i hc Cambridge. Trong bi bo nh ny chng ta s
lit k mt cch ngn gn v cc kt qu m cc nh ton hc c th gip cc ngh s xic trong
vic to ra cc m hnh tung hng v cc li ch do s hp tc lin ngnh ny c th em li cho
cc nh ton hc.

26

Tp ch Epsilon, S 10, 08/2016

Hnh 1: Nhng hnh nh c nht v tung hng cch y khong 4000 nm.
Bc u tin l to ra cc m hnh ton hc cho cc k thut tung hng c th ni v chng
mt cch chnh xc. Trong cc m hnh n gin nht, ta gi s thi gian l ri rc (chnh xc
hn, thi gian l mt chui cc thi im 1, 2, 3, . . .), v rng ngh s tung hng c hai tay, mi
tay ch c th gi c nhiu nht mt vt trong mi thi im. Khng mt tnh tng qut ta gi
s cc vt dng tung hng l cc qu bng. Cc tay thay i nhau lin tc, c ngha l mt
tay s lun bt (hng) v tung bng cc thi im l: 1, 3, 5, ... (ta gi l tay l), cn tay th
hai (tay chn) th lun tung v hng cc thi im chn: 2, 4, 6, ....
V by gi s l vn th v hn: lm th no m hnh ha cc cch tung bng khc nhau
v ng thi cho nhiu qu bng khc nhau? Phng php thng dng nht l phn loi cc
cch tung bng theo thi gian (s khonh khc) m qu bng bay trn khng. iu c ngha
rng nu ti thi im i ta tung bng theo kiu t ( vi t l mt s t nhin no ) th bng s
ri (vo mt trong hai tay) vo thi im i + t. Lu rng vi cch k hiu ny, nu bng c
tung theo kiu chn bng mt trong hai tay th n s ri vo ng tay , cn theo kiu l th
bng s ri vo tay th hai. Ta cng k hiu kiu 0 cho trng hp khng tung bng, tc l mt
trong hai tay c gi l tung bng theo kiu 0 ti thi im i nu tay khng c bng ti thi
im ny.
Hnh 2 l v d ca mt dy cc cch tung hng bng theo trnh t thi gian. Trong v d ny
ngi ngh s tung hng dng ba qu bng c nh du bng ba mu. Ngoi ra trn hnh v
cn c mt s thng tin b sung (v cu hnh) v s c gii thch sau.
Hot ng ca ngi ngh s tung hng trong v d ny c th c m t dng mt dy cc
cch tung bng trong tng thi im, l h5, 3, 1, 4, 5, 3, 0, 5, 5, 2, 0, 5, 3, 1, 4, 5, 3, 0i. Cch k
hiu ny c gi l phng php dy hon i (ting Anh l siteswap model). Tuy nhin, trong
thc t cc ngh s tung hng thng quan tm n cc m hnh tung hng (ting Anh gi l
juggling pattern) hn l ci dy di dng dc nh trn. Tt nht l nu ta tm c cc dy tun
hon mt dy con hu hn c th lp i lp li nhiu ln cho n v cng. V d: mt trong
27

Tp ch Epsilon, S 10, 08/2016

Hnh 2: V d mt dy cc cch tung hng bng hai tay


cc m hnh ph bin nht m mi ngi thng bt u hc l tung hng 3 qu bng theo kiu
thc nc. Kiu tung hng ny tng ng vi dy h3, 3, 3, 3, 3, 3, 3...i.

=
trnh trng lp cc thng tin v ch, ta thng k hiu cc dy tun hon bng cc thng tin
trong mt chu k ca n. Nh vy m hnh tung hng kiu thc nc l dy tun hon (3).
Ti y ta c th thy hng lot cc vn ton hc c t ra. Phi chng tt c cc dy hon
i u kh thi (hp thc)? S qu bng nh hng nh th no n m hnh? Tn ti bao nhiu
m hnh tung hng nu ta c nh mt s yu t nh s lng qu bng hoc s ln tung bng?
Trc khi tm hiu cc cu hi trn ta hy lm quen vi mt s tnh cht ca m hnh tung hng
dng dy hon i (siteswap patterns):
28

Tp ch Epsilon, S 10, 08/2016

nh ngha 1 Gi P = ht0 tk1 i l dy hon i vi di k. Dy P c gi l dy tung


hng khi v ch khi hm s
: {0, . . . , k 1} {0, . . . , k 1} xc nh bi (i) = i + ti

mod k

l mt hon v ca tp {0, . . . , k 1}.


nh ngha trn ch mun ni rng trong dy tung hng l dy hon i ca qu trnh tung hng
thc s. Khi tung hng, khng tay no c bt hai qu bng cng mt lc, iu co ngha
rng cc qu bng u phi ri vo mt trong hai tay cc thi im khc nhau. D thy mi
dy c di k = 1 u l dy tung hng v mi dy tun hon dng (n) cng l dy tung hng
tng ng vi tung hng kiu thc nc dng n qu bng.
B 1 (v gi tr trung bnh) Trong mi dy hon i tung hng tun hon a = (a0 , a1 , ..., ad1 )
d1
c di d, gi tr trung bnh ca dy a = a0 +a1 +...+a
l mt s t nhin v bng ng s
d
qu bng c dng tung hng.
Xin b qua phn chng minh b 1 v mong bn c yu thch ton hc coi y l mt bi tp
th v.
B trn y c th coi nh l iu kin cn kim tra dy hon i c phi l dy tung hng
khay khng: nu gi tr trung bnh ca dy khng phi l s nguyn th khng phi l dy
tung hng. V d:
Dy (5, 2, 1) c gi tr trung bnh bng 83 , v vy y khng phi l dy tung hng.
Dy (3, 2, 1) c gi tr trung bnh bng 2 nn c th l dy tung hng cho 2 qu bng. Tuy
nhin (1) = (2) = (3) = 0, v vy theo nh ngha y khng phi l dy tung hng.
Cc dy (4, 4, 1), (5, 3, 1), (4, 4, 1, 3) (5, 5, 5, 0, 0) l cc dy tung hng cho 3 qu bng;
cn (6, 4, 5, 1), (7, 3, 3, 3), (7, 1) l cc dy tung hng cho 4 qu bng. Hy vng sau khi
c xong bi vit ny, bn c c th d dng kim tra cc thng tin trn.

Hnh 3: V d ca dy tung hng tun hon (4, 4, 1, 3).


Benot Guerville pht hin v chng minh kt qu tuyt vi lin quan n iu kin xc
nh dy tung hng nh sau:
29

Tp ch Epsilon, S 10, 08/2016

nh l 2 (v ti c cu) Nu dy s t nhin c gi tr trung bnh cng l s nguyn th ta c


th hon v dy thnh mt dy tung hng.
Quay li v d trn Hnh 2. Ti mi thi im ta hy quan st v tr" ca qu bng, tc l thi
gian t thi im cho n khi qu bng ri vo mt trong hai tay. V khng tay no c bt
hai qu bng cng mt lc nn ti mt thi im tt c cc qu bng phi cc v tr khc nhau.
Ta c th ni rng bng c tung ln v tr k thay v ni rng bng c tung theo kiu k.
Cu hnh ti mt thi im bt k l mt dy s0 s1 s2 . . . gm cc k hiu v . K hiu
sk = nu v tr ca mt trong cc qu bng bng k, cn sk = khi khng qu bng no
v tr ny. Chiu di ca cu hnh thng l s t nhin hu hn tng ng vi v tr cao nht
ca cc qu bng. Trong v d Hnh 2 cu hnh ti mi thi im u c chiu di bng 5 (ct
cui cng).
V tr u tin (k hiu s0 ) ca cu hnh m t tnh trng ca bn tay ch ng, tc l tay l
thi im l hoc tay chn thi im chn. iu c ngha l nu k hiu u tin ca cu
hnh l th bn tay tng ng khng c bng v tip theo tay s tung bng theo kiu 0.
Cn nu k hiu u tin l th qu bng trong tay s phi c tung ln mt trong cc v tr
c k hiu (v tr cn trng). Lu : ta lun c th tung bng ln v tr cao nht (ti sao?).
Nu cu hnh ti thi im i l Ci = s0 s1 s2 . . . sd1 v s0 = th cu hnh tip theo s l
Ci+1 = s1 s2 . . . sd1 . Hai cu hnh c ni vi nhau bng mi tn c trng s bng 0:
0

Ci = s1 s2 . . . sd1 Ci+1 = s1 s2 . . . sd1


Cn nu s0 = th cc cu hnh ti thi im tip theo s c hnh thnh nh sau:
1. Ko di cu hnh Ci thnh Ci0 = s0 s1 s2 . . . sd1 sd vi sd = ;
2. Nu sh = v qu bng c tung ln v tr h th ta s i sh = ;
3. Xa s0 t Ci0 . Cu hnh ti thi im i + 1 s l Ci+1 = s1 s2 . . . sd1 sd
4. Ta ni hai cu hnh bng mi tn c trng s bng h:
h

Ci Ci+1
Bng cch ny ta c th thit lp biu chuyn dch gia cc cu hnh. Hnh 4 l v d minh
ha ca biu hon chuyn gia cc cu hnh khi tung hng 3 qu bng v v tr cao nht l 5.
Biu ny, thc cht l th c hng v c trng s.
S dng cc cu hnh ta c th d dng kim tra xem mt dy hon i c l dy tung hng hay
khng. D dng nhn thy rng mi mt dy tung hng tng ng vi mt dy chuyn trong
biu . Hn na dy tung hng tun hon tng ng vi mt chu trnh trong biu . Cng c
th s dng biu ny to ra cc bi biu din tung hng mi v k l. Nhiu ngh s (v
d cng ty Gandini Juggling) s dng m hnh ny thit k cc chng trnh biu din
ca mnh.
B 1 c th tng qut ha nh sau:
30

Tp ch Epsilon, S 10, 08/2016

Hnh 4: Biu dch chuyn gia cc cu hnh.


nh l 3 (tng qut v gi tr trung bnh) Nu a = ha0 , a1 , a2 , . . .i l mt dy tung hng c
cao hu hn th gii hn
P
ai
iI
lim
|I| |I|

hi t v bng s qu bng, y gii hn c xc nh cho tp hp tt c cc khong


I = {b, b + 1, . . . , c} Z v |I| = c b + 1 l s s t nhin trong khong I.
rng cc kt qu ton hc trong bi bo ny khng s dng n gi thit v hai bn tay.
Cc kt qu trn y vn ng cho trng hp tung hng dng nhiu chi hn, v d: 2 chn,
2 tay, u, nhiu ngi. Nghin cu v cc dy hon chuyn vn l vn thi s v c tng
qut ha cho nhiu trng hp nh:
Thi gian ng b: nhiu chi cng bt bng v tung bng trong cng mt thi im;
Multiplexes: mt tay c th tung nhiu qu bng trong cng mt thi im.
Cc nghin cu c bn v ti ny cng c tin hnh v kh nhiu bi bo c ng,
ch yu l v cc vn t hp lin quan n m hnh tung hng. Mt iu th v l mt s
kt qu lin quan n tung hng li c th s dng trong cc ngnh khc trong ton hc. Mt
s nghin cu, k c mt s lun n tin s (v d nh Combinatorial aspects of juggling ca
Anthony Mays) pht hin rt nhiu s lin h gia cc dy hon i (siteswap) vi mt s
l thuyt tin tin nht trong ton hc nh tnh ton chui Poincare cho nhm affine ca Weyl
A d1 hoc chng minh nh l lin quan n s q-Stirling. Trc , nhiu ngi cho rng
cc l thuyt ny l cc m hnh ton hc tng tng, khng thc dng.

31

Tp ch Epsilon, S 10, 08/2016

Khng c l thuyt v dng trong ton hc: s lin kt gia cc kt qu ton hc v cuc sng
thng xut hin mt cch tnh c trong cc lnh vc m bn thn cc nh ton hc cng khng
ng ti.

Ti liu
[1] Tp ch Delta ca Ba lan. S 9. 2014
[2] Beek, Peter J. & Arthur Lewbel (1995), The Science of Juggling, Scientific American,
Vol. 273, No 5, November 1995, p9297.
[3] Beever, Ben (2002), Siteswap Bens guide to juggling patterns, available at:
www.jugglingdb.com/compendium/geek/notation/siteswap/bensguide.html.
[4] Buhler, Joe, David Eisenbud, Ron Graham & Colin Wright (1994), Juggling drops and descents, The American Mathematical Monthly, Vol. 101, No. 6, JuneJuly 1994, p507519.
[5] Cardinal, Jean, Steve Kremer & Stefan Langerman (2006), Juggling with pattern matching, Theory of Computing Systems, 39(3), June 2006, p425 437.
[6] Carstens, Ed (1992), The mathematics of juggling, online publication, available at:
www.juggling.org/papers/carstens/.
[7] Polster, Burkard (2003), The mathematical of juggling. Springer-Verlag, New York.
[8] Shannon Claude E. (1980), Scientific Aspects of Juggling. In N.J.A. Sloane and A. D.
Wyner (eds) (1993) Claude Elwood Shannon: Collected Papers. IEEE Press.
[9] Anthony Mays (2006), Combinatorial aspects of juggling. Ph.D Thesis at University of
Melbourne.

32

Tp ch Epsilon, S 10, 08/2016

H MT M KHA CNG KHAI DA TRN


NG CONG E LLIPTIC - M T S NG DNG
ng Minh Tun (Vietkey)

TM TT
Epsilon s 9, chng ti gii thiu tng quan v h mt m kha cng khai da
trn ng cong Elliptic qua phn u ca chuyn c cng tn gi ca tc gi ng
Minh Tun. Trong s ny, Epsilon trn trng gii thiu phn tip theo (v cng l phn
kt) ca lot chuyn thng qua cc ng dng ca h mt m kha cng khai da trn
ng cong Eplliptic.

1. Bi ton Logarithm ri rc
nh ngha 1.1. Bi ton Logarithm ri rc trn ng cong Elliptic (ECDLP): Cho ng cong
E trn trng hu hn Fq , im P E(Fq ) vi bc n (nP = O = ) v im Q E(Fq ), tm
s nguyn k [0, n 1] sao cho Q = kP . S nguyn k c gi l Logarithm ri rc ca Q
vi c s P , v thng c vit l k = logP Q.
Bt k mt h mt kha cng khai no cng phi s dng mt bi ton kh xy dng hm
mt chiu. ngha mt chiu y c ngha l tnh thun th d (thut ton gii trong thi gian
a thc) v tnh ngc th kh (thut ton gii vi thi gian khng phi l a thc - thng l
hm m hoc na m). Cc tham s ca H mt da trn ng cong Elliptic (ECC) cn phi
c la chn cn thn trnh c cc tn cng i vi bi ton ECDLP. Thut ton vt cn
gii bi ton ECDLP l ln lt tnh th cc im P, 2P, 3P, ... cho n khi im mi tnh
c ng bng im Q. Trong trng hp xu nht s phi cn n n bc th, trung bnh
thng l n/2 l t c im Q, do cn phi chn n ln bi ton vt cn l khng
kh thi (n 2160 ).
Thut ton tt nht hin nay tn cng bi ton ECDLP l s kt hp ca thut ton Pohlih
Hellman v Pollards rho, thut ton ny c thi gian tnh l O( p), vi p l c s nguyn t

ln nht ca n do phi chn s n sao cho n chia ht s nguyn t p ln nht c p ln


gii bi ton ny l khng kh thi.
Trong phn tip theo, mt s phng php tn cng bi ton Logarithm ri rc s c trnh
by, a s cc phng php ny c th p dng c cho mt nhm bt k. Chi tit c th tham
kho trong [3, 8, 21].
33

Tp ch Epsilon, S 10, 08/2016

Cho G l nhm cc im trn ng cong E. P, Q G l cc im trn ng cong E, chng


ta cn gii bi ton kP = Q, N l bc ca G.

1.1. Phng php bc nh, bc ln


Phng php ny do Shanks xut v c H. Cohen m t trong [22].

1:
2:
3:
4:
5:
6:
7:

Thut ton 1 Phng php bc nh, bc ln

Chn m N v tnh mP .
Tnh v lu tr danh sch iP vi 0 i < m
Tnh Q jmP vi j = 0, 1, . . . , m 1
if iP = Q jmP then
k = i + jm( mod N )
end if
Quay v bc 3

D dng nhn thy Q = iP + jmP hay Q = (i + jm)P t k = i + jm. im iP c tnh


bng cch cng thm P vo (i 1)P v gi tr ny c gi l bc nh. Q jmP c tnh
bng cch cng thm mP vo Q (j 1)mP v gi tr ny c gi l bc ln.

1.2. Phng php Pollards v


Phng php ny do Pollard xut trong [23].
nh ngha hm f : G G mt cch ngu nhin Pi+1 = f (Pi ) vi P0 cng c chn mt
cch ngu nhin. Bi v G l tp hu hn do s c cc ch s i0 < j0 m Pi0 = Pj0 , t ta
c:
Pi0 +1 = f (Pi0 ) = f (Pj0 ) = Pj0 +1
Tng t s c Pi0 +l = Pj0 +l vi l 0, t chui Pi l chui tun hon vi chu k l j0 i0 .
Hm biu din chui Pi thng ging ch ci Hi Lp v l l do ti sao phng php ny
c tn l phng php .
Hm f c chn nh sau: Chia tp G thnh s tp con khng trng nhau S1 , S2 , . . . , Ss c kch
thc tng ng nhau, s thng c chn l 20, chn 2s s ngu nhin ai , bi mod N . t:
Mi = ai P + bi Q
V nh ngha:
f (g) = g + Mi ,

34

g Si

Tp ch Epsilon, S 10, 08/2016

Biu din Pj di dng Pj = uj P + vj Q, khi Pi0 = Pj0 ta c:


uj0 P + vj0 Q = ui0 P + vi0 Q
(ui0 uj0 )P = (vj0 vxj0 )Q
k = (vj0 vxj0 )1 (ui0 uj0 )
Phng php ny cng tng t nh phng php trn cn
tr s nh hn.

(mod N )

N bc, tuy nhin khng gian lu

1.3. Phng php Pohlig-Hellman


Pohlig v Hellman xut phng php ny trong [24].
Nu c th phn tch bc N ca G thnh cc tha s nguyn t th c th vit:
Y
N=
qiei
i

tng ca phng php ny l tm k (mod qiei ) vi mi i, sau p dng nh l ng d


Trung Hoa tnh k (mod N ). Coi q l s nguyn t v q e l ly tha e ca q c chia ht bi
N , vit k di dng sau:
k = k0 + k1 q + k2 q 2 + ,

0 ki < q

L gii thut ton 2 nh sau:


N
N
Q=
(k0 + k1 q + ) P
q
q
N
N
= k0 P + (k1 + k2 q + ) N P = k0 P
q
q
Bi v N P = v t y c th tm c k0 . Tip theo:


Q1 = Q k0 P = k1 q + k2 q 2 + P
N
N
Q1 =
(k1 + k2 q + ) P
2
q
q
N
N
= k1 P + (k2 + k3 q + ) N P = k1 P
q
q

T tm c k1 , tng t nh vy chng ta s tm c k2 , k3 . . . Thut ton s dng khi


e = r + 1, khi N/q e+1 khng cn l s nguyn na v chng ta khng th nhn Qe vi mt
s hu t.

35

Tp ch Epsilon, S 10, 08/2016

1:
2:
3:
4:
5:
6:
7:
8:
9:
10:
11:
12:
13:
14:
15:
16:
17:
18:

Thut ton 2 Phng php Pohlig-Hellman


o
n 
| 0 j q 1.
Tnh T = j Nq P


N
N
Tnh q Q. l phn t k0 q P ca T .
if e = 1 then
Nhy n bc 15.
end if
Q1 Q k0 P


Tnh qN2 Q1 . l phn t k1 Nq P ca T .
if e = 2 then
Nhy n bc 15.
end if
Ln lt tnh c cc gi tr k0 , k1 , . . . , kr1 v Q0 , Q1 , . . . , Qr1
Qr Qr1 kr1 q r1 P


N
Qr = kr Nq P
Xc nh kr sao cho qr+1
if e = r + 1 then
k = k0 + k1 q + k2 q 2 + + ke1 q e1 ( mod q e )
Stop.
end if
Quay v bc 11.

1.4. Phng php tn cng MOV


Tn cng MOV l tn vit tt ca cc tc gi Menezes, Okamoto, v Vanstone [25], s dng cp
Weil chuyn i bi ton Logarithm ri rc trong E(Fq ) thnh bi ton Logarithm ri rc
trong F
q m . Bi v gii bi ton Logarithm ri rc trong trng hu hn s d dng v nhanh hn
gii Logarithm ri rc trong nhm cc im trn ng cong Elliptic. Chn m sao cho:
E[N ] F
qm
Bi v tt c cc im trong E[N ] u c ta trong Fq = j1 Fq j , nn m tn ti. Theo nh
ngha v cp Weil v cc thuc tnh ca cp song tuyn tnh:
2 = eN (Q, T1 ) = eN (kP, T1 ) = eN (P, T1 )k = 1k

1:
2:
3:
4:
5:
6:

Thut ton 3 Tn cng MOV


Chn im ngu nhin T E(Fqm ).
Tnh bc M ca T .
Cho d = gcd(M, N ) v cho T1 = (M/d)T c ngha l T1 c bc l d, chia ht bi N , do
T1 E[N ].
Tnh cc cp Weil 1 = eN (P, T1 ) v 2 = eN (Q, T1 ). C hai 1 , 2 d F
qm .

k
Gii bi ton Logarithm ri rc 2 = 1 trong Fqm , s tnh c k (mod N ).
Lp li vi im ngu nhin T cho n khi bi s chung nh nht ca cc s d l N , t
xc nh c k (mod N ).
36

Tp ch Epsilon, S 10, 08/2016

2. Tham s ca h mt ECC
Cc tham s ca h mt ECC cn c la chn k cng trnh cc tn cng nh MOV, trong
qu trnh la chn h ECC cn phi t c mt s tiu ch c m t trong chun [26].
nh ngha 2.1. Tham s h mt D = (q, F R, S, a, b, P, n, h) l mt tp hp gm:
1. Bc ca trng Fq l q.
2. Phng php biu din trng FR (field representation) c s dng cho cc phn t
ca Fq .
3. S l mm c s dng trong trng ng cong Elliptic c to ra mt cch ngu
nhin.
4. Hai h s a, b Fq c dng nh ngha ng cong E trn Fq (ngha l y 2 =
x3 + ax + b).
5. P l mt im c bc nguyn t n v gi l im c s P = (xP , yP ) E(Fq ).
6. ng h s h = #E(Fq )/n.

3. Trao i kha
Trong cc mc cn li, chuyn s cp n mt s thut ton ng dng trong trao i kha,
m ha v k s c bn. Chun do cng ty Certicom xy dng [26] m t chi tit vic trin khai
ng dng ECC. Tc gi D. Hankerson [18] phn tch vic trin khai ECC bng phn mm, trong
khi tc gi L. Cao [27] phn tch thc hin cc giao thc c bn ca ECC bng phn cng.

3.1. Trao i kha DiffieHellman ECDH


Nm 1998, Laurie v cng s xut giao thc trao i kha da trn ECC [28]. Sau giao
thc ny c a vo cc tiu chun ANSI X9.42, ANSI X9.63 v IEEE P1363.
Hai bn A v B cn to kha phin b mt trao i trong mt knh truyn cng khai, hai bn
cng tha thun im c s P trn E. Bn A to kha b mt dA v gi gi tr dA P cho bn B,
ngc li bn B to kha b mt dB nhn vi P sau gi li cho A. Khi kha phin ca bn
A s l KA = dA dB P , v ca bn B s l KB = dB dA P . D dng nhn thy KA = KB , kha
ny ch ring hai bn A v B c th tnh c. Xem s di y:
Bn A
dA
dB P
KA = dA dB P

Bn B
dA P

dA P

dB
K B = dB dA P

dB P

37

Tp ch Epsilon, S 10, 08/2016

nh gi bo mt: tm c kha chia s KA hoc KB , Hacker buc phi tm c c 2 kha


b mt dA , dB , trong khi ch c th bt c thng tin trn ng truyn l dA P v dB P , khi bit
P , Hacker buc phi gii bi ton Logarithm ri rc dA = logP (dA P ) v dB = logP (dB P ) v
y l bi ton kh khng gii c trong thi gian a thc.

3.2. To kha b mt chia s ECMQV


Tn y ca giao thc l Elliptic Curve Menezes-Qu-Vanstone. Thut ton c a vo
trong cc chun ANSI X9.63, IEEE 1363-2000, v ISO/IEC 15946-3. Theo cc tiu chun ny
im c s c k hiu l G thay v l P nh thng gp. Lc ny thng c s dng
khi cc bn A v B c cp kha cng khai v b mt c nh, tng ng l (a, aG) v (c, cG).
Bn A sinh cp s ngu nhin (b, bG) v bn B tng ng sinh cp s ngu (d, dG), v trao i
2 cp nay cho nhau gi tr bG v dG. K hiu hm x : E N, ly gi tr x ca mt im trn
ng cong E.
Thut ton 4 To kha b mt chia s ECMQV
INPUT: Cc tham s ca h mt (K, E, q, h, G), cc s a, b, aG, bG, cG v dG.
OUTPUT: Kha b mt chia s Q (chia s vi vi i tng c kha cng khai
cG).
1: n dlog2 (#k)e/2.
2: u (x(bG)(mod 2n ) + 2n .
3: s b + ua((mod q).
4: v (x(dG)(mod 2n ) + 2n .
5: Q s(dG + v(cG)).
6: if Q = then
7:
Quay li bc 1.
8: end if
9: Tr v kha Q.
Bn B c th tnh ra cng s Q bng cch thay (a, b, c, d) trong thut ton trn bng (c, d, a, b).
Bn A s c cc gi tr uA , vA , sA v bn B s c uB , vB , sB . D dng nhn thy [10]:
uA = vB
uB = vA
QA = sA (dG + vA (cG)) = sA (d + vA c)G
= sA (d + uB c)G = sA sB G
QB = sB (bG + vB (aG)) = sB (b + vB a)G
= sB (b + uA a)G = sB sA G
QA = QB = Q
nh gi bo mt: hack c kha chia s, Hacker cn phi tnh c cc gi tr a, b, c, d,
mun vy Hacker phi gii cc bi ton Logarithm ri rc a = logG (aG), b = logG (bG),
c = logG (cG), d = logG (dG). y l cc bi ton kh khng th gii c trong thi gian a
thc.
38

Tp ch Epsilon, S 10, 08/2016

4. Xc thc - ch k s
4.1. ECDSA(The Elliptic Curve Digital Signature Algorithm)
Nm 1999, ECDSA (The Elliptic Curve Digital Signature Algorithm) c ph duyt thnh
tiu chun ca ANSI (ANSI X9.62-1998 ECDSA, phin bn mi nht l X9.62-2005), nm
2000 ECDSA cng c IEEE v NIST ph duyt thnh tiu chun FIPS PUB 186-4 (DSS
- Digital Signature Standard), phin bn mi nht ban hnh 7-2013. ISO nm 2002 cng ban
hnh tiu chun ISO/IEC 15946-2:2002 trong c phn dnh ring v ECDSA. M t chi tit
v ECDSA c th tm thy trong [29].
Ngi k s chn s d lm kha b mt v to ra kha cng khai l Q = dP , s dng hm bm
H to ra gi tr tm lc vn bn e ca vn bn m. Ch k s s l cp (r, s) c tnh theo
thut ton 5.
Thut ton 5 Sinh ch k s ECDSA
INPUT: Tham s D = (q, FR, S, a, b, P, n, h), kha b mt d, thng ip m.
OUTPUT: Ch k s (r, s).
1: Chn ngu nhin k [1, n 1],
2: R kP = (x1 , y1 ) v chuyn i x1 x1 .
3: r x1 (mod n).
4: if r = 0 then
5:
Nhy n bc 1:
6: end if
7: e H(m).
8: s k 1 (e + dr) (mod n).
9: if s = 0 then
10:
Nhy n bc 1:
11: end if
12: Tr v (r, s)
Ngi xc thc ch k nhn c vn bn m0 v ch k s (r, s) ca ngi k, s tnh gi tr
tm lc e0 ca vn bn nhn c l m0 v p dng thut ton 6 xc nh s ph hp ca
ch k s vi vn bn nhn c, t c th khng nh vn vn c do ng ngi k k hay
c s gi mo t ngi khc hoc vn bn c b sa i hay b li do ng truyn hay khng.

39

Tp ch Epsilon, S 10, 08/2016

Thut ton 6 Xc thc ch k s ECDSA


INPUT: Tham s D = (q, FR, S, a, b, P, n, h), kha cng khai Q = dP , thng ip nhn c
m0 , ch k (r, s).
OUTPUT: Ch k hp l hoc khng hp l.
1: Kim tra r v s c phi l nhng s nguyn nm trong khong [1, n 1]. Nu khng tr v
return(Ch k khng hp l).
2: e0 H(m0 ).
3: w s1 (mod n).
4: u1 e0 w (mod n) v u2 rw (mod n).
5: R0 u1 P + u2 Q.
6: if R0 = then
7:
return(Ch k khng hp l).
8: end if
9: Chuyn i x1 ca R0 s nguyn x1 .
10: r 0 x1 (mod n).
11: if r 0 = r then
12:
return(Ch k hp l).
13: else
14:
return(Ch k khng hp l).
15: end if
Chng minh tnh ng n ca thut ton: cn phi chng minh rng nu m0 = m hay e = e0
th r0 = r. Thc vy:
w = s1 = k(e + dr) 1
R0 = u1 P + u2 Q = (u1 + u2 d)P = (e0 + rd)wP
= (e0 + rd)s1 P = k(e0 + rd)(e + rd)1 P
Nu e = e0 ta s c R0 = k(e + rd)(e + rd)1 P = kP = R l iu cn phi chng minh.
nh gi bo mt: gi mo c ch k, Hacker cn phi tm c gi tr k v kha b mt
d, tm c 2 gi tr ny Hacker buc phi gii 2 bi ton Logarithm ri rc k = logP R v
d = logP Q v y u l 2 bi ton kh, cha gii c trong thi gian a thc.

4.2. Ch k s ElGamal
Da trn lc k s do ElGamal xut nm 1984 [30], phin bn sa i c a vo
thnh chun v ch k s DSS (Digital Signature Standard) trong FIPS 186 [31].
nh ngha hm f nh sau:
f : E(Fq ) Z
C th chn hm f (x, y) = x, trong x l s nguyn 0 x < q. Cp kha b mt v cng
khai ca ngi k l (x, Y ) | Y = xP . N l bc ca im P thng l s nguyn t ln.
40

Tp ch Epsilon, S 10, 08/2016

Thut ton 7 Sinh ch k s Elgamal


INPUT: Kha b mt x, thng ip m.
OUTPUT: Ch k s (R, s).
1: Chn ngu nhin k [1, n 1],
2: R kP .
3: s = k 1 (m xf (R)).
4: Tr v (R, s)
Thut ton 8 Xc thc ch k s Elgamal
INPUT: Kha cng khai Y = xP , thng ip nhn c m0 , ch k (R, s).
OUTPUT: Ch k hp l hoc khng hp l.
1: Tnh V1 = f (R)Y + sR.
2: Tnh V2 = m0 P .
3: if V1 = V2 then
4:
return(Ch k hp l).
5: else
6:
return(Ch k khng hp l).
7: end if
Chng minh tnh ng n ca thut ton: khi m0 = m:
V1 = f (R)Y + sR = f (R)xP + k 1 (m xf (R))R = mP = m0 P = V2
nh gi bo mt: Mun gi mo ch k s, Hacker buc phi tnh c s, tnh c s buc
phi tnh c k v kha b mt x, tnh c 2 gi tr ny Hacker buc phi gii bi ton
Logarithm ri rc k = logP R v x = logP Y , l 2 bi ton khng gii c trong thi gian a
thc.

5. M ha - Gii m
5.1. M ha Massey-Omura
Massey-Omura l hai tc gi xut lc m ha c m t trong Patent [32] vo nm
1986. Lc m ha ny t c s dng trong thc t nhng n c ngha v mt lch s.
Bn A
Biu din thng ip nh M E(Fq )
Chn mA | gcd(mA , N ) = 1
M2
Tnh m1
A ZN

Bn B
M1 =mA M

M2 =mB M1

M1

Chn mB | gcd(mB , N ) = 1

M3
M = m1
B M3

M3 =m1
A M2

41

Tp ch Epsilon, S 10, 08/2016

D dng nhn thy:

1
m1
B mA mB mA M = M

nh gi bo mt: Mun ph kha trong lc ny, Hacker phi tm c gi tr mA , mB


tm c cc gi tr ny Hacker phi ln lt gii 2 bi ton Logarithm ri rc mA = logM M1
v mB = logM1 M2 , v y l 2 bi ton cha gii c trong thi gian a thc.

5.2. M ha ElGamal
Trn c s h mt ElGamal [30], lc m ha c pht biu nh sau:
Bn B
Chn cp kha (xB , YB ) | YB = xB P

Bn A
Thng ip M E(Fq )
Chn k, tnh M1 = kP
Tnh M2 = M + kYB

M1 ,M2

M1 , M2
M = M2 xB M1

Chng minh tnh ng n ca lc m ha:


M = M2 xB M1 = M + kYB xB M1 = M + k(xB P ) xB (kP ) = M
nh gi bo mt: gii m c vn bn M , Hacker buc phi tm c k v xB , do
Hacker cn phi gii 2 bi ton Logarithm ri rc k = logP M1 v xB = logP YB , v y l 2
bi ton kh.

5.3. M ha ECIES (The Elliptic Curve Integrated Encryption System)


ECIES do Bellare v Rogaway xut v l mt bin th ca m ha dng h mt ElGamal, sau
thut ton ny c a vo cc chun ANSI X9.63 v ISO/IEC 15946-3, IEEE P1363a
v [26].
Tham s D = (q, FR, S, a, b, P, n, h) c chn tng t nh vi ECDSA. y cn la chn
thm cc hm m ha/gii m i xng k hiu l Ek (m) v Dk (c). Trong m l bn r cn
m ha, c l bn c m. Thut ton m ha i xng c chn y phc v qu trnh
m ha/gii m c d dng hn v nhanh hn so vi cc thut ton bt i xng. Ngoi ra
thay v s dng hm bm n gin, ECIES s s dng hai hm bm sau:
Message authentication code M ACk (c):
M AC : {0, 1}n {0, 1} {0, 1}n

42

Tp ch Epsilon, S 10, 08/2016

Key derivation function KD(T, l):


KD : E N {0, 1}
l l di kha (k1 ||k2 ). {0, 1} l chui bit c gi tr 0, 1 c di n hoc khng xc nh
(*).
Ngi nhn c cp kha cng khai/b mt l (Y, x) trong Y = xP .
Thut ton 9 M ha ECIES
INPUT: Vn bn cn m ha m, kha cng khai Y .
OUTPUT: Vn bn c m ha (U, c, r).
1: Chn k [1, q 1].
2: U kP .
3: T kY .
4: (k1 kk2 ) KD(T, l).
5: M ha vn bn, c Ek1 (m).
6: Tnh gi tr M AC cho vn bn m ha r = M ACk2 (C)
7: Tr v return(U, c, r).

Bn gii m s nhn c tp hp (U, c, r) gm cc thnh phn sau:


U cn thit tnh kha phin DiffieHellman T .
c l bn c m ha.
r c dng xc thc m vn bn..
Thut ton 10 Gii m ECIES
INPUT: Vn bn m ha U, c, r, kha b mt x.
OUTPUT: Vn bn gii m m hoc thng bo vn bn m khng hp
l.
1: T xU .
2: (k1 kk2 ) KD(T, l).
3: Gii m vn bn, m Dk1 (c).
4: if r 6= M ACk2 (C) then
5:
xut thng bo vn bn m khng hp l
6: end if
7: Tr v vn bn c gii m m.

Kha phin T sau khi c tnh trong phn gii m s c gi tr ging nh trong phn m ha.
Thc vy:
TDecryption = xU = x(kP ) = k(xP ) = kY = TEncryption
43

Tp ch Epsilon, S 10, 08/2016

nh gi bo mt: ph kha c lc ny Hacker cn phi tm c kha b mt x hoc


gi tr k bng cch gii bi ton x = logP Y hoc k = logP U , v y l 2 bi ton kh cha
gii c trong thi gian a thc.
Mt s thut ton v giao thc khc s dng ng cong Elliptic ng dng trong mt m c
th xem thm trong [10, 21]. Ti liu Guide to Elliptic Curve Cryptography [19] vi rt nhiu
thut ton chi tit c th coi l cm nang trin khai cho nhng bi ton ng dng c th ca
ECC.

Ti liu
[1] J. W. Bos, J. A. Halderman, N. Heninger, J. Moore, M. Naehrig, and E. Wustrow, Elliptic
Curve Cryptography in Practice, Financial Cryptography and Data Security, vol. 8437,
pp. 157175, 2014.
[2] J. H. Silverman and J. T. Tate, Rational Points on Elliptic Curves - Second Edition.
Springer, 2015.
[3] L. C. Washington, Elliptic Curves Number Theory and Cryptography, Second Edition.
CRC Press, 2008.
[4] J. W. S. Cassels, Lectures on Elliptic Curves.
[5] S. Lang, Elliptic Curves Diophantine Analysis.

University of Cambridge, 1991.


Springer, 1978.

[6] C. Kenig, A. Ranicki, and M. Rockner, Elliptic Curves A Computational Approach. Walter de Gruyter GmbH & Co., 2003.
[7] H. Cohen and G. Frey, Handbook of Elliptic and Hyperelliptic Curve Cryptography.
Chapman Hall/CRC, 2006.
[8] J. H. Silverman, The Arithmetic of Elliptic Curves.

Springer, 2009.

[9] L. Berger, G. Bockle, L. D. M. Dimitrov, T. Dokchitser, and J. Voight, Elliptic curves,


Hilbert modular forms and Galois deformations. Birkhauser, 2013.
[10] I. F. Blake, G. Seroussi, and N. P. Smart, Advances in Elliptic Curve Cryptography. Cambridge University Press, 2005.
[11] I. Connell, Elliptic Curve Handbook.

McGill University, 1999.

[12] T. H. Otway, Elliptic Hyperbolic Partial Differential Equations.

Springer, 2015.

[13] Dang Minh Tuan, Che tao thiet bi VPN IPSec bang phan cung dau tien o Vietnam, Tap
chi CNTT & TT, no. 2, pp. 4145, 2014.
[14] H. Lenstra., Factoring Integers with Elliptic Curves, The Annals of Matematics, vol. 126,
no. 3, pp. 649673, 1987.
[15] V. S. Miller, Use of elliptic curves in cryptography, CRYPTO 85, pp. 417428, 1985.
44

Tp ch Epsilon, S 10, 08/2016

[16] N. Koblitz, Elliptic curve cryptosystem, Math.Comp, vol. 48, no. 16, pp. 203209, 1987.
[17] A. Enge, Elliptic Curves and Their Applications to Cryptography.
Publishers, 2001.

Kluwer Academic

[18] D. Hankerson, J. L. Hernandez, and A. Menezes, Software Implementation of Elliptic


Curve Cryptography over Binary Fields, CHES2000, vol. 1965, pp. 243267, 2000.
[19] D. Hankerson, A. Menezes, and S. Vanstone, Guide to Elliptic Curve Cryptography.
Springer-Verlag, 2004.
[20] R. Schoof, Elliptic Curves Over Finite Fields and the Computation of Square Roots,
Matematics of Computation, pp. 483495, 1985.
[21] I. Blake, G. Seroussi, and N. Smart, Elliptic Curves in Cryptography.
versity Press, 1999.

Cambridge Uni-

[22] H. Cohen, A Course in Computational Algebraic Number Theory. Springer-Verlag, 1993.


[23] J. M. Pollard, Monte Carlo Methods for Index Computations (mod p), Mathematics of
Computation, vol. 32, no. 143, pp. 918924, 1978.
[24] S. C. Pohlig and M. E. Hellman, An Improved Algorithm for Computing Logarithms over
GF(p) and its Cryptographic Significance, IEEE Transactions on Information Theory,
vol. 24, pp. 106110, 1978.
[25] A. J. Menezes, T. Okamoto, and S. A. Vanstone, Reducing elliptic curve logarithms to
logarithms in a finite field, IEEE Trans. Inform. Theory, vol. 39, no. 5, pp. 16391646,
1993.
[26] C. Research, Standards For Efficient Cryptography, SEC 1: Elliptic Curve Cryptography.
Certicom Corp, 2000.
[27] L. Gao, S. Shrivastava, and G. E. Sobelman, Elliptic Curve Scalar Multiplier Design
Using FPGAs, CHES99, vol. 1717, pp. 257268, 1999.
[28] L. Laurie, M. Alfred, Q. Minghua, S. Jerry, and V. Scott, An Efficient Protocol for Authenticated Key Agreement, Designs Codes and Cryptography, vol. 28, no. 2, 1998.
[29] D. Johnson, A. Menezes, and S. Vanstone, The Elliptic Curve Digital Signature Algorithm (ECDSA), 2001.
[30] T. E. Gamal, A Public Key Cryptosystem and a Signature Scheme Based on Discrete
Logarithms, CRYPTO 84, vol. 196, pp. 1018, 1985.
[31] NIST, Digital Signature Standard (DSS) FIPS 186-4. National Institute of Standards and
Technology, 2013.
[32] J. Massey and J. Omura, Method and apparatus for maintaining the privacy of digital
messages conveyed by public transmission, Jan. 28 1986, US Patent 4,567,600. [Online].
Available: https://www.google.com/patents/US4567600

45

Tp ch Epsilon, S 10, 08/2016

VT L, HNH HC V TRI T TRN


Nguyn i Vit
(Vin Cng ngh Thng tin, i hc Quc gia H Ni)
GII THIU
Nhc ti chuyn Tri t trn l ngi ta ngh ti cc nh vt l N.Copernicus, G.Galilei,
G.Bruno v thng ngh rng l mt vn thuc v vt l. Thc ra y l mt vn
ng dng hnh hc c tm ra trc gn 2000 nm bi nh bc hc Hy Lp
Erasthonenes. V sao nhn loi li b qun mt pht kin v i nh vy? V sao cc nh
bc hc Trung Quc, vn cha bao gi bit v ngh ti vic ny cho n khi b vn minh
phng Ty trn vo v ln t.

Hnh hc v Vt l
Thi Hy Lp c, khoa hc khng phn bit, chuyn mn ha nh by gi. Mt nh bc hc
thng nghin cu cc vn Trit hc, Ton hc, Vt l, Logic, Ngn ng v cc khoa hc
nhn vn. Cc ngnh khc nhau, nh ta bit by gi, ch l cc vn m h quan tm.
Tri vi ngy nay, khi hnh hc gn lin vi ng dng thc t thi Hy Lp c l vic o t
ai. Vt l l khoa hc siu hnh tru tng ch ginh ring cho cc nh hin trit ln nht gii
thch mi hin tng xy ra trong t nhin. Cc vn vt l m h bn lun ht sc xa thc
t nh ti sao mi tn khng c ngi y li bay c. Khi , hnh hc v vt l l hai vn
khoa hc song sinh, cng nghin cu tnh cht v quan h ca cc vt th trong khng gian
sng ca con ngi.
Dn dn, hnh hc c gng thot ra khi cc ni dung ng dng c th, ngy cng tru tng
hn. iu tt, gip hnh hc bt thin cn c th vn ti nhng khi nim hin hu nhng
bn ngoi suy din trc gic ca con ngi. Nh th phm vi ng dng ca hnh hc ngy cng
m rng.
Vt l pht trin theo chiu ngc li, c gng thot khi thp ng siu hnh hc, tm n vi
quan st thc tin, gii thch cc hin tng t nhin c cn c hn. V cng nh th, vt l
ngy cng gip con ngi hiu nhiu v bn cht ca v tr.
Tuy vy, do vic chuyn mn ha qu su, ngy nay ngi ta, k c cc nh khoa hc c chuyn
mn su, c nhiu ng nhn. H cho rng, Hnh hc l khoa hc v nhng hnh th khng tn
ti, chng lin quan g ti thc tin. Ngc li, vt l l phi gn lin vi s liu ca cc qu
trnh c th. Do , chnh cc nh khoa hc cng t bn khon, nghin cu cc khoa hc nh
hnh hc v vt l l thuyt s em li iu g.
46

Tp ch Epsilon, S 10, 08/2016

Hnh hc Trung Hoa v Hy Lp c


Ngi ta thng ni cc nh hin trit Trung Quc cng tm ra hnh hc ring theo cch ca
h. C ngi cn gn Kinh Dch, tm linh v cc yu t siu hnh khc vi cc t l L Ban quy
nh cho th mc.
Sau hn 2000 nm, mc d pht minh ra rt nhiu iu kinh ngc, lp c nhng bn thin
vn u tin ca nhn loi, kin thc hnh hc ca ngi Trung Quc vn ht sc th s, khng
tin g c thm so vi L Ban, th k 4 trc Cng nguyn.
Ni mt cch khc, hnh hc, nu c Trung Quc thi c, ch l mt dng tin thn khoa hc
ht sc s ng. Khi vic th phng cng bi cc t l hnh hc ca L Ban, cng nh vic
cc tn ca Pythagoras tin rng nh Thng h mi pht mnh c ra nh l v tam gic
vung v phi m ti 100 con b t n vi hy vng rng Thng s cho h c nhiu nh
l khc p hn. Chnh lng tin m qung v hn ch vi cc ng dng ngay trc mt
lm cc hiu bit v hnh hc Trung Quc tr nn thin cn v dm chn ti ch.
Mc d cc t duy s khai u c th cha ng rt nhiu minh trit, nhng chng vn cha
phi l khoa hc. Cho n khi vn minh phng Ty trn vo, ngi ng vn tin rng Tri
t hnh vung, bu tri hnh bn cu chp ln mt t nh mt ci lng bn, bn phng c
bn ct chng tri ton b ci lng bn.
Mt ch b lp hai ngy nay cng c th t rt nhiu cu hi lm m hnh v tr quan hnh hc
ny sp. Ti tin rng cc nh hin trit Trung Hoa cng tng t ra c cc cu hi nh
th. Nhng iu g khin h khng dm tr li thay i v tr quan ? Chng ta s quay li
vn ny sau khi nhn li cch ngh ca cc nh bc hc Hy Lp c.
Nh bc hc Hy Lp Erasthostene sng vo th k 3 trc Cng Nguyn, tc l sau L Ban hn
200 nm. ng ch dng suy lun v da trn mt s d liu quan st bit c Tri t c
hnh cu v tnh c chu vi ca hnh cu mt cch chnh xc. mi chnh l khoa hc
ch thc.
tng ca Erasthostene kh n gin so vi hiu bit ca chng ta ngy nay: Ngi Hy Lp,
ngi A rp c bit ch to ra cc ng h mt tri tng i chnh xc da trn bng ca
cc nh thp khi c mt tri. Vo thi im gia tra, bng ca cc cc ng vung gc trn
mt t l ngn nht. Nu gi thit cc tia sng n mt t u song song v mt t l phng,
cc hnh chiu ca cc cc cao nh nhau phi ging ht nhau. Tuy nhin, Erasthostene nhn
thy hnh chiu ca cc cc c cng di vo gia tra ti Syene v Alexandria khng ging
nhau. iu c ngha l cc cc khng song song v lp thnh mt gc nht nh vi nhau
. Nh vy, Tri t phi l hnh cu c chu vi gp 50 ln khong cch t Syene n
xp x 2
50
Alexandria. Bi ton trn c th gii trong trng hp cc tia sng khng song song v u xut
pht t mt im l Mt Tri. Tuy nhin do khong cch gia Mt Tri vo Tri t l qu ln
so vi cc d liu trong bi ton, ng gp ca s sai lch ny khng ng k.
iu quan trng y khng phi l gii mt bi ton m phng php suy lun, sng to,
tr tng tng v kh nng lin tng gia cc khi nim tru tng v thc t. Cc nh hiu
trit ng i trc, c rt nhiu u th, nhng thiu i nhng thnh phn quan trng nht
to ra sng to khoa hc c tm c thc s. Nu c ai v sn cho h cc tia sng, cc, ng
47

Tp ch Epsilon, S 10, 08/2016

dy cung gia Syene v Alexandria v tm Tri t, cc hc tr ca h cng c th tnh ton d


dng chu vi ca vng trn, cho d h cha bao gi bit n s .
im quan trng trong pht kin vt thi gian gn 2000 nm so vi minh trit phng ng
ca Erasthostene l c tng tng gn lin vi cc d liu c th quan st c. D liu quan
st c mi gip chng ta "nhn" thy nhng s vt m nhng ngi minh trit n u cng
khng nhn thy c. Cc gio iu, gi thit, nh kin c sn s bt mt lm cc nh khoa
hc m la. Ni mt cch khc, Erasthostene nhn ra Tri t trn t cc bng nng, iu
m cc nh hin trit ng khng th no lm c do phng php nhn thc ca mnh.
Nghin cu hnh hc khng ch bao gm vic suy lun ra cc tnh cht ca cc vt th da trn
mt s tin c sn m cn l vic nhn thy c s hin hu ca mt hnh hc xc nh qua
cc vt th v hin tng c trong thc t.

Hnh hc, vt l v ro cn v hnh


Hnh hc, xy dng trn nhng tin c nh, v c gng tm ra nhng tnh cht bt bin vi
thi gian. Vi ton hc, mt tam gic c gi thit l c sinh ra cng vi v tr v tn ti
mi mi nh th. Cc cu hi, ti sao mt vt li c hnh tam gic, hnh thnh nh th no, trc
l hnh g v trong tng lai s l hnh g, u c cho l khng thuc phm vi nghin cu
ca hnh hc.
Hnh hc nh mt bc tranh tnh, mt lt ct ng thi gian ca cc s vt v ng cng li
chng li nghin cu. Vt l, ngi anh em song sinh ca hnh hc l khoa hc ca vn ng,
nh n ton hc bit cc thuc tnh ca s vt ti mi thi im, nhng tm cc quy lut
iu khin vn ng, thay i ca s vt.
Thc ra, hnh hc c sc mnh hn l mt trt t tnh. Chnh cch nhn nhn, ca nh khoa hc,
b nh hng khng t bi mi trng sng, cng lun xung quanh, t hn ch mnh ging nh
cc hin trit ng thi c.
Khng t cc nh Ton hc ln nh Leibnitz, Gauss, Poincar, Hilbert, Cartan thot c
khi nhng tn iu . Leibnitz cng vi Newton pht hin ra tnh ton gii tch v xy dng
h thng v tr c hc ca Newton. Gauss, Poincar t nn tng cho h thng v tr vi cc
hin tng in t ca Maxwell. Hilbert, Cartan cng Einstein xy dng m hnh th gii
bng cc a tp Riemann cho cc hin tng hp dn.
Trong cc m hnh v tr quan nh th, hnh hc tr thnh cun phim sinh ng, m t nhng
v n ln, sng lan truyn t nhng khong cch khng thi gian ti hng t nm nh sng.
Khng c hng ro no cn tr s suy ngh, bit cc khi nim tru tng thnh chuyn ginh
ring ca mt s ngi, cng khng c hng ro no ngn cch hnh hc v vt l. Cc tn iu
u l con ngi tng tng ra ngn cm chnh mnh.

48

Tp ch Epsilon, S 10, 08/2016

ngha thc tin ca hnh hc v vt l


Ngy nay, nhn loi ang ng trc nhng pht kin ln lao vi nhng quan st mi v sng
hp dn, vt cht ti, lc th nm ha hn mt cuc cch mng mi v khoa hc cng ngh. C
nhng ngi v nhng dn tc s tr nn hng cng, c nhng ngi v nhng dn tc s l
chuyn tu lch s bi khng chu t b thi quen suy ngh ca chnh mnh.
Khoa hc cng ngh khng th pht trin da trn mt ng lc ng dng thin cn vo nhng
khi nim c sn v c nh xung quanh ta. C l ng lc tm n cc khi nim mi quan
trng hn chnh bn thn ni dung cc pht kin khoa hc.
Trong khi , nh mt tt yu, mc d khng phi l ng lc ban u, khoa hc lun mang
n nhng cng ngh mi. Khi sng in t c tm ra t li gii ca phng trnh Maxwell,
khng ai ngh c c mt ngy, sng v tuyn truyn hnh, Internet, in thoi di ng li trn
ngp khng gian sng ca chng ta nh ngy nay. Cng thc nng lng E = mc2 ca l thuyt
tng i em li ngun nng lng khng l cho nhn loi. Phng trnh Schrodinger v cc
h thc giao hon ma trn ca Heisenberg m u cng ngh vt liu mi, bn dn v cc linh
kin in t m ra cuc cch mng v cng ngh thng tin.
Ni v thnh tu do khoa hc nng ngha em li, chng ta cng khng bao gi qun nhng
vt cn ng do chnh chng ta to ra l h qu ca thi quen c h, nh kin sai lm, thc
dng thin cn. Chng ta thy bi hc ca khoa hc thi c ng. kt thc ti xin
nhc li mt cu chuyn ni ln cch suy ngh thc dng thin cn c th gy nn mt sai
lm quyt nh vn mnh ca c mt dn tc nh th no.
Trc chin tranh th gii ln th 2, nc c l trung tm ca khoa hc th gii, ni pht minh
ra C hc Lng t v Thuyt tng i. V mt cng ngh, nc c i u v hon ton sn
sng ch to thnh cng bom nguyn t. Do chnh sch ca Hitler, rt nhiu nh khoa hc gii
ri khi c, do khng mun hp tc vi chnh quyn pht xt. Vi lc lng cn li, nc
c vn kh nng lm ra bom nguyn t trc M v Lin X. Tuy nhin, Hitler ban hnh
mt chnh sch l mi nghin cu khoa hc phi c kt qu thc tin trong vng 6 thng. Chnh
iu lm nc c khng th ch to bom nguyn t v khng cc nghin cu c bn cn
thit.
C th l mt may mn cho nhn loi, nhng khng h ngu nhin. Mt s ngu dt v chnh
sch cng ch l h qu tt yu ca cch ngh pht xt m thi. c ti, phn bit chng tc bao
gi cng s t km hm chnh mnh. Chnh v th m nhn loi cn tn ti.
Bn ngh sao nu L Ban cng tm ra c Tri t hnh cu nh Erasthostene, hnh hc Euclide
c tm ra v cch mng c kh n ra ti Trung Quc? Vi cch ngh ng, cho d c nhng
kin thc khoa hc v i nht, liu c phi l may mn hn cho nhn loi hay khng?

49

Tp ch Epsilon, S 10, 08/2016

ST MAYONNAISE V BU C TNG THNG M


Nils Berglund (i hc Orleans, Cng ha Php)
Ngi dch: Dng c Lm (i hc Sussex, Vng quc Anh)

LI NGI DCH
Nils Berglund l gio s ton hc ca i hc Orlans, Php. Lnh vc nghin cu chnh
ca ng l vt l ton, l thuyt h ng lc v l thuyt xc sut. Ngoi chuyn mn
ng cn c s thch trt tuyt v nu n. ng cng c kh nhiu bi vit ph bin khoa
hc, mt s ng trn cc tp ch, website khoa hc ca Php, in hnh l Images des
Mathmatiques. Bi vit sau y c dch t nguyn bn ting Php "Mayonnaise et
lections amricaines" ng trn tp ch Dossier Pour La Science s 91, thng 4 - 6
nm 2016. Bn dch c s cho php ca tc gi. Mt s thut ng, do cha c
dch ra ting Vit mt cch thng nht, hoc cho c gi c th tm hiu r hn, chng
ti ch thch thm di dng ting Anh. Tt c cc ch thch l ca ngi dch.

Chc hn l bn bit n phng trnh vi phn, nhng liu bn c bit phng trnh vi phn
o hm ring ngu nhin? C ngun gc t nhng vn sng ng quanh ta, chng rt hu
ch cho vic nghin cu cc hin tng khc nhau trong t nhin cng nh trong chnh x hi
loi ngi. N l lnh vc nghin cu trung tm dn n gii thng Fields ca mt nh ton
hc nm 2014.
Hy bt u vi hai bi ton thc t sau y. Trn ln mt hn hp du n vi nc, quan st
(vi cc cng c h tr ty theo la chn ca bn) s lng ng ca cc phn t trn b mt, v
so snh n vi tr chi xp hnh Tetris1 . Theo di s thay i quan im chnh tr ca ngi M
trong tin trnh ca cuc bu c tng thng s din ra vo thng 11 nm nay. Tht ngc nhin
l hiu c nhng hin tng khc nhau t nhng ngun gc rt khc nhau nh th, chng
ta li cn n s h tr ca cng mt cng c ton hc. l phng trnh o hm ring ngu
nhin (c k hiu l SPDE2 ), mt cng c ton hc m tr nn hiu qu hn rt nhiu nh
cc nghin cu gn y ca Martin Hairer, Gio s i hc Warwick, Vng quc Anh. Vi
nhng ng gp quan trng , ng c trao tng Huy chng Fields danh gi nm 2014. Vy
SPDE l g?
im khc nhau cn bn gia SPDE vi phng trnh vi phn thng l ch, SPDE c p
dng cho cc h m hnh ton hc c chiu v hn vi mt i lng c gi l ting n3 (hay
cc nhiu lon ngu nhin). Nh chng ta c th nghin cu cc tnh hung phc tp ni c
s tng tc, giao thoa ca rt nhiu yu t, nh cc bt du, cc phn t vt cht hay nhng
1

Mt tr chi in t rt ph bin t cui nhng nm 90 ca th k XX


Stochastic Partial Differential Equations
3
noise
2

50

Tp ch Epsilon, S 10, 08/2016

Hnh 1: S pht trin quan im chnh tr ca ngi dn M c th c m t bi phng trnh


o hm ring ngu nhin!
ngi M! Chng ta s cng vn bc mn b mt v cc phng trnh ny, cng nh khng qun
tm hiu nhng tin b quan trng no trong cc cng trnh ca Martin Hairer a n cho
ng y mt gii thng tng ng vi gii Nobel. Nhng trc khi bt u, hy cm ly ngn
uc ca bn!
t nng mt thanh kim loi mt cch khng ng u, tc l ch t mt s ch nht nh
trn thanh. Lm th no xc nh c nhit mt v tr no ca thanh kim loi theo
thi gian? Chng ta c th tr li cu hi ny nh l thuyt phng trnh truyn nhit (xem Ph
lc 1 cui bi vit), c gii thiu bi Joseph Fourier vo nm 1811. Bin trong phng trnh
truyn nhit l hm T (x, t), m t nhit ti im x v thi gian t. y l mt phng trnh
vi phn o hm ring, n biu th s ph thuc ca cc thng s v s bin thin theo khng
gian v theo thi gian ca nhit . Ngi ta bit cch gii phng trnh ny, v do d
on, bit c s phn b nhit ti thi im ban u T (x, 0), nhit ti bt k im no
cng nh ti bt k thi gian no sau .

t nng thanh kim loi


Phng trnh truyn nhit c hai tnh cht quan trng. Tnh cht th nht, nguyn l chng cht
nghim4 , ni rng nu ta bit c cc nghim m t s phn b nhit ca thanh kim loi cho
bi hai ngun nhit khc nhau, chng hn t nng thanh ti im x hoc im y, th ta cng
bit c nghim m t s phn b nhit ca n khi t nng thanh cng lc ti hai im x
v y. Nghim ny ch n gin l nhn c bng cch cng hai nghim trc vi nhau.
Tnh cht th hai ni rng phng trnh truyn nhit c tnh chnh quy5 . Gi s s phn b nhit
ban u rt khc nhau, chng hn, t na tri ca thanh kim loi vo l nung 1000 C
(ri b ra), cn na phi nhit phng. Th th gn nh ngay lp tc, nhit na tri s
lan truyn mt cch u n v lin tc ra ton b thanh cho n khi ton b thanh t c mt
nhit n nh6 .
4

superposition principle
regularity, hay tnh trn
6
ni cch khc, ngay sau khi b thanh kim loi ra khi l, th nhit trong thanh l mt hm lin tc
5

51

Tp ch Epsilon, S 10, 08/2016

Chng ta cng m t c s truyn nhit trong cc i tng phc tp hn, khi thay thanh kim
loi bi mt tm kim loi hnh ch nht, hnh a, hay mt khi kim loi hnh lp phng. Vi
nhng hnh khi tng qut hn, mc d khng phi lc no ta cng tm c nghim mt cch
chnh xc, nhng nguyn l chng cht nghim v tnh chnh quy th vn lun ng cho mi
trng hp.
Mt s tng qut kh d khc cho phng trnh truyn nhit l khi thanh kim loi c tip tc
cung cp mt ngun nhit thay i theo thi gian, ta c phng trnh truyn nhit cng bc7
(cn gi l bi ton khng thun nht). Lc ny nghim ca bi ton nhn c nh p dng
nguyn l Duhamel, ni rng nghim thi im t c th vit nh mt s chng cht nghim
ca cc thi im trc .

Mn st mayonnaise tht bi v b mt ln sng ca


tm tn
C nhiu phng trnh o hm ring c dng tng t nh phng trnh truyn nhit, nhng
cha thm mt s i lng khc lm cho vic nghin cu chng tr nn kh khn hn rt nhiu.
Mt v d in hnh l phng trnh Allen - Cahn, m phng hin tng tch pha8 . Khi trn
mt hn hp nc vi du n vo mt ci bnh thy tinh v lc mnh, ta nhn c mt th nh:
chng khng th trn ln hon ton vo nhau, m hnh thnh nhng git nh li ti du v nc
c th quan st thy qua knh lp.
Nguyn l tng t cng xy ra vi nc st mayonnaise, mt hn hp nh tng ca du n
v dm c kt dnh vi nhau bng lng trng g. Thiu thnh phn cui cng ny (hoc
mt thnh phn c vai tr tng ng), th d bn c c gng cch my, cc git li ti du n v
gim cng s t hp dn dn thnh hai lp khc nhau! l s tch pha. Hin tng tng t
cng c quan st thy trong mt s loi hp kim.
m hnh ha hin tng ny, hy tng tng c mt chui ht cm c ni vi nhau bi
nhng chic l xo. t ngang chui ht ln trn mt tm tn ln sng gm hai rnh song song
cch nhau bi phn chm nh ln cao gia (xem Hnh 2). Di tc dng ca trng lc, mi
ht cm u c xu hng ln xung y mt trong hai rnh. Tuy nhin v c si l xo, cc
ht ln cn ht cng c xu hng b ko xung rnh theo. Cc tng tc ny l mt s m
phng tng t cho hn hp nh tng trn khi chng tch thnh hai phn du n v dm, v
xu hng chuyn ng ca cc phn t ca hai cht lng bao quanh cc phn t cng loi9 .
By gi ly cc chui vi s ht cm tng dn cn l xo th ngn dn10 . Ti im ti hn, chui
ht c th c c trng bi mt hm Y (x, t), cho bit s chuyn dch ngang ca chui ti
im x v ti thi gian t. Gi s hai rnh tng ng vi Y = 1 v Y = 1, trong khi phn chm
l Y = 0. Ngi ta chng minh c rng s chuyn dch ngang ny tun theo mt phng
7

forced
phase separation
9
Bn c c th xem mt minh ha th v cho hin tng ny y https://www.youtube.
com/watch?v=NDQHepkSeS8
10
ng nhin kch thc ht s phi b dn
8

52

Tp ch Epsilon, S 10, 08/2016

Hnh 2: Chui ht cm v mayonnaise. Cc ht gn vi nhau bi l xo c th chuyn ng


t do theo phng y nhng b c nh theo phng x vi mt khong cch u nhau (a). Khi
chiu di lo xo ngn li, ngn hn khong cch nh nht gia cc ht, chng s c xu hng
xp thnh hng (ng nt t). t chui ht ny ln mt tm tn (b). Mi ht cm s b thu
ht bi cc ht ln cn ca n v bi rnh tm tn. Khi s ht tng ln v hn, s chuyn ng
ca chui ht c m t bi phng trnh Allen - Cahn mt chiu.
trnh o hm ring, nhn c bng cch thm vo phng trnh truyn nhit mt i lng
Y Y 3 , biu th cho hiu ng y cc ht v mt trong hai rnh. Phng trnh ny c a ra
vo nm 1972 mt cch c lp bi hai nhm nghin cu, mt nhm gm Nathaniel Chafee v
Ettore Infante, nhm kia l John Allen v Sam Cahn (xem Ph lc 2).
Khc vi phng trnh truyn nhit, phng trnh Allen - Cahn khng c nghim hin, cng
khng tha mn nguyn l chng cht nghim. Nguyn nhn l bi c s xut hin ca i
lng phi tuyn Y 3 . Tuy vy, ta c th xy dng c mt nghim ca n bng phng php
xp x lin tip. Trc ht, hy tm qun i i lng Y Y 3 tr v vi phng trnh truyn
nhit, v gi nghim ca n l Y0 . tng l sau thay th i lng Y Y 3 bi Y0 Y03 v
nhn c mt phng trnh truyn nhit khng thun nht m ta bit cch gii. Gi nghim
ca n l Y1 , li thay th i lng trn bi Y1 Y13 , v c tip tc nh th, vi hi vng rng
chng ta s tin mt cch t t n nghim chnh xc ca phng trnh. V iu ny thc t l
ng, tht vy, phng php ny chng qua l mt bin th ca php lp Picard, n cung cp
mt con ng tm nghim ca phng trnh Allen - Cahn v vn hot ng rt tt trong
trng hp s chiu tng ln (xem Hnh 3).
Nhc li rng, php lp Picard l mt phng php gii phng trnh vi phn bng cc php xp
x lin tip, cng nhiu php lp cng chnh xc. Thc t quy trnh ny thng c s dng
chng minh s tn ti nghim ca mt phng trnh vi phn hay phng trnh o hm ring c
th no .
Trong m hnh tch pha, chng ta cng c th quan tm n s bin i ca kch c cc nhm
phn t cng loi theo thi gian, v cc mt phn cch11 gia chng (xem Hnh 4).
Cc hin tng tng t nh s tch pha cng c quan st thy trong cc m hnh nam chm
hay cc m hnh v sinh thi (chng hn m t s hnh thnh cc vt vn hay lm m trn lng
ng vt trong qu trnh chng ln ln). Thm mt cht tng tng, chng ta thm ch nhn
thy iu tng t trong mt h m t s pht trin ca cc quan im c nhn. Xt mt t
11

interface

53

Tp ch Epsilon, S 10, 08/2016

Hnh 3: S tch pha. Chng ta c th m phng hin tng ny vi mt "thm" ht cm c


sp xp theo mt mng li vung, lin kt vi nhau bi cc l xo (a). Chng c th chuyn
ng vung gc vi mt phng. S tin ha ca h ny (b) c cho bi nghim ca phng
trnh Allen - Cahn ngu nhin hai chiu, gii thch hin tng tch pha. Vng mu v xam
lam tng ng vi cc pha nguyn cht (du n hoc dm), cc vng mu cam, vng hoc xanh
l cy l cc vng trn ln vi t l khc nhau gia hai pha. Bn hnh nh m t trng thi ca
h sau 10, 50, 100 v 300 n v thi gian.
nc c hai ng i lp, v d nc M. Nhng im xanh v nhng im tng trng cho
mt cch tng ng nhng ngi tin vo ng Dn ch v tin vo ng Cng ha, nhng im
c mu trung gian tng trng cho nhng ngi cha quyt nh s theo bn no, vi mt thin
hng mnh hay yu hng n mt trong hai ng. Chng ta gi s y rng mi ngi dn
b nh hng bi nhng ngi hng xm ca h, v do hnh thnh nhng vng m trong
mi ngi cng chia s mt quan im chnh tr. Vy phe no s chin thng trong cuc bu
chn tng thng sp ti? Cu tr li s c vo thng 11 ny12 !
Tnh hung trong h ca chng ta s phc tp hn mt cht khi thm mt i lng ngoi lc
ngu nhin (nhiu lon) vo phng trnh. l kt qu ca chng hn s chuyn ng nhit
ca cc phn t khng kh bao quanh cc ht cm trong chui. Khi cc phn t kh tc ng
qua li vi nhau, ta c th m t tng tc gia chng bng mt khi nim gi l ting n trng
thi gian13 , m tc ng ca n ti h ti nhng thi im khc nhau l c lp vi nhau.
Khi ta i n mt gi tr ti hn m s ht trn chui tin ra v cng, ting n trng khi tr
thnh ting n trng khng-thi gian14 . N tc ng mt cch c lp ti nhng thi im v
v tr khc nhau. Chng ta v vy ang i mt mt SPDE, v thm vo mt i lng tng
ng vi ting n m t mt h v hn: l phng trnh Allen - Cahn ngu nhin. Chng
ta s thy rng ting n ny, hon ton khng chnh quy15 trong cc iu kin ca ta, cn tr
vic tm nghim ca phng trnh khi chiu ln hn hay bng 2.
12

Mt v d minh ho r nt khc v vn cn nng hi l cuc trng cu dn v vic li hay ri lin minh


chu u ca ngi Anh ngy 23 thng 6 va ri. Sau khi c kt qu kim phiu vi chin thng cho phe Brexit, rt
nhiu ngi dn t ra hi hn v l phiu ca h b nh hng bi nhng ngi xung quanh!
13
temporal white noise
14
spatiotemporal white noise
15
highly irregular

54

Tp ch Epsilon, S 10, 08/2016

Hnh 4: Bi ton s tch pha t m hnh cc ht cm ny sinh nhiu cu hi th v. Hnh dng,


kch thc ca cc nhm ht cng loi (tc cc vng cng mu) thay i nh th no theo thi
gian? Liu mt phn cch gia cc nhm cui cng c bin mt? C th m t c chuyn
ng ca cc b mt phn cch ny theo thi gian? Lc khng-thi gian s cho mt cu tr
li. Khng gian l b ngang, thi gian l b dc t cao xung thp. Cc v tr ht cm c
nh du bi mu sc nh trn hnh. Ti cc im trn cc phn "mm" nh ra s xy ra xung
t gia cc mt phn cch, ko theo s bin mt dn ca chng.

Tetris v mt phn cch


Mt v d khc v SPDE l khi ta phun cc phn t ln mt vt cht nn, c s dng chng
hn trong cng ngh ch to vt liu bng k thut epitaxy16 cho cc thit b in t. m hnh
ha qu trnh ny, hy bt u ngun cm hng t mt dng bin th ca n l tr chi Tetris.
Xt trong khng gian mt chiu vi mt vt th lm nn t nm ngang cho trc. Cc phn t
tm coi l nhng hnh vung nh, ri thng t mt v tr ngu nhin t trn xung. Quy lut l
khi hnh vung chm vo nn, hoc chm vo mt hnh vung khc bn cnh hay bn di n,
n s khng di chuyn c na. Tuy nhin, cc hnh vung c th, vi mt xc sut nh, di
chuyn sang hai bn mt khi n xut hin. Cu hi l, lm th no bit c mc g gh trn
b mt ca cc vt liu c sn xut nh vy? (Xem Hnh 5).
Vo nm 1986, Mehran Kardar, Giorgio Parisi v Yi-Cheng Zhang thit lp c mt SPDE
m t qu trnh ti hn nhn c khi kch c hnh vung tin dn v 0 (xem Ph lc 2). Phng
trnh ny, c k hiu l KPZ, t ch ci u ca tn ba tc gi, cha cc i lng ca phng
trnh truyn nhit, m t s chuyn ng ca cc phn t phun vo, cng nh mt ting n trng
khng-thi gian v mt i lng phi tuyn c trng bi hnh dng ca b mt nn phn cch.
Tht khng may, phng trnh ny li khng t chnh17 do tnh khng chnh quy ca i lng
ting n, v cho n gn y chng ta khng th a ra c mt li gii c ngha ton hc
no cho n.
hiu c bi ton, chng ta phi lng ha tnh khng chnh quy ca cc i lng c trong
phng trnh. Chng ta c th lin kt mi hm f vi mt s r m cng ln khi hm cng trn.
16
tm dch l k thut cy ghp, mt k thut cho php ch to mng mng n tinh th c tinh khit rt cao,
thc hin trong mi trng chn khng siu cao, c pht minh vo nhng nm 60 ca th k XX
17
ill-posed

55

Tp ch Epsilon, S 10, 08/2016

Hnh 5: Mt s n gin ha ca tr chi Tetris. Vic phun cc phn t ln b mt vt liu nn


c th m hnh ha bi mt qu trnh tng trng ngu nhin. cc phn t c biu din
bi cc ht hnh vung ri xung mt v tr ngu nhin trn mt phng nm ngang, tng t tr
chi Tetris. Mi ht s dng li khi n chm vo ht khc. Hn na, cc ht trn cao c th
di chuyn sang hai bn vi mt xc sut nh. th rp ca b mt tng ln khi xc sut ny
gim xung: b mt hnh (a) th hn hnh (b).
Nu th ca hm s c tip tuyn ti mi im th r ln hn hoc bng 1. Hn na nu ti
mi im ca f ta u xc nh c mt cong, th r t nht bng 2. Nu f chnh quy bc r,
th cc o hm ring ca f chnh quy bc r 1. Mt cch ngc li, nghim ca phng trnh
truyn nhit cng bc bi f c bc chnh quy cao hn r.

Phng trnh khng t chnh


Bc chnh quy r khng nht thit phi l mt s nguyn. Chng hn chuyn ng Brown, m t
qu o hn lon ca mt ht phn hoa gia nhng ht khc trong nc, c bc chnh quy nh
hn 1/2 mt cht18 . Cc qu o ca chuyn ng Brown l khng kh vi (chng khng c tip
tuyn ti bt c im no). Tuy nhin, ngi ta c th nh ngha o hm ca chng theo ngha
phn phi.
Phn phi19 , mt i tng tng qut hn hm s, l mt l thuyt c pht trin vo na u
th k XX bi cc nh ton hc Jacques Hadamard, Salomon Bocher, Sergei Sobolev v Laurent
Schwartz. Thay v xc nh gi tr ca phn phi ti mt im c th, ta xc nh gi tr trung
bnh ca n trong mt ln cn nh xung quanh im n. Mt s ton t i s c nh ngha
18
19

thc t th c th xem n l mt s ty nh hn v gn bng 1/2


distribution, cn gi l hm suy rng

56

Tp ch Epsilon, S 10, 08/2016

trn hm phn phi: chng ta c th chng hn thc hin php cng v php ly o hm chng.
Tuy nhin, nhn chung ta khng th thc hin php nhn hai phn phi.
Ting n trng thi gian c th xem nh o hm theo ngha phn phi ca mt qu o Brown,
vi bc chnh quy hi b hn mt cht so vi 1/2 (chng ta s coi rng gi tr ny gn nh
bng 1/2). Ting n trng khng-thi gian, trong khi , c bc chnh quy ph thuc vo
chiu ca khng gian. Gi tr ny xp x 3/2 trong khng gian mt chiu, xp x 2 trong
khng gian hai chiu, v xp x 5/2 trong khng gian ba chiu. y l ni chng ta s tm ra
li gii ca bi ton.
Tht vy, chng ta chng minh c rng nghim Y0 ca phng trnh truyn nhit cng bc
bi mt ting n trng c bc chnh quy ln hn 2 n v, c th l xp x 1/2, xp x 0 hoc
xp x 1/2 ty theo s chiu ca khng gian.
Qu trnh lp Picard p dng cho phng trnh Allen - Cahn yu cu tnh ton, bc th hai,
i lng Y0 Y03 . Trong khng gian chiu 1, Y0 c bc chnh quy dng, vn khng ng
ngi. Vi khng gian chiu 2 hoc 3, Y0 lc ny l mt phn phi t vic n c bc chnh quy
m. Nh chng ta bit, khng th thc hin php nhn cc phn phi, i lng Y03 tr nn
v nh, v qu trnh lp do khng th thc hin c. Phng trnh KPZ, cng vy, cha
bnh phng o hm ca Y0 . o hm ny c bc chnh quy m thm ch vi chiu khng gian
1, php lp Picard khng th p dng cho phng trnh KPZ.

Ti chun ha v l thuyt cu trc chnh quy


n lc ny, chng ta c th t hi rng liu c thc s nn m u vo cc bi ton SPDE
khng t chnh nh vy? Thc t, trong hai v d m ta tho lun, chng ta bt u vi
mt m hnh ri rc vi mt s hu hn cc i tng (cc ht cm hay cc hnh vung), chng
ng nhin l hon ton xc nh. Vn ch xut hin khi ta chuyn qua gii hn khi cho kch
thc ca cc i tng ny tin dn mt cch lin tc v 0.
Tuy nhin ta s thy rng vic m u vo n l khng v ch, v l do cho nhng vic lm ny
ca chng ta n t khi nim v tnh ph qut. M hnh ri rc ca s pht trin ca b mt
phn gii l mt trng hp c bit gia rt nhiu m hnh c th c. Chng ta tuy vy bit
rng mt lng ln cc m hnh ny b iu chnh, trong din rng, bi phng trnh KPZ. y
chnh l mt c tnh ph qut, v s hiu bit v n s lm sng t cng mt lc ton b cc
m hnh cng loi.
Bi ton gii hn lin tc v ang xut hin trong vt l lng t, ni n c gi l s phn
k tia t ngoi20 . Thc t, phng trnh Allen - Cahn tng ng mt cch hnh thc vi mt
m hnh ca l thuyt trng, gi l m hnh 4 . T nhng nm 1930, cc nh vt l xut
vic gii bi ton ny bng mt phng php gi l ti chun ha21 . tng l a vo cc
tham s ca l thuyt t l 22 m trn h c th quan st c.
20

ultraviolet, cn gi l tia cc tm, hay tia UV


renormalization
22
scaling theory
21

57

Tp ch Epsilon, S 10, 08/2016

Trong trng hp phng trnh Allen - Cahn, phng php ny l lm cho phn sn gia chm
v rnh tm tn ngy cng dc khi khong cch gia cc ht cm gim dn (xem Hnh 2). Vi
s la chn thch hp cong ca phn chm, ta nhn c mt phng trnh t chnh23 ti
trng hp ti hn. iu ln cn duy nht cn li ca quy trnh ny l n c thit t hon
ton da trn cc tnh ton hnh thc m khng c mt lp lun ton hc chnh xc no.
Tuy nhin, vo nm 1997, Lorenzo Bertini v Giambattista Giacomin, bng vic s dng php
i bin rt kho lo, t c mt kt qu chng minh s hi t ca m hnh s pht trin
ca b mt phn cch n mt dng ti chun ha ca phng trnh KPZ. Sau , vo nm
2003, Giuseppe da Prato v Arnaud Debussche c th chng minh c s tn ti nghim
cho phng trnh Allen - Cahn ti chun ha hai chiu. Ch cn mt khim khuyt duy nht, cc
k thut ny khng s dng c na trong khng gian chiu ba.
ng gp quan trng ca Martin Hairer l pht trin mt l thuyt cho php x l mt cch c
h thng mt lng ln SPDE c in khng t chnh. N khng nhng cho php tm li cc
kt qu bit cho phng trnh KPZ v phng trnh Allen - Cahn hai chiu, m cn p dng
c cho phng trnh Allen - Cahn ba chiu v nhiu phng trnh khc.
Khi nim trung tm ca l thuyt ny l cu trc chnh quy24 . Cng vi phng php ti chun
ha, n cho php xy dng mt khng gian m trong cc php lp Picard vn hot ng tt.
Nt p cng nh sc mnh ca l thuyt l n cung cp mt lc tng qut, cho php x l
mt cch c h thng cc phng trnh thay v tm kim tng li gii ring r. L thuyt ny l
mt trong nhng bc tin quan trng hng ti vic chng minh mt s bi ton m lin quan
n tnh ph qut, v chng ta c th hi vng vo nhng tin b mnh m hn trong tng lai.

Ph lc
1 Phng trnh truyn nhit. Nhit T (x, t) ti im x v thi gian t ca thanh kim loi
c xc nh bi phng trnh T /t = D 2 T /x2 D l h s dn nhit. Trong
phng trnh ny, s hng bn tri, o hm ring ca hm nhit T i vi bin thi
gian t (khi xem x nh hng s), m t tc bin i ca nhit . S hng bn phi,
o hm ring cp hai ca hm nhit i vi bin
khng gian, c trng cho tnh
2
khng ng nht ca vt liu. Hm G(x, t) = ex /4DT / 4Dt l mt nghim ring ca
phng trnh, vi vic l tng ha thanh kim loi di v hn.
Khi c nh thi gian, G l mt ng cong Gauss c dng hnh chung, mt i tng

ng mt vai tr nn tng trong l thuyt xc sut. B rng ca chung, t l vi Dt,


tng ln theo thi gian, nhng bng 0 ti 0. Nghim ny cho ta bit rng, nhit ban
u bng 0 mi ni tr im 0, im ngun ni m thanh kim loi c tip xc ngun
nhit. Nu im ngun l mt im y no , th nghim s c cho bi G(x y, t).
Nguyn l chng cht nghim, mt tnh cht ca phng trnh truyn nhit, chR ra rng
nghim tng qut ca phng trnh truyn nhit vit c di dng T (x, t) = G(x
y, t)T (y, 0)dy.

23
24

well-posed
regularity structure

58

Tp ch Epsilon, S 10, 08/2016

Nu thay thanh kim loi bi mt a kim loi hai chiu, hm nhit s l T (x, y, t)
vi hai bin ta khng gian x v y v bin thi gian t. Phng trnh lc ny c dng
T /t = DT vi T l tng o hm ring cp hai ca T i vi x v y (ton t
Laplace). Phng trnh nhit ba chiu c dng tng t, vi ba ta khng gian.
Mt tnh cht khc ca phng trnh truyn nhit l tnh chnh quy (xem Hnh 6).
Hnh 6: S lan truyn u n ca
nhit . Ban u, nhit na tri
thanh st ang mc cao (mu ),
trong khi na phi ang nhit
phng. Sai khc ny mt dn khi
nhit lan dn ra ton thanh, v
khng c bc nhy nhit trong
thanh tr thi im ban u.
2 Phng trnh Allen - Cahn v phng trnh KPZ. Phng trnh Allen - Cahn ngu nhin
m t s tch pha c th vit di dng Y /t = DY + Y Y 3 + . y Y biu th
lin kt gia mi ht cm vi cc ht ln cn vi n thng qua l xo. i lng Y Y 3
c trng cho lc tng hp gia trng trng v phn lc ca tm tn, l nguyn nhn
lm cho ht cm c xu hng chy v cc v tr Y = 1 v Y = 1. S hng cui cng
m t ting n trng khng-thi gian, mt ngoi lc ngu nhin m s tc ng ca n
ti cc thi im v v tr khc nhau ti phng trnh l c lp vi nhau.
Phng trnh KPZ m t s pht trin ca mt b mt phn cch, c trng bi hm cao
ca n h(x, t) ti im x v thi gian t, c dng h/t = D 2 h/x2 +(1/2)(h/x)2 +.
S hng u tin bn tay phi xut hin do s hon i v tr ca cc phn t cnh nhau.
Ting n trng khng-thi gian m t ngoi lc ngu nhin tc ng ln chng. Hng
t cui cng (h/x)2 n t thc t rng mt phn cch khng nht thit phi pht trin
theo phng thng ng, m cn theo phng vung gc vi chnh n. Gi tr ny nhn
c bng cch chiu tc pht trin ca b mt ln phng thng ng, kt hp vi
nh l Pythagoras v mt s php xp x nhng gi tr nh.
3 L thuyt cu trc chnh quy. L thuyt pht trin bi Martin Hairer c ly cm hng t
mt khi nim gi l ng th25 do Terry Lyons a ra vo nm 1998 v m rng bi
Massimiliano Gubinelli. N c th xem nh mt s tng qut ha ca chui Taylor, cho
php xp x cc hm trn bi cc a thc. Chng hn, khai trin Taylor bc hai ca
hm f ti x c cho bi f (y) = f (x) + c1 (x)(y x) + c2 (x)(y x)2 + r(x, y),
phn d r(x, y) tin n 0 khi y tin dn v x. Cc h s c1 (x), c2 (x) ph thuc vo f ,
chng hn c1 (x) l o hm ca f ti im x. Kiu khai trin ny thng c s dng
xc nh nghim ca mt phng trnh vi phn gn mt im c th cho trc.
L thuyt ca Martin Hairer cho php c nhiu i tng tru tng phc tp hn a thc
trong khai trin Taylor ca nghim. Cc i tng ny bao gm chng hn nghim Y0 ca
phng trnh truyn nhit cng bc bi ting n trng khng-thi gian. Thay v xem xt
phng trnh gc, chng ta nghin cu cc phng trnh tha mn tt c cc h s trong
khai trin Taylor.
25

rough paths

59

Tp ch Epsilon, S 10, 08/2016

Phng php xp x lin tip Picard cho php chng minh s tn ti nghim trong khng
gian ca cc h s ny. Cui cng, mt nh l xy dng li lin kt mt phn phi vi
nghim ny. nh l ny s dng l thuyt sng nh wavelet, mt l thuyt cng c p
dng nhiu trong k thut ra nh, bao gm c nh k thut s.
4 Mt phin bn trc ca bi vit ny ta Quest-ce quune quation aux Drives Partielles Stochastique?26 (c mt s im khc, c bit c nhiu video minh ha), c
ng trn Images des Mathmatiques (mt tp ch ton hc online ca Trung tm Nghin
cu Khoa hc Quc gia Php - CNRS, trnh by cc nghin cu ton hc ra i chng
bng t ng v hnh nh, vi mc tiu tt c bi vit c vit bi cc nh ton hc nhng
khng c bi vit no c vit dnh cho cc nh ton hc!), c th c online ti a ch
http://images.math.cnrs.fr.

Ti liu
[1] M. Hairer, A theory of regularity structures, Inventiones Mathematicae, vol. 198, pp. 269504, 2014.
[2] G. Da Prato and A. Debussche, Strong solution to the stochastic quantization equations,
Ann. Probab., vol. 31, pp. 1900-1916, 2003.
[3] L. Bertini and G. Giacomin, Stochastic Burgers and KPZ equations from particle systems,
Comm. Math. Phys., vol. 183, pp. 571-607, 1997.
[4] M. Kardar et al., Dynamic scaling of growing interfaces, Physical Review Letters, vol. 56,
pp. 889-892, 1986.

26

do tc gi bi vit gi cho ngi dch

60

Tp ch Epsilon, S 10, 08/2016

HC CCH HC: MT BI HC QUAN TRNG


BC NHT ANG B B QUN
Dng Trng Tn

Nu bn gp mt bi ton kh, d c gng


nhiu m gii mi khng xong, th lm th
no? Cu tr li ph bin nht l c gng
tm ra ch sai trong lp lun ri i tip hoc
lm li t u. C hai phng n theo
kinh nghim ny khng ph hp vi nhng
li khuyn ca cc chuyn gia v no b.
Theo h, no b ca chng ta hot ng
theo hai c ch khc nhau: tp trung (focused
Hnh 1: Khi gii mt bi ton, no s tp trung vi mode) v th gin (diffused mode). Khi ta tp
s t cc t bo thn kinh.
trung cao vo gii quyt mt bi ton, no
s vo cuc s dng c ch tp trung vi s t
cc t bo thn kinh ti mt vng tp trung ca no b c huy ng. Khi ta ri vo th b nh
tnh hung dn, th d c c gng n my, cng ch c vng no tp trung c hot ng.
C ngha l chng ta c xu hng lp i lp li cc cch gii quyt vn , v kh lng thot
ra khi b tc. Khi hnh ng r sot li li gii hay i li tng bc t u khng c ch g
my. Nh thin ti Albert Einstein tng nhn xt i bn khng th gii bi ton theo 1000
cch ging nhau ri hy vng c li gii khc!. Trong nhng lc b bch nh th ny, cch tt
nht l tm ri xa bi ton y, i chi, th gin ri hng quay li vi bi ton. y khng phi
l li xi by v trch nhim. Vic bn tm ri bi ton i b, hng gi, hoc ngi thin t
pht s gip no b chuyn sang ch th gin, lc ny cc vng khc ca no b c kch
hot. Nu quay tr li gii ton, bn s c kh nng tm ra mt con ng khc, khng b tc
nh lc u.
Trn y ch l mt trong hng t v d cho thy nhng nghin cu v no b c th gip ci
thin ng k cch thc chng ta hc tp v lm vic. Tuy nhin, by lu nay chng ta vn
khng my khi ngh v vic tm hiu cc kin thc loi ny ci thin cch hc tp, v chng
ta thng ph mc cho thi quen sai khin trong cc hot ng ca mnh.
C th dn ra y mt thi quen tai hi khc vn chim ch trong trng hc ca chng ta: Cc
bi ging di. Bn c th gp bt k trng hc no cc tit hc ko di t 45 pht ti vi ting.
Bn cng d dng bt gp cnh tng hng t hc sinh l ng, ng gt, hoc ngi lm vic
ring trong lp v khng th ch tm vo bi ging. Trong khi hu ht cc gio vin li cho
cc c cu hc tr, th cc chuyn gia no b c mt li gii thch n gin cho hin tng ny:
61

Tp ch Epsilon, S 10, 08/2016

No chng ta ch c kh nng ch tm suy ngh trong mt thi gian rt ngn, chng 10 pht1 ,
sau l s n giai on mt tp trung. y l c ch phng v ht sc t nhin ca no ngi,
v vy hy phn chia cc bi ging thnh tng phn on ngn hn. Sau mi 10 pht tp trung,
hy thit k mt hot ng th gin v chuyn i sang phn on tip theo. Thc ra t
lu ngi ta bit dng k thut phn gi Pomodoro vi cc quy tc n gin k trn gia
tng ng k nng sut lm vic v hc tp.
Mt nghin cu ng trn Psychological Science in the Public Interest nm 20132 cho thy,
nhng phng php hc tp c s dng ph bin trong nh trng nh tm tt ni dung bi
ging, dng bt nh du on vn bn quan trng khi c sch, c i c li mt chng
sch ho ra li l nhng cch khng mang li my hiu qu v ghi nh. C nhng cch khc
hu hiu hn nhiu gip gia tng hiu qu hc tp nh: tch cc lm cc bi luyn tp, hay
hc tp cc kin thc theo hnh thc luyn tp phn tn vi cc khi kin thc c chia nh v
hc tp qua thi gian di.
Nhn t nhng tnh hung k trn, trng hc hin nay c v ang ph phm rt nhiu thi gian
ca hc tr ch v a thch kinh nghim m t quan tm ti vic tm hiu v vn dng cc kin
thc khoa hc v vic con ngi hc tp nh th no t xy dng cc hot ng gio dc
cho ti u.
Chng ta c th lin h vic hc tp nh cu chuyn ci cn cu v con c. Cch dy truyn
thng ph bin hin nay l dng cho i con c, trong khi nu ta trang b nng lc t hc cho hc
sinh th tc l cho h mt ci cn cu t lp sut i.
S thiu vng nhng bi hc lin quan n vic rn luyn k nng hc tp s mang li hu qu
m chng ta c chng kin l nhng th h hc tr th ng ch bit trng ch kin thc v
chn l t gio vin v nhng ngi i trc m khng ch ng t mnh xy dng tri thc
cho mnh. iu ny cng tr nn tai hi trong bi cnh thi i tri thc v s ha hin nay khi
m lng thng tin mi nm tng trng theo cp s m. Kin thc ngy hm nay cn ng,
ngy mai c th sai i nhiu. Ch c cch lm ch vic hc nh th no mi gip hc sinh
ng vng trong th gii ngy nay. Th cho nn, nhiu nh gio hin nay tha nhn rng tiu
chun xa m hin nay khng ch l bit c bit vit m cn phi tho cch t hc. Nhn theo
hng ny, chng ta c th d dng ng tnh vi nhn nh ca cha phng php Bn T
duy Tony Buzan: K nng t hc l k nng quan trng nht m mt ngi c th s hu.
Tht may mn l chng ta c th tm thy nhng sng kin mi trong mt s chng trnh gio
dc c ti vic rn luyn nng lc t hc trong chng trnh gio dc. Nh sng kin
Khung K nng th k XXI (p21.org) xp k nng hc tp v sng to thnh mt trong bn
hng mc chnh trong cc nng lc ct li m hc sinh th k XXI ny phi thnh thc.
Hay nh nhm Cnh Bum ch trng xy dng chng trnh gio dc tiu hc hin i xoay
quanh t tng ch o vi mt t duy nht: t hc. Theo hc sinh c rn luyn phng
php hc tp t tiu hc. Kt thc bc tiu hc, tr em c c nng lc t hc vng vng
sang bc hc cao hn cc em s s dng nng lc y t mnh n vi tri thc thay v ph
1

Medina, J. (2011). Brain Rules: 12 Principles for Surviving and Thriving at Work, Home, and School (Large
Print 16pt). ReadHowYouWant.com
2
Dunlosky, J., Rawson, K. A., Marsh, E. J., Nathan, M. J., & Willingham, D. T. (2013). Improving students learning with effective learning techniques promising directions from cognitive and educational psychology.Psychological Science in the Public Interest, 14(1), 4-58

62

Tp ch Epsilon, S 10, 08/2016

thuc vo s truyn ti thng tin mt chiu t nh trng. Nhm Cnh Bum xc quyt: Gio
dc tc l t gio dc, t lm ra chnh mnh!. Nhm i xa hn vic tuyn ngn vi bc vi
s quy trnh ha k thut tr em thc s xy dng c phng php hc cho mnh.
Ngi xa c cu, phm phi trong tnh hung kh lng th ly bt bin ng vn bin. i
vi vic hc tp, ci bt bin l phng php t hc, ci vn ng khng ngng l tri thc ca
thi i. Khng g bng trang b cho c ci bt bin ngi hc ca th k 21 c th
t mnh i trn i chn t do khm ph cnh ng tri thc ca nhn loi trong sut cuc i.
Thiu k nng thit yu ny th nhng khu hiu rn rng v x hi hc tp, hay hc tp sut i
ch cng lm l nhng li ni cho sang ming. Bi hc v cch hc cn phi l bi hc cn c
nht m mi hc sinh cn phi c luyn rn.

63

Tp ch Epsilon, S 10, 08/2016

LEONHARD EULER - NGI THY V I


Nguyn c Hng

GII THIU
Euler l mt nh ton hc, vt l hc, thin vn hc v i. ng l ngi t nn mng
cho khng bit bao nhiu l thuyt su sc gip gii quyt cho rt rt nhiu cc bi ton
thc t v qu tht ng l mt ngi thy v i ca tt c chng ta. Bi vit ny gii thiu
gin lc v cuc sng ca ng v tm tt cc cng trnh ca ng.

K 1 : Cuc sng thng trm ca Euler


Thi nin thiu
Leonhard Euler sinh ngy 15=04=1707 v l con trai c ca Paulus Euler v Margaretha Brucker.
Khi mi sinh Euler, cha ca ng l Paulus khi y cn ang l mt cha x ti nh th thnh Jakob
ngoi Basel. Mc d l mt cha x nhng ng li rt yu thch Ton hc, v vy trong hai
nm u i hc, ng ng k c hc cc mn Ton vi nh Ton hc ni ting Jakov
Bernuly. Sau , gia nh Euler chuyn ti Riehen, mt vng ngoi ca thnh ph Basel, ti
y cha ca Euler tr thnh mc s Tin Lnh ca gio x a phng cho ti cui i.
Nm 8 tui Euler c gi ti trng Latin hc tp, nhng trc Euler c hc Ton
v cc kin thc khc t cha ca mnh. Trong thi gian hc trng Latin, Euler sng cng vi
b ngoi v hc thm vi gia s Johannes Burckhardt, mt nh Thn hc tr tui v c bit say
m Ton hc. Thng 10=1720; tui mi ba 1 Leonhard theo hc ti i hc Basel vi chuyn
ngnh Trit hc, ng thi ng k hc phn ton s cp v c chnh Johann Bernoulli 2 ging
dy. Bng s am m v nhit huyt ca mnh trong vic hc tp m chng trai tr Leonhard
nhanh chng c Bernoulli v khuyn khch Leonhard hc tp v nghin cu Ton hc.
Nm 1723; Euler tt nghip thc s v c mt bi ging i chng 3 vi ch so snh trit
hc Descart v trit hc Newton.
Nm 19 tui, Euler ginh c mt gii thng t Vin Hn lm Khoa hc Paris vi l thuyt
v v tr ti u ca cnh bum trn cc con tu. iu c bit l trong sut qung thi gian trc
, Euler hu nh khng thy mt con tu no. Mt nm sau, khi chic gh gio s vt l ti i
1

13 tui Leonhard vo i hc khng phi l iu bt thng vo thi im .


L mt nh Ton hc ni ting vi khi nim v hn. ng cng l em trai ca Jakob Bernoulli thy ca cha
Leonhard.
3
Bi ging bng ting Latin.
2

64

Tp ch Epsilon, S 10, 08/2016

Hnh 1: Chn dung Euler, hin c treo Bo tng M thut Basel.


hc Basel b khuyt, Euler c Johann Bernoulli h tr rt nhiu c th ngi vo v tr
ny, nhng tht bi, cng d hiu bi khi Euler cn qu tr v thiu cc nghin cu c cng
b rng ri. Sau tht bi ny, Euler nhn li mi t Vin Hn lm Khoa hc St.Petersburg,
mi c thnh lp vi nm trc bi Nga Hong Peter I (cn nguyn ca li mi ny xut
pht t Johann Bernoulli v hai con trai ca ng bi tt c tng lm vic ti y).

Bc chuyn ln
u nm 1727; Euler chuyn ti St.Petersburg. Ti y ngoi vic nghin cu Ton, Euler cn
tham vn cho Nga v cc cu hi khoa hc v cng ngh trong bi kim tra trong cc k thi
dnh cho thiu sinh qun Nga.
Khc bit vi cc nghin cu vin nc ngoi ti vin, Euler nhanh chng thch nghi vi iu
kin sng khc nghit Bc u v ng thi Euler cng nhanh chng hc v s dng thnh
tho ting Nga phc v cho i sng v cc nghin cu ca mnh. Trong thi gian ny, Euler
sng cng vi Daniel Bernoulli v tr thnh bn thn ca Christian Goldbach 4 , th k ca Vin.
Nhng trao i gia Euler v Goldbach tr thnh nhng c liu quan trng cho lch s khoa
hc th k XVIII.
Khong thi gian Vin l khong thi gian Euler lm vic hiu qu v sng to nht, ng
cho ra nhiu kt qu c bit v chng mang li cho ng v tr v s ni ting ti Vin v trn
ton th gii sau ny.
Thng 1=1734; Euler kt hn vi Katharina Gsell, con gi ca mt ha s Thy S cng ang
ging dy trong Vin. Cuc sng gia nh ca Euler khng h sun s nh cng vic ca ng
ti Vin, h c ti 13 ngi con nhng ch c 5 trong s l pht trin khe mnh, trong c
4

Christian Goldbach nh Ton hc ngi c, ng ni ting vi gi thuyt Goldbach m cho ti ngy nay vn
cn ang l mt bi ton cha c li p.

65

Tp ch Epsilon, S 10, 08/2016

mt ngi tr thnh mt nh Ton hc v l tr l ca ng sau ny. Nm 1735 ng bnh nng,


v tng nh khng qua khi. Mt php mu gip ng vt qua n, nhng sut ba nm sau
bnh tt tip tc hnh h Euler v ln ny n khin ng mt con mt bn phi (c th nhn
thy rt r iu ny qua nhng bc chn dung ca Euler t khong thi gian ny).
Cng vi thi im cuc khng hong chnh tr Nga nm 1740; gy ra ci cht ca n hong
Nga Anna Ivanovna, Euler nhn c li mi t Quc vng Ph Frederick II n Berlin
gip thnh lp Vin Hn lm Khoa hc ti y v ng quyt nh ri St. Petersburg. Thm
na, ng cng vit trong hi k ca mnh nh sau: ... nm 1740; khi Nh vua Ph bt u
ln nm quyn hnh, ti nhn c mt li mi t Berlin, v ti nhn li khng cht ngn ngi,
sau khi n hong Anne b st hi v ko theo l s tr tr ca vng triu ...
Thng 6=1741; Euler cng vi v v hai ngi con ca mnh khi y l Johann Albrecht su tui
v Karl mi mt tui ri St. Petersburg ti Berlin.

Thi Berlin 1741 - 1766


Mt khi u c v khng thun li nh Euler ngh, v cn dang d cuc chin Silesia,
Frederick II cha th tp trung cho vic xy dng vin, v th mi ti tn 1746 vin mi chnh
thc ra i. Vin trng l nh ton hc Php, cn Euler ph trch ngnh Ton. Trong sut thi
gian di ch i, Euler hon thnh cun hi k vit ti 200 l th cng nm bi lun ln.
Tuy phi m trch rt nhiu cng vic ti Vin t qun l i thin vn, vn bch tho, lm
vic trc tip vi nhn vin, nghin cu vin, thm ch m trch c vic bn nhng cun nin
gim m bo ngun thu cho Vin, nhng khng v vy m nng sut lm Ton ca Euler b
suy gim.
Trong thi gian ny Euler tham gia cuc tranh lun v ngun gc ca nguyn l tc ng ti
thiu (principle of least action) 5 . Nm 1740; nguyn l ny c pht biu bi Pierre - Louis
Moreau de Maupertuis, nhng Johann Samuel cn c vo bc th ca Leibniz gi Jakob cho
rng ngi pht biu nguyn l ny u tin l Leibniz. Euler c li vo cuc tranh lun nhm
lm sng t vn , nhng do khng my ng tnh vi trit hc ca Leibniz, Euler ng v
pha Maupertuis v co buc Johann lm gi ti liu. Cuc tranh ci ny cng tr nn si ni
khi Voltaire tham gia cuc tranh lun v ng ng v pha Johann, Voltaire ch trch rt gay
gt c Euler v Maupertuis. Trc p lc d lun Maupertuis ri khi Berlin v Euler chu
trch nhim mi cng vic Vin.
Quan h ca Euler vi Fredric II khng c sun s, do s khc bit r rt v tnh cch cng
nh t tng. Fredric t tin, hi hc v qung giao cn Euler khim tn, sng kn o v l
mt tn theo o Tin Lnh. Mt khc sau khi Maupertuis ri Berlin, Euler l ngi cho
chng con thuyn Vin Hn lm nhng Ferderic khi pht l mi li gii thiu Euler vo
v tr vin trng v b qua tt c ri sau tuyn b chnh mnh mi l Vin trng Vin
Hn lm. Tt c cc iu trn cng vi vic khng c s ng h ca cc qu tc khc dn ti
Euler chp nhn mt li mi ca n hong Catherine II tr v St.Petersburg.
5

Nguyn l tc ng ti thiu l mt nguyn l bin phn c p dng rng ri trong vt l. p dng nguyn
l ny c th d dng tm ra c phng trnh qu o ca cc chuyn ng.

66

Tp ch Epsilon, S 10, 08/2016

St.Petersburg 1766 1783


Thi k ny cuc sng ca Euler c nhiu trc tr v mt c nhn, mt phi ca ng b c
thy tinh th (mt cn tt) v lm gim th lc rt nhiu. Nm 1771; sau ca phu thut, th lc
ca mt phi ca ng suy gim nhanh chng, v gn nh m hn. Cng trong nm ny, nh ca
Euler b chy trong trong v i ho hon St.Peterburg vo thng nm thiu ri hn 5000 ngi
nh, Euler c cu bi ngi hu ca mnh l Peter Grimm.

Hnh 2: Euler c Peter cu thot t ngi nh ca mnh trong trn i ho.


B p li mt phn kh khn ca Euler, n hong Catherine cho xy dng li mt cn nh
khc cho Euler.
Thm mt ni au nm 1773; v ng, Katharina Gsell cht. Euler ti hn ba nm sau
khng phi ph thuc vo con ci ca mnh.
Trong hon cnh gp nhiu kh khn nh vy nhng Euler khng h nn ch, ng vn lm vic
v say sa nghin cu vi s gip t nhng ngi khc, u tin l t n hong Catherine,
sau l Niklaus Fuss, mt ngi ng hng ti t Thu S, chu r tng lai ca Euler v
ngi con trai ca mnh. Gn mt na cc cng trnh khoa hc, cc bi bo ca Euler c vit
trong qung thi gian St.Petersburg ln th hai ny.
Leonhard Euler cht v t qu ngy 18=9=1783 trong khi chi vi mt trong nhng a chu
ca mnh. Cng chnh vo ngy ny, trn hai phin ln, ng vit ra cng thc din gii
bn cht Ton hc c lin quan ti vic chuyn ng ca khinh kh cu m chuyn bay u tin
do hai em nh Montgolfier thc hin vo ngy 5=6=1783: l bi vit cui cng ca ng, v
chun b xut bn bi con trai ca ng l Johann Albrecht. Nhng vic xut bn cc cng trnh
cng nh bi vit ca Euler vn cn ko di sut 50 nm k t sau ngy ng mt.

67

Tp ch Epsilon, S 10, 08/2016

1
GI TR NO CHO 1 + 1 + 1 + ? + HAY ?
2
L Ngc Tu
Mathworks, Inc.

LI GII THIU CA BAN BIN TP


1
Tng ca dy v hn 1 + 1 + 1 + = hay ? Hn phn ln bn c s nhanh
2
1
chng tr li l , nhng liu cng l mt cu tr li ng? Xa hn na, tng v hn
2
1
1 1 + 1 1 + l , l 0 hay ? Hn nhiu bn c s cn ngc nhin hn na
12
1
khi chng ti ni rng con s
khng ch khng v l m kt qu ny li lin quan
12
n l thuyt dy trong Vt l!
lm r vn th v ny, chuyn mc Ton hc Gii tr trn trng gii thiu n bn
c lot bi vit v cc chui v hn t tc gi L Ngc Tu.

1. Gii thiu
Sau khi bit c cc tnh cht ca cc php tnh c bn nh: cng, tr, nhn, chia vi 2 s thc
v th t thc hin, chng ta c th m rng ra tnh c gi tr ca mt biu thc bao gm
hu hn cc php ton trn. Bc pht trin t nhin tip theo s l tm cch gn gi tr cho mt
biu thc vi v s (m c) cc php tnh c bn, m trong y, dng biu thc n gin
nht ch bao gm php cng hoc tr, c gi l chui1 :
a0 + a1 + a2 + =

an

(1.1)

n=0

vi a0 , a1 , a2 , ... R l cc s thc.
Vic gn gi tr cho nhng chui s c thc hin bi cc nh khoa hc t cch y hn
2000 nm. Cch tnh gi tr ca nhng chui cp s nhn chng hn nh:
 n
X
1
1 1 1
= 1 + + + + = 2
2
2 4 8
n=0
1

series

68

Tp ch Epsilon, S 10, 08/2016

c bit n trong cc ghi chp t thi Hy Lp c i. Tuy nhin, mi n tn th k 17


v 18 vi s ra i ca gii tch, o hm v tch phn, vic tnh ton vi cc chui v hn mi
tr nn ph bin. Nhiu phng php bin i v gn gi tr cho chui c a ra, tuy nhin
cc kt qu t c li thng khng nht qun. Mc d cc nh ton hc ln thi y nh
Newton, Leibnitz, hay Euler dng nh bit c nhng chui no l an ton trong tnh ton,
h (c bit l Euler) vn s dng cc chui khng an ton thu c nhiu kt qu quan
trng. Nhng kt qu ny sau y c xc nhn li bng cc phng php khc mt cch
c lp.
Mi n tn th k 19, nh ngha chnh xc v tng qut v chui hi t mi c a ra bi
Cauchy [1]. nh ngha ca Cauchy, cng vi m rng gii tch2 cho n hin nay c th ni l
phng php chnh thng c s dng rng ri nht trong tnh ton chui v hm s. Tuy nhin
khng phi cc chui khng hi t theo Cauchy (gi l chui phn k) u v dng. C nhiu
phng php tnh ton c th c p dng cho cc chui phn k, gi chung l cc phng
php tnh tng3 . L thuyt tng qut v cc phng php tnh tng c xy dng v hon thin
vo cui th k 19, u th k 20, v hin nay nhng phng php ny c ng dng ph bin
trong l thuyt s v vt l.
Trong phn u ca lot bi v chui v hn ny, chng ta s gii thiu v 2 phng php ly
tng ca Cauchy v Abel, v mt s tnh cht mong mun cho phng php ly tng.

2. Chui hnh thc v Chui ly tha hnh thc


Vi mi dy s v hn (a0 , a1 , a2 , ...), ngoi chui (1.1), chng ta cn quan tm n dng chui
ly tha vi h s (a0 , a1 , a2 , ...) vi tm c nh sau:

X
n=0

an (x c)n = a0 + a1 (x c) + a2 (x c)2 + ...

Khi cha c gn gi tr, cc chui

an v

n=0

X
n=0

(2.1)

an (x c)n c to bi dy (a0 , a1 , ...) cn

c gi l cc chui hnh thc4 v chui ly tha hnh thc5 . Thng qua php gn cc chui /
chui ly tha vi dy cc h s:

X
n=0

X
n=0

an (a0 , a1 , ...),

an (x c)n (a0 , a1 , ...),

analytic continuation
summation methods
4
formal series
5
formal power series
3

69

Tp ch Epsilon, S 10, 08/2016

chng ta c c mi tng quan 1-1 gia khng gian cc chui hnh thc v khng gian cc
dy v hn m c6 . Thng qua mi tng quan ny, khng gian cc chui hnh thc / chui
ly tha hnh thc c th c thng hng cu trc ca mt khng gian vc t vi s chiu
l v hn m c, m trong y, cc php cng v nhn vi mt s thc c thc hin theo
tng thnh phn:

an +

n=0

bn =

n=0

X
(an + bn ),
n=0

an =

n=0

can .

n=0

Ngoi cu trc khng gian vc t ra, khng gian cc dy cn c php nhn tng phn t:
(a0 , a1 , a2 , ...) (b0 , b1 , b2 , ...) = (a0 b0 , a1 b1 , a2 b2 , ...),
tuy nhin nu chng ta a php nhn ny sang khng gian cc chui, kt qu thu c s khng
ging nh m rng ca tch ca 2 tng hu hn.
Cch m rng t nhin ca php nhn vi 2 tng hu hn ra chui v hn c nh ngha nh
sau:
!
!
!

X
X
X
X
ai bni .
bn :=
an
n=0

n=0

n=0

i=0

Tng t nh vy, cc chui ly tha hnh thc c mt php nhn t nhin c thng hng
/ m rng t php nhn a thc:
!
!
!

X
X
X
X
an (x c)n
bn (x c)n =
ai bni (x c)n .
n=0

n=0

n=0

i=0

Vi php nhn ny, tp cc chui ly tha hnh thc tr thnh mt vnh giao hon, thng c
k hiu bi R[[x c]]. Vnh cc chui ly tha hnh thc c th c k hiu bi R[[1]].
Bi tp 1. Tm phn t n v 1R[[x]] sao cho vi mi

an x n ,

n=0

1R[[x]]

an x

n=0

an x n .

n=0

Nhc li phn t a ca mt vnh giao hon R c gi l kh nghch7 nu nh tn ti b R sao


cho a b = 1R .
Bi tp 2. Chng minh rng mt chui hnh thc

an l mt phn t kh nghch khi v ch

n=0

khi an 6= 0 vi mi n N.
6

c th c xem nh l khng gian cc hm s RN := {f : N R}


invertible

70

Tp ch Epsilon, S 10, 08/2016

Bi tp 3. Chng minh rng mt chui ly tha hnh thc

X
n=0

v (kh nghch) khi v ch khi a0 6= 0.

an (x c)n l mt phn t n

Bi tp 4. Tm dy (an ) sao cho:

xn

n=0

an x n

n=0

= 1R[[x]] .

3. Chui hi t theo Cauchy


Vi mi chui hnh thc

an , chng ta c th xy dng dy cc tng tng phn (sn ) nh sau:

n=0

s 0 = a0 ,
s 1 = a0 + a1 ,
s 2 = a0 + a1 + a2 ,
..
.
s n = a0 + a1 + a2 + + an
..
.
nh ngha 5. Nu nh dy sn hi t v mt s thc s, k hiu l sn s hoc lim sn =
n

X
an l mt chui hi t, v gn gi tr s cho chui
lim sn = lim sn = s, chng ta s gi chui
n

X
n=0

an . Nu

n=0

an c gi l mt chui phn k nu nh n khng hi t.

n=0

y l cch gn gi tr cho chui ph bin nht, thng c k hiu bi

an = s. Tuy nhin,

n=0

trong lot bi ny, tin cho vic phn bit gia chui hnh thc v cc cch tnh tng khc
nhau, chng ta s k hiu cch gn gi tr cho chui hnh thc ny l:
!

n
X
X
(C)
an = lim sn = lim
ai = s,
n

n=0

i=0

theo tn ca Cauchy.

Bi tp 6. Chng minh rng nu nh

X
n=0

th:
(C) c

X
n=0

an + d

X
n=0

bn

an v

bn l 2 chui hi t, c, d l 2 s thc bt k,

n=0

= c (C)
71

X
n=0

an

+ d (C)

X
n=0

bn .

Tp ch Epsilon, S 10, 08/2016

Bi tp trn cho ta thy:


1. Tp cc chui hi t l mt khng gian vc t con.
2. Cch gn gi tr (C) l mt hm tuyn tnh t khng gian cc chui hi t vo R.
!
!

X
X
X
X
Bi tp 7. Tm 2 chui hi t
an v
bn sao cho chui
an
bn khng hi
n=0

t.

n=0

n=0

n=0

Nh vy tp cc chui hnh thc hi t khng phi l mt tp ng i vi php nhn, hay ni


mt cch khc, cch gn gi tr (C) khng tng thch vi php nhn cc chui hnh thc. Tuy

X
X
vy, nu nh an , bn u l cc s khng m, v cc chui
an v
bn hi t th tch ca
n=0

n=0

chng cng l mt chui hi t. Tnh cht c chng minh bi Cauchy vi nh ngha v kt


qu sau:
nh ngha 8. Chui

an c gi l hi t tuyt i8 nu nh chui

n=0

nh l 9.

(1)

an hi t tuyt i =

n=0

(2)

an v

n=0

bn hi t tuyt i =

n=0

X
n=0

|an | hi t.

an hi t.

n=0

an

n=0

X
n=0

bn

hi t tuyt i.

Hay ni mt cch khc, tp cc chui hi t tuyt i to thnh mt vnh con ng thi cng l
mt khng gian tuyn tnh con (hay cn gi l mt R-i s con) ca tp cc chui hnh thc.

4. Chui ly tha hi t v phng php ca Abel


nh ngha 10. Mt chui ly tha hnh thc
nh chui

X
n=0

X
n=0

an (x c)n c gi l hi t ti z R nu

an (z c) hi t.

C th d dng thy c rng cc chui ly tha c tm ti c u hi t ti c. Hn na, nh l


sau ch ra rng cc chui ly tha ch hi t trong mt on thng c tm c (bn knh c th
l +):
8

absolutely convergent

72

Tp ch Epsilon, S 10, 08/2016

nh l 11. Vi mi chui ly tha

n
X
n=0

an (x c)n , t (cho php bng +):

R=

(1) Vi mi |z a| < R, chui ly tha


(2) Vi mi |z a| < R, chui ly tha

1
lim sup|an |1/n

X
n=0

X
n=0

an (x c)n hi t tuyt i ti z,
an (x c)n phn k ti z.

S R c nh ngha nh trn c gi l bn knh hi t ca chui

X
n=0

 
a0 a1 a2
Bi tp 12. Chng minh rng nu dy , , , ... hi t, th
a1 a2 a3


an
.
R = lim
n an+1

Bi tp 13. Chng minh rng nu nh c 2 chui

n
X

n
X

bn (x c)n u c
an (x c) v
n=0
! n=0 n
!
n
X
X
an (x c)n +
an (x c)n v
n

bn knh hi t r > 0, th c 2 chui ly tha:


n=0
!
!
n
n
X
X
an (x c)n
an (x c)n u c bn knh hi t r.
n=0

an (x c)n .

n=0

n=0

Quan st k mt t, chng ta c th thy rng khi x c = 1, chui ly tha


tr thnh chui

X
n=0

lim

x1

th chng ta gi chui

an (x c)n s

an xn . Nu

n=0

an x n ,

n=0

an l Abel-kh tng9 (hoc (A)-kh tng), v gn gi tr ca gii hn

n=0

(A)

X
n=0

n=0

an . cho n gin, chng ta xt chui ly tha c tm 0

nh chui ly tha ny c bn knh hi t R 1, v tn ti gii hn:

ny cho chui:

an := lim
x1

Abel summable

73

X
n=0

an x n .

Tp ch Epsilon, S 10, 08/2016

Bi tp 14. Chng minh rng nu nh

an v

n=0

thc bt k, th:
(A) c

an + d

n=0

bn

n=0

bn l 2 chui Abel-kh tng, c, d l 2 s

n=0

= c (A)

an

n=0

+ d (A)

bn .

n=0

5. Chui phn k - Mt s tnh cht mong mun


Tr li vi v d u bi, xt chui

1 = 1 + 1 + 1 + ..., dy cc tng tng phn c th d

n=0

dng tnh ra c:

(s0 , s1 , s2 , ..., sn , ...) = (1, 2, 3, ..., (n + 1), ...),


v nh vy chui 1 + 1 + 1 + ... l phn k theo cch gn gi tr ca Cauchy:
lim sn = lim (n + 1) = .

Bi tp 15. Chng minh rng chui 1 + 1 + 1 + ... khng phi l mt chui Abel kh tng.
Vy liu c tn ti m cch gn gi tr cho chui (T ) no sao cho (T )(1 + 1 + 1 + ...) l hu
hn? iu ny ty thuc vo vic chng ta mun (T ) s hu thm nhng tnh cht no khc
na.
Chng ta tm thi lit k mt s tnh cht m (T ) nn c nh sau:
(1) T l hm tuyn tnh.
X
X
(2) Nu nh chui
an hi t th
an (T )-kh tng.
n=0

n=0

(2*) (T ) ng vi (C): nu nh chui

an hi t th

n=0

(T )

X
n=0

(3) Nu nh chui

an

= (C)

X
n=0

an .

an l (T )-kh tng th:

n=0

(T )

X
n=0

an

= (T )(a0 + a1 + a2 + ...) = a0 + (T )(a1 + a2 + a3 + ...).

74

Tp ch Epsilon, S 10, 08/2016

Tnh cht (3) tng ng vi vic phng php tnh tng v hn (T ) khng v thuc vo mt
s hu hn cc s hng u tin, c th xem nh l mt m rng ca php cng hu hn s
hng. Tnh cht ny tuy c v hp l, nhng li c th dn n mt s kt lun c v v l nh
sau:
Nu nh (T ) gn mt gi tr hu hn s cho tng 1 + 1 + 1 + ..., ng thi tha mn cc tnh
cht (3), th:
s = (T )(1 + 1 + 1 + ...) = 1 + (T )(1 + 1 + 1 + ...) = 1 + s.
T ta suy ra:
(T )(1 + 1 + 1 + ...) = s = + (!!!).
Nh vy gn c gi tr hu hn cho chui (1 + 1 + 1 + ...), chng ta s mt i tnh cht (3).
Mt trong nhng phng php ph bin gn gi tr hu hn cho chui (1 + 1 + 1 + ...) l s
dng m rng gii tch10 ca chui:

ns = 1s + 2s + 3s + ...

n=1

V mt hnh thc,

n0 = 10 + 20 + 30 + ... = 1 + 1 + 1 + ...

n=1

Tuy nhin, chui

ns ch hi t tuyt i khi s > 1 v phn k khi s = 1. Kh khn ny

n=1

c gii quyt bng cch m rng ra khng gian s phc C thay cho s thc. Khi y, chui

X
ns l i din ca hm gii tch (s) trn tp {s C : <(s) > 1}. Hm ny c mt m
n=1

rng gii tch duy nht ra C r {1}, v chng ta c th gn gi tr ca hm ny ti 0 ny cho


chui (1 + 1 + 1 + ...). Chng ta s k hiu phng php tnh tng ny bi (E):
1
(E)(1 + 1 + 1 + ...) = (0) = .
2

Trong phn sau, chng ta s i su hn vo chi tit ca m rng gii tch, mt trong nhng khi
nim v phng php rt quan trng ca ton hc.

Ti liu
[1] A. L. Cauchy, Analyse algbrique, Paris (1821).
[2] J. B. Conway, Functions of One Complex Variable, Graduate Texts of Mathematics 11,
Springer-Verlag (1973).
[3] G. H. Hardy, Divergent series, Oxford (1949).
10

analytic continuation

75

Tp ch Epsilon, S 10, 08/2016

TIP NI CU CHUYN V MT TNG LY THA


Trnh o Chin
(Cao ng S phm Gia Lai)

Gi s n v k l cc s nguyn dng. Trong [2], tc gi Trn Nam Dng cp nhng iu


th v v tng
n
X
Sk (n) =
j k = 1k + 2k + + nk .
(1)
j=1

Trong [3], Nguyn Mnh Linh tip tc cp mt s tnh cht ca tng (1), bng cch tip cn
cc a thc Sk (x) xc nh bi cc h thc truy hi.
Tip ni mch suy ngh trn, bi vit ny cp n mt s trng hp c bit ca tng
Sk (n) =

n
X
j=1

xj k = x1 k + x2 k + + xn k ,

(2)

mt dng tng qut ha trc tip ca tng (1).


Trc ht, ta nhc li ng nht thc Newton, mt ng nht thc quan trng, lin quan n
tng (2).

1. ng nht thc Newton


ng nht thc Newton, ln u tin c nh ton hc Newton thit lp vo Th k 17. T
n nay, c nhiu cch chng minh cho ng nht thc ny.
Vi n = 3, ng nht thc Newton c thit lp nh sau: Gi s x1 , x2 , x3 R. Th th
x1 , x2 , x3 l nghim ca phng trnh
(x x1 ) (x x2 ) (x x3 ) = 0,
hay
x3 (x1 + x2 + x3 ) x2 + (x1 x2 + x2 x3 + x3 x1 ) x x1 x2 x3 = 0.

76

Tp ch Epsilon, S 10, 08/2016

c1 = (x1 + x2 + x3 )
c2 = x 1 x 2 + x 2 x 3 + x 3 x 1

c3 = x1 x2 x3

Th th x1 , x2 , x3 l nghim ca phng trnh x3 + c1 x2 + c2 x + c3 = 0. t Sk = x1 k + x2 k +


x3 k , k = 1, 2, 3, 4, . . . Th th ta c ng nht thc sau

S1 + c1 = 0,

S2 + S1 c1 + 2c2 = 0,

S3 + S2 c1 + S1 c2 + 3c3 = 0,
S4 + S3 c1 + S2 c2 + S1 c3 = 0,

S5 + S4 c1 + S3 c2 + S2 c3 = 0,

...

Sk + Sk1 c1 + Sk2 c2 + Sk3 c3 = 0, k > 4.


ng nht thc trn c gi l ng nht thc Newton, trong trng hp n = 3.

T khi c thit lp, c nhiu cch chng minh cho ng nht thc Newton. Vi nhng gi
tr u tin ca s nguyn dng n, ng nht thc Newton c th c kim tra mt cch d
dng. Tuy nhin, trong trng hp n tng qut, cc cch chng minh u phi dng n vic
khai trin chui Taylor i vi mt s hm s quen thuc, trong khi khi nim chui v vic
khai trin ny cha c ph thng.
Di y l mt trong nhng phng php thit lp ng nht thc Newton trong trng hp
n = 3 v trng hp n bt k.

1.1. Thit lp ng nht thc Newton, trng hp n = 3


Ta c
(x x1 ) (x x2 ) (x x3 ) = x3 + c1 x2 + c2 x + c3 .

(3)

1
, ta c
x




1
1
1
1
c1
c2
x1
x2
x 3 = 3 + 2 + + c3 ,
x
x
x
x
x
x

Trong (3), thay x bi

hay

1
c1
c2
(1 x1 x) (1 x2 x) (1 x3 x)
= 3 + 2 + + c3 .
3
x
x
x
x
Mt cch tng ng, ta c
(1 x1 x) (1 x2 x) (1 x3 x) = 1 + c1 x + c2 x2 + c3 x3 .
77

(4)

Tp ch Epsilon, S 10, 08/2016

By gi, xt chui ly tha hnh thc

Sk xk .

k=1

Lu rng, bi khai trin Taylor, ta c

xk =

k=0

1
.
1x

Suy ra rng

1+

xk =

k=1

hay

xk =

k=1

1
1x

x
.
1x

(5)

Do , bi (4) v (5), ta c

Sk x =

k=1

k=1

x1 + x2 + x3
k

(x1 x) +

x =

(x2 x)k +


X

k=1

(x1 x) + (x2 x) + (x3 x)

(x3 x)k =

k=1

k=1

k=1

x1 x
x2 x
x3 x
+
+
1 x1 x 1 x2 x 1 x3 x

d
((1 x1 x) (1 x2 x) (1 x3 x))
c1 + 2c2 x + 3c3 x2
dx
= x
= x
(1 x1 x) (1 x2 x) (1 x3 x)
1 + c1 x + c2 x 2 + c3 x 3
c1 x + 2c2 x2 + 3c3 x3
.
=
1 + c1 x + c2 x 2 + c3 x 3
Suy ra rng

Sk xk

k=1

hay

Sk xk +

Ni cch khc, ta c

k=1



1 + c1 x + c2 x2 + c3 x3 = c1 x + 2c2 x2 + 3c3 x3 ,
Sk xk

k=1

k=1

Sk xk =

X
k=1

Sk xk



c1 x + c2 x2 + c3 x3 = c1 x + 2c2 x2 + 3c3 x3 .
!



c1 x + c2 x2 + c3 x3 c1 x + 2c2 x2 + 3c3 x3 .
78

(6)

Tp ch Epsilon, S 10, 08/2016

V tri ca (6) c khai trin nh sau


S1 x + S2 x2 + S3 x3 + S4 x4 + S5 x5 +

(7)

V phi ca (6) c vit li di dng


c1 x (S1 c1 + 2c2 ) x2 (S2 c1 + S1 c2 + 3c3 ) x3
(S3 c1 + S2 c2 + S1 c3 ) x4 (S4 c1 + S3 c2 + S2 c3 ) x5
So snh cc h s tng ng ca (7) v (8), ta thu c

hay

S1 = c1 ,

S2 = S1 c1 2c2 ,

S3 = S2 c1 S1 c2 3c3 ,
S4 = S3 c1 S2 c2 S1 c3 ,

S5 = S4 c1 S3 c2 S2 c3 ,

...

Sk = Sk1 c1 Sk2 c2 Sk3 c3 , k > 4.

S1 + c1 = 0,

S2 + S1 c1 + 2c2 = 0,

S3 + S2 c1 + S1 c2 + 3c3 = 0,
S4 + S3 c1 + S2 c2 + S1 c3 = 0,

S5 + S4 c1 + S3 c2 + S2 c3 = 0,

...

Sk + Sk1 c1 + Sk2 c2 + Sk3 c3 = 0, k > 4.

1.2. ng nht thc Newton, trng hp tng qut


Gi s x1 , x2 , . . . , xn l n s thc xc nh. t

c1 = (x1 + x2 + + xn )

c2 = x1 x2 + x1 x3 + + xn1 xn
c3 = (x1 x2 x3 + x1 x2 x4 + + xn2 xn1 xn )

...

c = (1)n x x x x
n
1 2 3
n
79

(8)

Tp ch Epsilon, S 10, 08/2016

v
Sk = x1 k + x2 k + + xn k ; k = 1, 2, 3, . . . , n, n + 1, n + 2, . . .
Ta c ng nht thc sau

S1 + c1 = 0,

S2 + S1 c1 + 2c2 = 0,
...

Sn + Sn1 c1 + + S1 cn1 + ncn = 0,

S + S c + + S c
k
k1 1
1 k1 = 0, k > n + 1.

ng nht thc trn c gi l ng nht thc Newton.

Da vo cch thit lp ng nht thc Newton trong trng hp n = 3, ta c cch chng minh
sau y cho ng nht thc Newton, trong trng hp n bt k.
Ta c

n
Y
i=1

Trong (9), thay x bi

(x xi ) = x +

n1
X

cni xi .

(9)

i=0

1
, ta c
x
n 
Y
1

X cni
1
,
= n+
x
xi
i=0

(1 xi x)

X cni
1
= n+
.
x
xi
i=0

i=1

hay

n
Q

i=1

xi

xn

Quy ng v rt gn, ta c
n
Y
i=1

n1

n1

(1 xi x) = 1 +

By gi, xt chui ly tha hnh thc

Sk xk .

k=1

80

n
X
i=1

ci x i .

(10)

Tp ch Epsilon, S 10, 08/2016

Bi (10), ta c

Sk xk =

k=1

n
X
X
i=1

k=1

n
X
i=1

x
=

xi k

xk =

ici xi1

i=1

1+

n
P

ci

xi

d
dx

n
Q

n
P

k=1

X
k

Sk x

Sk x +

k=1

Ni cch khc, ta c

X
k=1



n
P
i
1+
ci x
i=1

1+

n
P

ci x i

i=1

.
ci

S k xk =

1+

Sk x =

ici xi

i=1

1+

n
X

ci x

i=1

Sk x

k=1

n
P

n
P

.
ci x i

i=1

k=1

= x

d
dx

i=1

hay

k=1

xi

Tm li, ta c

Suy ra rng

i=1

(xi x)k

ici xi

i=1

1+

X
X

(1 xi x)

i=1

i=1


(1 xi x)

n
Q

i=1

n
P

(xi x)k

i=1

k=1

xi x
= x
1 xi x
n
P

n
X
X

X
k=1

Sk x

n
X

ci x

i=1

n
X

n
X
i=1

i=1

ci x

ici xi

n
X

ici xi .

i=1

n
X

ici xi .

(11)

i=1

Tng t cc bc nh trong trng hp n = 3, so snh cc h s tng ng 2 v ca (11), ta


thu c ng nht thc Newton.

2. Bi ton lin quan n tng Sk (n)


Nhng bi ton lin quan n tng (2) thng l nhng bi ton kh. Di y l mt trong
nhiu bi ton minh ha cho nhn nh ny, m gi thit c "gim nh" i rt nhiu.
Bi ton 1. Gi s x1 , x2 , . . . , xn l n s nguyn dng ty , xc nh trc. t
Sk = x1 k + x2 k + + xn k , k = 1, 2, 3, . . . , n, n + 1, n + 2, . . .
Bit rng Sk = k, vi k = 1, 2, 3, . . . , n. Tnh Sn+1 .
81

Tp ch Epsilon, S 10, 08/2016

Li gii. K hiu

1 = x1 + x2 + ... + xn ,

2 = x1 x2 + x1 x3 + ... + xn1 xn ,
3 = x1 x2 x3 + x1 x2 x4 + ... + xn2 xn1 xn ,

...

= x x x ...x .
n
1 2 3
n

Xt a thc

f (x) =

n
Y
i=1

trong

(x xi ) = x +

n
X

cj xnj ,

j=1

cj = (1)j j , j = 1, 2, ..., n.

Theo ng nht thc Newton, ta c


S1 + c1 = 0 1 1 = 0 1 = 1
v, vi mi k = 2, 3, ..., n
Sk + Sk1 c1 + Sk2 c2 + ... + S2 ck2 + S1 ck1 + kck = 0
Sk +

k1
X
j=1

Skj cj + kck = 0 k +

k1
X
j=1

(k j) cj + kck = 0

k1
X

k1

1X
kck = k
(k j) cj ck = 1
(k j) cj
k j=1
j=1
k1

(1)k k = 1
Vy

=1

k+1

= (1)

1X
(k j) (1)j j .
k j=1

k1
(1)k X
+
(1)j+1 (k j) j , k = 2, 3, ..., n
k j=1

Cng theo ng nht thc Newton, ta c


Sn+1 + Sn c1 + Sn1 c2 + ... + S2 cn1 + S1 cn = 0 Sn+1 +
Sn+1 +

n
X
j=1

(n j + 1)(1) j = 0 Sn+1 =

82

n
X
j=1

n
X
j=1

Sn j + 1 cj = 0

(n j + 1)(1)j j .

(12)

Tp ch Epsilon, S 10, 08/2016

Suy ra
Sn+1 =

n
X
j=1

(1)j+1 (n j + 1)j .

(13)

Lu rng, gi tr ca cc k khng ph thuc vo bin x. Do


n = (1)

n+1

(1)n
+
Sn .
n

(14)

Bi (13) v (14), ta c
Sn+1

(1)j
=
(n j + 1) .(1)j+1 . (1)j+1 +
Sj
j
j=1
n
P

Suy ra
Sn+1



Sj
=
(n j + 1) . 1
, n > 1,
j
j=1

vi S1 = 0.

n
X

(15)

Ngoi ra, (15) cn c vit li di dng


Sn+1

n1
X

Sj
=
(n j + 1) . 1
j
j=1

vi S1 = 0, S2 = 1.

Sn
+ 1
n

, n > 2,

Vi n > 2, t (16) , ln lt thay ch s n + 1 bi n, n 1, ta c




n1
X
Sj
Sn =
(n j) . 1
,
j
j=1

(16)

(17)


 n2


P
Sj
Sj
Sn1 =
(n j 1) . 1
=
(n j 1) . 1
+0
j
j
j=1
j=1



n2
P
Sj
Sn1
=
(n j 1) . 1
+ (n (n 1) 1) 1
.
j
n1
j=1
n2
P

Suy ra

Sn1

n1
X

Sj
=
(n j 1) . 1
j
j=1

Bi (16), (17), (18), ta c



 

Sn
Sj
Sn+1 +Sn1 =
2 (n j) . 1
+ 1
j
n
j=1

 


n1
P
Sj
Sn
Sn
2n 1
=2
(n j) . 1
+ 1
= 2Sn + 1
=1+
Sn .
j
n
n
n
j=1
n1
P

83

(18)

Tp ch Epsilon, S 10, 08/2016

Do , ta thu c phng trnh sai phn tuyn tnh cp hai, vi h s bin thin
(
2n 1
Sn+1
Sn + Sn1 = 1, n > 2
n
S1 = 0, S2 = 1.

(19)

gii phng trnh sai phn ny, ta xt hm sinh sau


g (x) =

Sn+1 xn .

n=1

Khi , bi (19), ta c
x
(1 x)2 .g (x) =

1x

Zx

g (t)dt.

(20)

Ly o hm, theo tng v ca (20), ta c


2 (1 x) .g (x) + (1 x)2 .g 0 (x) =

1
(g (x) g (0)) ,
(1 x)2

(21)

vi g(0) = 0.
Rt gn (21), ta c phng trnh vi phn tuyn tnh cp mt

1
(1 x)2 .g 0 (x) + (2x 1) .g (x) =
(1 x)2

g (0) = 0.

Gii phng trnh vi phn (22), ta c nghim

x
1e x1
g (x) =
.
(1 x)2

Khai trin hm ny theo Cng thc Taylor i vi chui ly tha, ta thu c


Sn+1 =

n
X
(1)k1

k!

k=1

Bi ton c gii quyt.

84

k+1
Cn+1
.

(22)

Tp ch Epsilon, S 10, 08/2016

3. Mt s ng nht thc lng gic lin quan n tng


Sk (n)
Trong phn ny, ta xt mt trng hp c bit ca tng (2), vi n l s nguyn dng ty v
xj l hm cosin theo bin j. C th tng t i vi nhng hm lng gic khc. Nhng kt qu
tm c l nhng ng nht thc lng gic thng gp trong chng trnh ton ph thng.
Cc kt qu di y s gip ta tnh c tng hu hn ca mt s hm cosin. Chng hn, tng
sau y

n 1

2
 
X
j
k
cos
,
(23)
n
j=1
trong bxc l k hiu ca s nguyn ln nht khng vt qu x.

Trc ht, cn lu mt kt qu quan trng sau m chng minh n c th xem, chng hn, trong
[5].
B 1. Nu f (x) = a0 + a1 x + +a2 x2 + ... + an xn + ... l hu hn hoc chui v hn hi
t, th vi 0 6 r < n, tng ar xr + ar+n xr+n + ar+2n xr+2n + ... c xc nh bi cng thc
X

n1

ar+jn xr+jn =

j>0

2i
trong z = e n l nghim th n ca 1.


1 X jr
z f zj x ,
n j=0

(24)

Ta chng minh cc kt qu sau


nh l 1. Gi s n v k l hai s nguyn dng. Th th

2
X

cos2k

j=1

j
n

1
n
= + 2k+1
2 2

n
X


k

j=

k+jn
C2k
.

 
k
Chng minh. p dng cng thc (24), vi r = k n
v f (x) = (1 + x)2k .
n
Khi , vi x = 1, ta c

k

n
X


k

j=

n1

k+jn
C2k

1X
=
z
n j=0


k

j kn

85

1 + zj

2k

(25)

Tp ch Epsilon, S 10, 08/2016

2i
trong z = e n .
Ta c
eir 1 + ei
=e

ir

ik
2

k

 i i

i
i k
= eir e 2 2 + e 2 + 2

i
2

+e

i
2

k

k
= 2k cosk ei( 2 r)
2

(26)

Hn na, ta c th vit
z


k

j kn

Khi , (25) tr thnh

1 + zj

n
X


k

j=

2k

k+jn
C2k

= 22k cos2k

j
n


k

i.2.j. .

 
n1
22k X 2k j
=
cos
.
n j=0
n

(27)

Bi tnh cht

(n j)
cos
n
nn nh l 1 c chng minh.

= cos

j
n

, J = 0, 1, 2, ..., n,

nh l 2. Gi s n v k l hai s nguyn dng sao cho n k (mod2). Th th

2
X

(1) cos

j=1

j
n

2n
X

1
n
= + k+1
2 2

j=

k+n

2n

Ck 2

+jn


1
.
trong [x] l k hiu ca s nguyn gn nht ca x, ngha l [x] = x +
2




k+n
k+n
Chng minh. p dng cng thc (24) vi r =
n
v f (x) = (1 + x)k .
2
2n
Khi , vi x = 1, ta c

+ n

2n
X

j=

k+n

+ n

2n

Ck 2

+jn

k + n k + n

n1
j
n
k
1X
2
2n
=
z
1 + zj ,
n j=0
86

Tp ch Epsilon, S 10, 08/2016

2i
trong z = e n .
Bi (26), ta suy ra rng

k + n k + n
  i.j.2 k + n 1

j
n
k
j
2
2n
2n
z
e
1 + z j = 2k cosk
.
n

Khi , ta c

k+n

2n
X

j=

k+n
+jn
2

Ck

k+n

2n

 
n1
2k X
j
j
k
=
(1) cos
.
n j=0
n

(28)

Mt khc, ta c

n1
X

(1) cos

j=0

1+

j
n

= 1+

2
X

(1) cos

j=1

2
X

(1) cos

j=1


j
+
n


j
+
n

2
X

n+kj

(1)

cos

j=1

nj

cos

(1)

j=1

2
X

j
n

= 1+2

2
X
j=1

(n j)
n


j
(1) cos
.
n
(29)
j

Bi (25) v (29), nh l 2 c chng minh.


nh l 3. Gi s n v k l hai s nguyn dng. Th th
n%
2
X

2k

cos

j=1

2j 1
.
n
2

1
n
= + 2k+1
2 2

n
X


k

j=

k+jn
(1)j C2k
.

Chng minh. chng minh nh l, ta s dng cng thc sau


$

n%
2
X
j=1

1
f (2j 1) =
2

n1
X
j=1

f (k)

87

n1
X
j=1

(1) f (k) ,

(30)

Tp ch Epsilon, S 10, 08/2016

vi f (k) = cos

2k


j
.
2n

Bi nh l 1 (nu thay n bi 2n) v nh l 2 (nu thay n v k bi 2n v 2k, theo th t), ta c

k
k
$ %

2n
2n
2

X
X
X
1 1
2n
1
2n
k+(2j+1)n
k+2jn
f (2j 1) = + 2k+1
C2k
C2k
+ 2k+1

2
2 2
2 2

k
k
j=1

j=
j=
2n
2n

22k+1

2n 
X

j=

k+2jn
C2k

k+(2j+1)n
C2k

2n

n
22k+1

k
bX
nc


k

j=

k+jn
.
(1)j C2k

nh l 3 c chng minh.
H qu 4. Gi s n l s nguyn dng. Th th

1.

2
P

cos

1
n
n
n
;
= + 2n + 2n+1 C2n
2 2
2

j
n

2j 1
.
n
2

2n

j=1

2.

j
n

2
P

(1) cos

j=1
$

3.

n%
2
P

cos

2n

j=1

n
1
= + n;
2 2

n
n
n
.
+ 2n+1 C2n
2n
2
2

H qu 5. Gi s n v k l hai s nguyn dng sao cho k < n. Th th

1.

2
P

cos

j=1

2.

3.

n%
2
P

j=1

1
n
k
= + 2k+1 C2k
;
2 2

j
n

2j 1
.
n
2

j
n

2
P

(1) cos

j=1
$

2k

cos

2k

1
= , vi n k (mod2) ;
2

n
22k+1

k
C2k
.

88

Tp ch Epsilon, S 10, 08/2016

Bi nh l 1, nh l 2, nh l 3, ta thu c mt s ng nht thc t hp sau


H qu 6. Gi s k l s nguyn dng. Th th
1.

k
P

j=0

kj
k
+ 22k1 ;
C2k
= 21 C2k

2.

3.

2
P

j=0

3
P

1 + 22k1
1 k
k3j
= C2k
C2k
+
;
2
3

4
P

1 k
k4j
+ 22k3 + 2k2 ;
C2k
= C2k
2

j=0

4.


k


j=0

5.

5
P

j=0

6.

k2j
k
= 12 C2k
+ 22k2 ;
C2k

6
P

j=0

k
k
5 + 3 5 + 23k1
;
5.2k

k5j
C2k

3+
1 k
= C2k
+
2

k6j
C2k

1 k
3k + 22k1 + 1
= C2k +
.
2
6

H qu 7. Gi s k l s nguyn dng. Th th
1.

k
P

j=1

2.

kj
C2k1
= 4k1 ;

3
P

j=1

3.

5
P

j=1

k3j
C2k3
=

k5j
C2k5

4k2 1
, k > 1;
3

4k3
=

5+1

2k5

51
, k > 2.
5.22k5

2k5

H qu 8. Gi s k l s nguyn dng. Th th
1.

k
P

j=0

kj
k
(1)j C2k
= 21 C2k
;

89

Tp ch Epsilon, S 10, 08/2016

2.

2
P

1 k
k2j
+ 2k1 ;
(1)j C2k
= C2k
2

3
P

1 k
k3j
+ 3k1 ;
(1)j C2k
= C2k
2

4
P

2+
1 k
+
= C2k
2

k
k
2 + 2 2
;
4

5+
1 k
= C2k
+
2

k
k
5 + 5 5
.
5.2k

j=0

3.

4.

5.

j=0

j=0

5
P

j=0

k4j
(1)j C2k

(1)

k5j
C2k

Ti liu
[1] Trnh o Chin, a thc ni suy Lagrange v ng nht thc Newton di gc nhn ca
ton ph thng, K yu hi tho khoa hc Nghin cu v Ging dy Ton hc ph thng,
Trng i hc Khoa hc T nhin Thnh ph H Ch Minh, 8-9/8/2014.
[2] Trn Nam Dng, Nhng iu th v v tng Pk (n) =

n
P

j=1

j k = 1k + 2k + + nk , Cc

phng php gii ton qua cc k thi Olympic, Nh xut bn i hc Quc gia Thnh ph
H Ch Minh, 2013.
[3] Nguyn Mnh Linh, Mt s vn xung quanh tng ly tha, K yu Gp g ton hc,
Vng Tu, 7/2015.
[4] Mircea Merca, A note on cosine power sums, Article 12.5.3 Journal of Integer Sequences,
Vol. 15 (2012).
[5] J. Riordan, Combinatorial Identities, John Wiley and Sons, 1968.

90

Tp ch Epsilon, S 10, 08/2016

V MT TON HAY TRN TP CH THTT


Trn Quang Hng, THPT chuyn KHTN, H Ni
Nguyn c Bo, THPT chuyn Phan Bi Chu, Ngh An

TM TT
Bi vit a ra cch chng minh v cc pht trin cho bi ton T12/465 ca tp ch Ton
hc v tui tr, ng thi cng ng dng cc bi ton vo cc bi ton khc nhau.

1. Mt s bi ton m u
Trn bo THTT s 465 [1] c bi ton T12 ca thy Nguyn Xun Hng nh sau
Bi ton 1. Cho tam gic ABC cn ti A c ng cao CD. Gi E l trung im ca BD, M
l trung im CE, phn gic ca BDC ct CE ti P . ng trn tm C bn knh CD ct
AC ti Q. Gi K l giao im ca P Q v AM . Chng minh rng tam gic CKD vung.

A
Q

D
E
P

M
C

Hnh 1.
Bin tp: Ng Quang Dng

91

Tp ch Epsilon, S 10, 08/2016

y l bi ton th v trn mt m hnh khng i xng. Bi ton c gii trn THTT s 469
[1] bi tc gi Nguyn c Bo. Chng ti s a ra thm cc khai thc v li gii trn m
hnh ca bi ton ny. Ta xt bi ton sau
Bi ton 2. Cho tam gic ABC cn ti A c ng cao CD. Gi E l trung im ca BD, M
l trung im CE, phn gic ca BDC ct CE ti P . ng trn tm C bn knh CD ct
AC ti Q. Gi K l giao im ca P Q v AM . F l hnh chiu vung gc ca D ln BC.
a) Chng minh rng tam gic CKD vung.
b) Ly S thuc P F sao cho ES k AM . Chng minh rng CS = CD.
c) Chng minh rng BP v ES ct nhau trn (C).

G
L K

Y
Z

I
J

E
H
B

M
X

F
S

T
R
Hnh 2.
Li gii. a) Gi T l giao ca DP vi ng trn tm C bn knh CD. Do DT l phn gic
CDE v BD l tip tuyn ca ng trn tm (C) nn DCT = 2DBT = 90 nn

92

Tp ch Epsilon, S 10, 08/2016

CT DC. Li c tam gic CDQ cn ti C nn ta c bin i gc


CQT = DQC 45 = 90

DCQ
90 BAC
BAC
45 = 45
=
2
2
2

do QT BC. Tam gic DEF v tam gic T CQ c cc cnh tng ng song song nn DT ,
F Q v CE ng quy ti P . Mt khc do EF k AC nn nu N l i xng ca F qua M th N
thuc AC. p dng nh l Menelaus cho tam gic QF N vi M , K, A thng hng, ta c
KQ
AQ
KQ AN M F

= 1 t
=
KF AQ M N
KF
AN
Gi BQ ct CK ti L. p dng nh l Menelaus cho tam gic QBF vi L, K, C thng hng
th
LB KQ CF

=1
LQ KF CB
vi ch EF k AC nn

CF
CB

AE
AB

v AN = AE ta thu c

LB
KF CB
AN AB
AB
=

=
LQ
KQ CF
AQ AE
AQ
Do vy AL l phn gic QAB nn LB = LC. Theo h thc lng trong tam gic vung th
CQ2 = CD2 = CF.CB suy ra 4CQF 4CBQ(cnh - gc - cnh) nn F QC = QBC =
LCB. T 4CF Q 4KF C (gc - gc) suy ra F KC = F CQ = DBC = F DC,
vy KCF D l t gic ni tip hay CKD = 90 .
b) Ly R i xng A qua M , do EF k AC v AE k CT nn d thy R nm trn EF v CT .
FE
FB
FS
=
=
SB k CK
FK
FR
FC
T chng minh trn ta c 4KF C 4CF Q 4CQB. Do

KF
KC

CQ
CB

CD
CB

KF 2
CD2
CF
KF
=
=
=
KC 2
CB 2
CB
KS
Suy ra KC 2 = KF.KS nn KSC = KCF = CBS. Vy CS 2 = CF.CB = CD2 nn
ta kt lun CS = CD.
c) AM ct BC ti X. p dng nh l Menelaus cho tam gic BCE vi A, M , X thng hng,
ta suy ra XB
= AB
= BC
. Gi T G l ng knh ca (C, CD). D thy B, P , G thng hng
XC
AE
CF
v tam gic CGQ cn ti C li c CG k AB nn QG k BC. BG ct CA ti Y .
YQ
QG
CQ
CD
=
=
=
YC
BC
CA
AC
QC
CD + AC
YC
CD
=

=
YC
AC
AC
CD + AC
Do vy YY CA = CD
. Gi I l trung im ca F Q. CI ct BQ ti Z. p dng nh l Menelaus
AC
ZQ
CF
= CB
. CZ ct AB ti O, p dng Menelaus
cho tam gic QBF vi Z, I, C thng hng th ZB
cho tam gic AQB vi O, I, C thng hng th

OB
CQ ZB
CD CF
=

OA
CA ZQ
AC CB
93

Tp ch Epsilon, S 10, 08/2016

XB Y C OA
BC CD CD.CF

=1
XC Y A OB
CF AC AC.CB
Nn AX, BY , CZ ng quy ti J. CL ct AB ti U . D thy QU k BC nn BU GC l hnh
bnh hnh nn BG i qua trung im W ca U C. Ta ch cc tam gic ng dng v bng
nhau 4CF Q 4CQB = 4BU C. M CI, BW l trung tuyn tng ng ca hai tam gic
CF Q v BU C nn ICQ = W BC = W GQ t gic CGQJ ni tip. Cng vi

KQU = QF C = BQC = QBA + QAB = KCA + QCG = KCG


nn t gic QKCG ni tip. Vy C, J, K, Q, G ng vin. BG ct (C, CD) ti H khc G. Ta
c HSQ = HGQ = SKJ. T HS k AM k ES nn H thuc ES.
Bn Xun Long lp 10 Ton, THPT chuyn KHTN xut mt li gii khc cho c) nh
sau
Qua C v CL k AB, khng kh thy rng L thuc BP v QL k BC. Gi QL ct KC
T th v tam gic DKC vung ti K v ta c T KQ = QCB = CQL = CLQ nn K
thuc AB suy ra trung im R ca BL nm trn (KM F ). Gi H l giao im ca BP vi (C),
ta s chng minh E, H, S thng hng. V HF CL l t gic ni tip v ch rng BS k CK
y ta coi S l im sao cho ES k AM th theo phn b) ta c S thuc (C) nn BHF S l
t gic ni tp, do vy ta quy v chng minh CBH = DKP . S dng nh l Pascal o
cho P , M , C thng hng ta c AK v CH ct nhau ti J thuc ng trn (KRU F H), suy ra
JKF = JHF = F LC = CBL v ta c iu phi chng minh.

94

Tp ch Epsilon, S 10, 08/2016

R
D

E
P

M
J

H
B

S
Hnh 3.

Nhn xt. C ba ca bi ton ny thc cht l cc bi ton chng minh ct nhau trn ng
trn m hai ng thng khng xut pht t hai im nm trn ng trn ny. y l mt dng
ton kh i hi phi dng thm cc im nm trn ng trn m hai ng thng i qua.
Nh vy ngoi bi ton 1 l phn a), chng ta c th tch ring cc phn b), c) thnh cc bi
ton di y
Bi ton 3. Cho tam gic ABC cn ti A c ng cao CD. Gi E l trung im ca BD, M
l trung im CE, phn gic ca BDC ct CE ti P . F l hnh chiu ca D trn BC. Ly S
thuc P F sao cho ES k AM . Chng minh rng CS = CD.

95

Tp ch Epsilon, S 10, 08/2016

D
P

M
F

S
Hnh 4.
Cch pht biu sau ca phn c) cng c trong [3]
Bi ton 4. Cho tam gic ABC cn ti A v ng cao CD. E, M l trung im ca BD, CE.
DP l phn gic ca tam gic CDE. H thuc BP sao cho EH k AM . Chng minh rng
CH = CD.

D
E
H

B
Hnh 5.

Trong [3] c a ra mt cch tip cn kh n gin cho bi ton 4 ny l dng phng php ta
. Tuy nhin chng ti khng a li gii ny vo v li gii ta khng p. Cng nh vy
vi vi cc bi ton 1 v bi ton 3, chng ti cng c nhn xt rng phng php ta l kh
hu dng trong vic chng minh trc tip cc bi ton ny. Nu so snh chng vi cc li gii
thun ty hnh hc m chng ti trnh by trong bi ton 2 th phc tp ca li gii vi
phng php ta gim i ng k. Nhng v d ny lm cho chng ta phn no thy c
nhng li th rt ln t phng php ta , tuy nhin v tnh p mt ca li gii th chng ti
vn u tin trnh by cc cch tip cn thun ty hnh hc.

96

Tp ch Epsilon, S 10, 08/2016

2. Mt s ng dng
Phn ny s ng dng ch yu l cc bi ton 1, 3, 4 v cch chng minh tng minh ca ba bi
ton ny c trong bi ton 2.
Khi thay i cch pht biu ca bi ton 1, ta thu c bi ton sau
Bi ton 5. Cho tam gic ABC vung ti A v phn gic AD. M , N l trung im ca BC,
AC. Dng im P sao cho N M = N P v M P k BN . F thuc AB sao cho CF k N P . E
thuc CF sao cho CE = AC. DE ct F M ti G. Chng minh rng AGC = 90 .
Li gii sau m phng cch lm ca tc gi Nguyn c Bo.

L
B

P
M
H

T
Hnh 6.
Li gii 1. Ly Q i xng A qua B. Ta thy tam gic QF C v M N P c cnh tng ng
song song nn tam gic QF C cn ti F . Gi T l giao ca AD vi ng tn tm C bn knh
CA. Do AT l phn gic CAB v QA l tip tuyn ca ng trn tm (C) nn ACT =
2AQT = 90 nn CT AC. Li c tam gic CAE cn ti C nn ta c bin i gc
CET = 135 AEC = 135

90 ACE
QF C 90
= 45 +
= 90 F CQ
2
2
97

Tp ch Epsilon, S 10, 08/2016

do ET QC. Gi H l hnh chiu ca A ln QC d thy tam gic BQH cn nn BH k F C,


khi tam gic ABH v tam gic T CE c cc cnh tng ng song song nn AT, HE v CB
ng quy ti D. Mt khc do BH k F C nn nu R l i xng ca H qua M th R thuc F C.
p dng nh l Menelaus cho tam gic EHR vi M , G, F thng hng, ta thu c
GE
FE
GE F R M H

=1
=
GH F E M R
GH
FR
Gi QE ct CG ti L. p dng nh l Menelaus cho tam gic EQH vi L, G, C thng hng
th
LQ GE CH

=1
LE GH CQ
= FF B
v F R = F B ta thu c
vi ch BH k F C nn CH
CQ
Q
LQ
GH CQ
FR FQ
FQ
=

=
LE
GE CH
FE FB
FE
Do F L l phn gic EF Q nn LQ = LC. Theo h thc lng trong tam gic vung th
CE 2 = CA2 = CH.CQ suy ra HEC = EQC = LCQ. T 4CHE 4GHC(gc
- gc) suy ra HGC = HCE = AQC = HAC, vy GCHA l t gic ni tip hay
CGA = 90 .
Li gii trc tip sau ca nickname PSJL s dng php nghch o, tham kho [2]

Hnh 7.
Li gii 2. ng thng qua C song song vi BN ct AB ti X. D thy 4F CX 4N P M
do F C = F X. Gi H l hnh chiu ca A ln CX, do BA = BX = BH nn BH k F C
suy ra,
BH
BA
BD
=
=
CE
CA
CD
98

Tp ch Epsilon, S 10, 08/2016

Do H, D, E thng hng. Gi Y l giao im ca BN v CF , Z l giao im ca ng


thng qua E song song CX vi F X. Do BH = BX = CY , BH k CY nn BHCY l
hnh bnh hnh suy ra H, M , Y thng hng. T p dng nh l Dersagues cho hai tam
gic thu x HM C v EF Z ta suy ra C, G, Z thng hng. Xt php nghch o cc C, bn
knh CA nn (CA) F X, HE (CXE) qua Z do G 7 Z suy ra G thuc (CA) hay
AGC = 90 .
Cng tng t cch lm trn, nu pht biu bi ton 1 theo cch khc ta c bi ton sau
Bi ton 6. Cho tam gic ABC vung ti A vi ng cao AH. AD l phn gic ca tam gic
AHB. Trung trc CA ct phn gic ABC ti M . N , P l trung im AC, M N . CP ct phn
gic ngoi gc A ti Q. DQ ct BP ti R. Chng minh rng ARC = 90 .

F
A

M
P
N

R
B

H
Hnh 8.

Li gii. D thy M nm trn ng trn ngoi tip tam gic ABC nn AM C = 90 . Gi


CM ct AB ti E khi tam gic BEC cn ti B c ng cao CA v CP i qua trung im
F ca AE. P cng l trung im CF . p dng trc tip bi ton 1 vo tam gic BEC cn ti
B th ta thu c ARC = 90 .
n y ta li thy rng ta hon ton c th pht biu bi ton cho phn gic trong gc A vi
cch chng minh hon ton tng t
Bi ton 7. Cho tam gic ABC vung ti A vi ng cao AH. AD l phn gic ca tam gic
AHB. Trung trc CA ct phn gic ngoi gc ABC ti M . N , P l trung im AC, M N .
CP ct phn gic gc BAC ti Q. DQ ct BP ti R. Chng minh rng ARC = 90 .

99

Tp ch Epsilon, S 10, 08/2016

A
N
D H
B

C
R
Q

M
Hnh 9.
n y vic kt hp c hai bi ton 6, 7 ny s cho ta mt bi ton ng quy th v sau
Bi ton 8. Cho hnh ch nht ABCD. P i xng vi C qua BD. (K) l ng trn ng
knh P C. E, F l trung im ca KB, KD. CE, CF ln lt ct phn gic ngoi v phn gic
trong BP C ti M, N . on thng AE, AF ln lt ct ng trn (K) ti S, T . Chng minh
rng M S, N T v AC ng quy.

P
M

A
S

E
K

T
N F
D

C
Hnh 10.

Bi ton sau c th coi l mt ng dng c rt ra t bi ton 1, tuy nhin nu cha c bit


bi ton 1 th bi ton ny cng l mt thch thc ln
Bi ton 9. Cho tam gic ABC nhn vi ng cao AH sao cho trung im ca AH nhn BC
di mt gc vung. Ly D i xng H qua trung im BC. E, F l trung im ca CA, AB.
Cc im P, Q ln lt thuc on DE, DF sao cho AP H = AQH = 90 . HK, HL l
phn gic ca cc tam gic HCA, HAB. Chng minh rng KP, LQ v AD ng quy.
100

Tp ch Epsilon, S 10, 08/2016

L
R
Q
M

Hnh 11.
Li gii. Gi I l trung im AH th BIC = 90 . T gi M , N l cc im nm trn
ng thng BC sao cho AM k IB, AN k IC. T tam gic AM N vung ti A v B, C
ln lt l trung im HB, HC nn DN = DC + CN = BH + HC = M B + BD = DM .
T D l trung im M N nn cc tam gic DAM v DAN cn ti D. p dng trc tip bi
ton 2 phn a) vo cc tam gic cn DAM, DAN ta thy LQ, KP cng i qua im R thuc
AD tha mn AR = AH.
Bi ton sau c th c coi l mt h qu bi ton 1 nhng c gii c lp nh sau
Bi ton 10. Cho tam gic ABC cn ti A c ng cao CD. Gi E, M l trung im ca BD,
CE. R thuc AB sao cho BR = CD. CR ct AM ti K. Chng minh rng CKD = 90 .

101

Tp ch Epsilon, S 10, 08/2016

Hnh 12.
Li gii. p dng nh l Menelaus cho tam gic REC vi ba im M , K, A thng hng, ta
suy ra
KR
AR
M C KR AE

= 1 t
=
M E KC AR
KC
AE
Ta cn chng minh
DR2
DR2
AR
KR
=
tc
l
chng
minh
=
KC
DC 2
DC 2
AE
Gi H l chn ng vung gc k t A xung BC, ch 4BDC 4BHA (gc - gc) ta
suy ra
BC.AH
AB 2 BC.AH
AR = AB CD = AB
=
AB
AB
2
2
BD
BH
AB BH 2
AE = AB
= AB
=
2
AB
AB
Ch ng thc trn thu c do 2BH = BC. Nn ta suy ra

Mt khc

AR
AB 2 BC.AH
=
AE
AB 2 BH 2

DR2
(CD DB)2
(AH BH)2
=
=
DC 2
DC 2
AH 2
Do vy ch cn chng minh
AB 2 BC.AH
(AH BH)2
AH 2 + BH 2 2AH.BH
=
=
AB 2 BH 2
AH 2
AH 2
102

Tp ch Epsilon, S 10, 08/2016

iu ny tng ng vi chng minh (AB 2 BC.AH).AH 2 = (AH 2 +BH 2 2AH.BH)(AB 2


BH 2 ). ng thc trn hin nhin ng do AB 2 BH 2 = AH 2 . Vy CKD = 90 .
Bi ton sau c xy dng da trn phn b) ca bi ton 2
Bi ton 11. Cho tam gic ABC c ng cao AD. Trn tia BC, CB ly cc im G, H sao
cho BG = BA, CH = CA. E, F l trung im ca DG, DH. ng trn (A, AD) ct CA,
AB ti P , Q. M , N , Y , Z ln lt i xng A qua C, B, P , Q. N E ct ng trn (A, AD) ti
K sao cho K, N khc pha ng thng qua A vung gc KN . M F ct ng trn (A, AD)
ti L sao cho L, M khc pha ng thng qua A vung gc LM . Ly S, T thuc M F , N E
sao cho Y S k P L v ZT k QK. Chng minh rng DS = DT .

K
L

U
Q V
H

P
E

S
Z

T
N

M
Hnh 13.

Li gii. Gi U , V l trung im ca AS, AT . Nh vy U thuc LP M F k CU , p dng


bi ton 2 phn b) trn vo tam gic cn CAH th AU D = 90 . Tng t AV D = 90 . Vy
DA = DS = DT .
103

Tp ch Epsilon, S 10, 08/2016

Bi ton 12. Cho tam gic ABC vung ti A. Trung trc CA ct phn gic ngoi gc ABC
ti M . N , P l trung im AC, M N . CP ct phn gic gc BAC ti Q. CQ ct AB ti R. K
l hnh chiu ca A trn CM . D thuc QK sao cho RD k BM . Chng minh rng CD = CA.

D
K

R
A

M
P
N

B
Hnh 14.

Li gii. Gi CK ct AB ti E d thy tam gic BCE cn ti B v c ng cao CA ng


thi R, P l trung im ca AE v CR. T p dng bi ton 3 vo tam gic cn BCE ta thu
c CD = CA.
Bi ton 13. Cho tam gic ABC vung ti A vi ng cao AH. M , N l trung im ca HB,
HC. HP , HQ l phn gic ca tam gic HAC, HAB. Trn on BQ, CP ly cc im K, L
sao cho AK = AH = AL. LN ct KM ti R. Chng minh rng ABRC l hnh ch nht.

V P
L

Q
K
B

R
Hnh 15.
104

Tp ch Epsilon, S 10, 08/2016

Li gii. Gi O l trung im BC th cc tam gic OCA, OAB cn ti O v u c ng cao


AH. Gi khi nu U , V l trung im ca AM , AN th theo bi ton 3 ta c LN k OV v
KM k OU . T theo tnh cht php v t tm A t s 2 th LN , KM i qua i xng ca A
qua O nn R l i xng ca A qua O. T ABRC l hnh ch nht.

3. Mt s bi ton luyn tp
Bi ton 14. Cho tam gic ABC cn ti A, ng cao CD. E, M ln lt l trung im BD,
CE. DP l phn gic trong ca tam gic CDE. H thuc BP sao cho EH k AM . Gi F l
hnh chiu ca D ln BC. R, Q ln lt l giao im ca F P vi DH v AC. Chng minh
rng BR, DP v ng thng qua Q song song vi BC ng quy.
Bi ton 15. Cho tam gic ABC cn ti A, ng cao CD. E, M ln lt l trung im BD,
CE. DP l phn gic trong ca tam gic CDE. Gi F l i xng ca D qua BC. S thuc
P F sao cho ES k AM . SP ct AC ti Q. R l giao ca DS v ng thng qua Q song song
vi BC. Chng minh rng RP , ES v ng thng qua Q vung gc vi BC ng quy.
Bi ton 16. Cho tam gic ABC nhn c ng cao AH sao cho trung im AH nhn BC
di gc vung. E, F l trung im CA, AB. Trn on BC ly cc im K, L sao cho
BK = AH BH v CL = AH CH. P , Q l hnh chiu ca H ln AL, AK. Chng minh
rng EP , QF v BC ng quy.

P
Q
B

K R L

Hnh 16.

Ti liu
[1] Tp ch ton hc v tui tr s 465 v s 469

105

Tp ch Epsilon, S 10, 08/2016

[2] Topic Right angle


http://artofproblemsolving.com/community/q1h1276419p6696477
[3] Topic Equal segments in isosceles triangle
http://artofproblemsolving.com/community/q1h1280275p6732257

106

Tp ch Epsilon, S 10, 08/2016

MT B V PHN GIC
Nguyn Trn Hu Thnh
Trng THPT chuyn L T Trng, Cn Th

TM TT
Bi vit xoay quanh mt b hay c nhiu m rng vi cch gii quyt bng cc cng
c hnh hc phng thun ty.

1. M u
Khi tm hiu v vn phn gic trong tam gic, tc gi pht hin c mt cu hnh kh p
v c nhiu ng dng, c th ta c bi ton sau [1]
Bi ton 1. 4ABC ni tip ng trn (O) c cc phn gic trong BE, CF ct nhau ti I.
EF ct (O) ti M , N . Cc ng thng M I, N I ct (O) ln lt ti P , Q khc M , N . Khi
P Q song song BC.
Bi ton trn c nhiu li gii, sau y tc gi xin nu ba cch:

Bin tp: Ng Quang Dng

107

Tp ch Epsilon, S 10, 08/2016

Li gii 1. ng phn gic ngoi ca gc A trong 4ABC ct BI, CI theo th t ti L, K


nn ta c L, K ln lt l tm ng trn bng tip gc B, C ca 4ABC. Vi ch rng t
gic AICL ni tip, ta c:
EN EM = EA EC = EI EL
Do t gic N IM L ni tip. Tng t N IM K ni tip nn nm im K, N , I, M , L cng
thuc mt ng trn.
Ko di M P , N Q ct BC ln lt ti G, H. Ta c:
(CK, CG) = (CA, CI) = (LA, LI) = (LK, LI) = (M K, M I)
Suy ra t gic KM CG ni tip, t :
(GI, GH) = (KM, KI) = (N M, N I) = (P I, P Q)
Nh vy P Q song song BC. Ta c iu phi chng minh.

Li gii 2. Cng nh Li gii 1, ta c a gic KLM IN ni tip ng trn . Gi d l tip


tuyn ti I ca . Ta c (IK, Id) = (LK, LI) = (CI, CB) nn d song song BC. Mt khc
(QI, QP ) = (M N, M I) = (IN, Id) suy ra d song song P Q. Nh vy P Q song song BC. Vy
bi ton c gii xong.
Li gii 3 tc gi xt b sau:
B . (IMO 2010) 4ABC ni tip ng trn , P l mt im nm trong ng trn.
AP , BP , CP ln lt ct ti K, L, M . Tip tuyn ti C ca ct AB ti D. Chng minh
rng nu DC = DP khi v ch khi M K = M L.

108

Tp ch Epsilon, S 10, 08/2016

Chng minh b . Do 4LP M 4CP B v 4KP M 4CP A nn:


BC
MK
AC
ML
=
v
=
MP
BP
MP
AP
Suy ra:

BC
AC
BC
BP
=

=
BP
AP
AC
AP
CA
Do P thuc ng trn Apollonius t s
dng trn on AB. Hn na D chnh l tm
CB
ca ng trn ny nn M K = M L khi v ch khi DC = DP .
MK = ML

Quay tr li bi ton,

Li gii 3. Gi O1 , O2 ln lt l tm ng trn ngoi tip 4ABI v 4ACI. Ta bit O1 ,


O2 nm trn (O). Gi K, L ln lt l tm ng trn bng tip gc B, C ca 4ABC. Do O1 ,
109

Tp ch Epsilon, S 10, 08/2016

O2 theo th t l trung im ca LI, KI nn (IO1 O2 ) tip xc (IKL) (IM N ). Gi G l


giao im ca O1 O2 v M N . Do O1 N M O2 ni tip nn:
PG/(IO1 O2 ) = GO1 GO2 = GM GN = PG/(IM N )
Tc l G nm trn trc ng phng ca (IO1 O2 ) v (IM N ), suy ra GI l tip tuyn chung
ca hai ng trn ny. Mt khc G thuc O1 O2 l ng trung trc ca AI nn GA2 = GI 2 =
GM GN = PG/(O) , suy ra GA l tip tuyn ca (O). p dng b cho 4AM N vi AI,
M I, N I ct (O) theo th t ti D, P , Q c P D = QD, suy ra P B = QC, nh vy P Q song
song BC.
Vy bi ton c gii xong.
T Li gii 1 ca Bi ton 1, ta c th pht biu bi ton li nh sau
Bi ton 2. Cho 4ABC v tm ng trn ni tip I. Gi Ib , Ic ln lt l tm ng trn
bng tip gc B, C ca 4ABC. (IIb Ic ) ct (ABC) ti M , N . M I, N I ct (ABC) ln th hai
theo th t ti P , Q. Chng minh rng P Q song song BC.
Bi ton 3. Cho 4ABC, ng cao AD, BE, CF ng quy ti H. (BHC) ct (DEF ) ti
M , N . M H, N H ct (DEF ) ln th hai theo th t ti P , Q. Chng minh rng P Q song song
EF .
Ta c th pht trin Bi ton 3 thnh bi ton sau:
Bi ton 4. Cho 4ABC, trc tm H. Ly D l mt im nm trn cnh BC. (BHD) ct
AB ti E. (CHD) ct AC ti F . ng thng HE ct (CHD) ti G, ng thng HF ct
(BHD) ti I. (HGI) ct (DEF ) ti M , N . Ko di M H, N H ct (DEF ) ln th hai ln lt
ti P, Q. Chng minh rng P Q song song EF .

110

Tp ch Epsilon, S 10, 08/2016

Chng minh. p dng nh l Miquel trong 4ABC ta c AEHF ni tip. T y d dng


suy ra H l tm ng trn ni tip 4DEF . p dng bi ton u cho 4DEF ta kt lun P Q
song song EF . Vy bi ton c gii xong.
Hoc t Li gii 3 ca Bi ton 1, bi ton c th tr thnh:
Bi ton 5. Cho 4ABC ni tip ng trn (O), ng phn gic trong AD. I l mt im di
chuyn trn ng thng AD. Tip tuyn ti A ca (O) ct ng trung trc ca AI ti X. Mt
ng thng qua X ct (O) ti M , N . Cc tia M I, N I theo th t ct (O) ti P , Q. Chng
minh P Q song song BC.
Sau y ta xt mt s m rng v ng dng ca bi ton ny.

2. Khai thc bi ton


Bi ton gc l mt cu hnh p v c nhiu tnh cht th v. Bi ton sau l mt kt qu v
vic chia i on thng.
Bi ton 6. Cho 4ABC ni tip ng trn (O) c cc phn gic BE, CF ct nhau ti I.
EF ct (O) ti M , N . Cc ng thng M I, N I ct (O) ln lt ti P , Q. Khi P Q chia
i BI v CI.
Tnh cht ny c pht hin khi tc gi tm thm nhng li gii khc cho Bi ton 1. Chia i
on thng cng l mt kiu bi ton ang ph bin hin nay, li gii sau c xut bi anh
Nguyn L Phc.
Chng minh. Gi X, Y , Z ln lt l tm ng trn bng tip gc A, B, C ca 4ABC. Ko
di IP , IQ ct BC theo th t ti L, K. Gi D l giao im ca AI vi (O).
Nh chng minh tnh cht 1, ta c t gic ZM CL ni tip, kt hp vi vic bn im A,
C, X, Z cng nm trn mt ng trn, ta c IL IM = IZ IC = IA IX. Do t gic
AM XL ni tip, suy ra (LI, LX) = (AM, AI) = (P I, P D). Nh vy P D song song LX. M
D l trung im ca IX nn P l trung im ca IL. Tng t Q l trung im ca IK hay
P Q l ng trung bnh ca 4ILK. T ta suy ra P Q chia i IB v IC.

111

Tp ch Epsilon, S 10, 08/2016

Vy bi ton c gii xong.


Chia i on thng l mt tnh cht hay, thm ch ta c th m rng bi ton thnh chia on
thng theo t s k, sau y ta c mt khai thc nhiu bi ton kh p nh sau:
Bi ton 7. Cho 4ABC ni tip ng trn (O) c cc phn gic BE, CF ct nhau ti I.
EF ct (O) ti M, N . Cc ng thng M I, N I ct (O) ln lt ti P , Q. Gi X, Y , Z ln
lt l tm ng trn bng tip gc A, B, C ca 4ABC. IP , IQ theo th t ct ng thng
BC ti L, K. Chng minh rng nm im X, Y , Z, L, K cng nm trn mt ng trn.

Bi ton 7 c suy ra trc tip t Bi ton 6 cng vi tnh cht: Php v t tm I t s


(XY Z) thnh (ABC).

112

1
bin
2

Tp ch Epsilon, S 10, 08/2016

Bi ton 8. Cho 4ABC ni tip ng trn (O) c cc phn gic BE, CF ct nhau ti I.
EF ct (O) ti M , N . Cc ng thng M I, N I ct (O) ln lt ti P , Q. Gi X, Y , Z ln
lt l tm ng trn bng tip gc A, B, C ca 4ABC. IP , IQ theo th t ct ng thng
BC ti L, K. Chng minh rng t gic M N LK ni tip ng trn tm X.

Chng minh. Nh ta thy, chng minh mt im l tm ca t gic th hai hng thng gp


nht l bin i gc hoc bin i cnh, y tc gi s dng phng php bin i cnh thng
qua vic chng minh AN v AK l hai ng ng gic, v chng minh LX = N X =
M X = KX th u tin ta c th chng minh KX = N X v LX = M X sau kt hp vic
LX = KX chng minh h qu trn ta suy ra bn cnh bng nhau. T ta ngh n vic
chng minh cc tam gic cn thng qua vic bin i gc, ch do t gic N AKX ni tip
nn (AN, AX) = (KN, KX) v (N X, N K) = (AX, AK). Nu tam gic N KX cn ti X
th (KN, KX) = (N X, N K), do ta phi chng minh (AN, AX) = (AX, AK) hay AN v
AK l hai ng ng gic ca 4ABC.
Ch t gic AN XK ni tip nn, m (AN, AX) = (AX, AK) nn XN = XK. Hon ton
tng t, XM = XL. V t tm I t s 2 bin D, P , Q thnh X, L, K nn XL = XK. Vy K,
L M , N thuc ng trn tm X.
Bi ton 9. Cho 4ABC ni tip ng trn (O) c cc phn gic BE, CF ct nhau ti I.
EF ct (O) ti M , N . Cc ng thng M I, N I ct (O) ln lt ti P , Q. Gi X, Y , Z ln
lt l tm ng trn bng tip gc A, B, C ca 4ABC. Tia IP , IQ theo th t ct ng
thng BC ti L, K. Gi S l im i xng ca K qua XY , T l im i xng ca L qua
XZ. Chng minh rng cc b ba im (Z, T, K), (A, T, B), (A, S, C), (Y, L, S) thng hng.

113

Tp ch Epsilon, S 10, 08/2016

Chng minh. Ta c (ZB, ZT ) = (ZL, ZB) = (ZB, ZK) nn Z, T , K thng hng.


Ta c (BZ, BT ) = (BL, BZ) = (BC, BX) = (BZ, BA) nn A, T , B thng hng.
Tng t A, S, C v Y , S, L thng hng.
Bi ton sau l mt tnh cht th v v hai ng thng ct nhau trn ng trn:
Bi ton 10. Cho 4ABC ni tip ng trn (O) c cc phn gic BE, CF ct nhau ti I.
EF ct (O) ti M , N . Cc ng thng M I, N I ct ng thng BC theo th t ti L, K.
LN , KM ct (O) ln th hai ti T . Chng minh rng T l im chnh gia cung BAC.
Li gii sau c pht hin khi tc gi c suy ngh tm mt ng dng cho nh l Brokard:

114

Tp ch Epsilon, S 10, 08/2016

Chng minh. Gi X, Y , Z ln lt l tm ng trn bng tip gc A, B, C ca 4ABC. Gi


J l giao im ca ng thng Y Z v BC. Theo bi ton gc c Y M IN Z ni tip ng
trn v Y ZBC ni tip. Theo bi ton 8 cn c KLN M ni tip nn trc ng phng ca
cc cp trong ba ng trn (O), (KLM N ), (Y ZM N ) ng quy nn M N i qua J.
Ta d dng chng minh JLN A v JKM A ni tip nn A l im Miquel ca b bn ng
thng (M N, LK, M K, LN ). Mt khc T l giao im ca M K v LN nn A cng thuc
(T M N ) hay T thuc (AM N ) (O). p dng nh l Brokard cho t gic M N LK ni tip
(X) ta c IA IX JT . M IA Y Z JZY nn T nm trn ng thng Y Z. Vy
bi ton c gii quyt.
T bi ton gc ban u ta thu c nhiu kt qu p, phn tip theo tc gi s m rng bi
ton gc ra tng qut hn.

3. M rng bi ton
Bi ton gc l mt kt qu rt p ca hnh hc phng v vn phn gic. Li gii 2 ta ch
s dng tnh cht ca ng phn gic gc A ca 4ABC. Li gii 3 li lin quan n vn
tip tuyn ti im A ca (ABC) nn cc bi ton tip theo ta s c nhng m rng cho Bi
ton 1.
Bi ton 11. 4ABC ni tip ng trn (O), ng phn gic trong AD. I l mt im di
chuyn trn AD sao cho I, A cng nm trn mt mt phng b BC. Gi E, F l giao im ca
BI vi AC, CI vi AB. EF ct (O) ti M , N . M I, N I theo th t ct (O) ti P , Q. Chng
minh rng P Q song song BC.
Trng hp ny ta thy khi v ng phn gic ngoi ca gc A ct BI, CI th cho hai im
khng cn c bit na. Nhng thy BI, CI ct ln lt cc ng trn (ACI) v (ABI)
th quan h gia cc gc s r rng hn.

115

Tp ch Epsilon, S 10, 08/2016

Chng minh. Gi S, T ln lt l giao im ca BI v (ACI), CI v (ABI), t y:


FI FT = FA FB = FN FM
Nn t gic N IM T ni tip. Tng t t gic N IM S ni tip.
Ta c:
(T B, T C) = (T B, T I) = (AB, AI) = (AI, AC) = (SI, SC) = (SB, SC)
Do t gic T BCS ni tip.
Gi d l tip tuyn ti I ca ng trn i qua cc im N , I, M , S, T , ta c:
(BC, BI) = (T I, T S) = (Id, IS)
Suy ra d song song BC. Mt khc:
(P I, P Q) = (N M, N I) = (IM, Id)
Suy ra d song song P Q. Do ta kt lun P Q song song BC.
Vy bi ton c gii xong.
Nhn xt. Li gii trn vn s dng k thut bin i gc cng vi phng tch. Tuy nhin r
rng trong bi ton m rng n c dng kho lo vn dng ht cc d kin m ca bi
ton.
Bi ton tip theo l mt m rng ca bi ton trn khi khai thc d kin hai ng trn ngoi
tip 4ABI v 4ACI:
Bi ton 12. Cho 4ABC, im D nm trn phn gic trong d ca gc A. Trong 4ABC, gi
d1 , d2 l hai ng thng i qua A i xng nhau qua d sao cho d1 nm trn mt phng cha
B b d. Qua D k ng thng song song vi BC ct d1 , d2 ln lt ti X, Y . BX ct AC ti
E, CY ct AB ti F . ng thng EF ct (ABC) ti M , N . Tia M Y , N X ct (ABC) ln
th hai ti P, Q. Chng minh P Q song song BC.

116

Tp ch Epsilon, S 10, 08/2016

Chng minh. Ko di BE ct (AXC) ti H, CF ct (ABY ) ti G. Khi do:


(GB, GC) = (AB, AY ) = (AX, AC) = (HB, HC)
Nn t gic GHCB ni tip. T y suy ra GHY X ni tip. Mt khc do:
FG FY = FA FB = FN FM
Nn GN Y M ni tip, tng t HM XN ni tip. Do su im G, N , X, Y , M , H cng
thuc mt ng trn. T iu ny ta c:
(XY, XQ) = (M P, M N ) = (QP, QX)
Nh th XY song song P Q. M XY song song BC nn P Q song song BC.
Vy bi ton c gii xong.
Ngoi ra, Bi ton 12 cn mt khai thc v ng thng song song vi BC na bi ton sau:
Bi ton 13. Cho 4ABC, im D nm trn phn gic trong d ca gc A. Trong 4ABC, gi
d1 , d2 l hai ng thng i qua A i xng nhau qua d sao cho d1 nm trn mt phng cha
B b d. Qua D k ng thng song song vi BC ct d1 , d2 ln lt ti X, Y . BX ct AC ti
E, CY ct AB ti F . ng thng EF ct (ABC) ti M, N . Tia M D, N D ct (ABC) ln
th hai ti P , Q. Chng minh P Q song song BC.
Trong Bi ton 11, ta ln lt c B, I, E v C, I, F thng hng v ta thy ng ni
tm ca (ABI) v (ACI), ng thng EF , tip tuyn ti A ca (ABC) ng quy. Vy trong
trng hp B, I, E v C, I, F khng cn thng hng nhng vn gi nguyn tnh cht ba ng
thng y ng quy th nh th no? Ta c bi ton th v nh sau:
Bi ton 14. Cho 4ABC ni tip ng trn (O), ng phn gic trong AD. I l mt im
di chuyn trn ng thng AD. Gi O1 , O2 ln lt l tm ca (ABI) v (ACI). Tip tuyn
ti A ca (O) ct ng thng O1 O2 ti X. Mt ng thng qua X ct (O) ti M , N . Cc tia
M I, N I theo th t ct (O) ti P , Q. Chng minh P Q song song BC.
bi ton ny ta thy r rng khng th s dng vic ko di BI, CI c na v M , N di
chuyn ty nn kh nh hng c vic xi phng tch, nhng vi b c nu Li
gii 3 ca bi ton gc th bi ton tr nn r rng hn.
Chng minh. Khng mt tnh tng qut, gi s N nm gia X v M . Ta c X nm trn ng
trung trc ca AI nn XA = XI. p dng B ca Bi ton 1 cho tam gic AM N vi AI,
M I, N I ct (ABC) ln lt ti Y , P , Q v ch BY = CY .

117

Tp ch Epsilon, S 10, 08/2016

c P Y = QY suy ra P B = QC. T y d dng suy ra P Q song song BC.


Vy bi ton c gii xong.
Cui cng, tc gi xin a ra mt s bi tp bn c rn luyn thm v cc tnh cht th v
ny:
Bi ton 15. Cho 4ABC ni tip ng trn (O). I l mt im di chuyn trn phn gic
trong ` ca gc A sao cho I, A cng nm trn mt mt phng b BC. Gi E, F l giao im
ca BI vi AC, CI vi AB. EF ct (O) ti M , N . M I, N I theo th t ct (O) ti P , Q, ct
ng thng BC ti L, K.
a) Gi Y , Z ln lt l giao im ca BI v (ACI), CI v (ABI). Chng minh rng bn im
L, K, Y , Z cng nm trn ng trn tm X.
b) Gi D l giao im ca ` v (O). Chng minh rng khi P , Q ln lt chia IL, IK theo cng
t s k th D cng chia IX theo t s k.
T Bi ton 15 ta c mt khai thc nh sau:
Bi ton 16. Cho 4ABC, ng phn gic trong AD ca gc A (D thuc BC). I l mt im
di chuyn trn cnh AD vi I khc A v D. BI ct AC ti E, CI ct AB ti F . ng thng
EF ct (ABC) ti M , N . M I, N I ct ng thng BC ln lt ti L, K. Chng minh rng
cc ng thng M K, N L giao nhau ti mt im c nh khi I thay i.
Bi ton 17. Cho 4ABC, ng phn gic trong BE, CF . EF ct (O) ti M , N . Chng
minh rng M , N l tip im ca hai tip tuyn chung ca (ABC) v ng trn bng tip gc
A ca 4ABC.
Bi ton 18. Cho 4ABC, hai tip tuyn chung ngoi `1 , `2 ca (ABC) v ng trn bng
tip gc A. ng thng BC ct `1 , `2 ln lt ti D, E. Chng minh rng AD, AE l hai
ng ng gic ca 4ABC.
Bi ton 17 v Bi ton 18 c th tham kho [3].
118

Tp ch Epsilon, S 10, 08/2016

Ti liu
[1] Two parallels
http://www.artofproblemsolving.com/community/c6h550786
[2] Two parallels generalization problem.
http://www.artofproblemsolving.com/community/c6h1147664p5418246
[3] Common tangents problem.
http://www.artofproblemsolving.com/community/c6h385175p3753324
[4] Bisects segment problem.
https://www.facebook.com/groups/Loicenter/permalink/982284725178035

119

Tp ch Epsilon, S 10, 08/2016

CC NG TRN C HAI IM CHUNG


TRONG T GIC NI TIP
Trn Minh Ngc, TP.HCM

1. Gii thiu
Trong qu trnh dy i tuyn trng THPT chuyn H Long, ti pht hin bi ton sau:
Bi ton 1. Cho t gic ABCD ni tip (O). Mt ng thng d ln lt ct AC, BD, AB,
CD, AD, BC ti M , N , P , Q, R, S. Chng minh rng vi mi im X nm trn (O) th
(XM N ), (XP Q), (XRS), (O) c mt im chung khc X.
Trong bi vit ny, ti s gii thiu ba li gii bi ton v mt s tnh cht khc trong cu hnh
ca n. Trong phn cui cng, ti s gii thiu v ng dng ca nhng tnh cht va tm c
trong vic gii nhng bi ton khc.

2. Li gii
2.1. Li gii 1
Ta pht biu v chng minh b sau:
B 1. Cho ng trn (O) v ng thng d. Trn (O) ly X, trn d ly A, B, C, D ri ln
lt cho XA, XB, XC, XD ct (O) ti im th hai l A0 , B 0 , C 0 , D0 . Khi A0 B 0 , C 0 D0 , d
ng quy khi v ch khi (XAB), (XCD), (O) c mt im chung khc X.
Bin tp: Ng Quang Dng

120

Tp ch Epsilon, S 10, 08/2016

Hnh 1. B 1
Chng minh. Chng minh hai chiu.
Chiu thun. Gi s A0 B 0 , C 0 D0 , d ng quy ti Z. p dng nh l Miquel cho t gic
ton phn to bi bn ng thng AA0 , BB 0 , AB, A0 B 0 , ta c (XAB), (XA0 B 0 )
(O), (ZAA0 ), (ZBB 0 ) ng quy ti mt im Y . Do DZY = BZY = BB 0 Y =
DD0 Y nn t gic DD0 ZY ni tip. Suy ra Y DZ = Y D0 Z = CXY . Do theo
nh l Miquel, t gic CDY X ni tip.
Vy (XAB), (XCD), (O) c chung im Y 6= X.
Chiu nghch. Gi s (XAB), (XCD), (O) c chung im Y =
6 X. Gi Z l giao im
ca A0 B 0 , d.
D1 l giao im th hai ca ZC 0 , (O).
D2 l giao im ca XD1 , d.
Theo chiu thun, (XAB), (XCD), (O) ng quy. Suy ra D2 D, D1 D0 .
Vy A0 B 0 , C 0 D0 , d ng quy ti Z.

B 2. Cho t gic ABCD ni tip ng trn (O). Mt ng thng d ln lt ct AC,


BD, AB, CD, AD, BC ti M , N , P , Q, R, S. Trn (O) ly X ri ln lt cho XM , XN ,
XP , XQ, XR, XS ct (O) ti M 0 , N 0 , P 0 , Q0 , R0 , S 0 . Khi M 0 N 0 , P 0 Q0 , R0 S 0 , d ng quy.

121

Tp ch Epsilon, S 10, 08/2016

Hnh 2. B 2
Chng minh. p dng nh l Pascal cho lc gic CABM 0 XP 0 , ta c CA M 0 X = M ,
AB XP 0 = P , BM 0 CP 0 = T thng hng, hay T d . Chng minh tng t BQ0 CN 0 =
U d.
p dng nh l Pascal cho lc gic M 0 N 0 C 0 P 0 Q0 B 0 , ta c M 0 N 0 P 0 Q0 = V , N 0 C Q0 B =
U , CP 0 BM 0 = T thng hng, hay V d , hay M 0 N 0 , P 0 Q0 , d ng quy. Chng minh tng
t M 0 N 0 , R0 S 0 , d ng quy.
Vy M 0 N 0 , P 0 Q0 , R0 S 0 , d ng quy.
Tr li bi ton.

Hnh 3
122

Tp ch Epsilon, S 10, 08/2016

Vi M 0 , N 0 , P 0 , Q0 , R0 , S 0 nh ngha nh trong b 2.
Theo b 2: M 0 N 0 , P 0 Q0 , R0 S 0 , d ng quy.
Theo b 1: (XM N ), (XP Q), (XRS), (O) c mt im chung khc X.

2.2. Li gii 2
Ta pht biu v chng minh b sau:
AM
DN
=
. Gi E
AB
DC
l giao im ca AB, CD. Khi (EAD), (EBC), (EM N ) c mt im chung khc E.

B 3. Cho t gic ABCD. Trn AB, CD ln lt ly M , N sao cho

Hnh 4. B 3
Chng minh. Gi F l giao im th hai ca (EAD) v (EBC).
Do F AB = 180o F AE = 180o F DE = F DC, F BA = F CD nn 4F AB
FA
AB
AM
4F DC. Suy ra
=
=
. Do 4F AM 4F DN . Suy ra EM F = EN F
FD
DC
DN
nn EN M F ni tip. Vy (EAD), (EBC), (EM N ) c im chung khc E.
Tr li bi ton.

123

Tp ch Epsilon, S 10, 08/2016

Hnh 5
XM , XN ln lt ct (XP Q), (O) ti im th hai l M1 , M2 , N1 , N2 .

AI P M BN

= 1.
p dng nh l Menelaus cho 4IM N vi A, P , B thng hng, ta c
AM P X BI
DI QN CM
Tng t

= 1. Do
DN QM CI
DI QN CM
AI P M BN

AM P X BI
DN QM CI

ng thc ny tng ng :
P M QM CI
P N QN DI

AM CM BI
DN BN AI
CI
DI
P M QM
P N QN
M M1
=
(do 4CBI 4DAI). Nn
=
. Suy ra
=
BI
AI
AM CM
DN BN
M M2
M M1 M X
N N1 N X
N N1
M M1
N N1
=
=
. Theo hnh v, ta c
=
. Theo b 2.3, ta
M M2 M X
N N2 N X
N N2
M M2
N N2
c (XM N ), (XM1 N1 ) (XP Q), (XM2 N2 ) (O) c im chung khc X. Chng minh
tng t (XM N ), (XRS), (O) c im chung khc X.
Vy (XM N ), (XM N ), (XRS), (O) c im chung khc X.

2.3. Li gii 3
Ta pht biu v chng minh b sau:

124

Tp ch Epsilon, S 10, 08/2016

B 4. Cho t gic ABCD ni tip ng trn (O). Mt ng thng ln lt ct AC, BD,


AB, CD ti M , N , P , Q. Khi tn ti duy nht im T sao cho T M .T N = T P .T Q =
PT /(O) .

Hnh 6. B 4
Chng minh. Gi T l im m T M .T N = PT /(O) , U l im m T P .T U = PT /(O) . Khi
T , U l duy nht.
V l giao im ca BN , CU . E, F ln lt l giao im th hai ca T B, T C vi (O).
Do T M .T N = T P .T U = PT /(O) = T B.T E = T C.T F nn cc t gic BEM N , CF P U
ni tip. Hn na, nu gi G l giao im ca EM , F P v p dng nh l Pascal o cho
lc gic BACF GE vi BA F G = P , AC GE = M , CF EB = T thng hng v
A, B, C, E, F (O), ta c G (O). T , ta c bin i gc sau:
BV C = 180o (V N U + V U N ) = 180o (GEB + GF C)
_

BC
360o GB GC
=
=
2
2
Suy ra V (O). Do V D, U Q. Vy tn ti duy nht im T sao cho T M .T N =
T P .T Q = PT /(O) .
Tr li bi ton.

125

Tp ch Epsilon, S 10, 08/2016

Hnh 7
Gi Y l giao im th hai ca T X v (O). Theo b 2.4, tn ti duy nht im T sao cho
T M .T N = T P .T Q = PT /(O) = T X.T Y . Suy ra cc t gic M N Y X, P QXY ni tip. Do
(XM N ), (XP Q), (O) ng trc. Chng minh tng t (XM N ), (XRS), (O) c im
chung khc X.
Vy (XM N ), (XP Q), (XRS), (O) c im chung khc X.

3. Cc tnh cht khc


Gi Y l im chung khc X ca (XM N ), (XP Q), (XRS), (O).
Tnh cht 1. Gi I l giao im ca AC, BD. (IM N ) ln lt ct (IAB), (ICD), (IAD),
(IBC) ti E, F , G, H . Khi (XEF ), (XGH) qua Y .

126

Tp ch Epsilon, S 10, 08/2016

Hnh 8. Tnh cht 1


Chng minh. Php nghch o cc I , phng tch k 6= 0 bin A, B, C, D, M , N , E, F , X
thnh A0 , B 0 , C 0 , D0 , M 0 , N 0 , E 0 , F 0 , X 0 . Khi AC, BD, (IAB), (ICD), (IM N ) bin
thnh A0 C 0 , B 0 D0 , A0 B 0 , C 0 D0 , M 0 N 0 . Suy ra M 0 N 0 ln lt ct A0 C 0 , B 0 D0 , A0 B 0 , C 0 D0 ti M 0 ,
N 0 , E 0 , F 0 . M A0 , B 0 , C 0 , D0 , X 0 ng vin. Nn theo bi ton 1, (A0 B 0 C 0 D0 ), (X 0 M 0 N 0 ),
(X 0 E 0 F 0 ) c im chung khc X 0 . Do (O), (XM N ), (XEF ) c im chung khc X , hay
(XEF ) qua Y . Chng minh tng t (XGH) qua Y .
Ch 1. Ta c kt qu tng t vi J, K ln lt l giao im ca AB v CD, AD v BC.
Khi d qua mt trong ba im I, J, K , kt qu ch cn ng vi hai im cn li.
Tnh cht 2. Gi s d qua I v ln lt ct tip tuyn ti A, C, B, D ca (O) ti T , U , V , W .
Khi ng trn qua X v tip xc d ti I, (XT U ), (XV W ) qua Y .

127

Tp ch Epsilon, S 10, 08/2016

Hnh 9. Tnh cht 2


Chng minh. ng trn qua X v tip xc d ti I qua Y ch l trng hp c bit ca
(XM N ) qua Y . Tht vy, khi d qua I th M N I, (XM N ) suy bin thnh ng trn
qua X v tip xc d ti I . Ta ch chng minh (XT U ), (XV W ) qua Y .
K hiu ZZ l tip tuyn ti Z ca (O)
Gi I1 , T1 , U1 ln lt l giao im th hai ca XI, XT , XU vi (O).
p dng nh l Pascal cho lc gic U1 XI1 CCA ni tip (O), ta c U1 X CC = U ,
XI1 CA = I, I1 C U1 A = U 0 thng hng, hay U 0 d. Chng minh tng t I1 A T1 C =
T 0 d.
p dng nh l Pascal cho lc gic U1 AI1 I1 CT1 ni tip (O), ta c U1 A I1 C = U 0 ,
AI1 CT1 = T 0 , I1 I1 U1 T1 = L thng hng, hay L U 0 T 0 = d. Do I1 I1 , U1 T1 , d ng
quy ti L. T , p dng b 1 cho X (O) v T, U, I, I d, ta c ng trn qua X
v tip xc d ti I, (XT U ), (O) c im chung khc X, hay (XT U ) qua Y . Chng minh tng
t (XV W ) qua Y .
Ch 2. Ta c kt qu tng t vi J, K.
Tnh cht 2 c m rng cho lc gic nh sau.
Tnh cht 3. Cho lc gic ABCDEF ni tip (O) c AD, BE, CF ng quy ti I. Mt ng
thng d qua I, ln lt ct AB, DE, BC, EF , CD, F A ti M , N , P , Q, R, S. Khi vi mi
im X nm trn (O), ng trn qua X tip xc vi d ti I, (XM N ), (XP Q), (XRS), (O)
c mt im chung khc X.

128

Tp ch Epsilon, S 10, 08/2016

Hnh 10. Tnh cht 3


Chng minh. p dng nh l Pascal cho lc gic M1 XI1 BAD, ta c M1 X AB = M ,
XI1 AD = I, I1 B DM1 = M 0 thng hng, hay M 0 d. Chng minh tng t, ta c
I1 D BN1 = N 0 d.
p dng nh l Pascal cho lc gic BI1 I1 DM1 N1 , ta c BI1 DM1 = M 0 , I1 I1 M1 N1 =
L, I1 D N1 B = N 0 , hay L M 0 N 0 = d. Do M1 N1 , I1 I1 , d ng quy ti L. T , p
dng b 1 cho X (O) v M, N, I, I d, ta c ng trn qua X v tip xc d ti I,
(XM N ), (O) c im chung khc X. Chng minh tng t ng trn qua X v tip xc d ti
I, (XP Q), (XRS), (O) c im chung khc X. Ta i n kt lun bi ton!
Ch 3. Tnh cht 2 c th c m rng cho 2ngic ni tip ng trn c cc ng cho
ng quy theo cch tng t.

4. ng dng
Bi ton 2. Cho ng gic ABCDE ni tip (O). Mt ng thng d ct AC, BD, AB, CD
ti M , N , P , Q. (EM N ) ln lt ct AC, BD ti im th hai l M 0 , N 0 . (EP Q) ln lt ct
AB, CD ti im th hai l P 0 , Q0 . Chng minh rng M 0 , N 0 , P 0 , Q0 thng hng.

129

Tp ch Epsilon, S 10, 08/2016

Hnh 11. Bi ton 2


Chng minh. p dng bi ton 1 cho t gic ABCD ni tip (O) v ng thng d ln lt
ct AC, BD, AB, CD ti M , N , P , Q, ta c (EM N ), (EP Q), (O) c im chung khc
F 6= E.
p dng bi ton 1 cho t gic M N N 0 M 0 ni tip v ng thng AB ln lt ct M M 0 , N N 0 ,
M N , M 0 N 0 ti A, B, P , P1 , ta c (EAB) (O1 ), (EP P1 ), (M N N 0 M 0 ) c im chung
khc E. Suy ra F (EP P1 ). Do P1 P l giao im th hai ca AB, (P EF ), hay M 0 ,
N 0 , P 0 thng hng. Chng minh tng t M 0 , N 0 , Q0 thng hng. Vy M 0 , N 0 , P 0 , Q0 thng
hng.
Bi ton 3. Cho 4ABC ni tip (O). Mt ng trn (OA ) ln lt tip xc vi AB, AC ti
D, E v tip xc trong vi (O) ti F . DE ln lt ct BC v tip tuyn ti A ca (O) ti G,
H. Chng minh rng (F GH) tip xc vi (O), BC v tip tuyn ti A ca (O).

130

Tp ch Epsilon, S 10, 08/2016

Hnh 12. Bi ton 3


Chng minh. p dng bi ton 1 cho t gic AABC ni tip (O) v ng thng DE ln
lt ct AB, AC, BC, AA ti D, E, G, H , ta c vi mi im X nm trn (O), (XDE),
(XGH), (O) c im chung Y , X.
Khi (XDE) tip xc vi (O) ti F th X Y F . Suy ra (F GH) tip xc vi (O) ti F .
p dng bi ton 2 cho t gic AABC ni tip (O) v ng thng d ln lt ct AB, AC,
BC, AA ti D1 , E1 , G1 , H1 , ta c D2 , E2 , G2 , H2 thng hng, trong D2 , E2 , G2 , H2 ln
lt l giao im th hai ca (F D1 E1 ) vi AB, AC; (F G1 H1 ) vi BC, AA.
Khi (F D1 E1 ) ln lt tip xc vi AB, AC ti D, E th D1 D2 D, E1 E2 E2 E.
Suy ra G1 G2 G, H1 H2 H. Do (F GH) ln lt tip xc vi BC, AA ti G,
H.
Bi ton 4. Cho t gic ABCD ni tip (O). Mt ng trn (OA ) ln lt tip xc vi AC,
BD ti E, F v tip xc trong vi (O) ti G. Gi I l giao im ca AC, BD. (IEF ) ln
lt ct (IAB), (ICD) ti im th hai l H, K. Chng minh rng (GHK) tip xc vi (O),
(IAB), (ICD).
Chng minh. Ta pht biu v chng minh b sau.
B 5. Cho ng gic ABCDE ni tip (O). Gi I l giao im ca AC, BD. Trn AC, BD
ln lt ly M , N ri v ng trn (IM N ) ln lt ct (IAB), (ICD) ti P , Q. (EM N ) ln
lt ct AC, BD ti im th hai l M 0 , N 0 . (EP Q) ln lt ct (IAB), (ICD) ti im th
hai l P 0 , Q0 . Khi M 0 , N 0 , P 0 , Q0 ng vin.
Chng minh b : Qua php nghch o cc I, phng tch k 6= 0, ta nhn c cu hnh bi
ton 2.

131

Tp ch Epsilon, S 10, 08/2016

Hnh 13. Bi ton 4


Tr li bi ton.
p dng tnh cht 1 cho t gic ABCD ni tip (O) v ng thng EF ln lt ct AC, BD
ti E, F v (IEF ) ln lt ct (IAB), (ICD) ti im th hai l H, K, ta c vi mi im
X nm trn (O), (XEF ), (XHK), (O) c im chung Y 6= X.
Khi (XM N ) tip xc vi (O) ti G th X Y G. Suy ra (GHK) tip xc vi (O) ti G.
p dng b cho t gic ABCD ni tip (O) v ng thng E1 F1 ln lt ct AC, BD ti
E1 , F1 v (IE1 F1 ) ln lt ct (IAB), (ICD) ti im th hai l H1 , K1 , ta c E2 , F2 , H2 ,
K2 ng vin, trong E2 , F2 , H2 , K2 ln lt l giao im th hai ca (GE1 F1 ) vi AC, BD;
(GH1 K1 ) vi AB, CD.
Khi (GE1 F1 ) ln lt tip xc vi AC, BD ti E, F th E1 E2 E, F1 F2 F . Suy
ra H1 H2 H, K1 K2 K. Do (GHK) ln lt tip xc vi (IAB), (ICD) ti H,
K.
Bi ton 5. Cho t gic ABCD ni tip (O). Gi I l giao im ca AC, BD. Mt ng
thng d qua I ln lt ct (IAB), (ICD) ti E, F . Chng minh rng dy cung XY ca (O)
song song vi d khi v ch khi t gic XY F E ni tip.

132

Tp ch Epsilon, S 10, 08/2016

Hnh 14. Bi ton 5


Chng minh. Php nghch o cc I, phng tch k 6= 0, bin A, B, C, D, E, F , X, Y thnh
A0 , B 0 , C 0 , D0 , E 0 , F 0 , X 0 , Y 0 . Khi AC, BD, (IAB), (ICD), d, XY ln lt bin thnh
A0 C 0 , B 0 D0 , A0 B 0 , C 0 D0 , d, (IX 0 Y 0 ) v A0 , B 0 , C 0 , D0 , X 0 , Y 0 ng vin.
Chiu thun. Gi s dy cung XY ca (O) song song vi d. Suy ra (IX 0 Y 0 ) tip xc vi d ti
I.
p dng tnh cht 2 cho t gic A0 B 0 C 0 D0 ni tip v ng thng d qua I, ln lt ct A0 B 0 ,
C 0 D0 ti E 0 , F 0 , ta c ng trn qua X 0 v tip xc vi d ti I, (X 0 E 0 F 0 ), (A0 B 0 C 0 D0 ) c
im chung khc X 0 . Suy ra Y 0 (X 0 E 0 F 0 ). Do XY F E ni tip.
Chiu nghch. Gi s XY F E ni tip.
V dy cung XZ k d . Theo chiu thun, XZF E ni tip. Suy ra Z Y , hay XY k d.
Bi ton 6. Cho lc gic ABCDEF ni tip (O) c AD, BE, CF ng quy ti I. Mt ng
thng d qua I, ln lt ct tip tuyn ti A, D, B, E, C, F ca (O) ti M , N , P , Q, R, S. Khi
vi mi im X nm trn (O) th (XM N ), (XP Q), (XRS), (O) c mt im chung khc
X.

133

Tp ch Epsilon, S 10, 08/2016

Hnh 15. Bi ton 6


Chng minh. p dng tnh cht 2 cho t gic ACDF v ng thng d qua I ct AA, DD
ti M , N , ta c ng trn qua X v tip xc d ti I, (XM N ), (O) c im chung khc
X. Chng minh tng t ng trn qua X v tip xc d ti I, (XP Q), (XRS), (O) c im
chung khc X. Ta i n kt lun bi ton !
Tng qut ca bi ton 1 l b Poncelet m rng (tham kho [1]). S dng b 4, ta c
mt li gii cho n. Ta cng c th tng qut tnh cht 1, 2, 3 v bi ton 1, 5 theo cch tng
t. Mt kha cnh khc, cc bi ton 2, 3, 4 u c li gii ngn gn hn, tc gi xin dnh cho
bn c. Nhng vi cch tip cn thng qua bi ton 1 v mt s tnh cht khc trong cu hnh
ca n, ta thy c mi lin quan gia cc bi ton trn.

Ti liu
[1] Trn Minh Ngc, B Poncelet m rng v ng dng, Geometry Blog
https://tranminhngocctlhp.wordpress.com/
[2] Nguyn Vn Linh, ng dng ca t s phng tch, Euclidean Geometry Blog
https://nguyenvanlinh.wordpress.com/
[3] Lachlan, "Coaxal Circles", Ch. 13 in An Elementary Treatise on Modern Pure Geometry.
London: Macmillian, pp. 199-217, 1893
Email: tranminhngoc.ctlhp@gmail.com

134

Tp ch Epsilon, S 10, 08/2016

NH L CAUCHY - DAVENPORT V NG DNG


Trn Minh Hin
(Trng THPT chuyn Quang Trung, Bnh Phc)

GII THIU
Ni dung ca chuyn ny trnh by mt s cch chng minh nh l Cauchy - Davenport v nu mt s ng dng ca n. c c s so snh s khc nhau gia cc kt
qu trn Z v trn Zp (tp cc s d theo modulo p, p nguyn t), trong phn 1 trnh by
mt s kt qu kh quen thuc ca cc tp tng trn Z. Bn c quan tm n cc kt qu
su hn v ni dung ny c th tham kho ti liu [3]. V cc kt qu hay trn Zp bn
c c th tham kho cc kt qu trong [1], [2]. Trong ton b chuyn ny, thng nht
k hiu
A C A D fa C b j a 2 A; b 2 Bg; c  A D fc  a j a 2 Ag:

Li gii ca mt s bi tp c th tng i di, nhng trong cha nhiu gii thch


quan trng bn c hiu r hn v tng.

1. Mt s nh gi tp tng trn Z
nh l 1. Cho A l mt tp gm k s nguyn. Khi j2Aj > 2k
s nguyn v nu j2Aj D 2k 1 th A phi l mt cp s cng.

1. Nu A l mt tp gm k

Li gii. t A D fa0 ; a1 ; : : : ; ak 1 g vi a0 < a1 < a2 <    < ak 1 . Khi tp 2A s cha k


s nguyn 2ai , vi i D 0; 1; 2; : : : ; k 1 v k 1 s nguyn ai 1 C ai , vi i D 1; 2; : : : ; k 1.
Do
2ai
T y suy ra j2Aj > 2k
Nu j2Aj D 2k
ai

1 C ai

< ai

< ai

C ai < 2ai ;

8i D 1; 2; : : : ; 2k

1:

1.

1, khi mi phn t ca 2A hoc c dng 2ai hoc ai


1 C ai C1

< ai C ai C1 ;

ai

1 C ai

C ai C1 ) ai

ai

< 2ai < ai C ai C1 ;

C ai . Do

8i D 1; 2; : : : ; k

nn suy ra
2ai D ai

D ai C1

ai ; 8i D 1; 2; : : : ; k

iu ny chng t A l mt cp s cng. nh l c chng minh.


135

1:

Tp ch Epsilon, S 10, 08/2016

Tng qut hn, ta c nh gi sau y.


nh l 2. Cho n > 2 v A1 ; A2 ; : : : ; An l cc tp hp hu hn s nguyn. Khi
jA1 j C jA2 j C    C jAn j

.n

1/ 6 jA1 C A2 C    C An j 6 jA1 j  jA2 j    jAn j:

Li gii. Bt ng thc jA1 C A2 C    C An j 6 jA1 j  jA2 j : : : jAn j l hin nhin, do s biu


din dng a1 C a2 C    C an .ai 2 Ai / l jA1 j  jA2 j : : : jAn j.
Ta chng minh bt ng thc cn li bng quy np. Vi n D 2, t
A1 D fa0 ; a1 ; : : : ; ak 1 g;

A2 D fb0 ; : : : ; bl

1g

vi a0 < a1 <    < ak 1 ; b0 < b1 <    < bl 1 . Gi s jA1 j D k 6 l D jA2 j. Khi tp


A1 C A2 cha 2k 1 C .l k/ cc phn t phn bit sau
a0 C b0 < a0 C b1 < a1 C b1 < a1 C b2 <   
< ai C bi < ai C bi C1 < ai C1 C bi C1 <   
< ak 1 C bk 1 < ak 1 C bk < ak 1 C bkC1 <    < ak
Do

jA1 C A2 j > .2k

1/ C .l

k/ D jA1 j C jA2 j

Vy n D 2 nh l ng. Gi s kt lun ng cho mi 6 n


vi n tp th

C bl

1:

1:

1 tp hu hn cc s s. Khi

jA1 j C jA2 j C    C jAn j D j.A1 C A2 C    C An 1 / C An j


> jA1 C A2 C    C An 1 j C jAn j 1
> jA1 j C jA2 j C    C jAn 1 j .n 2/ C jAn j
D jA1 j C jA2 j C    C jAn j .n 1/:

Chng t kt lun ng vi n. Theo phng php quy np th bi ton ng cho vi mi n tp


hu hn cc s nguyn.
B 1. Cho A; B l hai tp hp hu hn cc s nguyn v jAj D jBj D k. Nu jA C Bj D
jAj C jBj 1 th A v B l hai cp s cng c cng cng sai.
Li gii. t A D fa0 ; a1 ; a2 ; : : : ; ak 1 g v B D fb0 ; b1 ; b2 ; : : : ; bk 1 g vi a0 < a1 <    <
ak 1 , b0 < b1 <    < bk 1 . Khi A C B cha mt dy gm 2k 1 s nguyn tng dn sau
a0 C b0 < a0 C b1 < a1 C b1 < a1 C b2 <   
< ai 1 C bi < ai C bi < ai C bi C1 C ai C1 C bi C1 <   
<    < ak 1 C bk 1 :
Do jAj C jBj D 2k

1, suy ra dy s nguyn trn cha ton b cc phn t ca tp A C B. Do


ai

C bi < ai C bi < ai C bi C1

v
ai

C bi < ai

C bi C1 < ai C bi C1

136

Tp ch Epsilon, S 10, 08/2016

dn n
ai

C bi C1 D ai C bi ) ai

ai

D bi C1

bi ; 8i D 1; 2; : : : ; k

.1/:

.2/:

Tng t, bt ng thc
ai

C bi

< ai

ai

C bi

< ai C bi

v
dn n
ai

C bi D ai C bi

T (1) v (2), t q D a1
ai

) ai

C bi < ai C bi
1

< ai C bi

ai

D bi

bi

D q; 8i D 1; 2; : : : ; k

bi

1 ; 8i

D 1; 2; : : : ; k

a0 th
ai

D bi

1:

Chng t A; B l hai cp s cng c cng cng sai q.


Tng qut hn ta c kt qu sau. Bn c quan tm chng minh ca n tham kho b 1.3,
trang 10, ti liu tham kho [3].
nh l 3. Cho A; B l hai tp hu hn cc s nguyn, vi jAj D k > 2; jBj D l > 2. Nu
jA C Bj D k C l 1 th A; B l hai cp s cng c cng cng sai.

2. nh l cauchy - davenport
Trong phn ny, nu khng ch thch g thm th tt c cc tp hp c ly l cc tp con trong
trng Zp , tp cha h thng d y modulo p.
nh l 4 (nh l Cauchy - Davenport). Cho p l s nguyn t v A; B l hai tp con ca
Zp . Khi
jA C Bj > minfp; jAj C jBj 1g:

2.1. Chng minh thng qua php bin i Davenport


Cho B l tp con ca Zp , 0 2 B; jBj > 2 v A C B Zp . Khi ta c mt s nhn xt sau:
 A C B l tp con thc s ca A C 2B . Tht vy, vi x 2 A C B, khi
x D a C b.a 2 A; b 2 B/ ) x D
a C
b C
0 2 A C 2B:
2A

2B

2B

Vy A C B  A C 2B. Mt khc nu A C B D A C 2B th ta s c A C 2B D A C 3B.


Tht vy:
137

Tp ch Epsilon, S 10, 08/2016

x 2 A C 2B th x D a C b1 C b2 .a 2 A; b1 ; b2 2 B/ nn
x D
a C b1 C b2 C
0 2 A C 3B:

2A

2B

2B

2B

Do x 2 A C 3B nn A C 2B  A C 3B.

x 2 AC3B th x D aCb1 Cb2 Cb3 .a 2 A; b1 ; b2 ; b3 2 B/. Do aCb1 Cb2 2 AC2B


m A C 2B D A C B nn xy ra
a C b1 C b2 D a0 C b 0 .a0 2 A; b 0 2 B/:
Khi x D a C b1 C b2 C b3 D
a0 C
b 0 C b3 2 A C 2B. Do

2A

A C 3B  A C 2B.

2B

2B

V hon ton tng t ta c A C B D A C 2B D A C 3B D    . Nhng khi vi a 2 A


v 0 b 2 B (b tn ti do jBj > 2) th a C nb 2 A C B; 8n D 0; 1; 2; : : :. iu ny
phi xy ra A C B D Zp , tri vi gi thit.
 t X D .A C 2B/n.A C B/, khi X ;. t
Bx D fb 2 Bjx

b 2 A C Bg;

Bx D BnBx :

Khi tp Bx c gi l bin i Davenport ca tp B.


Khi 0 62 Bx ;. Tht vy, nu 0 2 Bx th x 0 2 A C B hay x 2 A C B, mu
thun do x 2 X . Ngoi ra Bx ; v vi x 2 X , khi
x D
a C b1 C b2 ;

2A

do
x

2B

2B

b1 D a C b2 2 A C B ) b1 2 Bx :

Ta c 0 2 Bx  B, v .A C Bx / [ .x Bx /  A C B .1/ (tnh cht ny hin nhin


do nh ngha ca hai tp Bx ; Bx ). Ngoi ra ta c .A C Bx / \ .x Bx / D ; .2/.
Tht vy, gi s c
bx ) x

a C bx D x

2Bx

Do x
bx 2 Bx .

2A

2Bx B

bx 2 A C B ) bx 2 Bx (theo nh ngha ca Bx ), mu thun vi

T (1) v (2) ta c
V jx

2Bx

bx D
a C bx 2 A C B:

jA C Bj > jA C Bx j C jx

Bx j D jBx j D jBj

jBx j nn ta c

jA C Bj > jA C Bx j C jBj
138

Bx j:
jBx j ./:

Tp ch Epsilon, S 10, 08/2016

Chng minh nh l 2.1. Gi s nh l khng ng, khi tn ti hai tp con A; B ca Zp m


A C B Zp .3/ v
jA C Bj 6 jAj C jBj 2 .4/:

Kt qu (3), v (4) s khng thay i nu thay B bi b C B (vi b 2 B ty ) (Tht vy, (3)


suy ra cc phn t ca A C B khng lp thnh mt h thng d modulo p, khi cc phn t
ca tp A C . b C B/ chng qua l cc phn t ca tp A C B tnh tin sang tri b n v
nn cng khng th lp thnh mt h thng d y modulo p. Cn tnh cht .4/ hin nhin
khi thay B bi b C B v j b C Bj D jBj).
Do , ta c th gi s 0 2 B. Trong tt c cc cp A; B tha mn (3), (4) ta chn cp A; B m
jBj nh nht. Khi jBj > 2 (v nu jBj 6 1, khi jACBj D jAj, cn jAjCjBj 2 6 jAj 1
th (4) khng xy ra.) Vy ta c 0 2 B; jBj > 2; A C B Zp . Do tn ti php bin i
Davenport Bx ca B (*) xy ra. Do 1 6 jBx j < jBj. Do Bx  B nn A C Bx  A C B, suy
ra jA C Bx j < jA C Bj < p. Kt hp (*) v (4) ta c
jA C Bx j 6 jA C Bj

jBj C jBx j 6 jAj C jBx j

2;

tc (4) li ng cho Bx , m Bx  B, mu thun vi tnh nh nht ca jBj. Vy iu phn chng


l sai. nh l c chng minh hon ton.

2.2. Chng minh bng a thc nhiu bin


B 2. Cho a thc P .x1 ; x2 ; : : : ; xn / l mt a thc n bin h s thc vi bc ca P theo
bin xi ln nht l ti , 1 6 i 6 n. Gi S1 ; S2 ; : : : ; Sn l cc tp con ca R v jSi j D ti C1; 8i D
1; 2; : : : ; n. Nu P .s1 ; s2 ; : : : ; sn / D 0 vi mi .s1 ; s2 ; : : : ; sn / 2 S1  S2      Sn th P  0.
Li gii. Ta chng minh bng quy np theo n, s cc bin ca a thc P . Vi n D 1, P .x1 / l
a thc mt bin x1 , c bc l t1 . V P .x1 / D 0 ti jS1 j D t1 C 1 im, nn theo tnh cht ca
a thc mt bin th P  0. Vy b ng cho n D 1. Gi s b ng cho mi a thc
n 1 bin, n > 2. Xt a thc P .x1 ; x2 ; : : : ; xn / l a thc n bin. Ta vit l a thc ny nh
l a thc mt bin xn :
Pn .xn / D P .x1 ; x2 ; : : : ; xn / D

tn
X

Qi .x1 ; : : : ; xn 1 /xni :

i D0

V a thc Pn l a thc mt bin xn , c bc tn v bng 0 ti tn C 1 im trong Sn . Do li


theo tnh cht a thc mt bin ta c
Pn  0 ) Qi .x1 ; x2 ; : : : ; xn 1 / D 0; 8i D 0; 1; : : : ; tn :

(n y mi chng t c cc a thc n 1 bin Qi .i D 1; 2; : : : ; tn / bng 0 ti cc im


ca S1  S2      Sn 1 . Tip theo phi chng minh cc a thc ny ng nht 0) Nhng vi
mi i D 0; 1; : : : ; tn th
Qi .x1 ; x2 ; : : : ; xn 1 / D 0; 8.x1 ; x2 ; : : : ; xn 1 / 2 S1  S2      Sn

nn theo gi thit quy np th Qi  0; 8i D 0; 1; : : : ; tn . T chng t P  0. Kt lun bi


ton ng vi n. Theo phng php quy np, th b ng cho mi a thc n bin.

139

Tp ch Epsilon, S 10, 08/2016

nh l 5. Cho a thc P .x1 ; x2 ; : : : ; xn / l mt a thc n bin h s thc. Gi S1 ; S2 ; : : : ; Sn


l cc tp con ca R. nh ngha a thc
Y
.xi s/ :
Qi .xi / D
s2Si

Nu P .s1 ; s2 ; : : : ; sn / D 0 vi mi .s1 ; s2 ; : : : ; sn / 2 S1  S2      Sn , th tn ti cc a thc


R1 ; : : : ; Rn 2 Rx1 ; : : : ; xn vi deg Ri 6 deg P deg Qi sao cho
P D R1 Q1 C R2 Q2 C    C Rn Qn :

Li gii. t ti D jSi j 1. Ta xt thut ton sau y: "Nu c mt n thc cx1m1 : : : xn xnmn trong
P m mi > ti , vi mt ch s i no , th ta thay th i lng ximi bi ximi ximi .i C1/:Qi .xi / ".
 Do deg Qi D ti C 1 v Qi l a thc monic, khi n thc x1m1 : : : xn xnmn s c
thay th bi mt s n thc khc, nhng tt c cc n thc u c bc nh hn
m1 C m2 C    C mn .
 Do , sau mi bc ca thut ton ny, th tng cc bc ca tt c cc n thc trong P
s gim thc s. Nhng do tng ban u l hu hn, nn thut ton ny s kt thc sau
mt s hu hn ln thc hin.
Gi P l a thc cui cng nhn c sau mi ln thc hin thut ton trn. Khi php bin
i


cx1m1    ximi    xn xnmn 7! cx1m1    ximi ximi .i C1/ Qi .xi /    xnmn
tng ng l mt php tr ca a thc P cho mt a thc c dng Ri0 :Qi , vi
Ri0 D cx1m1    ximi

deg Ri0 D m1 C    C mn

.i C1/

: : : xnmn

.ti C 1/ 6 deg P

deg Qi :

Do qu trnh ny thc hin cho tt c cc n thc trong P , khi


P DP

R1  Q1 C    C Rn  Qn

( y Ri l tng ca cc a thc Ri0 trn (mi n thc trong P c mi > ti ta u c mt


Ri0 . Thc hin thut ton cho tt c cc n thc ny, ta s c cc Ri0 khc nhau). V Ri0 c
bc u 6 deg P deg Qi nn) ta c
deg Ri 6 deg P

deg Qi :

V P nhn c cui cng ca tht ton trn, nn mi bin xi trong P c ly tha cao nht
l ti . V
Qi .xi / D 0; 8xi 2 Si
nn suy ra

P .s1 ; s1 ; : : : ; sn / D P .s1 ; s2 ; : : : ; sn /;

Theo b 2.2 th P  0, do

8.s1 ; s2 ; : : : ; sn / 2 S1  S2      Sn :

P D R1 Q1 C R2 Q2 C    C Rn Qn :
140

Tp ch Epsilon, S 10, 08/2016

nh l 6 (Combinatorial Nullstellensatz). Cho a thc P .x1 ; : : : ; xn / 2 Rx1 ; : : : ; xn c


bc deg P D t1 C    C tn , vi ti l cc s nguyn khng m v h s ca n thc x1t1    xntn
khc 0. Nu S1 ; : : : ; Sn l cc tp con ca R tha jSi j > ti , khi tn ti mt b .s1 ; : : : ; sn / 2
S1  S2      Sn sao cho
P .s1 ; s2 ; : : : ; sn / 0 :
Li gii. Ta ch cn chng minh kt lun bi ton trong trng hp jSi j D ti C 1 (v nu
jSi j > ti C 1 th hin nhin kt lun bi ton ng, nu ta chng minh c khng nh vi
jSi j D ti C 1). Gi s kt lun bi ton khng ng, tc l
P .s1 ; s2 ; : : : ; sn / D 0; 8.s1 ; : : : ; sn / 2 S1  S2      Sn :
Khi , theo nh l 2.3 th
P D R1  Q1 C R2  Q2 C    C Rn  Qn
vi deg Ri 6 deg P deg Qi ; 8i D 1; 2; : : : ; n. Theo gi thit, h s ca x1t1    xntn v phi
khc 0. Tuy nhin, do deg Ri  Qi 6 deg P v mt n thc trong Ri  Qi m c bc t1 C    C tn
khi v ch khi (theo cch xy dng trong 2.3), chng ta ly Ri nhn vi xiti C1 trong khai trin
Y
:
Qi .xi / D
.x si / D xiti C1 C


s2Si
cc tha s cn li c bc 6 ti
T y suy ra mi n thc trong R1 :Q1 C    C Rn Qn m c tng bc bng t1 C    C tn
u phi chia ht cho xiti C1 , vi mt ch s i 2 f1; 2; : : : ; ng. Do n thc x1t1    xntn nm
trong R1  Q1 C    C Rn Qn c tng bc bng t1 C    C tn cng phi chia ht cho xiti C1 . iu
ny ch xy ra c khi h s ca x1t1    xntn bng 0. iu ny tri vi gi thit. Vy iu phn
chng l sai. nh l c chng minh.

Ch : Cc nh l trn ng cho a thc trn trng R, n cng vn ng trn trng Zp .


Trc khi p dng nh l 2.4 chng minh nh l 2.1 ta s thy mt ng dng p ca n
trong bi ton kh ni tip sau: IMO 2007.
V d 1 (IMO 2007). Cho n l mt s nguyn dng. t
S D f.x; y; z/ j x; y; z 2 f0; 1; : : : ; ng; x C y C z > 0g
l mt tp cha .n C 1/3 1 im trong khng gian ba chiu. Xc nh s mt phng t nht,
sao cho hp ca chng cha S , nhng khng cha im .0; 0; 0/.
Li gii. D dng nhn thy 3n mt phng c phng trnh x C y C z D i; 8i D 1; 2; : : : ; 3n
l t c yu cu bi ton. Ta chng minh 3n chnh l s mt phng nh nht cn tm. Gi s
ngc li, tn ti cc mt phng P1 ; P2 ; : : : ; Pk .k 6 3n 1/ m hp ca chng ph ht tp S
v khng cha im .0; 0; 0/. Mi mt phng Pi xc nh bi mt phng trnh
ai x C bi y C ci z C di D 0:
141

Tp ch Epsilon, S 10, 08/2016

Nhn thy di 0 do .0; 0; 0/ 2


6 Pi . Hp ca cc mt phng Pi ph ht cc im ca S nu v
ch nu
k
Y
P .x; y; z/ D
.ai x C bi y C ci z C di / D 0; 8.x; y; z/ 2 S:
i D1

V 0 62 Pi nn P .0; 0; 0/ 0. t S1 D S2 D S3 D f0; 1; 2; : : : ; ng. Khi


S [ f.0; 0; 0/g D S1  S2  S3 :

Xt a thc

Q.x; y; z/ D P .x; y; z/
vi c l hng s v bng c D

c:

k
Y

.x

i/.y

i/.z

i/;

i D1

P .0; 0; 0/
.
. 1/3n .n/3

 Nhn thy Q.x; y; z/ D 0; 8.x; y; z/ 2 S1  S2  S3

./.

 Li do k < 3n nn deg Q D 3n. H s ca x n y n z n trong Q l c 0. Theo nh l 2.4,


tn ti mt im .x0 ; y0 ; z0 / 2 S1  S2  S3 Q.x0 ; y0 ; z0 / 0, mu thun vi (*).
T chng t iu phn chng sai. Bi ton c chng minh.
Chng minh nh l 2.1 bng nh l 2.4. Nu jAj C jBj 1 > p. Khi A C B D Zp . Tht
vy, do jAj C jBj > p nn vi mi x 2 Zp th tp A \ .x B/ ; (v nu A \ .x B/ D ;
th A; e B u l tp con ca Zp nn p > jAj C jx Bj D jAj C jBj > p, v l). T mi
phn t x 2 Zp u c th vit di dng x D a C b; a 2 A; b 2 B hay x 2 A C B. Chng t
Zp  A C B, suy ra A C B D Zp , nh l ng trong trng hp ny.
Nu jAj C jBj < p. Gi s kt lun bi ton khng ng, tc l jA C Bj 6 jAj C jBj
C l tp hp sao cho A C B  C v jC j D jAj C jBj 2. Xt a thc
Y
.x C y c/
P .x; y/ D

2. t

c2C

th deg P D jAj C jBj

2. V A C B  C nn

P .a; b/ D 0; 8.a; b/ 2 A  B: ./



2
Mt khc, h s ca x jAj 1 y jBj 1 trong P l jAjCjBj
. V jAj C jBj 2 < p nn h s ny
jAj 1
khc trong trong Zp . Do theo nh l 2.4 tn ti .a; b/ 2 A  B sao cho P .a; b/ D 0,
mu thun vi (*). Vy iu phn chng khng ng. Chng t kt lun ng trong trng hp
ny.
nh l 7. Cho p l s nguyn t, A; B l hai tp con khc rng ca Zp . nh ngha
Khi

A B D fa C bja 2 A; b 2 B; a bg:
jA Bj > minfp; jAj C jBj

3g:

.1/

jA Bj > minfp; jAj C jBj

2g:

.2/

Nu jAj jBj th ta c kt qu mnh hn

142

Tp ch Epsilon, S 10, 08/2016

Li gii.
 Nu jAj D 1 hoc jBj D 1, khi bt ng thc (2) hin nhin. Xt jAj; jBj >
2. Trong trng hp jA D jBj, khi ta ly tp B 0 D Bnfbg, vi b l mt phn t bt k
ca B. Khi p dng (2) cho hai tp A v B 0 ta c bt ng thc (1).
 T ta ch cn chng minh (2).
Trng hp jAj C jBj 2 > p. Nu p > 2 hin nhin. Xt p > 2 th p l. Ta s
chng minh ACB D Zp . Tht vy, xt g 2 Zp l mt phn t ty . t C D g B,
th jC j D jBj. Khi jAj C jC j D jAj C jBj > p C 2. Theo cng thc
jX [ Y j C jX \ Y j D jX j C jY j
ta c

Zp C jA \ C j > jA [ C j C jA \ C j D jAj C jC j > p C 2 ) jA \ C j > 2:
Gi x; y l hai phn t phn bit ca A \ C . V C D g
bx ; by 2 B sao cho
x C bx D y C by D g:

B, nn tn ti hai phn t

Ta s c c kt lun bi ton nu ch ra c x bx hoc y by (khi


g 2 A B). Gi s c hai iu trn u khng xy ra, khi x C x D y C y D
:
g ) 2.x y/ D 0. Suy ra 2::p, v l do p nguyn t l.

Xt trng hp jAj C jBj 2 < p. Gi s kt lun ca nh l khng ng, tc l


jA Bj < jAj C jBj 2. Gi C l mt tp hp trong Zp c jAj C jBj 3 phn t
v A C B  C . Xt a thc
Y
P .x; y/ D .x y/
.x C y c/:
c2C

Nhn thy rng vi mi a 2 A; b 2 B th P .a; b/ D 0. Mt khc, ta c deg P D


jC j C 1 D jAj C jBj 2. Do jAj jBj, nn h s ca x jAj 1 y jBj 1 trong P l
!
!
mCn 3
.m C n 3/
.m C n 3/
mCn 3
D
.m 2/.n 1/ .m 1/.n 2/
m 2
m 1
D

.m C n 3/
.m
.m 1/.n 2/

n/;

l khc 0 (theo modulo p) do m C n 3 < p v m n. Do theo nh l 2.4 tn


ti .a; b/ 2 A  B sao cho P .a; b/ 0, mu thun. Vy iu gi s l sai. Trng
hp ny c chng minh.
Vy nh l c chng minh hon ton.
nh l 8 (nh l Cauchy - Davenport tng qut). Gi s p l s nguyn t v A1 ; A2 ; : : : ; An
l cc tp con ca Zp (n > 2). Khi
jA1 C A2 C    C An j > minfp; jA1 j C jA2 j C    C jAn j
143

.n

1/g:

Tp ch Epsilon, S 10, 08/2016

Li gii. Ta chng minh bng phng php quy np. Vi n D 2, th kt qu trn l nh l 2.1.
Gi s nh l ng vi n 1. Khi , vi n tp A1 ; : : : ; An th
jA1 C A2 C    C An j D j.A1 C : : : C An 1 / C An /j
> minfp; jA1 C : : : C An 1 j C jAn j 1g .p dng cho n D 2 tp/
> minfp; minfp; jA1 j C jA2 j C : : : C jAn 1 j .n 2/g C jAn j 1g .gi thit quy np cho n
- Nu minfp; jA1 j C jA2 j C    C jAn 1 j

1 tp/:

2/g D p./, khi

.n

minfp; minfp; jA1 jCjA2 jC  CjAn 1 j .n 2/gCjAn j 1g D minfp; pCjAn j 1g D p.1/.do jAn j > 1/:
Mt khc t (*) th jA1 j C jA2 j C    C jAn 1 j

.n

jA1 j C jA2 j C    C jAn 1 j C jAn j

.n

) minfp; jA1 j C jA2 j C    C jAn j

T (1) v (2) suy ra

2/ > p nn
1/ > p C jAn j
.n

1/g D p:

1>p
.2/

minfp; minfp; jA1 jCjA2 jC  CjAn 1 j .n 2/gCjAn j 1g D minfp; jA1 jCjA2 jC  CjAn j .n 1/g.3/:
- Nu minfp; jA1 j C jA2 j C    C jAn 1 j

.n

2/g D jA1 j C jA2 j C    C jAn 1 j

.n

2/ th

minfp; minfp; jA1 j C jA2 j C    C jAn 1 j .n 2/g C jAn j 1g


D minfp; jA1 j C jA2 j C    C jAn 1 j C jAn j .n 1/g: .4/
Trong c hai trng hp (3) v (4) ta u c
jA1 C A2 C    C An j > minfp; jA1 j C jA2 j C    C jAn j

.n

1/g

tc nh l ng cho n. Theo phng php quy np ta c iu phi chng minh.


nh l 9 (nh l Vosper). Cho p l s nguyn t v A; B l hai tp con ca Zp sao cho
jAj; jBj > 2 v jA C Bj 6 p 2. Khi jA C Bj D jAj C jBj 1 khi v ch khi A; B l hai
cp s cng c cng cng sai.
Ch : Bn c cn tm hiu chng minh nh l ny, c th tham kho mt chng minh kh
s cp trong nh l 3.2 ti liu tham kho [4]. Ngoi ra trong ti liu ny, nh l 3.1 cng
cung cp thm mt cch chng minh s cp bng quy np, theo qua php bin i e. Chnh cch
chng minh ny cho ta mt li gii cho bi tp 4.11.

3. Mt s ng dng
V d 2 (Poland 2003). Chng minh rng vi mi s nguyn t p > 3, tn ti cc s nguyn
x; y; k sao cho 0 < 2k < p v x 2 C y 2 D kp C 3.

144

Tp ch Epsilon, S 10, 08/2016

Li gii. Kt qu ca bi ton khng thay i nu thay x; y bi p x; p y. Do ta s ch


p
ra cc s x; y trong min 0 6 x; y < tha mn iu kin bi ton. Khi x; y nm trong min
2
ny th hin nhin phn thng k s tha 2k < p, v
x2 C y2 <

p2
p2
p2
C
D
:
4
4
2

t S l tp cha tt c bnh phng theo modulo p. Ch x 2  .p x/2 .mod p/; 8x 2


n
h p io
pC1
1; 2; : : : ;
. Do jS j D
. Theo nh l 2.1 th
2
2


pC1
jS C S j > minfp; 2jS j 1g D min p; 2:
1 D p ) jS C S j D p:
2
iu ny chng t mi thng d modulo p u c th vit c thnh tng ca hai s chnh
phng. V bi ton l trng hp c bit vi kt qu bng 3.
V d 3. Cho p l s nguyn t. Cho trc p 1 s nguyn sao cho khng c s no chia ht
cho p. Chng minh rng ta c th i du mt vi s trong chng tng ca cc s thu c
chia ht cho p.
Li gii. Gi p

1 s nguyn l a1 ; a2 ; : : : ; ap 1 . Vi mi i D 1; 2; : : : ; p

1, t

Ai D fai ; ai g .mod p/:


Khi jAi j D 2; 8i D 1; 2; : : : ; p

1 v ai 6

ai .mod p/. p dng nh l 2.6 ta c

jA1 C A2 C    C Ap 1 j > minfp; jA1 j C    C Ap

.p

2/g D minfp; pg D p:

T y suy ra
jA1 C A2 C    C Ap 1 j D p

hay tp A1 C A2 C    C Ap
ca bi ton.

cha mt h thng d y modulo p. y chnh l kt lun

V d 4. Gi s p > 2 l s nguyn t v cho p 1 s nguyn a1 ; a2 ; : : : ; ap 1 tha mn


:
:
a1    ap 1 6 :: p v a1 C a2 C    C ap 1 6 :: p. Chng minh rng c th chia tp ny thnh hai
nhm ri nhau tng cc phn t ca hai nhm ng d vi nhau theo modulo p.
:
Li gii. t Ai D f0; ai g .mod p/. Th do ai 6 :: p; 8i D 1; 2; : : : ; p
1; 2; : : : ; p 1. Theo nh l 2.6 th

1 nn jAi j D 2; 8i D

jA1 CA2 C  CAp 1 j > minfp; jA1 jC  CjAp 1 j .p 2/g D minfp; 2.p 1/ .p 2/g D p:
Do A1 C A2 C    C Ap

D Zp . Khi tn ti bi 2 Ai ; i D 1; 2; : : : ; p

b1 C b2 C    C bp

a1 C a2 C    C ap
2
145

.mod p/:

1 sao cho

Tp ch Epsilon, S 10, 08/2016

V mi phn t bi 2 Ai th bi hoc bng 0, hoc bng ai . Tuy nhin do a1 Ca2 C  Cap 1 6 0


.mod p/ nn tn ti mt s phn t bi khc 0. Gi cc phn t l aj1 ; : : : ; ajk , th thay vo
ta c
a1 C a2 C    C ap 1
.mod p/
aj 1 C    C aj k 
2
hay

2 aj1 C    C ajk  a1 C    C ap 1 .mod p/
hay

aj1 C    C ajk  .a1 C    C ap 1 /

bi ton c chng minh.

.aj1 C    C ajk /

.mod p/

V d 5 (USAMO 2009). Cho n l s nguyn dng. Xc nh tp con A ln nht ca tp


f n; n C 1; : : : ; n 1; ng sao cho trong A khng tn ti ba phn t a; b; c (khng nht thit
phn bit) m a C b C c D 0.
Phn tch v hng dn gii. Nhn xt 0 62 A (v nu 0 2 A th 0 D 0 C 0 C 0, mu thun vi
tnh cht ca tp A). t
AC D A \ f1; 2; : : : ; ng;

A D A \ f n; n C 1; : : : ; 1g:

 Nhn xt AC C A v A l hai tp con ri nhau ca f n; n C 1; : : : ; n


vy, nu .AC C A / \ . A/ ;, khi tn ti s c m
c D iCj D

.i 2 f1; 2; : : : ; ng\A;

j 2 f n; nC1; : : : ; 1g\A;

1; ng. Tht
k 2 A/ ) i Cj Ck D 0;

mu thun vi nh ngha ca tp A.
 V AC C A v A l hai tp con ri nhau ca tp f n; n C 1; : : : ; n
c 2n C 1 phn t) nn

1; ng (tp ny

jAC C A j C j Aj 6 2n C 1 ) 2n C 1 > jAC C A j C jAj:

DjAj

Ngoi ra theo nh l 1.2 th

jAC C A j > jAC j C jA j


Do

2n C 1 > jAC j C jA j 1 C jAj D 2jAj



DjAj

1:
1 ) jAj 6 n C 1:

n y d on tp A ln nht c n C 1 phn t. Vic tip theo cn ch ra mt tp A c n C 1


phn t. tha mn bi ton, ta ch cn ly tp A cc phn t ln nht v nh nht ca tp
bi. l tp
8
9

>

>
<
=
hni
hni
AD
n; n C 1; : : : ;
1;
C 1; : : : ; n :

>

2
>
:
;
phn m

phn dng

146

Tp ch Epsilon, S 10, 08/2016

Ly ba phn t a; b; c ty ca tp A ny. Nu c ba phn t cng thuc phn m, hoc phn


dng ca tp A th hin nhin a C b C c 0. Nu hai s thuc phn dng, gi s a; b, v
mt s c thuc phn m, khi c > n v
hni
h n
i
hni
n
C1C
C1D2
C 1 > 2  D n > c ) a C b C c > 0:
aCb >
2
2
2
2

Tng t kim tra nu hai s thuc phn m, mt s thuc phn dng th tp A ny tha mn
bi. Tr ngi duy nht cn li l tp A ny c n C 1 phn t ch khi n l. Cn nu n chn,
th tp A ny ch c n phn t, khng t c ti n C 1 phn t. Tuy nhin ta s ch ra nu
n chn th tp A khng th c n C 1 phn t c nh di y. Tht vy, gi s jAj D n C 1
vn ng vi n chn. Do A  f n; : : : ; 1g v AC  f1; 2; : : : ; ng nn
AC C A  f n C 1; n C 2; : : : ; n

2; n

1g:

Li v AC CA v A l hai tp hp ri nhau ca tp 2nC1 phn t f n; nC1; : : : ; n 1; ng


nn 2n C 1 D jAC C A j C j Aj ch c th xy ra khi n; n 2 A, tc l n; n 2 A. V
n 2 A, theo cu trc tp A v nh ngha tp AC , th trong mi cp phn t sau y (n chn)
o
nno
nn
n
1; C 1 ;
f1; n 1g; f2; n 2g; : : : ;
2
2
2

n
ch c nhiu nht mt phn t trong mi cp thuc vo AC . Dn n jAjC 6 , tng t
2
n
jA j 6 . Do
2
n n
jAj D jAC j C jA j 6 C D n
2
2
mu thun vi jAj D n C 1. Vy bi ton c chng minh hon ton.

V d 6 (Vit Nam TST 2012). Cho s nguyn t p > 17. Chng minh rng t D 3 l s
nguyn dng ln nht tha mn iu kin: Vi cc s nguyn bt k a; b; c; d sao cho s abc
cho po v a C b C c chia ht cho p th tn ti cc s nguyn x; y; z thuc tp
nkhng chia hht
pi
:
0; 1; 2; : : :
1 sao cho ax C by C cz C d ::p.
t
Li gii. Nhn xt Nu fu1 ; : : : ; uk g cha mt h thng d y modulo p th tp d C
fu1 ; : : : ; u
h kpgicng cha mt h thng d y modulo p.
t k D
1, v A D f0; 1; 2; : : : ; kg. t
t
S D fax C by C czj0 6 x; y; z 6 kg D aA C bA C cA

.mod p/:

 Vi t D 3, theo yu cu bi ton v nhn xt trn th bi ton tng ng vi vic


chng minh nu t D 3 th S cha mt h thng d y modulo p, tc cn chng
minh jS j D p. t C D fax C by C czj 1 6 x; y; z 6 1g: Do a C b C c  0 .mod p/
nn vi mi b .x; y; z 2 A/ th
ax C by C cz  a.x

k/ C b.y

147

k/ C c.z

k/ .mod p/:

Tp ch Epsilon, S 10, 08/2016

k; y  k; z k/, nn ta thc hin i


k
xng tp A s dng tp C , bng cch t L D
v xt tp
2

Tc l mi b .x; y; z/ 2 A tng ng vi b .x
S 0 D fax C by C czj

th

jS j > jS 0 j

L 6 x; y; z 6 Lg

.du bng xy ra khi v ch khi k chn/:

 Khi tp S 0 vit c di dng

S0 D C
C    C C :
C C
L ln

Theo nh l 2.6 ta c

jS 0 j > minfp; L:jC j

.L

1/g:

./

c c kt lun bi ton l jS j D p, ta cn ch ra jS 0 j D p, do theo (*) ta s cn


chng minh
LjC j .L 1/ > p ./:
 Vi ch a; b; c khng chia ht cho p nn tp f0; a; b; c; a; b; c; a b; b c; c ag
gm 10 phn t i mt phn bit theo modulo p nm trong tp C . Do jC j > 10. Vy
c (**) ta s chng minh
10L

.L

1/ > 0 , 9L C 1 > p:

.  /

V p nguyn t, nn xt hai trng hp

hp i
p dng p D 6a C 1. Do p > 17 nn a > 3. Khi k D
3


k 
1
LD 2 D a
D a 1. Do (***) tng ng
2
9.a

1 D 2a

1, nn

1/ C 1 > 6a C 1 ) a > 3

.ng/:
hp i
p dng p D 6a C 5. Do p > 17 nn a > 2. Khi k D
3
 
L D k2 D a. Do (***) tng ng
9.a/ C 1 > 6a C 5 , a >

1 D 2a, nn

4
) a > 2 .ng, do a nguyn/:
3

Vy trong mi trng hp ca p th (***) u ng. Vy bi ton c chng minh trong


trng hp t D 3.
p 1
 Ta chng minh t D 4 khng tha mn. Tht vy, chn a D b D 1; c D 2 v d D
.
2

p

Khi vi x; y; z 2 A D 0; 1; 2; : : : ; 4
1 th
h p i
 p 1
h p i
 p 1
p 1
2
1 C
6 x C y 2z C
62
1 C
4
2
2
2

p
p
xDyD0;zD 4 1
xDyD 4 1;zD0
148

Tp ch Epsilon, S 10, 08/2016

hay

3
p 1
5
6 x C y 2z C
6p
:
2
2
2


5
3
;p
khng c s no chia ht cho p, tc khng tn ti
Trong khong gi tr
2
2
x; y; z tha mn.

Bi ton c chng minh.


V d 7 (IMO Shortlist 2007). Cho X l tp hp gm 10000 s nguyn, khng c s no
trong chng chia ht cho 47. Chng minh rng tn ti mt tp con Y ca X gm 2007 phn t
:
sao cho a b C c d C e 6 :: 47; 8a; b; c; d; e 2 Y .
Phn tch v hng dn gii.
 Mt tp M gm cc s nguyn c gi l tt nu 47 6
j a b C c d C e; 8a; b; c; e; d 2 M .
 Nhn thy tp J D f 9; 7; 5; 3; 1; 1; 3; 5; 7; 9g l mt tp tt. Tht vy, vi mi
phn t a; b; c; d; e 2 J th s a b C c d C e l s l v
45 D . 9/ 9 C . 9/ 9 C . 9/ 6 a b C c

d C e 6 9 . 9/ C 9 . 9/ C 9 D 45

nhng khng c s nguyn l no chia ht cho 47 nm trong tp f 45; 43; : : : ; 43; 45g.
 Vi mi k D 1; 2; : : : ; 46, th cc tp hp Ak D fx 2 X j9j 2 J W kx  j .mod 47/g
u l tp tt. Tht vy, gi s tn ti mt tp Ak khng tt (k 2 f1; 2; : : : ; 46g). Khi
tn ti 5 phn t x1 ; x2 ; x3 ; x4 ; x5 2 Ak sao cho
x1

x2 C x3

x4 C x5  0

.mod 47/ ) k.x1

x2 C x3

x4 C x5 /  0

.mod 47/:

Dn n kx1 kx2 C kx3 kx4 C kx5  0 .mod 47/. Theo nh ngha ca x1 ; : : : ; x5 ,


th tn ti j1 ; : : : ; j5 2 J sao cho
j1

j2 C j3

j4 C j5  kx1

kx2 C kx3

kx4 C kx5  0

.mod 47/

chng t J l tp khng tt, mu thun.


 Mi phn t x 2 X thuc ng 10 tp Ak . Tht vy, v 47 l s nguyn t, tp f1; 2; : : : ; 46g
lp thnh mt h thng d thu gn modulo 47, do x 2 X nn .x; 47/ D 1, do
fx; 2x; : : : ; 46xg cng lp thnh mt h thng d thu gn modulo 47. Do vi j 2 J
(gm c 10 gi tr j ) th tng ng s tn ti 10 ch s k 2 f1; 2; : : : ; 46g (cc j khc
nhau th k cng khc nhau) kx  j .mod 47/. Do x s thuc vo ng 10 tp
Ak .k 2 f1; 2; : : : ; 47g/.
 Do

4
X

kD1

6jAk j D 10jX j D 100000:

Theo nguyn l trung bnh, tn ti mt ch s k sao cho


jAk j >

100000
> 2173 > 2007;
46
149

Tp ch Epsilon, S 10, 08/2016

tc tp Ak ny chnh l tp cn tm.
Ta c iu phi chng minh.
Trong li gii trn, ta khng thy s dng nh l Cauchy - Davenport. Hy nhn k li gii trn,
th bc u tin ch ra tp J l quan trng nht. Tp J l tp tt c ng 10 phn t. Thc s
th tp cha cc s d khc nhau khi chia cho 47 m l tp tt th ch cha ti a 10 phn
t. Tht vy, nu jJ j > 11, th theo nh l 2.6 th, do 5jJ j 4 > 5  11 4 D 51 > 47
jJ C J C J C . J / C . J /j > minf5jJ j

4; 47g D 47

:
do s tn ti a; b; c; d; e 2 J sao cho a b C c d C e :: 47, suy ra tp J khng tt. Vy
jJ j 6 10. jJ j D 10 th theo nh l 2.7, J phi l mt cp s cng.
V d 8 (nh l Erdos - Ginzburg - Ziv). Cho n > 1 v a0 ; a2 ; : : : ; a2n 2 l mt dy gm
2n 1 s nguyn (khng nht thit phn bit). Chng minh rng tn ti mt dy con gm n s
ai1 ; : : : ; ain sao cho
ai1 C ai2 C    C ain  0 .mod n/:
Phn tch v hng dn gii.
 Trc tin ta chng minh kt lun bi ton trong trng hp
n D p l s nguyn t. Gi ai0 2 Z; 0 6 ai0 < p sao cho ai  ai0 .mod p/ (thc hin ly
modulo cc phn t ca dy). Ta c th nh li ch s cho cc s ai
0
0 6 a00 6 a10 6 : : : 6 a2p

Nu ai0  ai0 Cp

vi mt ch s i 2 1; p

6p

1:

1 th theo th t trn xy ra

ai0  ai0 C1      ai0 Cp

.mod p/

dn n
ai C ai C1 C    C ai Cp
Nu ai0  ai0 Cp

vi mi i 2 1; p

2; 8i D 1; 2; : : : ; p

 pai  0

.mod p/:

1. Khi t Ai D fai0 ; ai0 Cp 1 g th jAi j D

1. p dng nh l 2.6 ta c

jA1 CA2 C  CAp 1 j > minfp; jA1 jC  CjAp 1 j .p 2/g D minfp; 2.p 1/ .p 2/g D p:
T y suy ra
A1 C A2 C    C Ap

D Zp :

T tn ti mt lp cc ng d aj0 i 2 Ai , i D 1; 2; : : : ; p 1, vi ji 2 fi; i Cp 1g
sao cho
a0  aj0 1 C aj0 2 C    C aj0 p 1 .mod p/:
Suy ra

aj1 C aj2 C    C ajp

C a0  0

Vy bi ton ng trong trng hp n nguyn t.

150

.mod p/:

Tp ch Epsilon, S 10, 08/2016

 Ta chng minh kt lun bi ton trong trng hp tng qut bng phng php quy np.
Nu n D 1, bi ton hin nhin ng. Gi s vi n > 1 v kt lun bi ton ng cho mi
s nguyn dng nh hn n. Ta s chng minh bi ton ng vi n. Nu n nguyn t, bi
ton ng theo trn. Nu n hp s, t
n D uv

.1 < u 6 v < n/:

Khi theo gi thit quy np, bi ton ng cho u v v.


T dy s a0 ; : : : ; a2n 2 c di 2n 1 D 2uv
tn ti mt dy con a1;i1 ; : : : ; a1;iv sao cho
a1;i1 C    C a1;iv  0

1, theo gi thit quy np, lun

.mod v/:

Khi cn li 2n 1 v D .2u 1/v 1 s nguyn nm trong dy ban u,


nhng khng thuc vo dy con trn. Do 2u 1 > 2, li p dng gi thit quy
np, t dy di .2u 1/v 1 > 2v 1 s nguyn, lun tn ti mt dy con
a2;i1 ; : : : ; a2;iv sao cho
a2;i1 C    C a2;iv  0

.mod v/:

Khi cn li 2n 1 2v D .2u 2/v 1 s nguyn trong dy v khng thuc


vo hai dy con trn. C tip tc qu trnh trn, vi j D 1; 2; : : : ; 2u 1, ta nhn
c 2u 1 cc dy con i mt ri nhau aj;i1 ; : : : ; aj;iv di v sao cho
aj;i1 C    C aj 2;iv  0

.mod v/:

Khi
aj;i1 C    C aj 2;iv D bj v.bj 2 Z/;

8j D 1; 2; : : : ; 2u

Do gi thit ng cho u, nn t mt dy b1 ; b2 ; : : : ; b2u


dy con bj1 ; : : : ; bju sao cho
bj1 C bj2 C    C bju  0

di 2u

1:
1 s c mt

.mod u/;

tc l
bj1 C bj2 C    C bju D cu .c 2 Z/:
Khi

u X
v
X
rD1 sD1

ajr ;is D

u
X
rD1

bjr v D cuv D cn  0

.mod n/:

nh l c chng minh hon ton.


V d 9 (China TST 2016). Cho m; n l cc s nguyn dng tha mn n > m > 1 v S l
mt tp hp gm c n s t nhin. Chng minh rng S cha t nht 2n mC1 tp con phn bit,
m mi tp c tng cc phn t chia ht cho m (tp rng xem nh l tp con tha mn).

151

Tp ch Epsilon, S 10, 08/2016

Trc khi chng minh, ta cn s dng mt s kt qu quen thuc cho bi cc nhn xt sau:
Nhn xt 1: Trong n s nguyn ty , lun tn ti mt s hoc mt vi s m tng ca chng
chia ht cho n. y l mt tnh cht quen thuc chng minh bng Dirichlet.
Nhn xt 2: Cho p l s nguyn t v Q D fa1 ; a2 ; : : : ; ap 1 g l tp hp cha cc s nguyn
khng chia ht cho p. Khi tp
X
R D fS.A/ D
xjA  Qg
x2A

cha mt h thng d y modulo p. y S.;/ D 0:


Chng minh. Ta c th ly cc phn t ca Q theo modulo p m khng lm thay i bi ton.
Xt cc tp
A1 D f0; a1 g; A2 D f0; a2 g; : : : ; Ap 1 D f0; ap 1 g
:
th do ai 6 :: p; 8i D 1; 2; : : : ; p 1 nn jAi j D 2; i D 1; 2; : : : ; p 1. Theo nh l 2.6 th
jA1 CA2 C  CAp 1 j > minfp; jA1 jC  CjAp 1 j .p 2/g D minfp; 2.p 1/ .p 2/g D p:
Chng t A1 C    C Ap 1 D Zp . Mt khc, cc phn t trong A1 C A2 C    C Ap 1 l cc phn
t trong R (phn t 0 C 0 C    C 0 trong A1 C    C Ap 1 ng vi vic chn tp A D ;  Q).
Li gii. tng ca bi ton l ta s chia tp S thnh hai tp A; B. Mi tp con A0 trong A
u tm c mt tp con B 0 trong B m tng cc s ca hai tp con ny chia ht cho m. ng
ngha vi vic S.A0 /  S.B 0 / .mod m/.
 Trc tin ta chng minh nu m D p nguyn t, bi ton hin nhin ng. Tht vy:
Nu trong S c t nht p 1 s nguyn khng chia ht cho m. Gi Q D fa1 ; a2 ; : : : ; ap 1 g
l tp cha p 1 s nguyn khng chia ht cho p. Khi
S D Q [ .SnQ/:
Tp S nQ c n .p 1/ D n p C 1 phn t nn c 2n pC1 tp con. Mi tp con
A  .SnQ/, do nhn xt 2, tn ti mt tp con B  Q sao cho
S.A/ 

S.B/

.mod p/ ) S.A/CS.B/  0

.mod p/ ) S.A[B/  0

.mod p/:

Vy c mt tp con A  .SnQ/, lun tm c mt tp dng A [ B  S


:
S.A [ B/ :: p. M c 2n pC1 tp A nn tng ng c 2n pC1 tp con trong S tha
mn bi.

Nu trong S c < p 1 s nguyn khng chia ht cho m. Dn n trong S c


> n p C 1 s chia ht cho m. Trong trng hp ny hin nhin mi tp con ca
tp cha n p C 1 s ny u chia ht cho p, tc l cng c t nht 2n pC1 tp tha
mn bi ton.

152

Tp ch Epsilon, S 10, 08/2016

 Trong trng hp m ty . Ta chng minh khng nh bi ton bng quy np. Vi m D 1


bi ton hin nhin ng. Gi s bi ton ng cho mi gi tr nguyn dng < m. Xt
vi s nguyn dng m. t m D p:k, vi p nguyn t v k > 1. Mt tp A  S c
:
:
gi l tp gn tt ti thiu nu S.A/ :: k nhng S.A/ 6 ::m v jAj nh nht. Khi theo
nhn xt 1 th mt tp A l gn tt ti thiu th jAj 6 k.

Nu trong tp S ch c ti a t < p 1 tp gn tt ti thiu. Gi cc tp ny l


S1 ; : : : ; St . Khi xt phn hoch ca S
S D S1 [ S2 [    [ St [ N

.N D S n.S1 [ S2 [    [ St //:

V mi tp jSi j ch c ti a k phn t. Nn tp N c
jN j > n

tp > n

.p

p dng gi thit quy np, th tp N cha


2jN j

kC1

1/k D n

> 2n

m C k > k:

mC1

tp con phn bit chia ht cho k. Tuy nhin v c tt a t tp gn tt ti thiu, do


2n mC1 tp con ny mc d chia ht cho k, khng khng th l tp gn tt ti thiu,
tc mi tp con ny c tng chia cht cho m. Kt lun bi ton c chng minh
trong trng hp ny.
Nu trong tp S c > p 1 tp gn tt ti thiu. Chn ra p 1 tp gn tt ti thiu
phn bit, k hiu l B1 ; : : : ; Bp 1 . Khi xt phn hoch ca S
S D B1 [ B2 [    [ Bp

[M

.M D S n.S1 [ S2 [    [ St //:

V mi tp jSi j ch c ti a k phn t. Nn tp M c
jM j > n

1/k D n

.p

p dng gi thit quy np, th tp M cha


2jM j

kC1

> 2n

m C k > k:

mC1

tp con phn bit chia ht cho k. Do nh ngha ca B1 ; : : : ; Bp 1 , t


S.B1 / D r1 k; S.B2 / D r2 ; : : : ; S.Bp 1 / D rp 1 :k .ri 2 ZC ; i D 1; 2; : : : ; p 1/
:
v cc ri 6 :: p; 8i D 1; 2; : : : ; p 1. t Q D fr1 ; : : : ; tp 1 g, theo nhn xt 2, tp
R D fS.A/jA  Qg

cha mt h thng d y modulo p. Do tp S.B1 C B2 C    C Bp 1 / s


cha h thng d fk; 2k; : : : ; pkg .mod m/. Do ly bt k mt tp con A  M
:
(khi S.A/ :: k), lun tn ti mt tp con B  B1 C    C Bp 1 sao cho
S.B/ 

S.A/

.mod m/ ) S.A [ B/  0

.mod m/;
:
tc tp A[B tha mn. V c > 2n mC1 tp A  M m S.A/ :: k, nn c > 2n
tp tha mn yu cu bi ton.

mC1

Tng hp cc kt qu trn, bi ton c chng minh hon ton.


Li gii cho trng hp m D p l mt trng hp ring cho li gii m tng qut. Tuy nhin tc
gi vn trnh by y, cho thy cch lm vi m tng qut, da vo cch lm vi m D p
nguyn t.
153

Tp ch Epsilon, S 10, 08/2016

4. Bi tp t luyn
Bi tp 1 (Bosnia and herzegovina TST 2012). Chng minh rng vi mi s nguyn t p
l, lun tn ti s nguyn dng m < p v cc s nguyn x1 ; x2 ; x3 sao cho
x12 C x22 C x32 D mp:
Bi tp 2. Cho k; n l cc s nguyn dng v p l s nguyn t. Chng minh rng tn ti cc
s nguyn a1 ; : : : ; ak sao cho
a1k C a2k C    C akk  n

.mod p/:

Bi tp 3 (Belarus MO). Cho s nguyn t p v gi x; y; z 2 f0; 1; 2; : : : ; p 1g tha mn


:
x 2 C y 2 C z 2 :: p. Gi S.p/ l s b ba .x; y; z/ tha mn iu kin trn. Chng minh rng
S.p/ > 2p

1.

Bi tp 4 (Ukrainian TST 2007). Tm tt c cc s nguyn t p, sao cho tn ti s nguyn


n, phng trnh sau v nghim nguyn x 3 C y 3  n .mod p/.
Bi tp 5 (Tuymaada 2008). Cho 250 s nguyn dng khng vt qu 501. Chng minh
rng vi mi s nguyn dng t , tn ti bn s nguyn a1 ; a2 ; a3 ; a4 trong 250 s cho m
:
a1 C a2 C a3 C a4 t :: 23:
Bi tp 6. Cho p > 3 l s nguyn t. Tp f1; 2; : : : ; p 1g c phn hoch thnh 3 tp
con ri nhau A; B; C . Chng minh rng tn ti x; y; z ln lt thuc ba tp A; B; C sao cho
:
x C y z :: p.
Bi tp 7 (APMO 2014). Tm tt c cc s nguyn dng n sao cho vi mi s nguyn k, tn
:
ti s nguyn a sao cho a3 C a k :: n.
Bi tp 8 (Crux 2013, Vol 39, CC9). Cho k > 3 l s nguyn. t n D

tp con ca Zn . Chng minh rng S C S Zn .

k.k C 1/
v S l
2

Bi tp 9 (IMO 2003 (general)). Cho n l s nguyn dng > 2. Cho A l mt tp con cha
n C 1 phn t ca tp S D f1; 2; : : : ; n3 g. Chng minh rng tn ti n phn t t1 ; t2 ; : : : ; tn trong
S sao cho cc tp
Aj D fx C tj jx 2 Ag; 8j D 1; 2; : : : ; n
i mt ri nhau.

Bi tp 10 (USA TSTST 2013). Cho p l s nguyn t. Cho graph G y 1000p nh.


Trn mi cnh ca graph ta gn mt s nguyn. Chng minh rng trong G tn ti mt chu trnh
m tng cc s c nh trn cc cnh nm trong chu trnh chia ht cho p.
Bi tp 11 (Tomanian TST 2010). Cho X; Y l cc tp con hu hn ca na khong 0; 1/
sao cho 0 2 X \ Y v khng tn ti x 2 X; y 2 Y sao cho x C y D 1. Chng minh rng tp
fx C y x C y W x 2 X; y 2 Y g c t nht jX j C jY j 1 phn t.
154

Tp ch Epsilon, S 10, 08/2016

Ti liu
[1] A.Frimu and M.Teleuca, Applications of combinatorial Nullstellensatz, Gazeta Mathematica 2011
[2] O.J.Rodseth, Sumsets mod p, lecture notes.
[3] M.B.Nathanson, Additive number theory - Inverse baitapblems and the Geometry of Sumset, Springer, 1996.
[4] H.Lee, Combinatorial Number Theory, lecture notes.
[5] Website: http://artofproblemsolving.com.

155

Tp ch Epsilon, S 10, 08/2016

S DNG MODULO TRONG PHNG TRNH


NGHIM NGUYN V BI TON CHIA HT
L Anh Dng
(Trng THPT chuyn Hunh Mn t - Kin Giang)
La chn modulo l mt trong cc k nng hay dng trong vic gii phng trnh
nghim nguyn v bi ton chia ht. Chn modulo no, c s no chn modulo?
Bi vit cung cp mt s c s l thuyt v cc k nng nh trong vic suy lun tm
modulo.

1. Cc kin thc c s s dng


1.1. Hm CARMICHAEL
Theo nh l Euler th a'.n/  1 .mod n/ vi mi s nguyn dng a nguyn t cng nhau vi
n. Tuy nhin, '.n/ khng phi l s nguyn dng nh nht nh nht tha at  1 .mod n/.

nh ngha 1. Cho s nguyn dng n .n > 2/. .n/ l s nguyn dng nh nht tha
mn a.n/  1 .mod n/ vi mi s nguyn dng a m .a; n/ D 1. Hm .n/ gi l hm
Carmichael theo n.
nh l 1. (Tnh cht ca hm Carmichael)
i) Vi mi s nguyn dng a, b, ta c .a:b/ D .a/; .b/.
ii) .2/ D 1I .4/ D 2I .2k / D 2k 2 vi mi k > 3.
.p k / D p k 1 .p 1/ vi mi s nguyn
t p, p > 3.


sm
sm
Nu n D p1s1 :p2s2 :::pm
th .n/ D .p1s1 /; .p2s2 /; :::; .pm
/

Nhn xt. Trong gii ton ta thng dng cc nhn xt sau:

 Nu a2  1.mod p s / th hoc a  1.mod p s / hoc a 


nguyn t ln hn 2.

1.mod p s / vi p l s

 Nu a2  1.mod 2p s / th hoc a  1.mod 2p s / hoc a 


nguyn t ln hn 2.

1.mod 2p s / vi p l s

T nhn xt trn, ta c cc modulo hay dng p s ; 2p s ; 4p s


156

Tp ch Epsilon, S 10, 08/2016

1.2. S m ng
nh ngha 2. Cho p l s nguyn t, a l s t nhin. s m ng ca a theo p, k hiu l
vp .a/ nu v ch nu p j a v p C1 khng l c ca a.
nh l 2. Ta c cc tnh cht sau
1. vp .ab/ D vp .a/ C vp .b/
2. vp .an / D nvp .a/


3. vp .a C b/ > min vp .a/; vp .b/ ng thc xy ra khi vp .a/ vp .b/.

4. Vi s nguyn dng n, s nguyn t p th


 Nu p 2 v pj .x

y/ th

 Nu p D 2 v 4j .x

y/ th

 Nu p D 2 v 2j .x

y/, n l s chn th

vp .x n
v2 .x n
v2 .x n

y n / D vp .x

y/ C vp .n/:

y n / D v2 .x

y/ C v2 .n/:

y n / D v2 .x 2

y 2 / C v2 .n/

1:

 Nu pj .x C y/ v n l s nguyn dng l th

vp .x n C y n / D vp .x C y/ C vp .n/:

1.3. Cp ca phn t
nh ngha 3. Cho s nguyn dng n > 1 v s nguyn a nguyn t cng nhau vi n, s
k c gi l cp ca a modulo n (k hiu l ordn a) nu k l s nguyn dng nh nht
ak  1 .modn/.
nh l 3. Ta c cc tnh cht sau
i) ordn aj .n/
ii) Nu ah  1 .modn/ th ordn aj h
iii) ax  ay .mod n/ , ordn aj .x
iv) ordn as D

y/ hay x D t:ordn a C y vi .a; n/ D 1.

ordn a
.ordn a;s/

vi mi k > 3.


vi) Cho k l s nguyn dng, v p l mt s nguyn t l. Xt tp S D ak a 0  Zp .
p 1
Khi jS j D .k;p
.
1/
v) Nu x  3.mod 8/ hoc x  5.mod 8/ th ord2k x D 2k

vii) Cho p l s nguyn t v d j .p


nghim trong Zp .

1/. Khi phng trnh x d  1. mod p/ c ng '.d /


157

Tp ch Epsilon, S 10, 08/2016

1.4. Thng d bnh phng


nh ngha 4. Gi s m l s nguyn dng v a l s nguyn tha .a; m/ D 1. Nu phng
trnh ng d x 2  a .mod m/ c nghim th ta ni a l thng d bnh phng ca m. Ngc
li, ta ni a l khng thng d bnh phng ca m.
p 1
thng d bnh phng.
Nu p l s nguyn t l th trong dy 1; 2; :::; p 1 c ng
2
Ta thng s dng k hiu Legendre nh sau:
Cho p l s nguyn t l v s nguyn a khng chia ht cho p. Ta k hiu
(
hai
1 nu a l thng d bnh phng
D
:
b
1 nu ngc li
nh l 4. Ta c cc tnh cht sau
i) (Tiu chun Euler) Gi s p l s nguyn t l v khng l c ca s nguyn a. Khi :
p 1
 
a
a 2
p

.mod p/:

ii) Nu p l s nguyn t l v a v b khng chia ht cho p th


   
a
b
+) a  b .modp/ )
D
p
p
    
ab
a
b
+)
D
p
p p
 2
a
+)
D1
p
  
1
1 khi p  1 .mod 4/
+) Nu p l s nguyn t l th
D
1 khi p  1 .mod 4/
p

p2 1
 
2
D . 1/ 8
+) Nu p l s nguyn t l th
p
 
 
2
2
+)
D 1, nu p  1.mod8/;
D 1, nu p  3.mod8/
p
p

iii) (Lut thun nghch bnh phng) Nu p, q l cc s nguyn t l khc nhau th


p 1 q 1
   
:
p
q
2 :
:
D . 1/ 2
q
p
 
 
 
 
q
p
q
p
Do ,
D
, nu p  1 _ q  1.mod4/ v
D
, nu p  q 
q
p
q
p
3.mod4/.
158

Tp ch Epsilon, S 10, 08/2016

2. Mt s bi ton p dng
Bi ton 1. (IMO Shortlist 2002) Tm s nguyn dng t nh nht sao cho tn ti cc s
nguyn x1 ; x2 ; :::; xt tha
x13 C x23 C ::: C xt3 D 20022002 :
Phn tch v li gii. Ta thy
1
1
1
1
1
3 D :6 D .7/ D .9/ D  .14/ D .18/;
2
2
2
2
2
l cc moulo u tin ta xt ti.
Theo nhn xt trn, trc ht ta kho st cc modulo 9 ca x 3 ,20022002 .
Nu 3j x th x 3  0.mod 9/.
Nu .x; 3/ D 1 th x 3 ng d 1 hoc 1 modulo 9.
Nh vy, x13 C x23 C ::: C xt3 ng d t t n t modulo 9.
20022002  42002  46:333 :44  44  4

.mod 9/:

Nh vy, nu t 6 3 th x13 C x23 C ::: C xt3 khng ng d 4 modulo 9 nn ng thc sau


x13 C x23 C ::: C xt3 D 20022002
khng xy ra.
Vi t D 4, ta th tm 1 nghim ca phng trnh
x13 C x23 C x33 C x43 D 20022002 :

3
20022002 D 2002667 :2002, nh vy ta ch cn th xy dng cc nghim dng
xi D mi 2002667 , vi
m31 C m32 C m33 C m34 D 2002:
Ta chn m1 D m2 D 10I m3 D m4 D 1 tha yu cu ny.

Bi ton 2. (USAMO) Xc nh tt c nghim khng m .x1 ; x2 ; :::; x14 / ca phng trnh


4
x14 C x24 C ::: C x14
D 15999:

Phn tch v gii. Ly tha 4, gi cho ta xt modulo no? .n/ D 4 D 22 ! n D 16.


Ta c x 4 ng d 0 hoc 1 modulo 16.
4
Do x14 C x24 C ::: C x14
ng d 1; 2; :::; 14 modulo 16.
M 15999  1.mod 16/ (mu thun).
Vy phng trnh trn v nghim.
Bi ton 3. (USAMO 2005) Chng minh rng h phng trnh
(
x 6 C x 3 C x 3 y C y D 147157
x 3 C x 3 y C y 2 C y C z 9 D 157147
khng c ngim nguyn x, y, z.
159

Tp ch Epsilon, S 10, 08/2016

Phn tch v li gii. Cng hai phng trnh ta c


.x 3 C y C 1/2 C z 9 D 147157 C 157147 C 1:
i vi ly tha 9, ta xt modulo no?
1
1
1
1
9 D 18 D .19/ D .27/ D .54/;
2
2
2
2
nn ta c th xt theo cc moulo ny trc tin.
Ly tha 2, ta xt cc modulo 3, 4, 5, 8; s dng thng d bnh phng kim tra nghim.
Ta bt u vi modulo p D 19. Ta c
z 9  0; 1

.mod 19/

147157  . 5/18:8C13 

513  2

157147  .5/18:8C3  53  11

.mod 19/

.mod 19/:

Suy ra
.x 3 C y C 1/2 D 147157 C 157147 C 1
p dng tnh cht ta tnh c
 
13
D
19

1I


14
D
19

z 9  13; 14; 15

1I


15
D
19

.mod 19/:

nn iu ny khng xy ra.
Bi ton 4. (USAJMO 2013) Tm tt c cc s nguyn a, b sao cho a5 b C 3 v ab 5 C 3 u l
lp phng ca mt s nguyn.
Phn tch v li gii. Gi s

vi x; y 2 N. Suy ra

a5 b C 3 D x 3
ab 5 C 3 D y 3

.ab/6 D .a5 b/.ab 5 / D .x 3

3/.y 3

3/:

./

Ly tha 6, 3 gi cho ta xt cc modunlo 9 v 18. .9/ D 6 nn .ab/6  0; 1 .mod 9/.


x 3 ; y 3  0; 1 .mod 9/: Kt hp (*) ta c
8 3
8
< x  0 .mod 9/
< 3j x
3
y  0 .mod 9/ ) 3j y
:
:
.ab/6  0 .mod 9/
3j ab

Nu 3j a th t a5 b C 3 D x 3 ta c iu v l v 27j x 3 I 27j a5 b.
Nu 3j b th t ab 5 C 3 D y 3 ta c iu v l v 27j y 3 I 27j ab 5 .
Vy khng tn ti cc s nguyn a, b tha iu kin bi ton.

Bi ton 5. (MOP 2002) Chng minh rng phng trnh x 6 D y 5 C 24 khng c nghim
nguyn.
160

Tp ch Epsilon, S 10, 08/2016

Phn tch v li gii. Ta tm mt modulo chung cho hai ly tha 5, 6. Vi mong mun s ng
d hai v l ti thiu. Nh vy, 5:6j .p 1/, p D 31 l trng hp u tin ta xt.
30
30
x 6 c 1 C
D 6 ng d; y 5 c 1 C
D 7 ng d, nh vy kh nng cc ng d
.6; 30/
.5; 30/
ca hai v khng bng nhau
y 5 c cc ng d 0; 1; 5; 6; 25; 26; 30 theo modulo 31.
Suy ra y 5 C 24 c cc ng d 24; 25; 29; 30; 18; 19; 23 theo modulo 31.
x 6 c cc ng d 0; 1; 2; 4; 8; 16 theo modulo 31.
Vy phng trnh trn khng c nghim nguyn. Tip theo, ta phn tch mt s k nng chn
modulo vi cc phng trnh dng ly tha cha bin trn m.
Bi ton 6. Tm cc nghim nguyn dng ca phng trnh
2x C 3y D 5z :
Phn tch v li gii.
+ D on nghim phng trnh .xI yI z/ D .1I 1I 1/; .4I 2I 2/.
+ i vi ly tha c c s xc nh, thng phng n u tin ta th phn tch nhn t v
dng A:B D ax , xt c chung ca A, B a v h. phn tch nhn t, thng ta quan
tm n cc hng ng thc cng s m.
+ Cc modulo thng dng trc ht xt ly tha l anC1 vi n l nghim ln nht ca phng
trnh, cc modulo c ng d 1.
+ 5  1 .mod 2/2 ; 3  1 .mod 2/2 , 32  1 .mod 2/3 . Do , ta c th xt cc modulo
4, 8. xt cc modulo ny ta chia thnh 2 trng hp.
TH1: x D 1, phng trnh tr thnh
2 C 3y D 5z :
Phng trnh c nghim .1I 1I 1/.
Xt z > 2, suy ra 3y  5z 2  2 .mod 25/.
r
1
2
3
4
5
6
7
8

r
9
10
11
12
13
14
15
16

3r .mod 25/
3
9
2
6
18
4
12
11

3r .mod 25/
11
8
24
22
16
-2
-6
21

.25/ D 20 do ord3 25j 20 , suy ra ord3 25 D 20.


3y  2  31 3 .mod 25/, theo tnh cht cp phn t suy ra y D 13 C 20k .
k
3y D 313C20k D 313 : 320 ta xt modulo no? Modulo 11 l modulo xt trc c nh s
ng d ca 3y .
5z  2 C 310

1C2k

:33  7

161

.mod 11/:

Tp ch Epsilon, S 10, 08/2016

z
1
2
3
4
5

5z .mod 11/
5
3
4
9
1

Do khng c gi tr no ca z
5z  7

.mod 11/:

TH2. x D 2, phng trnh tr thnh


4 C 3y D 5z ;
suy ra z > 2. Suy ra
3y  5z

4  21  316

Khi
5z  4 C 310
TH3: x > 3. Khi
1z

1C2k

. 1/y  5z

.mod 25/ ) y D 16 C 20k:


.mod 11/ v l:

:36  7

3y  2x  0.mod 4/ ) y D 2n:

Nn
5z  3y C 2x  9n  1

.mod 8/;

m ord8 5 D 2 suy ra z D 2m. Suy ra


2x D 52m
Suy ra

hay

32n D .5m C 3n /.5m




vi r > s , r C s D x. Nn s D 1,ta c


3n /

5m 3n D 2s
5m C 3n D 2r
2:5m D 2r C 2s
2:3n D 2r 2s
5m D 2r
3n D 2r

1
1

C1
1

v 5m 3n D 2.
Phng trnh c nghim m D n D 1 ( gii TH1).
Vy phng trnh cho ch c 2 nghim .xI yI z/ l .1I 1I 1/ v .4I 2I 2/.
162

Tp ch Epsilon, S 10, 08/2016

Bi ton 7. Tm nghim nguyn dng ca phng trnh


2x C 3 D 11y :
Phn tch v li gii.
Phng trnh c nghim .xI y/ D .3I 1/.
Trong phng trnh modulo u tin ta c th s dng l modulo 11.
V 25  32  1 .mod 11/ nn ord2 11 D 10.
23  3 .mod 11/ nn 2x  3  23 .mod 11/. Suy ra x D 3 C 10n.
iu ny cha s dng c.
Vi x > 4,
11y  2x C 3  3 .mod 2x /:

v 11  3.mod 8/ nn ord24 11 D 22 D 4 ; 113  3.mod 24 / do y D 4k C 3, dng


modulo ny nh s d trng lp ng d.
Ta tng modulo ln 25 .
Ta chng minh phng trnh khng c nghim x > 5 bng cch la chn modulo theo s
nh bi trn.
Bc 1: S dng modulo 25 tm dng ca y:
11y  2x C 3  3

.mod 32/:

T ord25 11 D 25 2 D 8 ; 117  3 .mod 32/; 11y  117 .mod 32/ suy ra y D 8k C 7.


Phng trnh tr thnh
y
2x D 118yC7 3 D 118 :117 3:

Bc 2: Tm s nguyn t p sao cho hai v khng cng ng d, mun vy cch d nht l chn
modulo p sao cho 118  1 .mod p/ v ord2 p l nh.
1
Ch 8 D .17 1/ v 24 D 16  1 .mod 17/, ta xt mod 17.
2
2x c 4:2 D 8 ng d; v phi c 2 ng d, vy kh nng khng trng l ln.
112  62  2.mod 17/ ; 114  4.mod 17/ ; 118  1.mod 17/ ; 117  3.mod 17/
y
y
Suy ra 2x  .118 / :117 3  0.mod 17/ hoc 2x  .118 / :117 3  6.mod 17/.
Ta kho st cc ng d modulo 17 ca 2x vi x D 3 C 10n.
x
1
3
5
7

2x .mod 17/
2
8
2
8

Vy hai v khng cng ng d vi modulo 17.


Bi ton 8. (n ) Tm nghim nguyn dng ca phng trnh
7x D 3y C 4:

163

Tp ch Epsilon, S 10, 08/2016

Phn tch v li gii.


Phng trnh c nghim .1I 1/ v y D 2 khng tha phng trnh.
Ta c 3y  7x 4  3 .mod 7/ m 33  1.mod 7/, ord7 3 D 6 nn y D 1 C 6t.
Ta chng minh y > 3, khng tha.
Bc 1: T phng trnh suy ra
7x  3y C 4  4.mod 33 /:
Trc ht ta xc nh ord3n 7. t ord3n 7 D 3i :k vi .k; 3/ D 1, i 6 n 1.
Ta c
 i


v3 73 :k 1 D v3 .7 1/ C v3 3i :k D i C 1 > n:
Suy ra i > n 1 nn i D n 1.
Vy ord3n 7 D 3n 1 , do ord33 7 D 9.
Li c 78  4  7x .mod 27/, suy ra x D 9q C 8.
Phng trnh tr thnh
q
79 :78 4 D 3y :

Bc 2: Tm s nguyn t p sao cho 79  1.mod p/ v ordp 3 l nh. V


79

1 D .73

1/.76 C 73 C 1/:.73

1/:3:37:1063

nn ta chn p D 37.
Ta c:
72  12

.mod 37/I 74  122 

.mod 37/I 78  16

.mod 37/:

q
Vy 79 :78 4  12 .mod 37/.
Ta kho st cc ng d modulo 37 ca 3y vi y D 1 C 6t
y
1
7
13
19
25
31

3y .mod 37/
3
4
30
25
21
28

Vy hai v khng cng ng d vi modulo 37.


i vi cc phng trnh ly tha c biu thc au 1 th cc b LTE v kin thc v cp l
thc s hu dng. Ta xt cc v d sau y.
Bi ton 9. (1995 Czech Slovak Match) Gii phng trnh nghim nguyn dng p x
1 vi p l s nguyn t l.

164

yp D

Tp ch Epsilon, S 10, 08/2016

Phn tch v li gii.


Phng trnh c vit li p x D y p C 1, suy ra pj .y C 1/.
p dng b LTE ta c
x D vp .y p C 1/ D vp .y C 1/ C vp .p/ D 1 C vp .y C 1/ :
Suy ra y C 1 D p x
Ta c
Hay

y3 C 1

yp C 1
D p.
yC1

.y C 1/2 D y 3

y2

yC1<
vi mi y > 3.

2y D y.y 2

2/ > 0 8y > 3:

y3 C 1
yp C 1
6
:
yC1
yC1


xD2
pD3
Vy phng trnh c nghim x D 2; y D 2; p D 3.

Vy ta phi c y D 2 , suy ra p

x 1

D3,

Bi ton 10. (IMO ln 31) Gii phng trnh nghim nguyn dng
2x C 1 D x 2 y:
Phn tch v li gii.
Phng trnh c nghim .xI y/ D .1I 3/; .3I 1/.
Xt x > 3, t phng trnh suy ra x, y l cc s l.
Phng trnh c ly tha c au 1 c th s dng cp xt. Mt trong cc modulo hay dng
l c nguyn t ca nhn t.
Ta th xt theo modulo p, vi p l c ca x. 2x D x 2 y 1  1 .mod p/ nn 22  1
.mod p/ .
t u D ordp 2 > 1, ta c uj .p 1/. Suy ra uj 2x.
Vy p chn nh th no?
+ Nu p D 3 th hai iu ny l nh nhau. Vy ta phi xt ti s m chnh xc ca 3 trong x l
bao nhiu ?
+ Nu p > 3, th chn p nh th no trong s cc c ca x, cc c khng trng nhau, ta
s chn p l c nguyn t nh nht khc 3 ca x.
v3 .x 2 y/ D v3 .2x C 1x / D v3 .2 C 1/ C v3 .x/:
Suy ra 2v3 .x/ C v3 .y/ D 1 C v3 .x/ t y ta c v3 .x/ C v3 .y/ D 1.
t x D 3k :d vi k D 0 hoc 1; .d; 3/ D 1. Ta s chng minh d D 1.
V 2x D x 2 y 1  1 .mod x/, gi ta c th s dng nh l v cp ca 2.
Nu d > 1, gi p l c nguyn t nh nht ca d , cng l c nguyn t khc 3 nh nht ca
x. Do .d; 3/ D 1 nn p > 5.
t u D ordp 2 > 1, uj .p 1/. 2x D x 2 y 1 D 32k d 2 y 1  1 .mod p/ nn 22x  1
.mod p/. Suy ra uj 2x.
V p l c nguyn t khc 3 nh nht ca x v .x; p 1/ D 1 nn u 2 f2I 3I 6g.
M pj 2u 1 nn p D 7:
Suy ra 2x C 1  0 .mod 7/ iu ny mu thun v bng ng d sau.
165

Tp ch Epsilon, S 10, 08/2016

2x .mod 7/
2
4
1

x
1
2
3

Bi ton 11. (Trung Quc 2009) Tm tt c cc cp s nguyn t p, q tha


pqj 5p C 5q :
Phn tch v li gii.
Cc ly tha cng c s c th to dng au 1 :
am an D an .am

1/ :

Nu p D q, ta c p 2 2:5p , suy ra p D q D 5.
Nu trong hai s p, q c 1 s bng 5, gi s p D 5 q, suy ra qj 55 C 5q .
Theo Fermat 5p  5 .mod q/, suy ra 0  55 C 5q  55 C 5 .mod q/, suy ra qj 55 C 5 nn
q D 2 hoc 313.
Nu trong hai s p, q c 1 s bng 2, gi s p D 2 q, suy ra qj 52 C 5q .
Theo Fermat 5p  5.mod q/, suy ra 0  52 C 5q  52 C 5 .mod q/, suy ra qj 52 C 5 nn
q D 3 hoc 5.
Nu p q v p, q khc 5, khc 2. Ta c 0  5p C 5q  5 C 5q .mod p/, suy ra 5q 1  1
.mod p/.
Tng t 5p 1  1 .mod q/. Suy ra

 2p 2

5
 1 .mod q/
ordq 5 2p 2
)
:
52q 2  1 .mod p/
ordp 5 2q 2
Mt khc 5p 1  1 1 .mod q/ nn ordq 5 khng l c ca p 1.
Suy ra v2 .ordq 5/ D v2 .2p 2/ D 1 C v2 .p 1/.
Tng t
v2 .ordp 5/ D v2 .2q 2/ D 1 C v2 .q 1/:
Mt khc t

suy ra

Kt hp iu trn suy ra

ordq 5 q
ordp 5 p

1
;
1

v2 .ordq 5/ 6 v2 .q
v2 .ordp 5/ 6 v2 .p


1 C v2 .p
1 C v2 .q

1/ 6 v2 .q
1/ 6 v2 .p

1/
:
1/

1/
(v l):
1/

Bi ton 12. (Bulgaria) Tm tt c cc s nguyn t p; q tha mn


pqj .5p

2p / .5q

166

2q / :

Tp ch Epsilon, S 10, 08/2016

Phn tch v li gii.


Ta to dng au 1 nh th no ? Hai ly tha cng s m, thng ta nhn phn t nghch o.
5p  2p .mod q/, ta nhn vo hai v phn t nghch o ca 2 s to ra dng au 1. R rng,
p; q f2; 5g.
* Nu pj 5p 2p , theo Fermat ta c
5p
suy ra p D 3. Lc ny
hay

2p  5

23

3qj 53


23 .5q

qj 39 .5q

.mod p/;
2q /

2q / ;

tng t nh trn suy ra q D 3 hoc q D 13.


* Nuqj 5q 2q , ta cng c p D 3 hoc q D 13.
qj 5p 2p
, p; q 3.
* Xt
pj 5q 2p
Do .2; q/ D 1 nn tn ti m, n sao cho 2m C nq D 1 hay 2m  1 .mod q/.
Suy ra .5:m/p  .2m/p  1 .mod q/, do ordq .5m/ 2 f1; pg v ordq .5m/ p 1.
Nu ordq .5m/ D 1, hay 5m  1 .mod q/. Suy ra 2  2.5m/  5:2m  5 .mod q/, do
q D 3 (mu thun).
Vy ordq .5m/ D p, suy ra pj q 1 (*).
Tng t cng t 5q  2q .mod p/, ta c qj p 1 iu ny mu thun vi (*).
Vy .p; q/ D .3I 3/; .3I 13/; .13I 3/:
Bi ton 13. (USA TST 2003) Tm tt c cc s nguyn t p, q, r sao cho
pj q r C 1; qj r p C 1; rj p q C 1:
Phn tch v li gii.
Nu p; q; r u khc 2. T (*) suy ra
8
8
< pj q 2r 1
<ordp q 2 f1; 2; rg
2p
qj r
1 ; do ordq r 2 f1; 2; pg :
:
:
rj p 2q 1
ordr p 2 f1; 2; qg

./

+ Nu ordp q; ordq r; ordr p 2 f1; 2g th


8
8 2
<q  1 .mod p/
<q  1 .mod p/
2
p  1 .mod r/ ) p  1 .mod r/ :
:
: 2
r  1 .mod q/
r  1 .mod q/
8

<q 1 > 2p
Suy ra p 1 > 2r , iu ny v l v p; q; r > 3.

:
r > 2q
+ Vy trong 3 s ordp q; ordq r; ordr p phi c t nht mt s ln hn 2, gi s ordp q D 2r.
Suy ra 2rj p 1, do p  1 .mod r/, p q C 1  2 .mod r/ tri gi thit.
* Vy trong 3 s p, q, r phi c t nht mt s bng 2. Gi s p D 2. Suy ra q, r l cc s
167

Tp ch Epsilon, S 10, 08/2016

nguyn t l, do tng t chng minh trn ta cng c ordr 2 2 f2; 2qg.


Nu ordr 2 D 2 th r D 3 v qj 32 C 1 nn q D 5.
Nu ordr 2 D 2q suy ra 2qj r 1, do r  1 .mod 1/, v vy r 2 C 1  2 .mod q/ mu
thun gi thit).
Vy .p; q; r/ D .2; 3; 5/ v cc hon v ca n.
Cc bi tp sau ta phn tch mt s k nng chn modulo khng c sn. Cc modulo thng
l cc nhn t nguyn t no .
Bi ton 14. Cho p > 5 l s nguyn t, n l s nguyn dng sao cho cc s p 1; pn; n C 1
i mt khng c chung c ln hn 2. Chng minh rng phng trnh sau khng c nghim
nguyn dng x; y
2 C x C x 2 C : : : C x p 1 D y nC1 :
Phn tch v li gii.
Gi s tn ti cc s nguyn dng x; y tha phng trnh
2 C x C x 2 C ::: C x p

D y nC1 :

Vit li (1) :
1 C x C x 2 C ::: C x p

Nu x D 1, ta c

p D y nC1

1 D .y

D y nC1

1/.y n C y n

1:

C ::: C y C 1/;

suy ra y 1 D 1 v p D 2nC1 1.
t ordp 2 D u , u > 3, uj .p 1/ M 2nC1  1 .mod p/ nn uj .n C 1/.
Vy .n C 1; p 1/ > u > 2 mu thun gi thit.
 x > 2, ta c
xp

C xp

xp 1
D y nC1 1
x 1
D .y 1/.y n C y n 1 C ::: C y C 1/:

C ::: C x C 1 D

xp 1
.
x p1
x
1
Xt q l mt c nguyn t bt k ca
.
x 1
Nu x  1 .mod q/ th

Ta kho st cc c nguyn t ca

xp 1
0
D 1 C x C x 2 C ::: C x p
x 1

p

.mod q/

suy ra p D q.
Nu x 6 1 .mod q/, t x p  1 .mod q/ v p l s nguyn t th p D ordq x.
Mt khc x q 1  1 .mod q/ suy ra pj .q 1/.
xp 1
Vy tt c cc c nguyn t q ca
th q D p hoc q D 1 .mod p/.
x 1
Dn ti hai s y 1 v y n C y n 1 C ::: C y C 1 hoc chia ht cho p hoc c s d l 1 khi chia
cho p. (*)
168

Tp ch Epsilon, S 10, 08/2016

TH1: y

1  0 .mod p/ , y  1 .mod p/, suy ra


yn C yn

C ::: C y C 1  n C 1

.mod p/:

Theo (*) suy ra n C 1  1 .mod p/ hoc pj .n C 1/ iu ny mu thun gii thit cc s


p 1; pn; n C 1 i mt khng c chung c ln hn 2.
TH2: y

1  1 .mod p/ , y  2 .mod p/. Khi


yn C yn

C ::: C y C 1  2nC1

.mod p/:

Theo (*) suy ra hoc


2nC1

10

.mod p/

2nC1

11

.mod p/:

hoc
M 2nC1

1  1 .mod p/ cho ta 2nC1

.1/

2  0 .mod p/, suy ra

2n  1

.mod p/:

.2/

t ordp 2 D u; u > 3, uj .p 1/.


T (1) suy ra uj .n C 1/I uj .p 1/ mu thun gi thit.
T (2) suy ra uj nI uj .p 1/ mu thun gi thit.
Tm li trong cc trng hp ta u c iu gi s ban u l sai. Bi ton c chng minh.
Bi ton 15. (Turkey TST 2013) K hiu '.n/ l phi hm Euler ca n. Gii phng trnh
nghim nguyn dng ( n > 2) sau
2n C .n

1/ D nm C 1:

'.n/

Phn tch v li gii.


Ta c:
2n

1 D nm

.n

'.n/

1/:

Ta chn s nguyn t c ca n, m bo pj .n '.n/


n '.n/ 1 > p.
t n D p1r1 :::psrs , vi p1 < p2 < ::: < ps , xt p D p1 .
n

'.n/

Nu s > 2 th

1 D p1r1 :::psrs

p1 :::ps
do n

'.n/

Nu s D 1, n

.p1

n D p1r1

1/:::.ps

:::psrs

p1 :::ps

1/ > p1 :::ps

1 > p.
'.n/

1Dp

r 1

.p

1/

1 < p khi

1/ ta phi xt xem khi no

.p1

1/:::.ps

p1 p2 :::ps 1 .ps


r D1
r D 2; p D 2

TH1 : n D p l s nguyn t, 2p C 1 D p m C 1 suy ra p D 2; m D 2.


TH2: n D 22 , suy ra 24 C 1 D 4m C 1 suy ra m D 2.
169

1/

1/ > p1

1:

Tp ch Epsilon, S 10, 08/2016

TH3: n khng phi l s nguyn t v n > 4.


Gi p l c nguyn t nh nht ca n, gi li chng minh trn ta c n
ra
pj .n '.n/ 1/
2 n D nm C 1

.n

'.n/

1/  1

'.n/

1 > p , suy

.mod p/:

t ordp 2 D u , u > 2, uj .p 1/, m 2n  1.mop p/ nn uj n iu ny v l v p l c


nguyn t nh nht ca n.
Vy phng trnh c 2 nghim .n; m/ D .2I 2/; .4I 2/.
Bi ton 16. (IMO SL 2012) Cho x, y l cc s nguyn dng. Chng minh rng nu x 2
chia ht cho 2n y C 1 vi mi s nguyn dng n th x D 1.

Phn tch v li gii.

n
Nu x 1 v p l mt c nguyn t ca 2n yC1 th pj x 2 1. Suy ra ordp x UCLN.2n ; p
1/.
M UCLN.2n ; p 1/ xc nh trong trng hp c bit p D 4hC3 v UCLN.2n ; p 1/ D 2.
Lc ny
x 2 1  0 .mod p/

Nh vy, ta s i kho st cc c nguyn t dng 4h C 3 ca 2n y C 1. Khi n thay i, liu c


v s s p dng 4h C 3 khng ?
Ta chng minh vi m l s l, c v s s c nguyn t dng 4h C 3ca 2n m C 1 vi n no .
V 2m C 1 D 2.2s C 1/ C 1 D 4s C 3 do 2m C 1 c c nguyn t dng 4h C 3.
Gi s ch c hu hn s nguyn t p1 ; p2 ; :::; pk dng 4h C 3 m pi j 2n m C 1, trong
p1 ; p2 ; :::ps l tt c cc c nguyn t dng 4h C 3 ca 2m C 1.
t
2m C 1 D p1r1 p2r2 :::psrs :q11 :::qtt :
T cc s 2n m C 1, ta c th to dng ly tha quen thuc 2s 1 bng php hiu t nhn t
chung:
2n :m C 1 .2m C 1/ D 2m.2n 1 1/;
chia ht s w, ta ch cn chn n 1 D '.w/, vy w l s no?
Ta c
wj 2n :m C 1 .2m C 1/ , 2n :m C 1  2m C 1 .mod w/;

nh vy trong w nu c nhn t psC1 ; :::pk th psC1 ; :::pk khng l c ca 2n m C 1, do


2n m C 1 ch c cc c nguyn t dng 4h C 1 v pi ; qj , .1 6 i 6 s/. Vy ta chn
w D .2m C 1/:p1 p2 :::ps :psC1 :::pk :q1 q2 :::qt > 2m C 1:
Xt w D .2m C 1/:psC1 :::pk v n D '.w/ C 1. Suy ra
2n :m C 1  2m C 1 .mod .2m C 1/:p1 p2 :::ps :psC1 :::pk :q1 q2 :::qt /

./

Suy ra psC1 ; :::pk khng l c ca 2n m C 1, do 2n m C 1 ch c cc nhn t nguyn t dng


4h C 1 v pi ; qj , .1 6 i 6 s/.
Cng t (*) suy ra 2n :m C 1 D .2m C 1/M , trong M ch gm cc nhn t nguyn t dng
4h C 1. Suy ra
2n m C 1  2m C 1  3 .mod 4/ (v l):
170

Tp ch Epsilon, S 10, 08/2016

Vy c v s s nguyn t p dng 4h C 3 m p l c ca 2n m C 1 vi n no .
Nu y chn ta cng c c iu ny bng cch t y D 2u :m.
Vy c v s s nguyn t p pj x 2 1, do x D 1.
Bi ton 17. Tm tt c cc s nguyn dng n sao cho vi mi s nguyn dng k u tn ti
mt s nguyn dng a sao cho a3 C a k chia ht cho n.
Phn tch v gii.
Vi n D 1 hin nhin tha yu cu bi.
Nu n > 1, gi p l mt c nguyn t ca n. Khi , cho k chy t 0 n p

3

a C a 0 6 a 6 p 1
l h thng d y modulo p (*).
Do a3 C a l s chn nn p > 2.
Nu p D 3,th
a3 C a  a C a  2a

nn

l h thng d y modulo p.
Xt p > 3. Do (*) nn phng trnh

1 ta c

.mod 3/


a3 C a 0 6 a 6 2

a3 C a  03 C 0

.mod p/

khng c nghim a 0 .mod p/; hay phng trnh


x2 
v nghim. Vy tp

.mod p/

x2 1 6 x 6 p

l tp tt c cc khng thng d bc hai modulo p.


Do (*) nn phng trnh
a3 C a  .ka/3 C ka

.mod p/

v nghim vi mi 1 6 a; k 6 p
a2
Hay
Vy phng trnh

1 hay phng trnh



k 2 C k C 1 C 1  0 .mod p/:

k2 C k C 1 

1.a 1 /

k2 C k C 1 

c2

.mod p/:
.mod p/

v nghim .k; c/. Suy ra k 2 C k C 1 l thng d bc hai vi mi k.


Xt tp



p 1

A D f .x/ mod p 0 6 x 6
2
171

.1/

Tp ch Epsilon, S 10, 08/2016

vi f .x/ D x 2 C x C 1.
V f .x/

f .y/ D .x

y/.x C y C 1/, v x C y C 1 < p vi 0 6 x; y 6

1
2

; x y nn

f .x/ mod p f .y/ mod p .


pC1
, suy ra A l tp tt c thng d bc hai modulo p v 0.
Vy jAj D
2
Vi mi b D z 2 2 A, do p l nn tn ti w sao cho z  2wC1 .mod p/. Vy 4w 2 C4wC1 2 A.
Ta c b C 3 D 4.w 2 C w C 1/ D 22 .w 2 C w C 1/ l thng d bnh phng nn b C 3 2 A.
Vy A c tnh cht
8b; b 2 A ) b C 3 2 A:
2
pC1
Do 0; 1 2 A nn jAj > .p 1/ >
v l.
3r
2
Vy p D 3, do n D 3 .
* Vi n D 3r , ta chng minh

3

a C a 0 6 a 6 3r
l h thng d y modulo 3r .
Vi 0 6 a; b 6 3r 1; a b ta c
.a3 C a/

.b 3 C b/ D .a

b/.a2 C b 2 C ab C 1/

4.a2 C ab C b 2 C 1/ D .2a C b/2 C 3b 2 C 4  .2a C b/2 C 1 0

.mod 3/:

Vy
Suy ra

.a3 C a/

.b 3 C b/ D .a

b/.a2 C b 2 C ab C 1/ 0.mod 3r /:


a 3 C a 0 6 a 6 3r

l h thng d y modulo 3r .
Vy gi tr cn tm n l n D 3r . Cui cng chng ti a ra mt s bi tp bn c luyn tp.
Bi tp 1. (Bosnhia TST 2015) Chng minh rng c v s s nguyn dng n, n khng l s
nguyn t sao cho
nj 3n 1 2n 1 :
Bi tp 2. (Bosnhia TST 2014) Tm tt c cc s nguyn khng m x, y sao cho 7x
1.

2:5y D

Bi tp 3. (Php TST 2012) Cho p l s nguyn t. Tm tt c cc s nguyn dng a, b, c


tha
ap C b p D p c :
Bi tp 4. (c TST 2010) Tm tt c cc s nguyn dng m, n tha
3m

7n D 2:

Bi tp 5. (Iran TST 2012) Tm tt c cc s nguyn dng a, b, c sao cho a2 C b 2 C c 2 chia


ht cho 2013.ab C bc C ca/.
Bi tp 6. (Romanian Master in Mathematics 2012) Chng minh rng tn ti v s s nguyn
n
dng n sao cho 22 C1 C 1 chia ht cho n, nhng 2n C 1 khng chia ht cho n.
172

Tp ch Epsilon, S 10, 08/2016

Ti liu
[1] Olympiad Number Theory, Justin Stevens.
[2] http://www.artofproblemsolving.com

173

Tp ch Epsilon, S 10, 08/2016

CHNG MINH BT NG THC BNG


PHNG PHP PHN TCH BNH PHNG VI
S TR GIP CA MY VI TNH
Nguyn Quc Anh
(Thnh ph H Ch Minh)

GII THIU
Vi kh nng hin nay, my tnh gip ta gii c rt nhiu bi ton kh m trc
kia thng b tay. Mc d vy, vn cn mt s ln cc bi ton rt th v nhng cha
c "thut gii" hp l gii chng. Trong s , bi ton phn tch v dng tng bnh
phng trong chng minh bt ng thc l bi ton thng xuyn gp phi. Trong bi
vit ny, tc gi s gii thiu v vic ng dng cc lnh c sn trong phn mm Maple a
(http://www.maplesoft.com/, mt cng c kh quen thuc vi mi ngi,
x l mt s bi bt ng thc; trn c s , xy dng mt chng trnh dng Maple
gii quyt mt s bi ton phc tp hn. b
a

Maple, cng vi cc ngn ng lp trnh khc nh Mathematica, Mathlab, ... thuc th h th


t (Fourth-generation Programming Language) vi cc hm buildin v cng mnh m c th h
tr, gii quyt hu ht cc tnh ton cn thit, t n gin cho n phc tp.
b
Email ca tc gi: bdtilove@live.com)

1. Bi ton m u.
Ta s bt u vi bi ton sau y. Cho cc s thc a, b chng minh rng:
a2 + b2 2ab.
Ta c ngay: a2 + b2 2ab = (a b)2 0.
Xut pht t bt ng thc hin nhin ng: x2 0, li gii ny l v cng d hiu vi mi
ngi. Tuy nhin, vic quy mt a thc nhiu bin bt k v dng tng bnh phng khng phi
l iu d dng cht no, k c vi mt a thc i xng thun nht ba bin.

174

Tp ch Epsilon, S 10, 08/2016

2. Mt vi bi ton hai bin s.


hiu r vai tr ca my vi tnh trong vic phn tch cc bt ng thc v dng bnh phng
ta s bt u vi cc bi ton bt ng thc hai bin s, ni my tnh mang mt sc mnh "p
o" gn nh tuyt i.

V d 1 Cho a, b l cc s thc dng. Chng minh rng:


f (a, b) =

1
1
4
32(a2 + b2 )
+
+

0
a2 b 2 a2 + b 2
(a + b)4

Li gii. Ch vi lnh f actor n gin M aple cho ta phn tch nh sau:


(a4 + 8 a3 b + 6 a2 b2 + 8 ab3 + b4 ) (a b)4
0
f (a, b) =
a2 b2 (a2 + b2 ) (a + b)4

V d 2 Cho a, b l cc s thc dng v ab 1. Chng minh rng:


f (a, b) =

1
1
2
+

0
1 + a2 1 + b2 1 + ab

Li gii. Bng lnh f actor ta thu c kt qu sau:


(ab 1) (a b)2
f (a, b) = 2
(a + 1) (b2 + 1) (ab + 1)
vi ab 1 ta c:
f (a, b) =

1
2
1
+

0
2
2
1+a
1+b
1 + ab

V d 3 Cho a, b l cc s thc dng. Chng minh rng:


f (a, b) =

6
2
2

2
(a + b)
a + b2

Li gii. Ta c:
f (a, b) = 12 .

1
0
(a + b)2
ab +
4

ab (a b)2
0
(a + b)2 (a2 + b2 ) (a2 + 6 ab + b2 )
175

Tp ch Epsilon, S 10, 08/2016

V d 4 Cho cc s thc dng a, b. Chng minh rng:


a2 b
2 a2 + 2 ab
f (a, b) =
+
0
2 a3 + b 3 3 2 a2 + b 2
Li gii. Ta c phn tch nh sau:
f (a, b) =

(a + b) (a b)4
2
.
3 (2 a3 + b3 ) (2 a2 + b2 )

V d 5 Cho cc s thc a, b tha mn a + b = 1. Chng minh rng:

5 10 14
4
4
ab(a + b )
27
Li gii. Nu gi nguyn nh th, ta s khng th no phn tch c, ta s c mt bc chuyn
nho nh l ng bc ha c th phn tch bi ton, s dng iu kin u bi a + b = 1 ta c
php ng bc v phn tch nh sau:



10 14
f (a, b) =
(a + b)6 ab a4 + b4
27

2



5 10 14 2 a2 + 6 10 ab + 2b2 a2
10 + 2 ab + b2
=
54
P
Trong phn tip theo, ta quy c k hiu f (a, b, c) l tng i xng tnh theo bin a, b, c i
vi hm s f .
5

3. Phn tch bnh phng cho cc bi ton ba bin s.


Nh thy mc trn, my tnh c sc mnh p o i vi lp cc bi ton hai bin s.
Nhng vi lp cc bi ton ba bin s th sao?

3.1. Phn tch trc tip


Vi mt s bi ton tng i n gin ta c th phn tch bng hm f actor nh i vi cc bi
ton hai bin s trn.
V d 6 Cho cc s thc a, b, c. Chng minh rng:
X
X
X
X
a4 +
a3 b +
ab3 3
a2 bc
176

Tp ch Epsilon, S 10, 08/2016

Li gii. Ta c th chng minh d dng bng bt ng thc AM GM , tuy nhin y l cc


bin thc. Vi s tr gip ca Maple ta c phn tch nh sau:
X
X
X
X

a4 +
a3 b +
ab3 3
a2 bc = a2 + b2 + c2 ab bc ca (a + b + c)2 0
ng thc xy ra khi a = b = c hoc a + b + c = 0.

V d 7 Cho cc s thc a, b, c tha mn a + b + c 0. Chng minh rng:


X
X
X
X
a3 b 2 +
a2 b 3
a3 bc +
a2 b 2 c
Li gii. Li l mt bi ton vi cc bin thc "kh chu" khin ta khng th p dng AM GM
nh thng l. Ta c phn tch nh sau vi s tr gip ca Maple:
X

X
X
X
X
X
a3 b 2 +
a2 b3
a3 bc +
a2 b2 c =
a2 b 2
a2 bc (a + b + c) 0
ng thc xy ra khi a = b = c hoc a + b + c = 0.
Tuy nhin ta s khng bn nhiu v cc bt ng thc dng ny.

3.2. Mt vi kiu phn tch hn hp


Nu c s dng mt hm f actor v c ngay kt qu s gy nhm chn v trn thc t khi chng
minh mt bt ng thc khng bao gi n gin nh nhng bi ton trn.
V d 8 Cho cc s thc khng m x, y, z tha mn x + y + z = 32. Tm gi tr ln nht ca
biu thc sau:
f (x, y, z) = x3 y + y 3 z + z 3 x
Phn tch. Cho x = 24, y = 8, z = 0 ta c f (x, y, z) = 110592 ta s chng minh y l gi tr
ln nht ca f (x, y, z). Tuy nhin nu c nguyn v c chng minh th rt kh, ta s c mt
bc chuyn nho nh l ng bc ha bt ng thc ny thnh:
27
(x + y + z)4 x3 y + y 3 z + z 3 x
256
Ti y, liu bn c tng h m? R rng cch phn tch trc tip s khng hiu qu vi
bi ton c du bng lch ti bin nh th ny. Nhng hy li du ng thc x = 3y, z = 0.
T y gi cho ta phn tch bi ton v dng:
(x 3y)2n g(x, y, z) + z h(x, y, z) 0.

177

Tp ch Epsilon, S 10, 08/2016

Li gii v thao tc my tnh.


Khng mt tnh tng qut gi s x = max{x, y, z}


> f:=27 (x + y + z)4 256 x3 y + y 3 z + z 3 x ;

f := 27(x + y + z)4 256(x3 y + y 3 z + z 3 x)

d thao tc vi bt ng thc ny ta s khai trin hon ton bt ng thc vi lnh expand.


> g:=expand(f);

g := 27 x4 148 x3 y + 108 x3 z + 162 x2 y 2 + 324 x2 yz + 162 x2 z 2 + 108 xy 3 + 324 xy 2 z +


324 xyz 2 148 z 3 x + 27 y 4 148 y 3 z + 162 y 2 z 2 + 108 yz 3 + 27 z 4
Nh trong bi ta bit ng thc xy ra khi z = 0, thay z = 0 vo g bng lnh subs.
> h:=subs(z=0,g);

h := 27 x4 148 x3 y + 162 x2 y 2 + 108 xy 3 + 27 y 4


> h1:=factor(h);

h1 := (27 x2 + 14 xy + 3 y 2 ) (3 y + x)2
Vy l d on ca chng ta ng c mt na. Ta s tip tc vi na cn li.
> g1:=g-h;

g1 := 108 x3 z + 324 x2 yz + 162 x2 z 2 + 324 xy 2 z + 324 xyz 2 148 z 3 x 148 y 3 z +


162 y 2 z 2 + 108 yz 3 + 27 z 4
> factor(g1);

z (108 x3 + 324 x2 y + 162 x2 z + 324 xy 2 + 324 xyz 148 xz 2 148 y 3 + 162 y 2 z + 108 yz 2 + 27 z 3 )
D thy bt ng thc ny lun ng. Tht vy do:
324x2 y 148y 3 v 162x2 z 2 148xz 3 .
Ta c th vit li li gii ngn gn nh sau:
Khng mt tnh tng qut gi s x = max{x, y, z}, ta c:
f :=27(x + y + z)4 256(x3 y + y 3 z + z 3 x)

= 27 x2 + 14 xy + 3 y 2 (3 y + x)2 +

+ z 108x3 + 176x2 y + 14x2 z + 176xy 2 + 324xyz + (148x2 z 148xz 2 )+

+ (148xy 2 148y 3 ) + 162y 2 z + 108yz 2 + 27z 3 0

ng do x = max{x, y, z}. ng thc xy ra khi x = 24, y = 8, z = 0.


178

Tp ch Epsilon, S 10, 08/2016

V d 9 Cho cc s thc dng x, y, z. Chng minh rng:





2
x2 + y 2 + z 2 (x + y + z) + 3xyz



2 x2 + y 2 + z 2 + (x + y + z)2 x3 y + y 3 z + z 3 x + xyz (x + y + z)

Li gii. Vic phn tch cm a thc cng knh ny thnh tng cc bnh phng tng i kh
v s xut hin ca cm i lng x3 y + y 3 z + z 3 x trong v phi khin khin vic phn tch tr
nn kh khn v gn nh "ph sn". Tuy nhin vi mt cht kho lo ta c phn tch nh sau:



2
x2 + y 2 + z 2 (x + y + z) + 3xyz



2 x2 + y 2 + z 2 + (x + y + z)2 x3 y + y 3 z + z 3 x + xyz (x + y + z)
2
= x3 x2 y + x2 z xy 2 + xyz + xz 2 y 3 y 2 z yz 2 + z 3 +

+ 4 (x y) (y z) (y + z) z x2 + xy + xz + y 2 + yz + z 2 0

Vi y l s nm gia x v z. Cn v lm sao phn tch c, xin dnh li cho bn c nh mt


bi tp rn luyn.
Qua hai v d minh ha trn, hn bn s c cht kh chu v khng c im chung gia hai
bi ton trn?

4. Phng php S.O.S, mt tiu chun chung


Trong phn ny ta s ni khi qut v phng php S.O.S. C th xem l mt tiu chun "chung"
cho vic phn tch cc bt ng thc v dng tng cc bnh phng.

4.1. Dng chnh tc v mt vi tiu chun ca phng php S.O.S


4.1.1. Dng chnh tc
V c bn khi ng trc mt bt ng thc bt k ca ba bin a, b, c ta s tm cch a chng
v dng tng ca cc bnh phng (a b)2 , (b c)2 , (c a)2 k hiu:
Sc (a b)2 + Sa (b c)2 + Sb (c a)2 0.
Phn a v dng chnh tc trn l bc u tin trong cch s dng phng php S.O.S. Nu
may mn c c Sa , Sb v Sc u dng th bi ton c chng minh. Tuy nhin khng phi
lc no ta cng may mn nh th.

179

Tp ch Epsilon, S 10, 08/2016

4.1.2. nh l S.O.S
Xt biu thc:
S = f (a, b, c) = Sa (b c)2 + Sb (c a)2 + Sc (a b)2 ,
Trong Sa , Sb , Sc l cc hm s ca a, b, c, khi
1. Nu Sa , Sb , Sc 0 th S 0.
2. Nu a b c v Sb , Sb + Sc , Sb + Sa 0 th S 0.
3. Nu a b c v Sa , Sc , Sa + 2Sb , Sc + 2Sb th S 0.
4. Nu a b c v Sb , Sc 0, a2 Sb + b2 Sa 0 th S 0.
5. Nu Sa + Sb + Sc 0 v Sa Sb + Sb Sc + Sc Sa 0 th S 0

4.2. Mt vi bi ton minh ha


Tip theo, tc gi xin gii thiu mt s bi ton kh th v, c kt hp dng chng trnh hsos
m tc gi t xy dng trn nn Maple gii quyt. tng thc hin v cch vn hnh ca
n s gii thiu mc sau.
Bi ton 1 Cho cc s thc a, b, c khng m sao cho ab + bc + ca > 0. Chng minh rng:
s
s
s
2
2

a + bc
b + ca
c2 + ab
+
+

6
b2 + bc + c2
c2 + ca + a2
a2 + ab + b2
Li gii. t
A=

a + bc
+
2
b + bc + c2

b + ca
+
2
c + ca + a2

c2 + ab
,
a2 + ab + b2

B = (a2 + bc)2 (b2 + bc + c2 )(2a + b + c)3 + (b2 + ca)(c2 + ca + a2 )(2b + c + a)3


+ (c2 + ab)(a2 + ab + b2 )(2c + a + b)3
p dng bt ng thc Holder ta c

3
A2 B (a2 + bc)(2a + b + c) + (b2 + ca)(2b + c + a) + (c2 + ab)(2c + a + b)

Do ta ch cn chng minh

hX
i3
X
2
(a + bc)(2a + b + c) 6
(a2 + bc)2 (b2 + bc + c2 )(2a + b + c)3
180

Tp ch Epsilon, S 10, 08/2016

n y bc ca bt ng thc kh cao v kh kh x l, v mt ln na my tnh li chng t


c sc mnh. S dng chng trnh hsos, ta c th phn tch bt ng thc trn thnh dng
S.O.S nh sau:
Sc (a b)2 + Sb (c a)2 + Sa (b c)2 0
Trong


Sc = 2(a7 + b7 ) + 9c(a6 + b6 ) + 7ab(a5 + b5 ) + 36abc(a4 + b4 ) + 9a2 b2 + 27abc2 (a3 + b3 )
+ 60a2 b2 c(a2 + b2 ) + 3a3 b3 (a + b) + 72a2 b2 c2 (a + b) + 72a3 b3 c + 6a2 b2 c3

Tng t vi Sa , Sb .
My tnh a ra phn tch ny trong 1.484s1 , mt khc d thy Sa , Sb , Sc 0 do a, b, c 0.
Chng minh hon tt. ng thc xy ra khi a = b = c.
Bi ton 2 Cho cc s thc dng a, b, c hy chng minh bt ng thc sau lun ng:
f (a, b, c) = a3 + b3 + c3 + 3 abc ab (a + b) bc (b + c) ca (c + a) 0
Li gii. S dng phn mm hsos c vit trn nn Maple ta thu c phn tch nh sau:
f (a, b, c) = (a + b c) (a b)2 + (b + c a) (b c)2 + (c + a b) (c a)2
My tnh a ra phn tch ny trong 0.188s, mt khc khng phi Sa , Sb , Sc lun dng.
Khng mt tnh tng qut gi s a b c, ta c
Sb = a + c b 0 v Sb + Sc = 2a 0 v Sb + Sa = 2c 0
nn bt ng thc ny ng theo tiu chun 2.
ng thc xy ra khi a = b = c hoc a = b, c = 0 cng cc hon v tng ng.

4.3. Vi dng v chng trnh hsos


Trong mc ny ta s tm hiu s lc v chng trnh hsos v tng thut ton vn hnh n
nh th no.

4.3.1. Gii thiu khi qut v chng trnh hsos


Chng trnh hsos c vit bng ngn ng lp trnh Maple bi tc gi v mt s "ng nghip"
ngi Trung Quc vo nm 2009. Phin bn 1.0 ca chng trnh ch hot ng i vi cc bt
ng thc dng a thc, n phin bn 2.0 mi hot ng c vi cc bt ng thc dng phn
thc. Phin bn hon chnh 3.0 hon chnh vo nm 2011. Hin chng trnh vn cn trong giai
on pht trin, phin bn tip theo tp trung vo vic pht trin thut ton cho cc bi ton
dng cn. Tuy nhin, vn cn trong giai on kim th nn tc gi cha th cng b rng ri.
1

Tt c cc tnh ton c lin quan n thi gian u c thc hin trn cng mt my tnh s dng b x l
Intel(R) Core(TM) i7-4510U 2.00GHz, b nh Ram 8Gb.

181

Tp ch Epsilon, S 10, 08/2016

4.3.2. Thut ton


Bc mt: Kim tra iu kin cn v ca f (a, b, c):
+ iu kin cn: f (a, a, a) = 0.
+ iu kin : f (a, b, c) l a thc ng bc v i xng.
Nu tha mn c hai iu kin ta i n bc hai. Nu khng kt thc.
Bc hai: Khai sinh a thc c bc n 2, vi n l bc ca a thc f (a, b, c) cn phn tch gn
km vi h s t do.
Bc ba: Gii h t do, sau a ra kt qu nu h t do c nghim.

4.3.3. V d minh ha
V d. Hy thc hin php phn tch S.O.S cho cc a thc sau y:
1. a3 + b3 + c3 (a2 b + b2 c + c2 b)
2. a2 + b3 + c4 ab a2 b a3 c
3. a3 + b3 + c3 a2 b b2 c c2 a
Phn tch.
1. Bc mt: a thc khng c dng hon v vng quanh. Phn tch kt thc.
2. Bc mt: a thc khng c dng hon v vng quanh. Phn tch kt thc.
3. Bc mt: a thc ny tha mn c iu kin cn v , nn ta s thc hin phn tch
S.O.S cho n:
Bc hai: Sinh a thc t do f (a, b, c) c bc n = 3 2 = 1 l
f1 (a, b, c) = m1 a + m2 b + m3 c
T y ta c dng S.O.S cn tm l:
fsos = (am1 + bm2 + cm3 ) (a b)2 + (am3 + bm1 + cm2 ) (b c)2 +
(am2 + bm3 + cm1 ) (c a)2

Gii phng trnh f (a, b, c) = fsos ta c nghim (m1 , m2 , m3 ) = (2, 1, 0).


Vy ta c
a3 + b3 + c3 a2 b b2 c c2 a = (2a + b) (a b)2 + (2b + c) (b c)2 + (2c + a) (c a)2
182

Tp ch Epsilon, S 10, 08/2016

5. Li vn l cc php phn tch bnh phng?


Bn cm thy c cht chn nn v cc php phn tch bnh phng ny? Chng u m h v
rt kh nm bt, thm ch vi phng php S.O.S ta vn phi chng minh nhng bt ng
thc trung gian cc k phc tp d gim i hai bc so vi ban u. Nhng vn khng th ph
nhn s hp dn ca php phn tch bnh phng trong chng minh bt ng thc. p, hon
ho, trong sng v d hiu. Bi ton phn tch v cc dng tng bnh phng bi ton th 17
trong s 23 bi ton ca Hilbert:
Cho mt a thc nhiu bin lun nhn gi tr khng m trn trng s thc, liu n c
th c biu din di dng tng cc bnh phng ca cc hm hu t?
Li gii khng nh c chng minh nm 1927 bi Emil Artin. Sau Charles Delzell
tm ra mt thut ton cho bi ton ny. Tuy nhin li gii li s dng cc kin thc v ton cao
cp, khng tin trnh by trong bi bo ny. B li ta s nghin cu mt "gii thut" khc n
gin hn nhng u hiu qu. Phn ny ca bi bo xin trnh by s lc v cng trnh nghin
cu ca gio s SUI Zhen-lin tnh 2 cho vic phn tch mt a thc bt k v dng tng cc bnh
phng da trn thut ton "sinh a thc t hp".

5.1. Vi dng v lpsos v sosany


Chng trnh lpsos do gio s SUI Zhen-lin t xy dng thut ton v vit trn nn ngn ng
lp trnh Maple, cn sosany do tc gi xy dng li da trn thut ton ca gio s SUI Zhen-lin
cung cp. Tuy hai chng trnh u c vit trn nn ngn ng lp trnh Mapple tuy nhin hiu
nng tnh ton v hiu qu ca hai chng trnh l khc nhau do "trnh " v k nng "lp
trnh" ca hai tc gi l khc nhau.

5.2. Mt vi bi ton minh ha


Bi ton 1 Cho cc s thc x, y, z chng minh rng:
(x y)4 + (y z)4 + (z x)4 +

9
yz(y z)2 0
2

Li gii. Ta c phn tch sau:


X

9
9
3
1
(xy)4 + yz(y z)2 = (y z)2 (y +z)2 + (y z)2 (2xy z)2 + (2xy z)4 0.
2
8
4
8

Hin ang ging dy ti Cao ng du m nng cao Shengli, ng Dinh, Sn ng.

183

Tp ch Epsilon, S 10, 08/2016

Chng trnh lpsos cho ra kt qu sau 3.812s.


Chng trnh sosany cho ra kt qu sau 6.12s.

Bi ton 2 Cho cc s thc dng a, b, c chng minh rng:


s
a b c
11(a2 + b2 + c2 )
+ + +1
+5
b c a
ab + bc + ca
Li gii. Ta c phn tch nh sau:


a b c
+ + +1
b c a

2

11 (a2 + b2 + c2 )
5
ab + ca + bc

"
#
1 a (12ac + 3b2 + 4bc) (b c)2 b (12ab + 4ac + 3c2 ) (c a)2 c (3a2 + 4ab + 12bc) (a b)2
+
+
=
4
bc2 (ab + ac + bc)
ca2 (ab + ac + bc)
ab2 (ab + ac + bc)
"
#
2
2
2
b (2ab ac c2 )
c (a2 ab + 2bc)
1 a (2ac b2 bc)
+
+
0
+
4 bc2 (ab + ac + bc) ca2 (ab + ac + bc)
ab2 (ab + ac + bc)
Chng trnh lpsos cho ra kt qu sau 4.344s.
Chng trnh sosany cho ra kt qu sau 12.364s.

Bi ton 3 Cho cc s thc dng x, y, z, chng minh rng:


x2 + 2z + 1

y 2 + 2x + 1

 64
z 2 + 2y + 1 . (x + y + z) (xy + zx + yz)
9

Li gii. Ta c phn tch nh sau:



 64
y 2 + 2x + 1 z 2 + 2y + 1 . (x + y + z) (xy + zx + yz)
9


2
2
2
= 2x z + 1 (x 1) + 2y x + 1 (y 1)2 + 2z y 2 + 1 (z 1)2 +
4
4
4
+ .z (9y + 2) (x y)2 + .x (9z + 2) (y z)2 + .y (9x + 2) (z x)2 +
9
9
9
2
2
+ (xyz 1) + 3 (xy + xz + yz x y z) +

x2 + 2z + 1

+ (x 1)2 (y z)2 + (y 1)2 (z x)2 + (z 1)2 (x y)2 0

Chng trnh lpsos cho ra kt qu sau 0.75s.


Chng trnh sosany cho ra kt qu sau 2.422s.

184

Tp ch Epsilon, S 10, 08/2016

Bi ton 4 Cho cc s thc dng a, b, c tha mn a + b + c = 3. Chng minh rng:


r
r
r

a
b
c
+
+
3
2+b
2+c
2+a
Li gii. S dng bt ng thc Cauchy-Schwarz ta c:
X r
2 X
a
a X1+a

.
2+b
1+a
2+b
T y ta s c gng chng minh:

Tht vy ta c:

a X1+a
.
3
1+a
2+b

X1+a
a

=
1+a
2+b
P
X [a(7c2 + 3c + 2)(ab 1)2 + (bc + 2a + 4)(a 1)2 (b c)2 ]
Q
Q
0
(1 + a). (2 + a)

Chng trnh lpsos cho ra kt qu sau 2.01s.


Chng trnh sosany cho ra kt qu sau 3.12s.

Bi ton 5 Cho cc s thc x, y, z > 0. Chng minh rng:


X (z + x)(x + y)
18

2
(y + z)(3x + 4yz)
7(x + y + z)
Li gii. Ta c phn tch sau:
P

(z + x)(x + y)
18

=
2
(y + z)(3x + 4yz) 7(x + y + z)
P
1
1
Q
Q

z(1008x3 y + 1191x2 y 2 + 19x2 yz


2
126 (x + y + z) (x + y) (3x + 4yz)
+1008xy 3 + 19xy 2 z + 420xyz 2 + 66z 4 )(x y)4
1
+
63
P
1
Q
Q
z(420x2 z 2 + 254xz 3 + 19y 4 )(x y)2 (x 2z)2
2
(x + y + z) (x + y) (3x + 4yz)
1
+
63
P
1
Q
Q
z(19x4 + 420y 2 z 2 + 254yz 3 )(x y)2 (y 2z)2
(x + y + z) (x + y) (3x2 + 4yz)
0
Chng trnh lpsos cho ra kt qu sau 8s.
Chng trnh sosany cho ra kt qu sau 842.12s.
Trc khi chuyn qua phn tip theo ca bi vit l phn tch k hn v cch x l ng sau cc
dng bin i trn, liu bn c cht n tng g vi cc bi ton v li gii trnh by trn?
185

Tp ch Epsilon, S 10, 08/2016

5.3. Thut ton SUI


V c bn m ni thut ton ca SUI gm c 5 bc c bn:
1. Bc 1: Phng on a thc.
2. Bc 2: Khai sinh tp bin.
3. Bc 3: To nhn t.
4. Bc 4: T hp nhn t v gn h s.
5. Bc 5: Gii h v a ra kt qu.
Khc vi thut ton ca hsos vn ch p dng cho cc a thc i xng thun nht, thut ton
SUI p dng cho mt a thc bt k nn khng cn iu kin cn v nh hsos.

5.4. p dng vo bi ton phn tch bnh phng


cho d hiu ta s thao tc trc tip trn mt bi ton c th, xt a thc:
9
f (x, y, z) = (x y)4 + (y z)4 + (z x)4 + yz(y z)2
2
Bc 1: y l mt a thc bc 4 nu mun phn tch bi ton ny v dng tng cc bnh phng
khng m ta c th d dng phng on c dng ca phn tch m ta cn hng n c dng:

X
n=1

(fn (x, y, z)) +

(in (x, y, z))2 (kn (x, y, z))2

n=1

Bc 2: Khai sinh tp bin:


Nh thu c bc 1, tng bnh phng ca a thc ny l tch ca cc i lng bc mt
to nn, cho nn tp bin ca chng ta s n gin ch gm:
x, y, z
Bc 3: D thy ng thc xy ra khi v ch khi x = y = z. Ta c lng nhn t ca php
phn tch c kh nng s cha cc i lng sau:
x, y, z, x y, y z, z x
Bc 4: T hp v to cc nhn t, vi s tr gip ca Maple ta c t hp sau y:
tohop := [x, y, z, x, y, z, 2x + y, 2x + z, x y, x + y, x z, x y, x + y, x
z, x + z, 2x y, 2x z, 2y + x, 2y + z, y z, y + z, y z, y + z, 2y x, 2y z, 2z +
186

Tp ch Epsilon, S 10, 08/2016

x, 2z + y, z x, 2z x, 2z y, 2x + y + z, x y + z, x + y z, x + 2y z, x
2y + z, x y + z, x + y z, 2x y z, y + x z, y + 2z x, y 2z + x, y + z x]
Vi vi dng code n gin Maple cho ta mt a thc c to nn t tohop nh sau:
2
2
(x + y)2 + M56 (y + z x) (x z)2 + M57 (y + z x) (x + z)2
2
2
M58 (y + z x) (2 x y)2 + M59 (y + z x) (2 x z)2 +
2
2
M79 (y + x z) (y + 2 z x)2 + M80 (y + x z) (y 2z + x)2
2
M81 (y + x z) (y + z x)2 +
2
2
M10 (y) (2x + z)2 + M11 (y) (x y)2

M55 (y + z x)
+
+
+
+

2

Bc 5: Bng vic gii h trn v chn kt qu ph hp ta a ra c kt qu nh sau:


X

9
9
3
1
(x y)4 + yz(y z)2 = (y z)2 (y + z)2 + (y z)2 (2x y z)2 + (2x y z)4 0
2
8
4
8

Nh cc bn thy trn a thc c to ra c (422 )/2 = 882 phn t khc nhau, ni nm


na ta phi gii mt h c 882 bin s khc nhau. Mt con s khng khip, tuy nhin ni i th
cng phi ni li, bin s tuy nhiu nhng ch l h bc nht thun nht - "mt mnh t"
c khai thc trit .

6. Nhn xt
6.1. Li th n t my tnh v phn mm
S dng mt t kin thc Ton hc c bn v tn dng li th s mt ca my tnh l kh nng
tnh ton nhanh v chnh xc, ta c th gii quyt c mt bi ton kh, l phn tch mt
a thc khng m trn trng thc v cc dng tng bnh phng khng m.
Ngoi my tnh th cc phn mm ton hc cng ng vai tr cc k quan trng. K t sau khi
cc th h my tnh c nhn ra i, ri n h thng World Wide Web th vic ra i ca b phn
mm: Maple, Mathematica, Matlab v cc phn mm tnh ton tng t nh hng khng h
nh n vic hc v lm ton trn ton th gii. Vi th vin cc hm ton hc phong ph v
khng ngng pht trin qua tng nm, cc phn mm ni trn ngy cng hon thin hn khng
ch ng dng trong ton hc m cn l cng c c lc cho cc nh khoa hc thuc nhiu lnh
vc khc nhau nh vt l, ha hc, in...
Vi chng, ta c th tit kim rt nhiu thi gian v khng phi vit cc hm ring cho mnh na
m c th p dng trc tip thut ton theo ca mnh. Tt nhin, n vn i hi phi c mt
kin thc tng i v lp trnh.
187

Tp ch Epsilon, S 10, 08/2016

6.2. Nhng hn ch
D c tc tnh ton nhanh v chnh xc, tuy nhin nu "thut ton" khng ti u vn khng
th tn dng c ht sc mnh ca my tnh. Mt khc, cc phn mm ton hc u c pht
trin da trn mt ngn ng lp trnh trung gian nh Java, nn tc thc thi vn cn chm.
Khi bi ton ny c gii quyt th mt bi ton khc li ra i, gi ta c th vit c mt a
thc khng m trn trng thc v cc dng tng bnh phng, nhng lm sao ta bit c liu
a thc c lun dng trn trng thc m vit? Tht may mn, bi ton ny c gii
quyt hon ton bi Wen-Tsun Wu v cc ng nghip. Gio s Yang Lu v cc ng nghip
ca mnh da trn "gii thut" ca Wu v vit nn chng trnh bottema2009 cng l phn
mm hot ng trn nn Maple. Tuy nhin do vit trn nn Maple c chng trnh khng tng
thch trn cc phin bn Maple mi hin nay do c s thay i v c php lp trnh cc phin
bn. n nm 2014 th c chnh tc gi vit li ton b m ngun v hot ng trn tt c cc
phin bn ca Maple3 . Tuy nhin do khun kh bi bo c hn, tc gi s trnh by v chng
trnh ny trong mt bi bo khc.
Mt trong nhng hn ch khc na chnh l Bng n t hp4 nh thy trn mt a thc bc
bn ta phi lm vic vi 861 bin t do. Bi ton s 5 c bc 9 ngn ca my tnh tn 14p30
mi cho ra kt qu. R rng trong tng lai ta cn ci thin thut ton nhiu hn na. Tng hp
cc khuyt im trn ta thy chng trnh c s chm tr n t ba ngun khc nhau:
tr = Time(Thut ton cha ti u) + Time(Ngn ng trung gian) + Time(Bng n t hp)
Nu khc phc c nhng li ny trong tng lai chng trnh s hiu qu v hot ng nhanh
hn rt nhiu.
Mt khc xuyn sut bi bo ta ch lm vic vi cc bt ng thc dng a thc v phn thc,
ch c mt v d dng cn, nhng chng ta x l theo hng bnh phng ph cn. Vic
pht trin thut ton cho bt ng thc dng cn l v cng phc tp, tuy nhin trong tng lai
s c nhng thut ton cho vn ny.

7. Gii thiu mt chng trnh cng loi


V l do nu trn nn tc gi cha th gii thiu m ngun ca chng trnh hsos. Thay
vo , tc gi xin gii thiu m ngun ca chng trnh cng loi (cng vit trn nn Maple)
do tc gi xy dng gii quyt cc bi v cc bt ng thc a thc bc bn ba bin:
3

Phin bn Maple mi nht cho n khi bi bo c vit l 2016.1


Bng n t hp l thut ng dng m t s tng nhanh v t bin ca mt hm s do phi tnh ton ht
cc trng hp t hp khc nhau.
4

188

Tp ch Epsilon, S 10, 08/2016

print("==================================================");
print("prove4");
print("Chuong trinh duoc viet boi Nguyen Quoc Anh");
print("Day la mot chuong trinh ma nguon mo. (Open source code.)");
print("bdtilove@live.com");
print("[[[[3.0]]]]");
print("Copyright(C) 2013-2016");
print("[xprove4,yprove4]");
print("==================================================");
#################################################################
sgm:=proc(expr)
local rap,ex2,ex3,ex:
rap:={a=b,b=c,c=a}:
ex2:=subs(rap,expr):
ex3:=subs(rap,ex2):
ex:=expr+ex2+ex3:
RETURN(ex)
end:
#################################################################
pro:=proc(expr)
local rap,ex2,ex3,ex:
rap:={a=b,b=c,c=a}:
ex2:=subs(rap,expr):
ex3:=subs(rap,ex2):
ex:=expr*ex2*ex3:
RETURN(ex)
end:
#################################################################
prove4:=proc(ineq)
local exp,i,sj,ff,tt,ff1,g,f:
if whattype(ineq)= = then print("This is not an inequality!")
else
g:=rhs(ineq)-lhs(ineq):f:=convert(g,+):sj:=time():exp:={}:tt:=0:
ff1:=unapply(f,a,b,c):
for i from 2 to nops([op(expand(numer(f)))]) do
if degree([op(expand(numer(f)))][i]/subs(a=1,b=1,c=1,
[op(expand(numer(f)))][i]))<>degree([op(expand(numer(f)))][1]
/subs(a=1,b=1,c=1,[op(expand(numer(f)))][1])) then tt:=tt+1:fi:od:
if

tt>0 then print("ERROR, this polynomial is not


homonegeous!"):
elif tt=0 and nops(expand({ff1(a,b,c),ff1(a,c,b)
,ff1(b,c,a),ff1(c,a,b),ff1(c,b,a),ff1(b,a,c)}))>2 then

189

Tp ch Epsilon, S 10, 08/2016


print("ERROR,This form is not circle symmetric!")
elif tt=0 and
nops(expand({ff1(a,b,c),ff1(b,c,a),ff1(c,a,b)}))=1 then if
type(f,symmfunc(a,b,c)) then print("This is a symmetric
polynomial!"):check1(ineq):else print("This is a cyclic
symmetric polynomial!"):check2(ineq):fi:fi:fi:
end:
################################################################
solve01:=proc(ff1)
local m,n,p,g,gg:
m:=coeff(subs({a=a,b=1,c=1},ff1),a^4);
n:=coeff(subs({a=a,b=1,c=0},ff1),a^2);
p:=coeff(subs({a=a,b=1,c=0},ff1),a^3);
g:=coeff(subs({a=a,b=0,c=1},ff1),a^3);
if subs({a=a,b=a,c=a},ff1)=0 and 3*m*(m+n)-p^2-p*g-g^2>=0 and m>0
and p^2+p*g+g^2<>0 then
gg:=sgm((3*m*(a^2-b^2)+(p-g)*a*b-(2*p+g)*b*c+(p+2*g)*c*a)^2/(18*m))
+sgm((3*m*(m+n)-p^2-p*g-g^2)*((p-g)*a*b-(2*p+g)*b*c
+(p+2*g)*c*a)^2/(18*m*(p^2+p*g+g^2))):
else print("Cant give a solution."):fi;
end:
################################################################
solve02:=proc(ff1)
local k,l,o,ff7,ff6,Mm,i,j,gg:
ff7:=sgm((k*a+l*b+o*c)^4);
Mm:=solve(subs(a=1,b=1,c=1,
{op(collect(ff1-ff7,[a,b,c],distributed))}),{k,l,o});
gg:=remove(hastype, {Mm}, {And(complexcons, Not(realcons)),
specfunc(anything, RootOf)});
if gg<>{} then subs(gg[1], ff7) else print("Cant give a
solution."):fi
end:
################################################################
solve03:=proc(ff2)
local m1,m2,m3,m4,m5,deg2,g,amu4,amu3b,amu3c,amu2bmu2,amu2bc,gg:
m1 := simplify(coeff(subs({a = a, b = 1, c = 1}, ff2), a^4));
m2 := simplify(coeff(subs({a = a, b = 1, c = 0}, ff2), a^2));
m3 := simplify(coeff(subs({a = a, b = 1, c = 0}, ff2), a^3));
m4 := simplify(coeff(subs({a = a, b = 0, c = 1}, ff2), a^3));
m5 := simplify(coeff(subs({a = a, b = 1, c = 1}, ff2), a^2)-2*m2);
amu4 :=(x^2+y^2+z^2);
amu3b :=2*(x*m+z*p+y*n);
amu3c :=2*(x*p+z*n+y*m);

190

Tp ch Epsilon, S 10, 08/2016


amu2bmu2 :=(2*x*y+2*y*z+2*z*x+m^2+n^2+p^2);
amu2bc := 2*(x*n+m*p+z*m+n*p+y*p+m*n);
g := sgm((x*a^2+y*b^2+z*c^2+m*a*b+n*b*c+p*c*a)^2);
deg2 := solve({x=1,amu2bc = m5, amu3b = m3, amu3c = m4, amu4 = m1,
amu2bmu2 = m2}, {m, n, p, x, y, z});
gg:=remove(hastype, {deg2}, {And(complexcons, Not(realcons)),
specfunc(anything, RootOf)});
if gg<>{} then subs(gg[1], g) else print("Cant give a
solution."):fi:
end:
################################################################
solve04:=proc(ff)
local k,l,ff7,ff5,Mm,gg:
ff7:=k*(sgm(a^2)-l*sgm(a*b))^2;
Mm:=solve(subs(a=1,b=1,c=1,
{op(collect(ff-ff7,[a,b,c],distributed))}),{k,l});
gg:=remove(hastype, {Mm}, {And(complexcons, Not(realcons)),
specfunc(anything, RootOf)});
if gg<>{} then subs(gg[1], ff7) else print("Cant give a
solution."):fi:
end:
#################################################################
check1:=proc(ineq)
local ff1;
ff1:=convert(rhs(ineq)-lhs(ineq),+);
if solve(subs(b=1,c=1,ff1)>=0,a)=a and
solve(subs(b=0,c=0,ff1)>=0,a)=a then print("This inequality is
true! Try to solving:
"):solve01(ff1),solve02(ff1),solve03(ff1),solve04(ff1);
else print("This inequality is false!"):fi:
end:
################################################################
check2:=proc(ineq)
local m,r,p,q,s,ff2,ff1:
ff1:=convert(rhs(ineq)-lhs(ineq),+);
m:=coeff(subs({a=a,b=1,c=1},ff1),a^4);
ff2:=expand(ff1/m);
r:=coeff(subs({a=a,b=1,c=0},ff2),a^2);
p:=-coeff(subs({a=a,b=1,c=0},ff2),a^3);
q:=-coeff(subs({a=a,b=0,c=1},ff2),a^3);
s := simplify(coeff(subs({a = a, b = 1, c = 1}, ff2), a^2)-2*r);
if s>=p+q-r-1 and s<=2*(r+1)+p+q-p^2-p*q-q^2 then print("This
inequality is true! Try to solving:

191

Tp ch Epsilon, S 10, 08/2016


"):solve01(ff1),solve02(ff1),solve03(ff1),solve04(ff1);
else print("This inequality is false!"):fi:
end:

8. Li cm n
Cui cng tc gi xin gi li cm n su sc n gio s SUI-Zhen Lin v cung cp thut gii
tc gi vit bi bo ny. Cm n em L Hong Long, THPT V Nguyn Gip, Qung Bnh v
chnh sa Latex cho bi bo ny. Cm n anh Hong Ngc Th v anh Nguyn Ngc Giang
cng cc anh, em Din n Ton hc VMF dnh thi gian c v gp cho bi bo hon
thin hn.

Ti liu
[1] Yang Lu, Recent advances in automated theorem proving on inequalities, Journal of
Computer Science and Technology, September 1999, Volume 14, Issue 5, pp 434-446.
[2] SHUI-Zhen Lin, Squares polynomial decomposition., Shengli Oilfield Advanced Training
College,Dongying,Shandong,257000,P.R.China
[3] http://diendantoanhoc.net/topic/157829-ch%E1%BB%A9ng-minh-c%
C3%A1c-b%C4%91t-%C4%91a-th%E1%BB%A9c-b%E1%BA%ADc-4-ba-bi%E1%
BA%BFn-th%E1%BB%B1c-tr%C3%AAn-m%C3%A1y-t%C3%ADnh
[4] Trang web http://mathlinks.ro.

192

Tp ch Epsilon, S 10, 08/2016

MT VI IM C BIT CA PHONG TRO


OLYMPIC TON CA M
Gary Antonick, ngi dch Nguyn V Duy Linh

Trong gii IMO, t vi chc nm nay, v tr s 1 ca i tuyn Trung Quc ti cc k IMO gn


nh khng cn phi bn ci. Thm ch c ngi cn ni na a, na tht Nu Trung Quc
em theo 40 hc sinh th h cng c 40 huy chng vng. V vy, nhng nm no m Trung
Quc khng xp th nht th i tuyn chim v tr ca h lun c c mt s quan tm c
bit. V d nh Bulgaria vo nm 2003; Nga vo nm 2007; Hn Quc vo nm 2012 hay M
vo nm 2015: Cc chuyn gia u tm mt cch no gii thch kt qu t bin ca i
son ngi Trung Quc. Th nhng, ti IMO 2016; khi on M ln th hai lin tip ot ngi
qun qun ti IMO, ngi ta bt u phi suy ngh mt cch nghim tc: Dng nh i tuyn
M khng xp th nht do t bin. Chng ta cng theo di cuc tr chuyn gia nh bo Gary
Antonick vi TS Po-Shen-Loh, trng on M ti IMO 2015 v IMO 2016:

Hnh 1: i tuyn IMO M: Ankan Bhattacharya, Allen Liu, Ashwin Sah, Michael Kural, Yuan Yao,
Junyao Peng, v hun luyn vin Po-Shen Loh - nh ca i hc Carnegie Mellon.

M ginh thng li ti k thi Olympic Ton Hc Quc T (IMO) ln th 57; cuc tranh ti
gii ton uy tn nht th gii dnh cho hc sinh trung hc.
Cuc thi c t chc t ngy 6 n ngy 16 thng 7 ti Hong Kong, bao gm cc on d thi
ca hn 100 nc tham gia. on M ginh thng li vi s im 214 trn im ti a l 252
vt qua Hn Quc .207/ v Trung Quc .204/: Ba on trn cng vi Singapore .196/; i
Loan .175/; Bc Triu Tin .168/; Lin bang Nga .165/; Anh .165/; Hong Kong .161/ v Nht
Bn .156/ l mi on dn u.
Chc mng on M: Cc sinh vin Ankan Bhattacharya, Michael Kural, Allen Liu, Junyao
Peng, Ashwin Sah, and Yuan Yao, hun luyn vin trng Po-Shen Loh, v hun luyn vin ph
Razvan Gelca.
193

Tp ch Epsilon, S 10, 08/2016

Tun ny chng ta s im qua hai bi ton c sc trong s su bi ca nm nay. Cuc tho


lun ca chng ta s c dn dt bi chnh Po-Shen Loh.
y l hai bi ton th thch tun ny ca chng ta.
Thch th nht l Bi 2 ca IMO 2016:
Tm tt c s nguyn dng n sao cho mi ca mt bng n  n c th c in bng mt
trong ba ch ci I, M v O theo cch sau y:
 Trn mi hng v mi ct, mt phn ba cc l I, mt phn ba l M v mt phn ba l O.
 Trn mi ng cho, nu s trn ng cho l bi s ca ba th mt phn ba l I, mt
phn ba l M v mt phn ba l O.
Th thch th hai ca chng ta kh mt cht. y l phn bnh lun ca tin s Loh:
im c sc ca IMO nm nay l s lng ln cc bi ton khng chun mc kt hp nhiu
lnh vc ca ton hc vo trong cng mt vn . Bi ton kh nht ha ra l bi s 3; mt bi
kt hp gia i s, hnh hc v l thuyt s. V bi ny, Hoa K t c s im tng cng
cao nht so vi cc nc khc, gp phn quyt nh trong chin thng cui cng, hai ln lin
tip ot ngi qun qun (2015 v 2016), chm dt 21 nm khng ot gii nht k t ln cui
ot gii nht vo nm 1994: V y l ln u tin i tuyn M ot gii nht 2 nm lin tip.
Chng ta hy th xem qua bi ny. y l bi s 3 ca IMO 2016:
Cho P D A1 A2    Ak l mt a gic li trn mt phng. Cc nh A1 ; A2 ; : : : ; Ak c ta
nguyn v nm trn mt ng trn.Gi S l din tch ca P: Bnh phng di cc cnh ca
P chia ht cho n (n l mt s nguyn dng l). Chng minh rng 2S l mt s nguyn chia ht
cho n:
l cc thch thc cho tun ny. Nu mun, bn c th th sc vi bn bi cn li ca cuc
thi IMO nm nay. ( thi, li gii v bnh lun c th xem bi ca Nguyn Tin Dng s
bo ny, TS). t im cao, bn phi tr li ng mi cu hi v ng thi chng minh cu
tr li ca mnh l ng.

Cuc tr chuyn ngn vi hun luyn vin on M PoShen Loh


Ti c may mn hin din ti Hong Kong tun ri nhn cuc thi IMO, v c gp hun luyn
vin on M sau s kin ny. Sau y l trch dn t cuc hi thoi ca chng ti:
Gary Antonick (G.A): Chc mng chin thng ca anh. Anh c bt ng khi ginh chin thng?
Po-Shen Loh (P-S.L): Chin thng l mt bt ng v gi y n c ngha l Hoa K s mnh
hn trong tng lai. Mt vi iu th v xy ra trong nm nay: Cc hc sinh theo hc chng
trnh bi dng ca M c nh gi kh nng ginh thng li ti cuc thi IMO. nh gi ca
194

Tp ch Epsilon, S 10, 08/2016

h l 40%: i vi ti, con s rt ln. Nu anh nh gi kh nng ny ba nm trc y, anh


s khng c con s phn trm cao n nh vy. C rt nhiu i mnh ti cc cuc thi ny.
iu ng ni l chin thng nm ngoi mang li mt tc ng t tin rt ln.
Anh hon tt mt iu g nh th no khng ch l mt hm s ca ci m anh bit m l anh
tham gia mt hot ng mnh m nh th no. Nm nay, on M i thi vi mt thi nng
nhit CHNG TA S LM C IU . Hoa K thng trc , do ginh chin
thng l iu c th.
Nm nay ng l nm ti kin thit cho Hoa K - c bn thnh vin ca on u khng phi l
hc sinh nm cui ca trng nhng h vn c th chin thng.

Hnh 2: Ni chuyn vi hun luyn vin on M Po-Shen Loh ti Hong Kong sau cuc thi Olympic
Ton Hc Quc T 2016:

G.A: C v nh bi thi nm nay mang nhiu tnh sng to hn nhng nm trc. Theo anh cc
bi thi sng to th no?
P-S.L: Cc bi thi nm nay c bit l bi s 3 m chng ti chia s trn kt hp nhiu
lnh vc khc nhau mt cch sng to. Bt c ni no m chng ta c s kt hp, nh l trong
nh bp, th ni c s sng to.
G.A: on M c chn lc v bi dng nh th no?
P-S.L: Cc hc sinh c chn t vi trm ngn hc sinh thng qua mt chui nhng bi kim
tra chn lc do Hip Hi Ton Hc M (MAA), t chc chnh thc ca M tham gia IMO, chu
trch nhim. i gm su hc sinh c tp hp li trc k thi IMO c bi dng trong
ba tun ri ti i hc Carnegie Mellon, cng do chnh MAA t chc.
Tham gia vo tri bi dng l su em thi nm nay cng vi nm mi bn hc sinh khc c
th tham d vo cc nm sau. Nm nay, chng ta cng bi dng nhng hc sinh quc t - hc
sinh trong cc on IMO ca cc quc gia khc. Chng ta tr chi ph my bay, khch sn, n
ung v hc hnh cho h.
G.A: Xem no, cc anh bi dng hc sinh ca cc on IMO khc?
P-S.L: Mi trong s h, bao gm mt vi hc sinh ot huy chng vng. Chng hn nh hai
trong s bn huy chng vng ca i Singapore c bi dng cng vi on M. Mt trong
s h bnh lun rng mt trong s nhng k thut anh ta dng gii cc bi thi nm nay l hc
t lp bi dng. Thnh cng ln l nm sau chng ti s mi 30 hc sinh quc t.
195

Tp ch Epsilon, S 10, 08/2016

G.A: Nhng nc khc c lm nh vy khng?


P-S.L: Khng vi mc nh vy. iu ny l khc thng. Th nht, cho nhng hc sinh quc
t vo hc mang n cho nhng hc sinh ng u ca M nhng ngi bn bng vai phi v.
Ngi ta lun lun ni vi anh rng nu anh l ngi ti ba nht trong lp hc th anh ang
hc nhm lp. Nn chng ti mang n nhng ngi ngang mc vi su em dn u.
Th hai, khi nhng hc sinh n d thi IMO, h s khng b sc v vn ha. Bt thnh lnh anh
p xung v anh nhn chung quanh - ly cha ti. l on Hn Quc. l on Trung
Quc. l on Singapore. Bn h phi rt l lng.
Ti nh li khi ti tham d IMO, chnh l cm gic m ti tri qua. Nhng nu anh bi
dng chung vi nhau, anh s c ci kinh nghim quc t ba tun ri.
G.A: Ai ngh ra ci tng bi dng nhiu thnh vin ca nhiu on khc nhau?
P-S.L: tng ca ti. Ti ngh ra mt vi iu kh k l.
G.A: Vic bi dng thi IMO trng nh th no?
P-S.L: Chng ti c lp ton t 8 W 30 sng n 3 W 00 chiu, tnh lun gi n tra. Sau chng
ti c seminar vo lc 7 W 30 PM khong mt gi.
C cch mt ngy, chng ti li lm vic n 6 W 00 v chng ti c mt bi kim tra.
y l mt video v kha bi dng ny: An Inside Look at the MAAs Mathematical Olympiad
Summer Program.
Ti s kin m chng ti t chc l mt bu khng kh khc hn. Anh khng phi
nh mt ai . Cc anh ch lm vic vi nhau trong 3 tun ri.
G.A: Chng trnh bi dng ca on M khc vi nhng chng trnh khc nh th no?
P-S.L: Chng trnh bi dng ca on M khng ch trng n nhng k thut. iu tht
s lm vi ngi lo lng l nu bn n vi chng trnh bi dng ca on M, bn c th
khng c hun luyn y .
G.A: iu g tch bit gia nhng em gii nht ca IMO so vi nhng em cn li?
P-S.L: Ti khng mun ni l di truyn. Rt nhiu ngi ni rng anh phi sinh ra vi mt
ci g rt c bit. Trong mt lc no , bn c th thy mt iu g nh vy. Nhng mi
ngi cn bn m ni u nh nhau.
y l mt v d. Gi s ti bo bn nh cu sau y: First of all, what is gravity?
By gi, vi cu tip theo. Bn c th nh v vit li y chang nh vy? Khng th c.

196

Tp ch Epsilon, S 10, 08/2016

Nhng iu ny khng c g l khng th c. Tt c mi ngi Bangladesh c th lm c.


iu m ti mun ni l bn c vi khi nim trong b c ca bn gi l ch ci v t. Khi
bn nhn phin bn Anh vn ca cu hi ny v hp dn, b no ca bn khng nh nhng
ng ngon ngoo u. B no ca bn sn sng vi cc khi nim v sau nn thng
tin li. i vi bn, khng c vn g ng k.
Vy th u l ch khc bit gia nhng em gii nht v nhng em cn li? Nu bn xem li
ton hc nu bn xy dng cu trc khi nim, khi bn l gii vn , bn ang l lun vi
nhng khi nim ln. Bn lm vic vi nhng khi nim ln v sp t chng vi nhau.
khng phi l iu k diu. l liu b no ca anh phn chia bn khi nim, v anh c
th lm vic vi ton b cc khi nim nh nhng i s nguyn thy nh th p t lm vic
vi tng ch ci nh b trong tng lc.
G.A: Cm n anh, Po.
gii thiu nhng khi nim ca k thi IMO nm nay, Hun Luyn Vin Quc Gia Po-Shen
Loh nhn mnh mt vi cu vui ca anh tun ny trn Expii, xem ti y.
Quan tm n k thi Olympic Ton nm sau? Mi tun, Po-Shen Loh cho ng nm cu hi trn
Expii, vi mc t d ti kh. Cng tc vi b phim gn y The Man Who Knew Infinity v
thin ti ton hc Srinivasa Ramanujan, ngi vt qua tnh lp d khng th ng c
thay i tng lai ca ton hc, h lng sc th gii tm kim nhng ti nng ton hc cha
c pht hin, trong Spirit of Ramanujan Talent Search.
Tun ny nh vy n y l ht. Nh thng l, mt khi bn c th c bnh lun cho nhng bi
ng ny, hy s dng Gary Hewitts Enhancer xem cng thc v hnh nh mt cch ng
n nht v gi n cc cu ca bn n: gary.antonick@NYTimes.com.

Li gii
Xem bnh lun ca c gi vo th su v li gii v tm tt ca Po-Shen Loh.

197

Tp ch Epsilon, S 10, 08/2016

BNH LUN THI OLYMPIC TON QUC T


(IMO) 2016
Nguyn Tin Dng
(i hc Toulouse, Php)

thi ngy th nht c 3 bi: Bi 1 l hnh phng, bi 2 dng t hp v bi 3 l hnh t hp.


Nh vy trong mt ngy m c n 2 bi lin quan n t hp, l kiu ton va d va kh
khng cn kin thc g nhiu, nhng cn nhn nh v tm chin lc gii cho tt.
Bi 1 c nh gi l tng i d, ni chung hc sinh no ca Vit Nam cng phi gii c.
Bi ton 1. Cho tam gic BCF vung ti B: Gi A l im nm trn ng thng CF sao
cho FA D FB v F nm gia A v C: Ly im D sao cho DA D DC v AC l phn gic
ca DAB: Ly im E sao cho EA D ED v AD l phn gic ca EAC: Gi M l trung
im ca CF: Gi X l im sao cho AMXE l hnh bnh hnh (AM==EX v AE==MX).
Chng minh rng cc ng thng BD; FX v ME ng qui.
Li gii. D thy E; D; X thng hng v CDA D 180
tm ni tip tam gic ABC:

2; CBA D 90 C suy ra D l

198

Tp ch Epsilon, S 10, 08/2016

Do
DB D DC D DA ) DBF D DFA D ) D 2 .BCF / ) CDF D 90 :
Suy ra DFA D 90 C m DEA D 180 2 nn E l tm ni tip tam gic DFA: Do
C EA nn tam gic EFA cn ti E suy ra EFA D EAF D 2 D CFB: Cho nn ba
im B; E; F thng hng. T d dng chng minh c BMEA l hnh thang cn, suy ra
EB D MA C EX v
FB D FA D MA

MF D EX

MB D EX

EA D EX

ED D DX

kt hp vi MB D EA D MX ta suy ra iu phi chng minh.


Bi ton 2. Cho mt bng vung n  n: Ngi ta mun in cc ch ci I; M; O vo cc
ca bng (mi mt ch ci) sao cho trong mi hng, mi ct th s lng ca mi ch ci
ng bng 31 tng s, v trong mi ng cho c di chia ht cho 3 th cng nh vy. Hi
vi nhng s n no th c th lm c vic ?
(ng cho hiu theo ngha nh nhng ng cho ca bn c).
Bi ny li gii khng di, nhng tm c li gii khng d lm. Nhn xt hin nhin u tin
l n phi chia ht cho 3; th mi c 13 ca n l s nguyn. Ngoi ra cn nhng iu kin g?
Th xt trng hp n gin nht n D 3 thy khng th c.
Trng hp n D 6 th sao? Sau khi th xp nhiu ln khng c, th ta a ra gi thuyt l
n D 6 khng c v s th chng minh.
Trng hp n D 9 th sao? Khi n chia ht cho 9: V c th xp c, thm ch chng minh
c l vi mi s n D 9k chia ht cho 9 u xp c.
Mt cch xp nh sau: Thay v vit I; M; O ta s vit 0; 1; 1 .modulo 3) cho tin. nh s cc
bng cp s .i; j / ch hng i; ct j (i; j i t 1 n n).
 
S vit .i; j / l s 3i C j modulo 3 ( y x l k hiu ch phn nguyn).

Quay li cc trng hp n chia ht cho 3 nhng khng chia ht cho 9: Mun chng minh rng
khng th xp c.
Tng t nh trng hp n chia ht cho 9; ta mun dng ng d theo 3 chng minh. K
hiu cc ch thnh 0; 1; 1 nh lc ny.
Gi s xp c, th th tng tt c cc s bng 0 (thc s bng 0; ch khng ch ng d vi 0
2
modulo 3). By gi ch tt c cc vi ch s c dng .3i C 2; 3j C 2/. C tng cng n3
cc nh vy, v con s ny l mt s nguyn khng chia ht cho 3:
Qua bt k mt no khc th c ng 1 ng hoc l cho vi di chia ht cho 3 hoc l
ngang hoc l dc m s i qua mt trong cc trn. Tng cc s trn cc ng , c tnh c
lp bng 0 (trn mi ng u bng 0) ng thi n bng tng tt c cc s ca bng vung
cng thm 3 ln tng tt c cc s trn cc c nh du pha trn. Suy ra tng cc s trn
cc c nh du bng 0:
199

Tp ch Epsilon, S 10, 08/2016

Cho n y th mi th OK. Nhng mu thun nm ch ny: By gi ch no ghi 0 th thay


bng 1; ch no ghi 1 th thay bng 1; ch no ghi 1 th thay bng 0 (hon v tun hon). Khi
bng mi vn tha mn cc iu kin nu ra, v nh vy tng cc s ca cc c nh
du trn bng mi cng phi bng 0:
Mt khc, khi lm nh vy th mi s c thay i C1 modulo 3; v nh vy tng theo modulo
2
3 phi thnh n3 modulo 3; v khc 0: Mu thun ny cho thy khng tn ti cch xp khi n
khng chia ht cho 9:
Bi ton 3. Cho a gic li ni tip ng trn vi cc nh u l im nguyn v di bnh
phng ca cc cnh u chia ht cho mt s t nhin l N no . Chng minh rng 2 ln
din tch ca a gic cng chia ht cho N:
Bi ny c coi l kh nht ca ngy 1. Thc ra li gii cng khng di nu tm c ng
hng (v nguyn tc chung, khng c bi thi no i hi li gii qu di, v nu qu di th hc
sinh khng lm ni trong thi gian thi).
gii, u tin lm trng hp tam gic. Trong trng hp ny c th chng minh trc tip
bng cch vn dng mt cng thc no tnh din tch tam gic khi bit ta cch nh
(iu kin ni tip ng trn l t tha mn v khng c tc dng g i vi tam gic).
Trng hp a gic: Tm cch quy v trng hp tam gic bng quy np. Tc l tm cch chia
a gic thnh nhiu tam gic, v lm sao vn dng c iu kin ni tip.
C li gii rt hay v ngn gn ca Phm Ngc Mai nh sau
Vi n D 3 ta c

16S 2 D 2.a2 b 2 C b 2 c 2 C c 2 a2 /

a4

b4

c4:

:
:
T bnh phng ca a; b; c u chia ht cho n ta suy ra 16S 2 :: n cho nn 2S :: n (2S nguyn).
Xt k > 4: R rng ta ch cn xt trng hp n D p m .m > 1/ vi p l mt s nguyn t l.
Ta chng minh khng nh sau: Tn ti mt ng cho c bnh phng chia ht cho p:
Ly ng cho e chia a gic cho thnh hai a gic c v s cnh nh hn k: Gi a; b l hai
cnh lin tip ti u mt ca e (xem hnh v)

200

Tp ch Epsilon, S 10, 08/2016

e
d

Theo nh l Ptolemy, ta c
2

ac C bd D ef ) .abcd / D

e2f 2

c 2 a2
2

b2d 2

2

(1)

T cch chn e nn ta c c 2 ; d 2 u chia ht cho p do v tri ca (1) chia ht cho p 2mC2 :


Suy ra e 2 f 2 c 2 a2 b 2 d 2 chia ht cho p mC : Theo gi thit ta c a2 ; b 2 chia ht cho p m :
V vy ta nhn c e 2 f 2 chia ht cho p mC : Suy ra f 2 chia ht cho p m : Nh vy khng nh
c chng minh.
p dng mnh ny bng cch s dng phng php quy np theo s cnh k ca a gic ta
d dng suy ra kt lun ca bi ton.
Bi ton 4. t P .n/ D n2 C n C 1: Tm s t nhin b > 1 nh nht sao cho tn ti dy s
lin tip vi b phn t P .a C 1/; P .a C 2/; : : : ; P .a C b/ (a l s t nhin) sao cho bt k s
no trong dy cng c mt c s chung khc 1 vi mt s khc trong dy.
Li gii. Li gii bi ny thuc loi b ci, tng i d nhng c kh nhiu trng hp cn
phi xt.
Trc ht, rng vi mi n th P .n/ l s l, v P .n/ v P .n C 1/ nguyn t cng nhau
gcd.n2 C n C 1; .n C 1/2 C .n C 1/ C 1/ D gcd.n2 C n C 1; 2n C 2/
D gcd.n2 C n C 1; n C 1/
D gcd.n2 ; n C 1/ D 1:

T suy ra b khng th bng 2; b cng khng th bng 3; v nu dy 3 s th s gia s


nguyn t cng nhau vi 2 s cn li. Vy b D 4 c khng?
Nu b D 4 th s th 3 phi c c chung khc 1 vi s th 1; s th 2 phi c c chung khc
1 vi s th 4: Nh vy cn xt gcd.P .n/; P .n C 2//: Lm tng t nh trn th tnh ra (thut
201

Tp ch Epsilon, S 10, 08/2016

ton Euclid) c chung ln nht l 1 hoc 7; v l 7 khi v ch khi n ng d vi 2 modulo 7:


Nhng khng th c a C 1 v a C 3 u ng d vi 7 modulo 7; nn b D 4 cng khng c.
xem b D 5 c c khng, phi xt n gcd.P .n/; P .n C 3// n khc 0 khi v ch khi n
ng d 1 modulo 3: T , lm tng t nh trn, suy ra cng khng c.
n b D 6 th c. Khi a s phi tha mn cc iu kin sau:
 a C 1 ng d vi 1 modulo 3 ( cho P .a C 1/; P .a C 4/ u chia ht cho 3).
 a C 2 ng d vi 7 modulo 19 ( cho P .a C 2/ v P .a C 6/ u chia ht cho 19).
 a C 3 ng d vi 2 modulo 7 ( cho P .a C 3/ v P .a C 5/ u chia ht cho 7).
Theo nh l thng d th tn ti a nh vy.
Bi ton 5. Cho ng nht thc
.x

1/.x

2/    .x

2016/ D .x

2/    .x

1/.x

2016/:

Xa i N nhn t bc nht t hai bn ca ng nht thc ny, sao cho mi bn cn t nht 1


nhn t, v c mt phng trnh khng c nghim thc. Hi s N nh nht lm c nh
vy l bao nhiu?
Li gii. D thy l N > 2016; v nu chng hn li x
phi, nu khng s c nghim x D 1:

1 bn tri th phi xa n i bn

Phn kh hn l chng minh rng ch cn xa ng 2016 nhn t l . Cch xa c v khng


duy nht. Mt cch xa li hai bn P .x/ v Q.x/ nh sau
P .x/ D .x

2/.x

3/.x

6/.x

7/    .x

2014/.x

2015/;

Q.x/ D .x

1/.x

4/.x

5/.x

8/    .x

2013/.x

2016/:

C th chng minh P .x/ > Q.x/ vi mi s thc x:

Tht vy, trng hp P .x/ > 0 v Q.x/ < 0 (v d nh 3 < x < 4) th hin nhin P .x/ >
Q.x/: Trong trng hp m P .x/ v Q.x/ u dng (v d nh l 4 < x < 5), th ta c cc
bt ng thc
.x 2/.x 3/ > .x 1/.x 4/ > 0;
.x

6/.x

7/ > .x

Nhn vo vi nhau ta c P .x/ > Q.x/:

5/.x

8/ > 0;



Trng hp m c P .x/ v Q.x/ u m, v d 2 < x < 3; khi nhm li theo kiu khc
.x

4/.x

5/ > .x

3/.x

6/ > 0;

.x

8/.x

9/ > .x

7/.x

10/ > 0;

202

Tp ch Epsilon, S 10, 08/2016


.x

2012/.x
.x

2013/ > .x

1/.2016

x/ > .x

2011/.x

2014/ > 0;

2/.2015

x/ > 0:

Nhn vo vi nhau ta c Q.x/ > P .x/; tc l P .x/ > Q.x/ (khng c trng hp no
m P .x/ < 0 < Q.x/ v cc on m ca P .x/ nm trn trong cc on m ca Q.x/).
Xin mi th kim tra vi hai a thc khc
P .x/ D .x 506/.x 507/.x 508/    .x
Q.x/ D tch ca cc nhn t cn li:

5113/;

Bi ny thuc din kh va phi, nhiu bn gii c.


Bi ton con ch (bi s 6) c coi l bi kh nht ca ngy th hai, v l bi chm i tuyn
Vit Nam. Nu nh phn ln cc bn trong on gii c hai bi s 4 v s 5; th bi s 6
khng bn no gii c: Ch c mt bn c 3 im trn 7; v cc bn cn li u 0 im.
Bi ton 6. C N on thng ct nhau tng i mt trn mt phng sao cho khng c 3 on
no ng quy. Nh vy trn mi on c N 1 im nt ct v hai nt u ui. Thy Minh
(Nguyn Khc Minh) chi tr sau: t N con ch vo N u ca N on thng (mi on
1 con). Ri thy v tay N ln. C mi ln thy v tay th con ch nhy t mt nt n nt tip
theo trn on thng ca n (theo hng c nh t u t ch n u kia). Chng minh rng
i) Nu N l th lun c th t ch sao cho khi nhy nh vy, khng c ln no m 2 con
ch u nhy cng vo 1 nt.
ii) Nu N chn th t ch kiu g cng c lc hai con ch nhy cng vo 1 nt mt ln v
tay no .
Bi ny kh ch n l, v hc sinh ca ta ch gii gii cc bi thuc nhng dng quen thuc
cy i ba li cn gp bi l ni chung l ng ngc khng bit phi lm th no. Nu c
c chin lc tt i mc vi cc bi ton l nh bi ny, th chng s khng cn phc tp
qu na (nu qu phc tp ngi ta khng chn lm bi thi). Li gii thc ra cng kh ngn
gn, thm ch c khi cn ngn hn nhng bi khc ca thi. Mt li gii nh sau:
Li gii. Gm c 4 bc:
Bc 1: Ly 1 vng trn to cha tt c cc im ct nhau ca cc on thng bn trong.
Ko di cc on thng sao cho cc im u ui ca chng nm trn vng trn. Sau khi lm
th ta c th nh s th t cc im u ui ny theo vng trn (chng hn theo chiu kim
ng h): Ta nh s k hiu chng t A1 n A2N : D thy l cc im Ai v AN Ci l cng
thuc v mt on thng vi mi i (iu ny suy ra t tnh cht cc on thng u ct nhau).
Bc 2: Quan st rng nu ch t Ai v Ai C1 (hai im st nhau trn vng trn) th th no
chng cng cng u nhau sau mt s bc nhy, v s nt tnh t hai u n giao im ca
hai ng th i v th i C 1 l bng nhau. y l quan st then cht ca bi ton.
203

Tp ch Epsilon, S 10, 08/2016

Bc 3: Gi s ta mun xp ch sao cho chng khng b nhy vo nhau v khng mt tnh tng
qut, gi sa con u tin xp A1 : Khi khng c xp A2 ; suy ra phi xp AnC2 cho
ng th 2; suy ra khng c xp AnC3 ; suy ra phi xp A3 cho ng th 3: Tc l c
phi xp cch mt im 1 ! 3 ! 5 !   
Nu N D 2k l s chn th 1 ! 3 ! 5 !    ! 2k C 1 D N C 1; tc l thnh ra xp 2 con
2 u on thng th nht, v l. Suy ra l khng th xp ch sao cho chng khng nhy vo
nhau trong trng hp ny.
Bc 4: N D 2k C 1 l th sao? C mi 1 cch xp nh trn
1; 3; : : : ; 2k C 1 D N; N C 2; N C 2; : : : ; N C 2k D 2N

1:

Cn chng minh rng cch ny OK. chng minh, ta dng tnh chn l. rng s im
u - ui gia hai con ch bt k theo cch xp ny l mt s l. T d suy ra nu s bc i t
mt con ch n mt nt no l s chn th s bc i t con ch ng cn li n nt
l s l v ngc li. Do chng khng th no nhy n nt ct ti cng mt thi im.
Nhn xt. Lm sao tm c li gii?
Cn c nhng chin lc chung p dng vi nhng bi ton l nh th ny, gm nhng bc nh:
 To ra ci g m bu vu vo (nh trong bc 1, xp th t cc ng m bu vu vo th
t , phn tch tr nn d hn).
 Xt cc trng hp ring n gin tm quy lut chung (xt vi 3; 4; 5; 6 con).
 To ra cc gi thuyt trung gian v tm cch chng minh cc gi thuyt . V d, chng minh
c th xp c ch khi N l, to gi thuyt v chn - l, tc l xp sao cho tnh chn l ca hai con
n 1 u mt l khc nhau. Gi thuyt ny thc ra c th chng minh trc tip c m khng
cn n bc 2 quan st pha trn.
C mt gi thuyt khc cng kh th v ( chng minh l khi N chn th khng xp c): Tn
ti mt im mt m s bc t n c 4 im u ui ca 2 on ct ti n u bng N2 :
Nu gi thuyt ny ng, th c ngha l sau N2 bc s c 2 con ng u. Rt tic l gi thuyt
ny thc ra khng ng, c phn v d khi N D 6: Bi vy khi c cc gi thuyt, th cng cn th
kim tra (vi N nh) xem c ng khng nu khng chng minh c cho N tng qut.
 Quan st tm quy lut. Nh l bc 2 pha trn l mt bc mu cht, lm cho bi ton tr nn
n gin hn.
 Quy np i vi cc bi t hp trong c N ty . Trong bi ton con ch ny, nu quy np
theo s on thng, bng cch mi ln b bt i 1 hay 2 on, th rt kh khn, v cc s bc
nhy n t im u ui n im mt thay i kh phc tp mi khi thm 1 on. Tuy nhin
cc iu nu trong bc 2 v bc 3 pha trn, vit ra mt cch t m, th u c th vit bng
quy np.

204

Tp ch Epsilon, S 10, 08/2016

BI TON HAY LI GII P


Trn Nam Dng
(i hc Khoa hc T nhin - HQG TP.HCM)

GII THIU
Chuyn mc ny c ly cm hng t bi vit ca thy Nguyn Duy Lin v bi ton s
6 trong k thi IMO 2001 vi 5 cch gii khc nhau. Mc ny s dnh vit v nhng
bi ton hay, li gii p v nhng cu chuyn th v xung quanh nhng bi ton v li
gii .
Tn ca chuyn mc c mn t tn ca mt nhm nhng ngi yu ton trn Facebook
do anh Nguyn Vn Li sng lp Bi ton hay Li gii p am m ton hc.
Chuyn mc ghi nhn cc c ca bn c v s chn ng mi k 1; 2 bi ton.
S ny chng ti gii thiu bui tr chuyn ca TS Trn Nam Dng ti Tri h Phng
Nam nm 2016 nhn mt bi ton kh d ca k thi ny, bi s 1.

Bi ton 1 (Olympic Tri h Phng Nam 2016). Gii phng trnh


13.1 2x 2 / 9.1 C 2x 2 /
C p
D 0:
p
1 C x2
1 x2

.1/

Bi ton ny rt n gin, v nhn k, n ch l mt phng trnh bc ba ca x 2 ; v phng trnh


li c nghim c bit. Nhiu bn hc sinh gii c bi ny, v y l li gii m i a
s hc sinh tm ra:
iu kin jxj < 1: Ta thc hin bin i tng ng
2

9.1 C 2x /
13.2x
1/
D p
,
p
1 C x2
1 x2

2x 2

169.2x 2

1>0

1000x 6

1>0

2x 2

750x 2 C 88 D 0

1/2 .1 C x 2 / D 81.1 C 2x 2 /2 .1
2x 2
2.5x 2

1>0
4/.100x 4 C 80x 2

Ta thy trong iu kin 2x 2 1 > 0 th 100x 4 C 80x 2


5x 2 4 D 0; tc l x D p25 :1
1

11/ D 0

11 > 0 nn t y suy ra phi c

Khi c gii thiu bi ny, bn V Quc B Cn cng c xut thm li gii sau y:

205

x2/

Tp ch Epsilon, S 10, 08/2016

Cu chuyn c l dng li v cng khng c g bnh lun. Mt phng trnh v t bnh


thng c gii bng mt phng php bnh thng khng c g c bit. Th nhng ti (Trn
Nam Dng) t cu hi.
Bi ton c gii xong ri. Nhng by gi mi l cu hi kh: Bi ton ny lin quan n mt
bi ton quen thuc no?
Do cc em hc sinh khng on c (qu kh on), ti gi : l bi ton bt ng
thc trong thi Olympic 30/4 nm 1996, khi lp 10.
Bi ton 2 (Olympic 30/4, 1996, khi lp 10). Cho 0 6 x 6 1: Chng minh rng

 p
p
2
2
x 13 1 x C 9 1 C x 6 16:

.2/

Ti t cu hi: Bit l lin quan ri , nhng lin quan th no?


Sau vi giy, mt bn hc sinh tr li D tha thy, nu gi v tri ca bt ng thc l f .x/
th phng trnh f 0 .x/ D 0 chnh l phng trnh bi ton ca chng ta .
ng l nh vy. chng minh (2), ta ch cn chng minh gi tr ln nht ca hm s v tri
bng 16: Nu bit o hm, iu ny s quy v vic gii phng trnh f 0 .x/ D 0 ri xt du
f 0 .x/ tm cc tiu. S gii quen thuc ny nu ln ngha quan trng ca o hm v
cng gii thch cc phng trnh xut hin t u v v sao ta phi hc gii phng trnh.
Ti li t tip cu hi Gii bng o hm th n gin ri, nhng y l bi ton cho khi lp
10. Vy lm sao cc bn y gii c. Ti sao BTC li chn bi ton ny? Ti xin bt m cho cc
bn l bi ton c chn do li gii ch c 1 dng, v khng cn dng n o hm. Cc bn
bit im ri ti x D p25 ; cc bn c th phc dng li li gii 1 dng ca p n khng?.
Sau vi pht, c mt bn hc sinh ln trnh by li gii sau: Theo bt ng thc AM-GM,
 13 
p
p

13 
13x 1 x 2 D
x  2 1 x2 6
x 2 C 4.1 x 2 / ;
2
4

 3
p
p

3
9x 1 C x 2 D
3x  2 1 C x 2 6 9x 2 C 4.1 C x 2 /
2
4

Cng tc bt ng thc trn li, ta c iu phi chng minh. Du bng xy ra khi v ch khi
x > 0; x 2 D 4.1 x 2 / v 9x 2 D 4.1 C x 2 / tc l khi x D p25 :
Ta bin i phng trnh v dng
13.2x 2 1/
D9
2x 2 C 1


1 x2
, 13 1
1 C x2

2
1 C 2x 2

D9

2
1 C x2

1:

n y, ch cn rng v tri l hm lin tc v tng theo n x 2 ; cn v phi l hm lin tc v gim theo x 2 ;


ta chng minh c x 2 phi bng 45 :

206

Tp ch Epsilon, S 10, 08/2016

Ti ni rng chnh l li gii ca p n, ch khc l p n vit gp li nn chng minh


chnh ch c mt dng!
C l v li gii ngn gn m bi ton c chn, li c xp vo v tr bi ton. . . d.
S tht din ra cho thy l mt nhn nh sai lm: ch c duy nht mt th sinh ca k thi gii
c bi ny, l em V c Ph. Em gii bng bt ng thc Cauchy-Schwarz.
Li gii nh sau: Ta c
2
 p
p

2
2
6 .13 C 27/ 13.1
13 1 x C 9 1 C x

T suy ra
 p
x 2 13 1

2
p
x 2 C 9 1 C x 2 6 80x 2 .8


x 2 / C 3.1 C x 2 / D 80.8

5x 2 / 6 4.5x 2 C 8

5x 2 /:

5x 2 /2 D 256:

Ly cn bc hai hai v, ta d c iu phi chng minh.


Trong khi 3 hc tr cng ca ti l L Quang Nm, Nguyn L Lc, Lu Minh c b
tay. Nm cn ni Em th dng o hm m cng khng c. Chc cu y tnh sai, ch
nu tnh ng s dn n phng trnh (1) v gii c ri.

Quay tr li vi hai li gii trn, mt bn hc sinh li thc mc: C c cc li gii ny l do


ta bit im ri x D p25 v tm cch cn bng h s thch hp khi p dng AM-GM. Nhng gi
tr ny u d on ra. Vy phi lm th no?
Ti ni: y chnh l cu hi m ti mun nghe. Li gii 1 dng p n tuy p v ng
ngng m, c th v tay nhng ta cha hc nhiu c , v cc hng s khi p dng AMGM vn l b n. Lm sao tm ra cc hng s ny vi iu kin cha bit im ri. y l
cu tr li: ta dng phng php h s bt nh. Vi hai s dng ; bt k, ta c
 13
p
13 
2
D
x  1 x 6
. 2 x 2 C 1 x 2 /;
13x 1

2


p
p
9
9
9x 1 C x 2 D
x  1 C x 2 6
. 2 x 2 C 1 C x 2 /:

2
p

x2

Cng cc bt ng thc v theo v, ta c


p
13x 1

p
13 2 2
x 2 C 9x 1 C x 2 6
. x C 1
2

Du bng xy ra khi v ch khi 2 x 2 D 1

x2/ C

x2; 2x2 D 1 C x2:

tm gi tr ln nht ca v tri, ta cn chn ; sao cho


i) V phi ca (3) khng ph thuc vo xI
ii) Tn ti x sao cho 2 x 2 D 1

9 2 2
. x C 1 C x 2 /:
2

x2; 2x2 D 1 C x2:


207

.3/

Tp ch Epsilon, S 10, 08/2016

T y ta d dng tm c iu kin i vi ; l h phng trnh

13

. 2

1/ C

2 C 1 D 2

9 2
. C 1/ D 0
2
1

Gii h ny ra, ta c (cng l mt phng trnh bc ba ca ) D 12 ; D 32 : T m c


li gii nh trn. Ch h phng trnh rt ging hay chnh xc hn l tng ng vi phng
trnh (1). Mt ln na l do ta phi hc gii phng trnh, h phng trnh c gii thch.
Cui cng, ti ngh cc bn hc sinh p dng cc phng php tng t gii quyt bi
ton 3 ca k thi:
Bi ton 3 (Olympic Tri h Phng Nam 2016). Mt nh a cht ang v tr A trong sa
mc, cch con ng thng 10 km .AN D 10 km). Trn con ng th xe ca nh a cht c
th chy vi vn tc 50 km/h nhng trn sa mc th n ch chy c vi vn tc 30 km/h. Nh
a cht ang rt kht nc v ng bit rng c mt trm xng P v tr xui theo ng 25
km .NP D 25 km) v c x x Chng Dng p lnh.
A

a) Nh a cht tn bao nhiu pht i t A n P theo ng sa mc?


b) Nu nh a cht i t A n N; sau s dng con ng n P th c nhanh hn i
t A n P theo ng sa mc khng?
c) Hy tm mt cch i nhanh hn cho nh a cht. Cch ca bn l nhanh nht cha?
Cc bn hc sinh Tin Giang gii quyt rt tt bi ton ( ni cu cui tm phng n ti
u) m khng dng n o hm. Cn cc bn th sao?

208

Tp ch Epsilon, S 10, 08/2016

LI GII THI TON QUC T


FORMULA OF UNITY - THE THIRD MILLENNIUM
(TIP THEO)
Ban bin tp Epsilon

Tip theo Epsilon s 9, Ban bin tp xin gii thiu vi bn c li gii ca thi ca k thi Formula of
Unity. Phn 2 gm cc ca khi lp 9, 10, 11.

1. thi dnh cho Khi lp R9


Bi 1 Cc nh ca mt a gic u 12 cnh c t mu xanh v . Bit rng trong 3 nh
bt k to thnh mt tam gic u, c t nht 2 nh mu . Chng minh rng ta c th chn 4
nh to thnh mt hnh vung vi t nht 3 nh .
Li gii.

209

Tp ch Epsilon, S 10, 08/2016

Ta thy c tt c 4 tam gic u ri nhau v 3 hnh vung ri nhau trong a gic cho.
Gi s rng khng tn ti hnh vung no tha mn bi th vi mi hnh vung, c khng
qu 2 nh . Do , tng cng c khng qu 6 nh .
Trong khi , mi tam gic u ng gp t nht 2 nh nn s c tng cng t nht 8 nh
. iu mu thun ny cho ta pcm.
Bi 2 Ta ni mt s nguyn dng l p nu n l tch cc giai tha ca cc s nguyn t
(khng nht thit phi phn bit). Ta gi mt s hu t dng l tt nu n l t s gia hai s
nguyn dng p. Chng minh rng tt c cc s hu t dng u tt.
Li gii. Trc ht, ta thy rng nu mt s hu t dng l tt th tch v thng ca chng
cng u tt.
Ngoi ra, mi s nguyn dng n u c th vit thnh

n!
.
(n1)!

T , ta a bi ton v chng minh mi s nguyn dng u tt v ta s thc hin iu ny


bng quy np.
Vi n = 1, ta c 1 =

2!
2!

l s tt.

Vi n = 2, ta c 2 =

2!2!
2!

Vi n = 3, ta c 3 =

3!
2!

cng l s tt.

cng l s tt.

Khi , vi n 4, nu n l hp s, ta c th vit n thnh tch ca cc s nguyn t nh hn


v theo gi thit quy np, n cng l s tt.
Nu n 4 l s nguyn t, ta vit n =
suy ra n l s tt.

n!
(n1)!

v n 1 l hp s, khi n cng l s tt nn

Theo nguyn l quy np th nhn xt c chng minh. Bi ton c gii quyt.


Chng hn 5 =

5!
4!

5!
234

5!
23 3

5!
(2!)3 3!
2!

5!
.
(2!)2 3!

Bi 3 C 27 con gin tham gia mt cuc chy ua. Trong mi vng s c ba con gin chy.
Mi con gin chy vi tc c nh, khng i gia cc vng ua, v tc ca cc con gin
l i mt khc nhau. Sau mi vng, ngi ta ghi li th t v ch ca cc con gin tham gia
vng ua . Hi 14 vng ua c xc nh chnh xc theo th t hai con gin chy nhanh
nht khng?
210

Tp ch Epsilon, S 10, 08/2016

Li gii. Cu tr li l khng nh.


Ta dng 9 vng u loi 9 con chm nht. 3 vng tip theo chn ra con nhanh nht trong 9 con
nhanh nht v loi 3 con chm nht. Vng 13 chn ra con nhanh nht (v ch) trong 3 con
nhanh nht. Lc ny ch cn 3 con c th ng nh l con ng nh vng 13, con ng nh
trong cuc ua vi con v ch vng 3 trn v con ng nh cuc ua vi con v ch 9
vng u. Dng trn 14 tm ra con th nh t 3 con ny.
Bi 4 Cho tam gic ABC vi B = 30 , C = 105 v D l trung im on thng BC. Tm
gc BAD?
Li gii. H CH vung gc vi AB th suy ra CHD l tam gic u v AHC l tam gic
vung cn ti H. T y suy ra tam gic AHD cn ti H v
BAD = HAD = 15 .
Bi 5 John c 12 que g vi di mi que l mt s nguyn dng khng vt qu 56. Chng
minh rng John c 3 que c th to thnh mt tam gic.
Li gii. Gi s ngc li, khng c 3 que to thnh tam gic. Xp th t chiu di cc que g
a1 a2 ... a12 th t gi thit, ta s suy ra an+2 an+1 + an . T y ln lt suy ra
a3 a2 + a1 2, a4 a3 + a2 3, a5 a4 + a3 5.
C nh th
a6 8, a7 13, a8 21, a9 34, a10 55, a11 89.
iu ny mu thun, suy s John s lun to c tam gic.
Bi 6 Hy tm mt s nguyn dng sao cho tch cc c t nhin ca n l 1090.
Li gii. Ta tm s di dng 2m 5n . Cc c ca s ny c dng
2a 5b vi 0 a m, 0 b n.
T tch cc c bng 10

mn(m+1)(n+1)
4

nn ta a v

mn(m + 1)(n + 1) = 360.


Chn m = 3, n = 5 th ta c mt s tha mn yu cu bi ton l 23 55 = 25000.

211

Tp ch Epsilon, S 10, 08/2016

Bi 7 Tt c chng ta u bit 32 + 42 = 52 . Bn cnh , khng phi ai cng bit rng


102 + 112 + 122 = 132 + 142 . Liu c tn ti hay khng 2015 s nguyn dng lin tip sao cho
tng bnh phng ca 1008 s u tin bng tng bnh phng ca 1007 s sau ?
Li gii. Cu tr li l khng nh, khi thay 1007, 1008 bi cc s lin tip ty .
Gi s s u tin ca dy l n + 1. Ta c:
1008
X

(n + i) = 1008n + 2n

i=1

c ng thc

2015
X

1008n + 2n
n2 + 2n

1008
X

i=1

2015
X
i=1

Thay 1008 = a, ta c

i=1

i 2

i2

1008
X
i=1

1008
X

i2 =

i+

i=1
1008
X

n2 2n

2015
P

i2 v

i=1

i+

2015
X

i2 .

i=1009

(n + i)2 , ta cn xt phng trnh:

i=1009

2015
X

1008
X

i=1009

i=1

i=1

2015
X

(n + i)2 = 1007n2 + 2n

(n + i)2 =

2015
X

i+

i=1

i=1009
1008
P

1008
X

i=1

1008
X

i = 1007n + 2n

i=1
2015
X

i=1009
1008
X

i2

i=

i=1

1008
X

i2

i=1
2015
X
i=1

2015
X

i+

i=1009
2015
X

2015
X

i2

i=1009

i2 = 0

i=1009
1008
X

i2 2

i=1

i2

=0

(2a 1)2a 2a(a + 1)


= a(a 2) v
2

(2a 1)2a(4a 1) 2a(a + 1)(2a + 1)


= a2 (2a 3).
6

Do , ta c phng trnh n2 a(2a 4)n a2 (2a 3) = 0. Phng trnh ny c 2 nghim c


nh l n = a, n = 2a2 3a. Vy ta tm c nghim dng l n = 2029104.

2. thi dnh cho Khi lp R10


Bi 1 Hai ch th Bugs v Roger c cc xem ai nhanh hn. xc nh ngi chin thng,
hai bn quyt nh t chc mt cuc thi. Mi bn th s nhy 50 mt theo mt hng v sau
quay li nhy ngc li. Bit rng, di mi bc nhy ca Bugs l 50 cm v ca Roger l
60cm, nhng th Bugs nhy c 6 bc trong khi Roger ch nhy c 5 bc. Hi ai s ginh
c chin thng?
212

Tp ch Epsilon, S 10, 08/2016

Li gii. Gi s rng thi gian nhy mt bc ca th Bugs l 5 th theo gi thit, thi gian
nhy c mt bc ca th Roger l 6.
nhy c ht 50 2 = 100 mt (i ra l 10000 cm) trong hai lt, Bugs cn thi gian l:
10000 : 50 5 = 1000.


10000
Trong khi , th Roger cn phi thc hin
= 168 bc (v 10000 khng chia ht cho
60
60 nn th cn nhy thm 1 bc). Thi gian th Roger cn dng l 168 6 = 1008.
Do , th Bugs chin thng.
Bi 2 Vi nhng gi tr no ca n th ta c th chia mt hnh vung thnh n hnh ch nht ng
dng sao cho c t nht hai trong s chng l khng bng nhau?
Li gii. R rng n = 2 khng tha mn. Ta s chng minh rng mi n 3 u tha mn.

Trc ht, ta chia hnh vung thnh 2 phn, mt phn hnh ch nht ln bn phi v phn cn
li chia thnh n 1 hnh ch nht nh ng dng vi nhau.
T l cnh ca hnh ch nht mu vng l x+y
, t l cnh ca hnh ch nht mu xanh l
x
cc hnh ch nht ng dng th phi c
x+y
x+y
=
x = (n 1)y.
x
(n 1)y
T l ny chn c nn lun tn ti cch chia tha mn bi.
213

x+y
.
(n1)y

Tp ch Epsilon, S 10, 08/2016

Bi 3 C tn ti hay khng cc s nguyn dng a v b sao cho


lcm(a, b) = lcm(a + 2015, b + 2016)?
y, lcm(a, b) c k hiu cho bi chung nh nht ca hai s a v b.
Li gii. Cu tr li l khng nh. Chng hn, ta chn a = 2015 th cn tm b sao cho
lcm(2015, b) = lcm(4030, b + 2016)
Ta thy rng 2015 = 5 13 31 v 2016 = 25 32 7 nn c th chn b l c ca 2016 vi dng
2a 3b 7 c tng b + 2016 cng ch c c nguyn t thuc {5, 13, 31, 2, 3, 7}.
Ta chn c b = 168. Khi bi chung nh nht ca c hai cp s u l 338520.
Bi 4 Cho tam gic ABC vi B = 30 , C = 105 v D l trung im on thng BC. Tm
gc BAD?
Li gii. Xem li gii phn trc.
Bi 5 Ngi ta in vo cc ca bng vung 10 10 cc s nguyn dng phn bit sao cho
tng cc s trn mi hng, mi ct l bng nhau v nh nht c th. Bit rng, cc s 1, 2, . . . , 9
v 2015 c in trc trn mt ng cho. Hi tng c th nh nht l bao nhiu?
Li gii. Ta xt m hnh sau:
1
2
3
4
5
6
7
8
a1

a2

a3

a4

a5

a6

214

a7

a8

9
a9

b1
b2
b3
b4
b5
b6
b7
b8
b9
2015

Tp ch Epsilon, S 10, 08/2016

t cc s hng cui (tr 2015) l a1 , a2 , . . . , a9 v cc s ct cui (tr 2015) l b1 , b2 , . . . , b9 .


Ta cn c
9
9
X
X
ai =
bi .
i=1

i=1

Ch rng

9
X

ai +

i=1

9
X
i=1

bi 10 + 11 + 12 + + 27 = 333.

Tuy nhin, tng ny phi chn nn ta c th chn


9
X

ai +

i=1

9
X
i=1

Ta xy dng c trng hp

9
P

ai =

9
X

bi = 334

i=1

9
X

bi = 167.

i=1

ai = 10 + 11 + 12 + 13 + 24 + 25 + 26 + 28 + 18 = 167 cn

i=1
9
X

bi = 14 + 15 + 16 + 17 + 19 + 20 + 21 + 22 + 23 = 167.

i=1

Tng ca hng cui v ct cui l 167 + 2015 = 2182 . Khi , ta c th khng qu kh khn
in thm vo cc cn li cc s ln hn 28 v tha mn iu kin bi ton.
Vy tng nh nht l 2182.
Bi 6 ng trn ni tip tam gic ABC tip xc vi cc cnh AB, BC v AC ti cc im
C1 , A1 v B1 tng ng. Chng minh rng
CB
BA
AC
+
+
> 4.
AB1 CA1 BC1
Li gii.
, CA1 = a+bc
, BC1 =
t BC = a, CA = b, AB = c th AB1 = b+ca
2
2
minh rng
2b
2a
2c
+
+
>4
b+ca a+bc c+ab
.
b
a
c

+
+
>2
b+ca a+bc c+ab
Theo BT Cauchy-Schwarz th

c+ab
.
2

b2
a2
c2
(a + b + c)2
+
+
2
.
b(b + c a) a(a + b c) c(c + a b)
a + b2 + c 2
215

Ta cn chng

Tp ch Epsilon, S 10, 08/2016

Ta cn c

(a + b + c)2
> 2 2(ab + bc + ca) > a2 + b2 + c2 .
2
2
2
a +b +c

BT cui ng v c th vit thnh a(b + c a) + b(c + a b) + c(a + b c) > 0.


Bi 7 Tt c chng ta u bit 32 + 42 = 52 . Bn cnh , khng phi ai cng bit rng
102 + 112 + 122 = 132 + 142 . Khng nh sau ng hay sai: Vi mi s nguyn dng k, c
2k + 1 s nguyn dng lin tip sao cho tng bnh phng ca k + 1 s u tin bng tng
bnh phng ca k s cn li?
Li gii. Xem li gii phn trc.

3. thi dnh cho Khi lp R11


Bi 1 Hai ch th Bugs v Roger c cc xem ai nhanh hn. xc nh ngi chin thng,
hai bn quyt nh t chc mt cuc thi. Mi bn th s nhy 50 mt theo mt hng v sau
quay li nhy ngc li. Bit rng, di mi bc nhy ca Bugs l 50 cm v ca Roger l
60cm, nhng th Bugs nhy c 6 bc trong khi Roger ch nhy c 5 bc. Hi ai s ginh
c chin thng?
Li gii. Xem li gii phn trc.
Bi 2 Vi nhng gi tr no ca n th ta c th chia mt hnh vung thnh n hnh ch nht ng
dng sao cho c t nht hai trong s chng l khng bng nhau?
216

Tp ch Epsilon, S 10, 08/2016

Li gii. Xem li gii phn trc.


Bi 3 C tn ti hay khng cc s nguyn dng a v b sao cho
lcm(a, b) = lcm(a + 2015, b + 2016)?
y, lcm(a, b) c k hiu cho bi chung nh nht ca hai s a v b.
Li gii. Xem li gii phn trc.
Bi 4 Cho tam gic ABC vi B = 30 , C = 105 v D l trung im on thng BC. Tm
gc BAD?
Li gii. Xem li gii phn trc.
Bi 5 Ti mi im c ta nguyn trn mt phng ta trng mt cy vi ng knh 106 .
Mt bc tiu phu n cy ti gc ta (0, 0) v ng trn gc cy. Hi phn mt phng m
anh ta nhn thy c b gii hn hay khng? y, cc cy c coi nh l mt ct hnh tr v
hn vi cc trc cha cc im nguyn ca mt phng ta .
Li gii. Gi s anh tiu phu nhn theo hng ca ng thng : y = ma vi m (0; +)
(trng hp cn li chng minh tng t).
Khong cch t mt im K(a, b) vi a, b Z+ th
|b ma|
d(K, ) =
12 + m2
. Do , nu nh hng nhn b che bi gc cy ti K th
|b ma|
1

< 6 |b ma| <


2
10
1+m

1 + m2
.
106

Ta c b sau: Vi mi s v t dng m v vi s > 0 nh ty , lun tn ti hai s nguyn


dng a, b sao cho |b ma| < .
Do , nu m l s v t th s lun tn ti im K che hng nhn trn.
Xt m l s hu t v t m = pq vi p, q Z+ , (p, q) = 1. Khi ng thng ny s i qua
im K(q, p), cng khng tha.
Vy phn mt phng m anh ta nhn thy lun b gii hn.
217

Tp ch Epsilon, S 10, 08/2016

Bi 6 Hy ch ra mt b 4 s dng khng th l bn knh ca bn hnh cu i mt tip xc.


Li gii. Gi R1 , R2 , R3 , R4 l bn knh ca cc hnh cu. Trc ht, ta chn R1 = R2 = R3 = 1
th cc hnh cu u phi i mt tip xc ngoi.

Mt phng i qua cc tm ca 3 hnh cu ct chng to thnh m hnh nh bn di:

Hnh cu th 4 mun tip xc c vi cc hnh cu c th ch c hai kh nng l tip xc v


bn ngoi hoc tip xc pha trong. Ta s chn R4 nh n khng th tip xc trong.

2 3
D dng tnh c t s gia bn knh ng trn nh so vi cc ng trn ln l
1
3
1
0.155. Ta chn R4 =
th b (R1 , R2 , R3 , R4 ) tha mn bi ton.
10
Bi 7 Tt c chng ta u bit 32 + 42 = 52 . Bn cnh , khng phi ai cng bit rng
102 + 112 + 122 = 132 + 142 . Khng nh sau ng hay sai: Vi mi s nguyn dng k, c
2k + 1 s nguyn dng lin tip sao cho tng bnh phng ca k + 1 s u tin bng tng
bnh phng ca k s cn li?
Li gii. Xem li gii phn trc.
218

Tp ch Epsilon, S 10, 08/2016

CC VN C IN V HIN I
Ban bin tp

GII THIU
Chuyn mc ny dnh cho cc vn c in v hin i c trnh by di dng cc
bi ton xu chui. c th l chui cc bi gii bi ton ng chu, chng minh ng
2
thc Euler k diu 1 C 212 C 312 C    D 6 ; mt chui bi ton vn tr ... Cch trnh by
xut pht t nhng vn n gin, d hiu, nhng khi nim mi s c nh ngha
lun trong bi c th c tng i c lp. V mi mt chui bi s nu ra nhng vn
nht nh, c th l gii quyt mt bi ton kinh in hay nu ra nhng gi thuyt mi,
nhng vn mi. Li gii v tho lun v cc bi ton s c ng s N C 3:

219

Tp ch Epsilon, S 10, 08/2016

GII THIU CUC THI HC THUT


ENTROPY KHAI PH D LIU
Ln I, Nm 2016

1. Mc ch
Vi mc tiu khai ph tim nng tri thc ca th h tr Vit Nam v lnh vc khoa hc d liu
(Data Science), vin John von Neumann (JVN), i din khi HQG Tp. HCM t chc cuc
thi Entropy ln 1 nm 2016 nhm to c hi cho cc bn sinh vin, hc vin cao hc c pht
huy nng lc chuyn mn v t duy sng to. y l mt sn chi hc thut chuyn nghip, gip
cc bn sinh vin, hc vin cao hc c c hi thc hnh, pht trin kh nng nghin cu cng
nh tch lu thm kinh nghim trong chuyn ngnh v lnh vc khoa hc my tnh v phn tch
d liu.
Bn cnh , cuc thi Entropy ln u tin t chc ti Vit Nam s l c hi cc bn tr yu
khoa hc c bit n lnh vc khoa hc d liu ang rt nng ti cc nc trn th gii. T
, cuc thi tm kim v mang n nhng ngun nhn lc tim nng cho nn kinh t x hi Vit
Nam trong tng lai. y cng chnh l c hi vin JVN gii thiu v mi trng hc tp,
nghin cu cng cc c hi v hc bng du hc ti cc trng danh ting trn th gii.

2. Th l cuc thi v gii thng


i tng tham gia cuc thi l cc sinh vin, hc vin cao hc t tt c cc trng i hc ti
Vit Nam vi tui t 20 35 tui. Cuc thi c chia l hai bng: Bng A dnh cho i tng
l cc sinh vin nm cui v bng B dnh cho cc hc vin cao hc. Cc th sinh tham d c
th ng k trc tip ti vin JVN hoc ng k trc tip thng qua website ca vin trong thi
gian t ngy 08/03/2016 n ngy 31/05/2016.
Thi gian t chc cuc thi l t ngy 02/04/2016 n ngy 03/07/2016 v tri qua ba vng: Vng
loi (04=06=2016), vng bn kt (18=06=2016) v vng chung kt (02=07=2016 v 03=07=2016).
Nhng th sinh xut sc nht s c c hi nhn c phn thng l cc hc bng ton phn
ti vin JVN v ti cc trng i hc danh ting trn th gii (ParisTech, France v Trinity
College, Dublin, Ireland).

3. Din bin cuc thi


Chnh thc c pht ng vo thng 03 nm 2016: Cuc thi thu ht trn 250 th sinh (c
bng A v bng B) t cc trng i hc trn c nc v c th sinh ang hc tp v nghin cu
ti nc ngoi.
220

Tp ch Epsilon, S 10, 08/2016

a) Vng s loi: c din ra vo ngy 04=06=2016


Hnh thc thi: Online.
Tng s bi thi nhn c qua h thng : 173 bi lm.
Sau khi nhn c kt qu Ban T chc chn ra 150 th sinh tip tc vo vng bn kt.
b) Vng bn kt: Vng bn kt ca cuc thi c t chc ti Vin John von Neumann
Thi gian: 18=06=2016
Hnh thc thi: Gm c 2 phn thi.
 Phn 1 W Th sinh lm bi thi trn giy bng cch tr li nhng cu trc nghim gii ton
v cu hi Story Telling nhm nh gi kh nng phn tch v suy lun ca th sinh.
 Phn 2 W Phn thi Coding, nhm kim tra cc kin thc c bn v lp trnh cng nh kh
nng gii quyt vn .
Sau vng thi Bn kt Ban T chc chn ra 10 th sinh bng A v 6 th sinh bng B tranh ti ti
vng chung kt.
c) Vng thi chung kt:
Thi gian : 02 v 03=07=2016
Hnh thc thi: Trnh by trc hi ng Ban gim kho v bo v ti.
Cc thi sinh vo vng chung kt c nhn mt b d liu v c 01 tun tin hnh phn tch
v gii quyt cc yu cu theo bi a ra da trn b d liu c cung cp.
Cc th sinh c 30 pht (bng A) v 40 pht (Bng B) trnh by cng nh tr li cc cu hi
a ra t cc thnh vin Ban Gim kho.

4. Kt qu cuc thi
Bng A
Gii nht: L Quc Thng i hc Khoa hc T nhin (HQG TP.HCM).
Gii nh: L T ng Khoa i hc FPT.
Gii ba:
 L Vn Duyt i hc Cng ngh Thng tin (HQG TP.HCM).
 Phan Trng Bu i hc Quc t (HQG TP.HCM).
221

Tp ch Epsilon, S 10, 08/2016

 Nguyn c Tr i hc Bch khoa (HQG TP.HCM).


Gii khuyt khch:
 Phm Thng Hi i hc Khoa hc T nhin (HQG TP.HCM).
 Nguyn Nht Nam i hc Bch khoa (HQG TP.HCM).
 Phm Minh Chu i hc Bch khoa (HQG TP.HCM).
 Phm Th Thu Phng Vin John von Neumann.
 Ph Ngc ng Khoa i hc Khoa hc T nhin (HQG TP.HCM).
Bng B
Gii nht: L V Hong Vin John von Neumann.
Gii nh: Hong Nh Thnh i hc Kinh t TP.HCM.
Gii ba: Nguyn Ngc Tun i hc Bch khoa (HQG TP.HCM).
Gii khuyn khch:
 Hong Thanh Tng i hc Cng ngh - HQG HN.
 Phc Ho i hc Bch Khoa Nng.
 Trn Anh Duy i hc Khoa hc T nhin (HQG TP.HCM).

5. thi
Bng A
Phn I phn tch d liu phi cu trc
Mt cng ty A hot ng trong lnh vc nghin cu th trng tin hnh thu thp d liu t
cc trang bo in t Vit Nam kho st xem th hiu ca ngi dn v cc ch x hi v
i sng nh th no. T h tr cho cc cng ty bn hng lm chin lc marketing hiu qu
hn. D liu c ly v, lu trn mt c s d liu di nh dng file vn bn (.txt) m cha
qua bt k khu x l no. Do trong qu trnh ly d liu, cc k thut vin ca cng ty A s
sut qun ghi nh ch cho tng bi vit khi c ti v. Nhng g cng ty A hin c l mt
th mc cha hn 28:000 file vn bn (text), mi file vn bn l ni dung mt bi vit trn mt
trang bo no .

222

Tp ch Epsilon, S 10, 08/2016

Cu hi: Vi s lng bi vit ln nh vy (hn 28:000 bi vit), bn hy tm cch no


nhm cc bi vit theo nhng ch khc nhau. Bn hy xut mt phng php c th
t tn cho tng ch mt cch hp l nht. Kt qu cng ty A mong i s l mt file dng
csv gm 2 ct: Ct 1 l tn bi bo, ct 2 l tn ch tng ng.
File d liu c gi km (Tn file: phan1.zip).

Phn II Phn tch d liu c cu trc


Phn kin thc chun b
Cng ty QK l mt cng ty chuyn sn xut cc loi thc phm M, trong c sn phm tht
trn. Sn phm tht trn c ng gi trong mt lp giy bc cha b ln gin km bt v cc
gia v khc nhau. Ngi mua c th trn ln cc thnh phn nh trng v tht b to ra phn
tht trn. S trn ln ny c tc dng lm tng hng v ca sn phm.
QK l mt thng hiu c uy tn, mc d doanh thu ca cng ty khng qu ln nhng cng ty
lun c li nhun n nh. Gi s bn l gim c thng mi ca QK, v phi xem xt li k
hoch sn xut mt hng tht trn. Nhim v u tin ca bn l chun b d on doanh thu bn
hng, v kinh ph cho khuyn mi v qung co cho nm sau. Bn c d liu lch s ca cng
ty. D liu ny bao gm doanh s bn hng cng nh l cc chi ph dnh cho phn khuyn mi
v qung co trong 24 qu va qua (n v mt ngn USD). Ngoi ra, d liu cng bao gm
ch s index kinh t trong th trng bn tht trn. Gi tr ch s ny cao th hin thi k kinh t
tt. Sn phm tht trn c bn thng qua cc i l thc phm ti Texas, Ohio, Utah, v New
York. Chi ph qung co thng c dng tr cc tp ch v thc phm v sc khe. Chi ph
khuyn mi th tp trung chi tr cho cc i l phn phi v cc qun l ca hng. Cc chi ph
ny bao gm cc khuyn mi c bit, v d nh mua bn tng mt, tng hoa hng cho i l
vi doanh thu cao hay cc cuc thi bn hng gia cc i l vi gii thng l mt chuyn i du
lch Hawaii.
Bn s xem xt d liu lch s v c th thy nhng bin i ln trong doanh s bn hng gia
cc qu, v s khc nhau cho cc chi ph qung co v khuyn mi. Trong mt cuc hp, ph
ch tch bn hng gii thch rng trc y c mt chnh sch chung l ch nn chi tr cho qung
co hoc khuyn mi. Tuy nhin, c mt tranh ci lu di trong cng ty v hiu qu tng
tc gia khuyn mi v qung co i vi doanh s tht trn. Ngi tin nhim c gng th
nhiu phng php so snh khc nhau nhng cha th xc nh c qung co hay khuyn mi
l tt hn.
Mt s kin hoi nghi rng vic dnh chi tiu cho khuyn mi v qung co l lng ph bi
v chng khng nh hng lm n vc bn hng. Mt s ngi khc li cm thy rng vic
khuyn mi c tc ng lm gim doanh s bn hng trong tng lai. Ngha l, h cm thy cc
i l v qun l ca hng mua rt nhiu trong thi gian khuyn mi v sau khng t hng
cc giai on tip theo cho n khi h cn. Tc ng ca qung co cng khng r rt, v doanh
s bn hng thng thay i rt nhiu trong cc giai on m chi ph qung co nh nhau. V
d, trong hai qu 23 v 24 (xem bng d liu), chi ph qung co gn bng nhau (36; 000 USD
v 39; 000 USD) nhng doanh thu tng ng l 648; 000 USD v 343; 000 USD.
223

Tp ch Epsilon, S 10, 08/2016

Ngoi ra, mt chuyn vin thuc phng ti chnh nhn mnh rng th trng tht trn l th
trng phn chu k (counter-cyclical) kinh t, ngha l sn phm bn tt hn trong thi k
kinh t i xung, v ngc li. Anh ta cho rng tht trn r tin hn cc loi thc phm khc,
cho nn ngi ta thng mua nhiu hn trong thi k kh khn. Hn na, anh ta cho rng doanh
thu bn hng c tnh cht ma v, vi nhiu sn phm bn c trong nhng thng lnh hn l
nhng thng nng nh ma h. Ma lnh M l Qu 4 v Qu 1, ma nng ri vo Qu 2 v
Qu 3:
Obs
1
2
3
4
5
6
7
8
9
10
11
12
13
14
15
16
17
18
19
20
21
22
23
24
Mean

Sales
504.72
406.59
398.55
587.76
598.92
703.62
387.24
365.67
388.71
372.96
603.30
614.73
484.38
227.76
329.13
308.25
433.86
514.98
404.70
245.43
433.20
627.24
647.61
342.81
455.51

Prom
15.6
22.2
0.0
0.0
0.0
31.8
21.3
3.9
0.0
8.4
45.3
50.1
39.6
4.2
0.0
0.0
0.0
13.8
17.7
0.0
17.4
37.8
42.3
11.4
16.0

Adv.
30
36
45
57
39
21
12
6
6
30
30
33
6
33
6
3
45
48
0
15
9
54
36
39
26.6

Index
100
102
104
104
104
100
98
96
98
103
105
107
107
107
108
105
103
108
110
112
113
112
113
114
105.5

Ch thch
 Obs (Observation) l d liu thu thp tng qu, bt u t Qu 1:
 Sales l doanh s bn hng ca tht trn theo qu ca QK (ngn USD).
 Prom l chi tiu dng trong cc hot ng khuyn mi trong tng qu (ngn USD).
 Adv l chi tiu dng trong vic qung co trong tng qu (ngn USD).
 Index l ch s kinh t ca th trng.
224

Tp ch Epsilon, S 10, 08/2016

Phn cu hi
1: xut mt m hnh hi quy tuyn tnh (linear regression) d on doanh s bn tht trn
cho QK.
2: Nu bn c $1:000 dnh cho mt trong hai vic qung co v khuyn mi, th bn nn
chn ci no v ti sao? C nhng tc ng nh th no n vic s dng $1:000 trong mi vic
qung co hoc khuyn mi?
3: Bn c ng vi kin ca chuyn vin phng ti chnh rng th trng tht trn c tnh
cht phn chu k (counter-cyclical) so vi ch s kinh t? Ti sao?
4: Bn c ngh rng c tnh cht ma v trong doanh s bn hng hay khng? Ti sao?
Gi tr li cu hi: Cc bn th cn nhc cc yu t sau y:
- Ma nng tng ng vi qu 2 v 3; ma lnh tng ng vi qu 1 v 4:
- iu kin kinh t thay i nh th no.
- nh hng ca Khuyn mi v Qung co c ko di hay khng.

Bng B
Phn I phn tch d liu phi cu trc
Mt cng ty A hot ng trong lnh vc nghin cu th trng tin hnh thu thp d liu t
cc trang bo in t Vit Nam kho st xem th hiu ca ngi dn v cc ch x hi v
i sng nh th no. T h tr cho cc cng ty bn hng lm chin lc marketing hiu qu
hn. D liu c ly v, lu trn mt c s d liu di nh dng file vn bn (.txt) m cha
qua bt k khu x l no. Do trong qu trnh ly d liu, cc k thut vin ca cng ty A s
sut qun ghi nh ch cho tng bi vit khi c ti v. Nhng g cng ty A hin c l mt
th mc cha hn 28:000 file vn bn (text), mi file vn bn l ni dung mt bi vit trn mt
trang bo no .
Cu hi:
1: Vi s lng bi vit ln nh vy (hn 28:000 bi vit), bn hy tm cch no nhm
cc bi vit theo nhng ch khc nhau. Bn hy xut mt phng php c th t tn
cho tng ch mt cch hp l nht. Kt qu cng ty A mong i s l mt file dng csv gm
2 ct: Ct 1 l tn bi bo, ct 2 l tn ch tng ng.
2: Ngoi ra, cng ty A mun bn chn ra mt ch no v nh bn xut mt phng
php t ng nh gi mt bi bo bt k trong ch theo ba mc khc nhau (tch
cc, tiu cc v trung ha). Bn s lm mt chng trnh hon chnh gip cng ty gii quyt
vn ny. Kt qu cng ty A mong i s l mt file dng csv gm 2 ct: Ct 1 l tn bi bo,
ct 2 l nh gi tng ng vi bi bo .
225

Tp ch Epsilon, S 10, 08/2016

Gi : Cc bn xem xt s dng k thut Sentiment Analysis gii quyt vn ny.


File d liu c gi km (Tn file: phan1.zip).

Phn II Phn tch d liu c cu trc


Phn kin thc chun b
Cng ty QK l mt cng ty chuyn sn xut cc loi thc phm M, trong c sn phm tht
trn. Sn phm tht trn c ng gi trong mt lp giy bc cha b ln gin km bt v cc
gia v khc nhau. Ngi mua c th trn ln cc thnh phn nh trng v tht b to ra phn
tht trn. S trn ln ny c tc dng lm tng hng v ca sn phm.
QK l mt thng hiu c uy tn, mc d doanh thu ca cng ty khng qu ln nhng cng ty
lun c li nhun n nh. Gi s bn l gim c thng mi ca QK, v phi xem xt li k
hoch sn xut mt hng tht trn. Nhim v u tin ca bn l chun b d on doanh thu bn
hng, v kinh ph cho khuyn mi v qung co cho nm sau. Bn c d liu lch s ca cng
ty. D liu ny bao gm doanh s bn hng cng nh l cc chi ph dnh cho phn khuyn mi
v qung co trong 24 qu va qua (n v mt ngn USD). Ngoi ra, d liu cng bao gm
ch s index kinh t trong th trng bn tht trn. Gi tr ch s ny cao th hin thi k kinh t
tt. Sn phm tht trn c bn thng qua cc i l thc phm ti Texas, Ohio, Utah, v New
York. Chi ph qung co thng c dng tr cc tp ch v thc phm v sc khe. Chi ph
khuyn mi th tp trung chi tr cho cc i l phn phi v cc qun l ca hng. Cc chi ph
ny bao gm cc khuyn mi c bit, v d nh mua bn tng mt, tng hoa hng cho i l
vi doanh thu cao hay cc cuc thi bn hng gia cc i l vi gii thng l mt chuyn i du
lch Hawaii.
Bn s xem xt d liu lch s v c th thy nhng bin i ln trong doanh s bn hng gia
cc qu, v s khc nhau cho cc chi ph qung co v khuyn mi. Trong mt cuc hp, ph
ch tch bn hng gii thch rng trc y c mt chnh sch chung l ch nn chi tr cho qung
co hoc khuyn mi. Tuy nhin, c mt tranh ci lu di trong cng ty v hiu qu tng
tc gia khuyn mi v qung co i vi doanh s tht trn. Ngi tin nhim c gng th
nhiu phng php so snh khc nhau nhng cha th xc nh c qung co hay khuyn mi
l tt hn.
Mt s kin hoi nghi rng vic dnh chi tiu cho khuyn mi v qung co l lng ph bi
v chng khng nh hng lm n vc bn hng. Mt s ngi khc li cm thy rng vic
khuyn mi c tc ng lm gim doanh s bn hng trong tng lai. Ngha l, h cm thy cc
i l v qun l ca hng mua rt nhiu trong thi gian khuyn mi v sau khng t hng
cc giai on tip theo cho n khi h cn. Tc ng ca qung co cng khng r rt, v doanh
s bn hng thng thay i rt nhiu trong cc giai on m chi ph qung co nh nhau. V
d, trong hai qu 23 v 24 (xem bng d liu), chi ph qung co gn bng nhau (36; 000 USD
v 39; 000 USD) nhng doanh thu tng ng l 648; 000 USD v 343; 000 USD.
Ngoi ra, mt chuyn vin thuc phng ti chnh nhn mnh rng th trng tht trn l th
trng phn chu k (counter-cyclical) kinh t, ngha l sn phm bn tt hn trong thi k
226

Tp ch Epsilon, S 10, 08/2016

kinh t i xung, v ngc li. Anh ta cho rng tht trn r tin hn cc loi thc phm khc,
cho nn ngi ta thng mua nhiu hn trong thi k kh khn. Hn na, anh ta cho rng doanh
thu bn hng c tnh cht ma v, vi nhiu sn phm bn c trong nhng thng lnh hn l
nhng thng nng nh ma h. Ma lnh M l Qu 4 v Qu 1, ma nng ri vo Qu 2 v
Qu 3:
Obs
1
2
3
4
5
6
7
8
9
10
11
12
13
14
15
16
17
18
19
20
21
22
23
24
Mean

Sales
504.72
406.59
398.55
587.76
598.92
703.62
387.24
365.67
388.71
372.96
603.30
614.73
484.38
227.76
329.13
308.25
433.86
514.98
404.70
245.43
433.20
627.24
647.61
342.81
455.51

Prom
15.6
22.2
0.0
0.0
0.0
31.8
21.3
3.9
0.0
8.4
45.3
50.1
39.6
4.2
0.0
0.0
0.0
13.8
17.7
0.0
17.4
37.8
42.3
11.4
16.0

Adv.
30
36
45
57
39
21
12
6
6
30
30
33
6
33
6
3
45
48
0
15
9
54
36
39
26.6

Index
100
102
104
104
104
100
98
96
98
103
105
107
107
107
108
105
103
108
110
112
113
112
113
114
105.5

Ch thch
 Obs (Observation) l d liu thu thp tng qu, bt u t Qu 1:
 Sales l doanh s bn hng ca tht trn theo qu ca QK (ngn USD).
 Prom l chi tiu dng trong cc hot ng khuyn mi trong tng qu (ngn USD).
 Adv l chi tiu dng trong vic qung co trong tng qu (ngn USD).
 Index l ch s kinh t ca th trng.

227

Tp ch Epsilon, S 10, 08/2016

Phn cu hi
1: xut mt m hnh hi quy tuyn tnh (linear regression) d on doanh s bn tht trn
cho QK.
2: Nu bn c $1:000 dnh cho mt trong hai vic qung co v khuyn mi, th bn nn
chn ci no v ti sao? C nhng tc ng nh th no n vic s dng $1:000 trong mi vic
qung co hoc khuyn mi?
3: Bn c ng vi kin ca chuyn vin phng ti chnh rng th trng tht trn c tnh
cht phn chu k (counter-cyclical) so vi ch s kinh t? Ti sao?
4: Bn c ngh rng c tnh cht ma v trong doanh s bn hng hay khng? Ti sao?
Gi tr li cu hi: Cc bn th cn nhc cc yu t sau y:
- Ma nng tng ng vi qu 2 v 3; ma lnh tng ng vi qu 1 v 4:
- iu kin kinh t thay i nh th no.
- nh hng ca Khuyn mi v Qung co c ko di hay khng.

Yu cu ca ban t chc
1: Phn I nh gi v kh nng chuyn mn trong lnh vc khoa hc d liu v kh nng trnh
by bo co. Phn II nh nh gi c v kh nng chuyn mn trong lnh vc khoa hc d
liu, kh nng sng to, kh nng trnh by (bng powerpoint) v kh nng phn bin trc Ban
Gim kho.
2: Phn I v II: Th sinh lm v vit bo co dng word (hoc chuyn thnh file dng pdf) gi
v BTC Cuc thi chm nng lc chuyn mn v trnh by, gi v BTC trc 17 gi ngy
30=6=2016 qua email TS.Nguyn Minh Trung trung.nguyenminh@jvn.edu.vn.
3: Phn II: Th sinh chun b bo co ny trn file dng powerpoint trnh by v tr li cu
hi ca Ban Gim kho trong ngy thi chung kt. Bng A thi ngy 02=07=2016: Bng B thi
ngy 03=07=2016:

228

You might also like